31.01.2015 Views

Woo Young Lee Lecture Notes on Operator Theory

Woo Young Lee Lecture Notes on Operator Theory

Woo Young Lee Lecture Notes on Operator Theory

SHOW MORE
SHOW LESS

You also want an ePaper? Increase the reach of your titles

YUMPU automatically turns print PDFs into web optimized ePapers that Google loves.

1<br />

Graduate Texts<br />

————————————————————————–<br />

<str<strong>on</strong>g>Woo</str<strong>on</strong>g> <str<strong>on</strong>g>Young</str<strong>on</strong>g> <str<strong>on</strong>g>Lee</str<strong>on</strong>g><br />

<str<strong>on</strong>g>Lecture</str<strong>on</strong>g> <str<strong>on</strong>g>Notes</str<strong>on</strong>g> <strong>on</strong><br />

<strong>Operator</strong> <strong>Theory</strong><br />

Spring 2010<br />

Seoul Nati<strong>on</strong>al University<br />

Seoul, Korea<br />

Copyright c 2010 by the Seoul Nati<strong>on</strong>al University


2<br />

.<br />

<str<strong>on</strong>g>Lecture</str<strong>on</strong>g> <str<strong>on</strong>g>Notes</str<strong>on</strong>g> <strong>on</strong> <strong>Operator</strong> <strong>Theory</strong><br />

<str<strong>on</strong>g>Woo</str<strong>on</strong>g> <str<strong>on</strong>g>Young</str<strong>on</strong>g> <str<strong>on</strong>g>Lee</str<strong>on</strong>g>


3<br />

.<br />

Preface<br />

The present lectures are based <strong>on</strong> a graduate course delivered by the author at<br />

the Seoul Nati<strong>on</strong>al University, in the spring semester of 2010.<br />

In these lectures I attempt to set forth some of the recent developments that had<br />

taken place in <strong>Operator</strong> <strong>Theory</strong>. In particular, I focus <strong>on</strong> the Fredholm and Weyl<br />

theory, hyp<strong>on</strong>ormal and subnormal theory, weighted shift theory, Toeplitz theory,<br />

and the invariant subspace problem.<br />

Seoul<br />

February, 2010<br />

The author


C<strong>on</strong>tents<br />

1 Fredholm <strong>Theory</strong> 7<br />

1.1 Introducti<strong>on</strong> . . . . . . . . . . . . . . . . . . . . . . . . . . . . . . . . . 7<br />

1.2 Preliminaries . . . . . . . . . . . . . . . . . . . . . . . . . . . . . . . . 8<br />

1.3 Definiti<strong>on</strong>s and Examples . . . . . . . . . . . . . . . . . . . . . . . . . 10<br />

1.4 <strong>Operator</strong>s with Closed Ranges . . . . . . . . . . . . . . . . . . . . . . 11<br />

1.5 The Product of Fredholm <strong>Operator</strong>s . . . . . . . . . . . . . . . . . . . 14<br />

1.6 Perturbati<strong>on</strong> Theorems . . . . . . . . . . . . . . . . . . . . . . . . . . 16<br />

1.7 The Calkin Algebra . . . . . . . . . . . . . . . . . . . . . . . . . . . . 18<br />

1.8 The Punctured Neighborhood Theorem . . . . . . . . . . . . . . . . . 23<br />

1.9 The Riesz-Schauder (or Browder) <strong>Theory</strong> . . . . . . . . . . . . . . . . 26<br />

1.10 Essential Spectra . . . . . . . . . . . . . . . . . . . . . . . . . . . . . . 32<br />

1.11 Spectral Mapping Theorems . . . . . . . . . . . . . . . . . . . . . . . . 33<br />

1.12 The C<strong>on</strong>tinuity of Spectra . . . . . . . . . . . . . . . . . . . . . . . . . 35<br />

1.13 Comments and Problems . . . . . . . . . . . . . . . . . . . . . . . . . 38<br />

2 Weyl <strong>Theory</strong> 39<br />

2.1 Introducti<strong>on</strong> . . . . . . . . . . . . . . . . . . . . . . . . . . . . . . . . . 39<br />

2.2 Weyl’s Theorem . . . . . . . . . . . . . . . . . . . . . . . . . . . . . . 41<br />

2.3 Spectral Mapping Theorem for the Weyl spectrum . . . . . . . . . . . 53<br />

2.4 Perturbati<strong>on</strong> Theorems . . . . . . . . . . . . . . . . . . . . . . . . . . 59<br />

2.5 Weyl’s theorem in several variables . . . . . . . . . . . . . . . . . . . . 66<br />

2.6 Comments and Problems . . . . . . . . . . . . . . . . . . . . . . . . . 73<br />

3 Hyp<strong>on</strong>ormal and Subnormal <strong>Theory</strong> 77<br />

3.1 Hyp<strong>on</strong>ormal <strong>Operator</strong>s . . . . . . . . . . . . . . . . . . . . . . . . . . . 77<br />

3.2 The Berger-Shaw Theorem . . . . . . . . . . . . . . . . . . . . . . . . 81<br />

3.3 Subnormal <strong>Operator</strong>s . . . . . . . . . . . . . . . . . . . . . . . . . . . 86<br />

3.4 p-Hyp<strong>on</strong>ormal <strong>Operator</strong>s . . . . . . . . . . . . . . . . . . . . . . . . . 97<br />

3.5 Comments and Problems . . . . . . . . . . . . . . . . . . . . . . . . . 101<br />

4 Weighted Shifts 105<br />

4.1 Berger’s theorem . . . . . . . . . . . . . . . . . . . . . . . . . . . . . . 105<br />

4.2 k-Hyp<strong>on</strong>ormality . . . . . . . . . . . . . . . . . . . . . . . . . . . . . . 109<br />

4.3 The Propagati<strong>on</strong> . . . . . . . . . . . . . . . . . . . . . . . . . . . . . . 113<br />

5


CONTENTS<br />

4.4 The Perturbati<strong>on</strong>s . . . . . . . . . . . . . . . . . . . . . . . . . . . . . 120<br />

4.5 The Extensi<strong>on</strong>s . . . . . . . . . . . . . . . . . . . . . . . . . . . . . . . 138<br />

4.6 The Completi<strong>on</strong> Problem . . . . . . . . . . . . . . . . . . . . . . . . . 147<br />

4.7 Comments and Problems . . . . . . . . . . . . . . . . . . . . . . . . . 153<br />

5 Toeplitz <strong>Theory</strong> 157<br />

5.1 Preliminaries . . . . . . . . . . . . . . . . . . . . . . . . . . . . . . . . 157<br />

5.1.1 Fourier Transform and Beurling’s Theorem . . . . . . . . . . . 157<br />

5.1.2 Hardy Spaces . . . . . . . . . . . . . . . . . . . . . . . . . . . . 160<br />

5.1.3 Toeplitz <strong>Operator</strong>s . . . . . . . . . . . . . . . . . . . . . . . . . 162<br />

5.2 Hyp<strong>on</strong>ormality of Toeplitz operators . . . . . . . . . . . . . . . . . . . 169<br />

5.2.1 Cowen’s Theorem . . . . . . . . . . . . . . . . . . . . . . . . . 169<br />

5.2.2 The Case of Trig<strong>on</strong>ometric Polynomial Symbols . . . . . . . . . 173<br />

5.2.3 The Case of Rati<strong>on</strong>al Symbols . . . . . . . . . . . . . . . . . . 176<br />

5.3 Subnormality of Toeplitz operators . . . . . . . . . . . . . . . . . . . . 179<br />

5.3.1 Halmos’s Problem 5 . . . . . . . . . . . . . . . . . . . . . . . . 179<br />

5.3.2 Weak Subnormality . . . . . . . . . . . . . . . . . . . . . . . . 189<br />

5.3.3 Gaps between k-Hyp<strong>on</strong>ormality and Subnormality . . . . . . . 195<br />

5.4 Comments and Problems . . . . . . . . . . . . . . . . . . . . . . . . . 197<br />

6 A Brief Survey <strong>on</strong> the Invariant Subspace Problem 199<br />

6.1 A Brief History . . . . . . . . . . . . . . . . . . . . . . . . . . . . . . . 199<br />

6.2 Basic Facts . . . . . . . . . . . . . . . . . . . . . . . . . . . . . . . . . 201<br />

6.3 Quasitriangular operators . . . . . . . . . . . . . . . . . . . . . . . . . 202<br />

6.4 <strong>Operator</strong>s whose spectra are Carathéodory regi<strong>on</strong>s . . . . . . . . . . . 204<br />

6


Chapter 1<br />

Fredholm <strong>Theory</strong><br />

1.1 Introducti<strong>on</strong><br />

If k(x, y) is a c<strong>on</strong>tinuous complex-valued functi<strong>on</strong> <strong>on</strong> [a, b] × [a, b] then K : C[a, b] →<br />

C[a, b] defined by<br />

(Kf)(x) =<br />

∫ b<br />

a<br />

k(x, y)f(y)dy<br />

is a compact operator. The classical Fredholm integral equati<strong>on</strong>s is<br />

λf(x) −<br />

∫ b<br />

a<br />

k(x, y)f(y)dy = g(x), a ≤ x ≤ b,<br />

where g ∈ C[a, b], λ is a parameter and f is the unknown. Using I to be the identity<br />

operator <strong>on</strong> C[a, b], we can recast this equati<strong>on</strong> into the form (λI − K)f = g. Thus<br />

we are naturally led to study of operators of the form T = λI − K <strong>on</strong> any Banach<br />

space X. Riesz-Schauder theory c<strong>on</strong>centrates attenti<strong>on</strong> <strong>on</strong> these operators of the<br />

form T = λI − K, λ ≠ 0, K compact. The Fredholm theory c<strong>on</strong>centrates attenti<strong>on</strong><br />

<strong>on</strong> operators called Fredholm operators, whose special cases are the operators λI −<br />

K. After we develop the “Fredholm <strong>Theory</strong>”, we see the following result. Suppose<br />

k(x, y) ∈ C[a, b] × C[a, b] (or L 2 [a, b] × L 2 [a, b]). The equati<strong>on</strong><br />

λf(x) −<br />

∫ b<br />

a<br />

k(x, y)f(y)dy = g(x), λ ≠ 0 (1.1)<br />

has a unique soluti<strong>on</strong> in C[a, b] for each g ∈ C[a, b] if and <strong>on</strong>ly if the homogeneous<br />

equati<strong>on</strong><br />

λf(x) −<br />

∫ b<br />

a<br />

k(x, y)f(y)dy = 0, λ ≠ 0 (1.2)<br />

has <strong>on</strong>ly the trivial soluti<strong>on</strong> in C[a, b]. Except for a countable set of λ, which has<br />

zero as the <strong>on</strong>ly possible limit point, equati<strong>on</strong> (1.1) has a unique soluti<strong>on</strong> for every<br />

g ∈ C[a, b]. For λ ≠ 0, the equati<strong>on</strong> (1.2) has at most a finite number of linear<br />

independent soluti<strong>on</strong>s.<br />

7


CHAPTER 1.<br />

FREDHOLM THEORY<br />

1.2 Preliminaries<br />

Let X and Y be complex Banach spaces. Write B(X, Y ) for the set of bounded linear<br />

operators from X to Y and abbreviate B(X, X) to B(X). If T ∈ B(X) write ρ(T ) for<br />

the resolvent set of T ; σ(T ) for the spectrum of T ; π 0 (T ) for the set of eigenvalues<br />

of T .<br />

We begin with:<br />

Definiti<strong>on</strong> 1.2.1. Let X be a normed space and let X ∗ be the dual space of X. If<br />

Y is a subset of X, then<br />

Y ⊥ = {f ∈ X ∗ : f(x) = 0 for all x ∈ Y } = {f ∈ X ∗ : Y ⊂ f −1 (0)}<br />

is called the annihilator of Y . If Z is a subset of X ∗ then<br />

. ⊥ Z = {x ∈ X : f(x) = 0 for all f ∈ Z} = ∩<br />

is called the back annihilator of Z.<br />

f∈Z<br />

f −1 (0)<br />

Even if Y and Z are not subspaces, and Y ⊥ and . ⊥ Z are closed subspaces.<br />

Lemma 1.2.2. Let Y, Y ′ ⊂ X and Z, Z ′ ⊂ X ∗ . Then<br />

(a) Y ⊂ . ⊥ (Y ⊥ ), Z ⊂ ( ⊥ Z) ⊥ ;<br />

(b) Y ⊂ Y ′ =⇒ (Y ′ ) ⊥ ⊂ Y ⊥ ; Z ⊂ Z ′ =⇒ . ⊥ (Z ′ ) ⊂ . ⊥ Z;<br />

(c) ( ⊥ (Y ⊥ )) ⊥ = Y ⊥ , . ⊥ (( ⊥ Z) ⊥ ) = ⊥ Z;<br />

(d) {0} ⊥ = X ∗ , X ⊥ = {0}, . ⊥ {0} = X.<br />

Proof. This is straightforward.<br />

Theorem 1.2.3. Let M be a subspace of X. Then<br />

(a) X ∗ /M ⊥ ∼ = M ∗ ;<br />

(b) If M is closed then (X/M) ∗ ∼ = M ⊥ ;<br />

(c) . ⊥ (M ⊥ ) = cl M.<br />

Proof. See [Go, p.25].<br />

Theorem 1.2.4. If T ∈ B(X, Y ) then<br />

(a) T (X) ⊥ = (T ∗ ) −1 (0);<br />

(b) cl T (X) = . ⊥ (T ∗−1 (0));<br />

(c) T −1 (0) ⊂ . ⊥ T ∗ (Y ∗ );<br />

(d) cl T ∗ (Y ∗ ) ⊂ T −1 (0) ⊥ .<br />

Proof. See [Go, p.59].<br />

8


CHAPTER 1.<br />

FREDHOLM THEORY<br />

Theorem 1.2.5. Let X and Y be Banach spaces and T ∈ B(X, Y ). Then the followings<br />

are equivalent:<br />

(a) T has closed range;<br />

(b) T ∗ has closed range;<br />

(c) T ∗ (Y ∗ ) = T −1 (0) ⊥ ;<br />

(d) T (X) = . ⊥ (T ∗−1 (0)).<br />

Proof. (a) ⇔ (d): From Theorem 1.2.4 (b).<br />

(a) ⇒ (c): Observe that the operator T ∧ : X/T −1 (0) → T X defined by<br />

x + T −1 (0) ↦→ T x<br />

is invertible by the Open Mapping Theorem. Thus we have<br />

T −1 (0) ⊥ ∼ =<br />

(<br />

X/T −1 (0) )∗ ∼ = (T X)<br />

∗ ∼ = T ∗ (Y ∗ ).<br />

(c) ⇒ (b): This is clear because T −1 (0) ⊥ is closed.<br />

(b) ⇒ (a): Observe that if T 1 : X → cl (T X) then T ∗ 1 : (cl T X) ∗ → X ∗ is <strong>on</strong>e-<strong>on</strong>e.<br />

Since T ∗ (Y ∗ ) = ran T ∗ 1 , T ∗ 1 has closed range. Therefore T ∗ 1 is bounded below, so that<br />

T 1 is open; therefore T X is closed.<br />

Definiti<strong>on</strong> 1.2.6. If T ∈ B(X, Y ), write<br />

α(T ) := dim T −1 (0) and β(T ) = dim Y/cl (T X).<br />

Theorem 1.2.7. If T ∈ B(X, Y ) has a closed range then<br />

α(T ∗ ) = β(T ) and α(T ) = β(T ∗ ).<br />

Proof. This follows form the following observati<strong>on</strong>:<br />

T ∗−1 (0) = (T X) ⊥ ∼ = (Y/T X)<br />

∗ ∼ = Y/T X<br />

and<br />

T −1 (0) ∼ = (T −1 (0)) ∗ ∼ = X ∗ /T −1 (0) ⊥ ∼ = X ∗ /T ∗ (Y ∗ ).<br />

9


CHAPTER 1.<br />

FREDHOLM THEORY<br />

1.3 Definiti<strong>on</strong>s and Examples<br />

In the sequel X and Y denote complex Banach spaces.<br />

Definiti<strong>on</strong> 1.3.1. An operator T ∈ B(X, Y ) is called a Fredholm operator if T X<br />

is closed, α(T ) < ∞ and β(T ) < ∞. In this case we define the index of T by the<br />

equality<br />

index (T ) := α(T ) − β(T ).<br />

In the below we shall see that the c<strong>on</strong>diti<strong>on</strong> “ T X is closed ” is automatically fulfilled<br />

if β(T ) < ∞.<br />

Example 1.3.2. If X and Y are both finite dimensi<strong>on</strong>al then any operator T ∈<br />

B(X, Y ) is Fredholm and<br />

indeed recall the “rank theorem”<br />

which implies<br />

index(T ) = dim X − dim Y :<br />

dim X = dim T −1 (0) + dim T X,<br />

index(T ) = dim T −1 (0) − dim Y/T X<br />

= dim X − dim T X − (dim Y − dim T X)<br />

= dim X − dim Y.<br />

Thus in particular, if T ∈ B(X) with dim X < ∞ then T is Fredholm of index zero.<br />

Example 1.3.3. If K ∈ B(X) is a compact operator then T = I − K is Fredholm of<br />

index 0. This follows from the Fredholm theory for compact operators.<br />

Example 1.3.4. If U is the unilateral shift operator <strong>on</strong> l 2 , then<br />

index U = −1 and index U ∗ = −1.<br />

With U and U ∗ , we can build a Fredholm operator whose index is equal to an arbitrary<br />

prescribed integer. Indeed if<br />

[ ] U<br />

p<br />

0<br />

T =<br />

0 U ∗q : l 2 ⊕ l 2 → l 2 ⊕ l 2 ,<br />

then T is Fredholm, α(T ) = q, β(T ) = p, and hence index T = q − p.<br />

10


CHAPTER 1.<br />

FREDHOLM THEORY<br />

1.4 <strong>Operator</strong>s with Closed Ranges<br />

If T ∈ B(X, Y ), write<br />

dist ( x, T −1 (0) ) = inf { ||x − y|| : T y = 0 } for each x ∈ X. (1.3)<br />

If T ∈ B(X, Y ), we define<br />

{<br />

}<br />

γ(T ) = inf c > 0 : ||T x|| ≥ c dist (x, T −1 (0)) for each x ∈ X :<br />

we call γ(T ) the reduced minimum modulus of T .<br />

Theorem 1.4.1. If T ∈ B(X, Y ) then<br />

T (X) is closed ⇐⇒ γ(T ) > 0.<br />

Proof. C<strong>on</strong>sider ̂X = X/T −1 (0) and thus ̂X is a Banach space with norm ||̂x|| =<br />

dist (x, T −1 (0)). Define ̂T : ̂X → Y by ̂T ̂x = T x. Then ̂T is <strong>on</strong>e-<strong>on</strong>e and ̂T ( ̂X) =<br />

T (X).<br />

(⇒) Suppose T X is closed and thus ̂T : ̂X → T X is bijective. By the Open<br />

Mapping Theorem ̂T is invertible with inverse ̂T −1 . Thus<br />

||T x|| = || ̂T ̂x|| ≥<br />

1<br />

|| ̂T −1 || ||̂x|| = 1<br />

|| ̂T −1 || dist (x, T −1 (0)),<br />

which implies that γ(T ) = 1<br />

|| ̂T −1 || > 0.<br />

(⇐) Suppose γ(T ) > 0. Let T x n → y. Then by the assumpti<strong>on</strong> ||T x n || ≥<br />

γ(T ) ||̂x n ||, and hence, ||T x n − T x m || ≥ γ(T ) ||̂x n − ̂x m ||, which implies that (̂x n )<br />

is a Cauchy sequence in ̂X. Thus ̂x n → ̂x ∈ ̂X because ̂X is complete. Hence<br />

T x n = ̂T ̂x n → ̂T ̂x = T x; therefore y = T x.<br />

Theorem 1.4.2. If there is a closed subspace Y 0 of Y for which T (X) ⊕ Y 0 is closed<br />

then T has closed range.<br />

Proof. Define T 0 : X × Y 0 → Y by<br />

T 0 (x, y 0 ) = T x + y 0 .<br />

The space X × Y 0 is a Banach space with the norm defined by<br />

||(x, y 0 )|| = ||x|| + ||y 0 ||.<br />

Clearly, T 0 is a bounded linear operator and ran (T 0 ) = T (X) ⊕ Y 0 , which is closed<br />

by hypothesis. Moreover, ker (T 0 ) = T −1 (0) × {0}. Theorem 1.4.1 asserts that there<br />

exists a c > 0 such that<br />

)<br />

||T x|| = ||T 0 (x, 0)|| ≥ c dist<br />

((x, 0), ker T 0 = c dist ( x, T −1 (0) ) ,<br />

which implies that T (X) is closed.<br />

11


CHAPTER 1.<br />

FREDHOLM THEORY<br />

Corollary 1.4.3. If T ∈ B(X, Y ) then<br />

T (X) is complemented =⇒ T (X) is closed.<br />

In particular, if β(T ) < ∞ then T (X) is closed.<br />

Proof. If T (X) is complemented then we can find a closed subspace Y 0 for which<br />

T (X) ⊕ Y 0 = Y . Theorem 1.4.2 says that T (X) is closed.<br />

To see the importance of Corollary 1.4.3, note that for a subspace M of a Banach<br />

space Y ,<br />

Y = M ⊕ Y 0 does not imply that M is closed.<br />

Take a n<strong>on</strong>-c<strong>on</strong>tinuous linear functi<strong>on</strong>al f <strong>on</strong> Y and put M = ker f. Then there<br />

exists a <strong>on</strong>e-dimensi<strong>on</strong>al subspace Y 0 such that Y = M ⊕ Y 0 (recall that Y/ker (f)<br />

is <strong>on</strong>e-dimensi<strong>on</strong>al). But M = ker f cannot be closed because f is c<strong>on</strong>tinuous if and<br />

<strong>on</strong>ly if f −1 (0) is closed.<br />

C<strong>on</strong>sequently, we d<strong>on</strong>’t guarantee that<br />

dim (Y/M) < ∞ =⇒ M is closed. (1.4)<br />

However Corollary 1.4.3 asserts that if M is a range of a bounded linear operator<br />

then (1.4) is true. Of course, it is true that<br />

M is closed,<br />

dim (Y/M) < ∞ =⇒ M is complemented.<br />

Theorem 1.4.4. Let T ∈ B(X, Y ). If T maps bounded closed sets <strong>on</strong>to closed sets<br />

then T has closed range.<br />

Proof. Suppose T (X) is not closed. Then by Theorem 1.4.1 there exists a sequence<br />

{x n } such that<br />

T x n → 0 and dis ( x n , T −1 (0) ) = 1.<br />

For each n choose z n ∈ T −1 (0) such that ||x n − z n || < 2. Let V := cl {x n − z n : n =<br />

1, 2, . . .}. Since V is closed and bounded in X, T (V ) is closed in Y by assumpti<strong>on</strong>.<br />

Note that T x n = T (x n − z n ) ∈ T (V ). So 0 ∈ T (V ) (T x n → 0 ∈ T (V )) and thus there<br />

exists u ∈ V ∩ T −1 (0). From the definiti<strong>on</strong> of V it follows that<br />

||u − (x n0 − z n0 )|| < 1 2<br />

for some n 0 ,<br />

which implies that<br />

dis ( x n0 , T −1 (0) ) < 1 2 .<br />

This c<strong>on</strong>tradicts the fact that dist ( x n , T −1 (0) ) = 1 for all n. Therefore T (X) is<br />

closed.<br />

12


CHAPTER 1.<br />

FREDHOLM THEORY<br />

Theorem 1.4.5. Let K ∈ B(X). If K is compact then T = I − K has closed range.<br />

Proof. Let V be a closed bounded set in X and let<br />

y = lim<br />

n→∞ (I − K)x n, where x n ∈ V. (1.5)<br />

We have to prove that y = (I − K)x 0 for some x 0 ∈ V . Since V is bounded and K<br />

is compact the sequence {Kx n } has a c<strong>on</strong>vergent subsequence {Kx ni }. By (1.5), we<br />

see that<br />

( )<br />

x 0 := lim x ni = lim (I − K)xni + Kx ni exists.<br />

i→∞ i→∞<br />

But then y = (I − K)x 0 ∈ (I − K)V ; thus (I − K)V is closed. Therefore by Theorem<br />

1.4.4, I − K has closed range.<br />

Corollary 1.4.6. If K ∈ B(X) is compact then I − K is Fredholm.<br />

Proof. From Theorem 1.4.5 we see that (I − K)(X) is closed. Since x ∈ (I − K) −1 (0)<br />

implies x = Kx, the identity operator acts as a compact operator <strong>on</strong> (I − K) −1 (0);<br />

thus α(I − K) < ∞. To prove that β(I − K) < ∞, recall that K ∗ : X ∗ → X ∗ is also<br />

compact. Since (I − K)(X) is closed it follows from Theorem 1.2.7 that<br />

β(I − K) = α(I − K ∗ ) < ∞.<br />

13


CHAPTER 1.<br />

FREDHOLM THEORY<br />

1.5 The Product of Fredholm <strong>Operator</strong>s<br />

Let T ∈ B(X, Y ). Suppose T −1 (0) and T (X) are complemented by subspaces X 0<br />

and Y 0 ; i.e.,<br />

X = T −1 (0) ⊕ X 0 and Y = T (X) ⊕ Y 0 .<br />

Define ˜T : X 0 × Y 0 → Y by<br />

˜T (x 0 , y 0 ) = T x 0 + y 0 .<br />

The space X 0 × Y 0 is a Banach space with the norm defined by ||(x, y)|| = ||x|| + ||y||<br />

and ˜T is a bijective bounded linear operator. We call ˜T the bijecti<strong>on</strong> associated with<br />

T . If T is Fredholm then such a bijecti<strong>on</strong> always exists and Y 0 is finite dimensi<strong>on</strong>al.<br />

If we identify X 0<br />

∼ = X0 × {0} then the operator T 0 : X 0 → Y defined by<br />

T 0 x = T x<br />

is a comm<strong>on</strong> restricti<strong>on</strong> of T and ˜T to X 0 (= X 0 × {0}).<br />

Note that<br />

1<br />

(a)<br />

|| ˜T = γ(T );<br />

−1 ||<br />

(b) If ̂T : X/T −1 (0) → T X then ̂T ∼ = ˜T .<br />

Lemma 1.5.1. Let T ∈ B(X, Y ) and M ⊂ X with codim M = n < ∞. Then<br />

in which case, index T = index T 0 + n.<br />

T is Fredholm ⇐⇒ T 0 := T | M is Fredholm,<br />

Proof. It suffices to prove the lemma for n = 1. Put X := M ⊕ span {x 1 }. We<br />

c<strong>on</strong>sider two cases:<br />

(Case 1) Assume T x 1 /∈ T 0 (M). Then T X = T 0 M ⊕ span {T x 1 } and T −1 (0) =<br />

T0 −1 (0). Hence<br />

β(T 0 ) = β(T ) + 1 and α(T 0 ) = α(T ). (1.6)<br />

(Case 2) Assume T x 1 ∈ T 0 (M). Then T X = T 0 M, and hence there exists u ∈ M<br />

such that T x 1 = T 0 u. Thus T −1 (0) = T −1<br />

0 (0) ⊕ span {x 1 − u}. Thus<br />

From (1.6) and (1.7) we have the result.<br />

β(T 0 ) = β(T ) and α(T 0 ) = α(T ) − 1. (1.7)<br />

Theorem 1.5.2. (Index Product Theorem) If T ∈ B(X, Y ) and S ∈ B(Y, Z) then<br />

S and T are Fredholm =⇒ ST is Fredholm with<br />

index (ST ) = index S + index T.<br />

14


CHAPTER 1.<br />

FREDHOLM THEORY<br />

Proof. Let ˜T be a bijecti<strong>on</strong> associated with T , X 0 , and Y 0 : i.e., X = T −1 (0) ⊕ X 0<br />

and Y = T (X) ⊕ Y 0 . Suppose T 0 := T | X0 . Since ˜T is invertible, S ˜T is invertible and<br />

index (S ˜T ) = index S. By identifying X 0 and X 0 × {0}, we see that ST 0 is a comm<strong>on</strong><br />

restricti<strong>on</strong> of S ˜T and ST to X 0 . By Lemma 1.5.1, ST is Fredholm and<br />

index (ST ) = index (ST 0 ) + dim X/X 0<br />

(<br />

)<br />

= index(S ˜T ) − dim X 0 × Y 0 /X 0 × {0} + α(T )<br />

= indexS − dim Y 0 + α(T )<br />

= index S − β(T ) + α(T )<br />

= index S + index T.<br />

The c<strong>on</strong>verse of Theorem 1.5.2 is not true in general. To see this, c<strong>on</strong>sider the<br />

following operators <strong>on</strong> l 2 :<br />

T (x 1 , x 2 , x 3 , . . .) = (0, x 1 , 0, x 2 , 0, x 3 , . . .)<br />

S(x 1 , x 2 , x 3 , . . .) = (x 2 , x 4 , x 6 , . . .).<br />

Then T ad S are not Fredholm, but ST = I. However, if ST = T S then we have<br />

ST is Fredholm =⇒ S and T are both Fredholm<br />

because T −1 (0) ⊂ (ST ) −1 (0) and (ST )(X) = T S(X) ⊂ T (X).<br />

Remark 1.5.3. For a time being, a Fredholm operator of index 0 will be called a<br />

Weyl operator. Then we have the following questi<strong>on</strong>: Is there implicati<strong>on</strong> that if<br />

ST = T S then<br />

S, T are Weyl ⇐⇒ ST is Weyl <br />

Here is the answer. The forward implicati<strong>on</strong> comes from the “Index Product Theorem”<br />

without commutativity c<strong>on</strong>diti<strong>on</strong>. However the backward implicati<strong>on</strong> may fail<br />

even with commutativity c<strong>on</strong>diti<strong>on</strong>. To see this, let<br />

T =<br />

[ U 0<br />

0 I<br />

]<br />

and S =<br />

[ ] I 0<br />

0 U ∗ ,<br />

where U is the unilateral shift <strong>on</strong> l 2 . Evidently,<br />

[ ] U 0<br />

index (ST ) = index<br />

0 U ∗<br />

but S and T are not Weyl.<br />

= index U + index U ∗<br />

= 0,<br />

15


CHAPTER 1.<br />

FREDHOLM THEORY<br />

1.6 Perturbati<strong>on</strong> Theorems<br />

We begin with:<br />

Theorem 1.6.1. Suppose T ∈ B(X, Y ) is Fredholm. If S ∈ B(X, Y ) with ||S|| <<br />

γ(T ) then T + S is Fredholm and<br />

(i) α(T + S) ≤ α(T );<br />

(ii) β(T + S) ≤ β(T );<br />

(iii) index (T + S) = index T .<br />

Proof. Let X = T −1 (0) ⊕ X 0 and Y = T (X) ⊕ Y 0 . Suppose ˜T is the bijecti<strong>on</strong> with<br />

T, X 0 and Y 0 . Put R = T + S and define<br />

˜R : X 0 × Y 0 → Y by ˜R(x 0 , y 0 ) = Rx 0 + y 0 .<br />

By definiti<strong>on</strong>, ˜T (x 0 , y 0 ) = T x 0 + y 0 . Since ˜T is invertible and<br />

|| ˜T − ˜R|| ≤ ||T − R|| = ||S|| < γ(T ) =<br />

1<br />

|| ˜T −1 || ,<br />

we have that ˜R is also invertible. Note that R 0 : X 0 → Y defined by<br />

R 0 x = Rx<br />

is a comm<strong>on</strong> restricti<strong>on</strong> of R and ˜R to X 0 . By Lemma 1.5.1, R is Fredholm and<br />

index R = index R 0 + α(T )<br />

= index ˜R − β(T ) + α(T )<br />

= index T<br />

which proves (iii). The invertibility of ˜R implies that X 0 ∩ R −1 (0) = {0}. Thus we<br />

have<br />

α(R) ≤ dim X/X 0 = α(T ),<br />

which proves (i). Note that (ii) is an immediate c<strong>on</strong>sequence of (i) and (iii).<br />

The first part of Theorem 1.6.1 asserts that<br />

the set of Fredholm operators forms an open set.<br />

Theorem 1.6.2. Let T, K ∈ B(X, Y ). Then<br />

T is Fredholm, K is compact =⇒ T + K is Fredholm with<br />

index (T + K) = index T.<br />

16


CHAPTER 1.<br />

FREDHOLM THEORY<br />

Proof. Let X = T −1 (0) ⊕ X 0 and Y = T (X) ⊕ Y 0 . Define ˜T , ˜K : X0 × Y 0 → Y by<br />

˜T (x 0 , y 0 ) = T x 0 + y 0 , ˜K(x0 , y 0 ) = Kx 0 + y 0 .<br />

Therefore ˜K is compact since K is compact and dim Y 0 < ∞. From ( ˜T + ˜K)(x 0 , 0) =<br />

(T + K)x 0 and Lemma 1.5.1 it follows that<br />

T + K is Fredholm ⇐⇒<br />

˜T + ˜K is Fredholm.<br />

But ˜T is invertible. So<br />

˜T + ˜K = ˜T ( I + ˜T −1 ˜K) .<br />

Observe that ˜T −1 ˜K is compact. Thus by Corollary 1.4.6, I + ˜T<br />

−1 ˜K is Fredholm.<br />

Hence T + K is Fredholm.<br />

To prove the statement about the index c<strong>on</strong>sider the integer valued functi<strong>on</strong><br />

F (λ) := index (T + λK). Applying Theorem 1.6.1 to T + λK in place of T shows that<br />

f is c<strong>on</strong>tinuous <strong>on</strong> [0, 1]. C<strong>on</strong>sequently, f is c<strong>on</strong>stant. In particular,<br />

index T = f(0) = f(1) = index (T + K).<br />

Corollary 1.6.3. If K ∈ B(X) then<br />

K is compact =⇒ I − K is Fredholm with index (I − K) = 0.<br />

Proof. Apply the preceding theorem with T = I and note that index I = 0.<br />

17


CHAPTER 1.<br />

FREDHOLM THEORY<br />

1.7 The Calkin Algebra<br />

We begin with:<br />

Theorem 1.7.1. If T ∈ B(X, Y ) then<br />

T is Fredholm ⇐⇒ ∃ S ∈ B(Y, X) such that I − ST and I − T S are finite rank.<br />

Proof. (⇒) Suppose T Fredholm and let<br />

X = T −1 (0) ⊕ X 0 and Y = T (X) ⊕ Y 0 .<br />

Define T 0 := T | X0 . Since T 0 is <strong>on</strong>e-<strong>on</strong>e and T 0 (X 0 ) = T (X) is closed<br />

T −1<br />

0 : T (X) → X 0 is invertible.<br />

Put S := T −1<br />

0 Q, where Q : Y → T (X) is a projecti<strong>on</strong>. Evidently, S(Y ) = X 0 and<br />

S −1 (0) = Y 0 . Furthermore,<br />

I − ST is the projecti<strong>on</strong> of X <strong>on</strong>to T −1 (0)<br />

I − T S is the projecti<strong>on</strong> of Y <strong>on</strong>to Y 0 .<br />

In particular, I − ST and I − T S are of finite rank.<br />

(⇐) Assume ST = I − K 1 and T S = I − K 2 , where K 1 , K 2 are finite rank. Since<br />

T −1 (0) ⊂ (ST ) −1 (0) and (T S)X ⊂ T (X),<br />

we have<br />

α(T ) ≤ α(ST ) = α(I − K 1 ) < ∞<br />

β(T ) ≤ β(T S) = β(I − K 2 ) < ∞,<br />

which implies that T is Fredholm.<br />

Theorem 1.7.1 remains true if the statement “I − ST and I − T S are of finite<br />

rank” is replaced by “I − ST and I − T S are compact operators.” In other words,<br />

T is Fredholm ⇐⇒ T is invertible modulo compact operators.<br />

Let K(X) be the space of all compact operators <strong>on</strong> X. Note that K(X) is a closed<br />

ideal of B(X). On the quotient space B(X)/K(X), define the product<br />

[S][T ] = [ST ],<br />

where [S] is the coset S + K(X).<br />

The space B(X)/K(X) with this additi<strong>on</strong>al operati<strong>on</strong> is an algebra, which is called<br />

the Calkin algebra, with identity [I].<br />

18


CHAPTER 1.<br />

FREDHOLM THEORY<br />

Theorem 1.7.2. (Atkins<strong>on</strong>’s Theorem) Let T ∈ B(X). Then<br />

T is Fredholm ⇐⇒ [T ] is invertible in B(X)/K(X).<br />

Proof. (⇒) If T is Fredholm then<br />

∃ S ∈ B(X) such that ST − I and T S − I are compact.<br />

Hence [S][T ] = [T ][S] = [I], so that [S] is the inverse of [T ] in the Calkin algebra.<br />

(⇐) If [S][T ] = [T ][S] = [I] then<br />

ST = I − K 1 and T S = I − K 2 ,<br />

where K 1 , K 2 are compact operators. Thus T is Fredholm.<br />

Let T ∈ B(X). The essential spectrum σ e (T ) of T is defined by<br />

σ e (T ) = {λ ∈ C : T − λI is not Fredholm}<br />

We thus have<br />

σ e (T ) = σ B(X)/K(X) (T + K(X)).<br />

Evidently σ e (T ) is compact. If dim X = ∞ then<br />

σ e (T ) ≠ ∅<br />

(because B(X)/K(X) ≠ ∅).<br />

In particular, Theorem 1.6.2 implies that<br />

σ e (T ) = σ e (T + K)<br />

for every K ∈ K(X).<br />

Theorem 1.7.3. If T ∈ B(X, Y ) then<br />

T is Weyl ⇐⇒ ∃ a finite rank operator F such that T + F is invertible.<br />

Proof. (⇒) Let T be Weyl and put<br />

Since index T = 0, it follows that<br />

X = T −1 (0) ⊕ X 0 and Y = T (X) ⊕ Y 0 .<br />

dim T −1 (0) = dim Y 0 .<br />

Thus there exists an invertible operator F 0 : T −1 (0) → Y 0 . Define F := F 0 (I − P ),<br />

where P is the projecti<strong>on</strong> of X <strong>on</strong>to X 0 . Obviously, T + F is invertible.<br />

(⇐) Assume S = T + F is invertible, where F is of finite rank. By Theorem 1.6.2,<br />

T is Fredholm and index T = index S = 0.<br />

19


CHAPTER 1.<br />

FREDHOLM THEORY<br />

The Weyl spectrum, ω(T ), of T ∈ B(X) is defined by<br />

ω(T ) =<br />

{<br />

}<br />

λ ∈ C : T − λI is not Weyl<br />

Evidently, ω(T ) is compact and in particular,<br />

∩<br />

ω(T ) =<br />

σ(T + K).<br />

K compact<br />

Definiti<strong>on</strong> 1.7.4. An operator T ∈ B(X, Y ) is said to be regular if there is T ′ ∈<br />

B(Y, X) for which<br />

T = T T ′ T ; (1.8)<br />

then T ′ is called a generalized inverse of T . We can always arrange<br />

indeed if (1.8) holds then<br />

T ′ = T ′ T T ′ : (1.9)<br />

T ′′ = T ′ T T ′ =⇒ T T ′′ T and T ′′ = T ′′ T T ′′ .<br />

If T ′ satisfies (1.8) and (1.9) then it will be called a generalized inverse of T in the<br />

str<strong>on</strong>g sense. Also T ∈ B(X, Y ) is said to be decomposably regular if there exists<br />

T ′ ∈ B(Y, X) such that<br />

T = T T ′ T and T ′ is invertible.<br />

The operator S := T −1<br />

0 Q, which was defined in the proof of Theorem 1.7.1, is a<br />

generalized inverse of in the str<strong>on</strong>g sense. Thus we have<br />

T is Fredholm ⇐⇒ I − T ′ T and I − T T ′ are finite rank.<br />

Generalized inverses are useful in solving linear equati<strong>on</strong>s. Suppose T ′ is a generalized<br />

inverse of T . If T x = y is solvable for a given y ∈ Y , then T ′ y is a soluti<strong>on</strong><br />

(not necessary the <strong>on</strong>ly <strong>on</strong>e). Indeed,<br />

T x = y is solvable =⇒ ∃ x 0 such that T x 0 = y<br />

Theorem 1.7.5. If T ∈ B(X, Y ), then<br />

=⇒ T T ′ y = T T ′ T x 0 = T x 0 = y.<br />

T is regular ⇐⇒ T −1 (0) and T (X) are complemented.<br />

20


CHAPTER 1.<br />

FREDHOLM THEORY<br />

Proof. (⇐) If X = X 0 ⊕ T −1 (0) and Y = Y 0 ⊕ T (X) then T ′ : Y → X defined by<br />

T ′ (T x 0 + y 0 ) = x 0 , where x 0 ∈ X 0 and y 0 ∈ Y 0<br />

is a generalized inverse of T because for x 0 ∈ X 0 and z ∈ T −1 (0),<br />

T T ′ T (x 0 + z) = T T ′ (T x 0 ) = T x 0 = T (x 0 + z).<br />

(⇒) Assume T ′ is a generalized inverse of T : T T ′ T = T . Obviously, T T ′ and T ′ T<br />

are both projecti<strong>on</strong>s. Also,<br />

which gives<br />

T (X) = T T ′ T (X) ⊂ T T ′ (X) ⊂ T (X);<br />

T −1 (0) ⊂ (T ′ T ) −1 (0) ⊂ (T T ′ T ) −1 (0) = T −1 (0),<br />

T T ′ (X) = T (X) and (T ′ T ) −1 (0) = T −1 (0),<br />

which implies that T −1 (0) and T (X) are complemented.<br />

Corollary 1.7.6. If T ∈ B(X, Y ) then<br />

T is Fredholm =⇒ T is regular.<br />

Theorem 1.7.7. If T ∈ B(X, Y ) is Fredholm with T = T T ′ T , then T ′<br />

Fredholm with<br />

index (T ′ ) = −index (T ).<br />

is also<br />

Proof. We first claim that<br />

ST is Fredholm =⇒ (S Fredholm ⇐⇒ T Fredholm) : (1.10)<br />

indeed,<br />

ST is Fredholm =⇒ I − (ST ) ′ (ST ) ∈ K 0 and I − (ST )(ST ) ′ ∈ K 0 ,<br />

which implies<br />

T is Fredholm ⇐⇒ I − T (ST ) ′ S ∈ K 0<br />

⇐⇒ S is Fredholm.<br />

Thus by (1.10), T ′ is Fredholm and by the index product theorem,<br />

index (T ) = index (T T ′ T ) = index (T ) + index (T ′ ) + index (T ).<br />

21


CHAPTER 1.<br />

FREDHOLM THEORY<br />

Theorem 1.7.8. If T ∈ B(X, Y ) is Fredholm with generalized inverse T ′ ∈ B(Y, X)<br />

in the str<strong>on</strong>g sense then<br />

Proof. Observe that<br />

which gives that β(T ) = α(T ′ ).<br />

index (T ) = dim T −1 (0) − dim (T ′ ) −1 (0).<br />

(T ′ ) −1 (0) = (T T ′ ) −1 (0) ∼ = X/T T ′ (X) ∼ = X/T (X),<br />

Theorem 1.7.9. If T ∈ B(X, Y ) is Fredholm with generalized inverse T ′ ∈ B(Y, X),<br />

then<br />

index (T ) = trace (T T ′ − T ′ T ).<br />

Proof. If T = T T ′ T is Fredholm then<br />

Observe that<br />

I − T ′ T and I − T T ′ are both finite rank.<br />

dim (I − T ′ T )(X) = dim (T ′ T ) −1 (0) = dim T −1 (0) = α(T );<br />

dim (I − T T ′ )(Y ) = dim (T T ′ ) −1 (0) = dim X/T T ′ (Y ) = dim X/T (X) = β(T ).<br />

Thus we have<br />

trace (T T ′ − T ′ T ) = trace ( (I − T ′ T ) − (I − T T ′ ) )<br />

= trace (I − T ′ T ) − trace (I − T T ′ )<br />

= rank (I − T ′ T )(X) − dim (I − T T ′ )(X)<br />

= α(T ) − β(T )<br />

= index (T ).<br />

22


CHAPTER 1.<br />

FREDHOLM THEORY<br />

1.8 The Punctured Neighborhood Theorem<br />

If T ∈ B(X, Y ) then<br />

(a) T is said to be upper semi-Fredholm if T (X) is closed and α(T ) < ∞;<br />

(b) T is said to be lower semi-Fredholm if T (X) is closed and β(T ) < ∞.<br />

(c) T is said to be semi-Fredholm if it is upper or lower semi-Fredholm.<br />

Theorem 1.6.1 remains true for semi-Fredholm operators. Thus we have:<br />

Lemma 1.8.1. Suppose T ∈ B(X, Y ) is semi-Fredholm. If ||S|| < γ(T ) then<br />

(i) T + S has a closed range;<br />

(ii) α(T + S) ≤ α(T ), β(T + S) ≤ β(T );<br />

(iii) index (T + S) = index T .<br />

Proof. This follows from a slight change of the argument for Theorem 1.6.1.<br />

We are ready for the punctured neighborhood theorem; this proof is due to Harte<br />

and <str<strong>on</strong>g>Lee</str<strong>on</strong>g> [HaL1].<br />

Theorem 1.8.2. (Punctured Neighborhood Theorem) If T ∈ B(X) is semi-Fredholm<br />

then there exists ρ > 0 such that α(T −λI) and β(T −λI) are c<strong>on</strong>stant in the annulus<br />

0 < |λ| < ρ.<br />

Proof. Assume that T is upper semi-Fredholm and α(T ) < ∞. First we argue<br />

Indeed,<br />

Next we claim that<br />

(T − λI) −1 (0) ⊂<br />

∞∩<br />

T n (X) =: T ∞ (X). (1.11)<br />

n=1<br />

x ∈ (T − λI) −1 (0) =⇒ T x = λx, and hence x ∈ T (X)<br />

=⇒ Note that λx = T x ∈ T (T X) = T 2 (X)<br />

=⇒ By inducti<strong>on</strong>, x ∈ T n (X) for all n.<br />

T ∞ (X) is closed:<br />

indeed, since T n is upper semi-Fredholm for all n, T n (X) is closed and hence T ∞ (X)<br />

is closed.<br />

If S commutes with T , so that also S(T ∞ (X)) ⊂ T ∞ (X), we shall write ˜S :<br />

T ∞ (X) → T ∞ (X). We claim that<br />

To see this, let y ∈ T ∞ (X) and thus<br />

˜T : T ∞ (X) → T ∞ (X) is <strong>on</strong>to. (1.12)<br />

∃ x n ∈ T n (X) such that T x n = y (n = 1, 2, . . .).<br />

23


CHAPTER 1.<br />

FREDHOLM THEORY<br />

Since T −1 (0) is finite dimensi<strong>on</strong>al and T n (X) ⊃ T n+1 (X),<br />

∃n 0 ∈ N such that T −1 (0) ∩ T n 0<br />

(X) = T −1 (0) ∩ T n (X) for n ≥ n 0 .<br />

From the fact that T n (X) ⊂ T n 0<br />

(X), we have<br />

x n − x n0 ∈ T −1 (0) ∩ T n 0<br />

(X) = T −1 (0) ∩ T n (X) ⊂ T n (X).<br />

Hence<br />

x n0<br />

∈ ∩<br />

n≥n 0<br />

T n (X) = T ∞ (X) and T x n0 = y,<br />

which says that ˜T is <strong>on</strong>to. This proves (1.12). Now observe<br />

dim (T − λI) −1 (0) = dim ˜ T − λI −1 (0) = index ˜ T − λI = index ˜T : (1.13)<br />

the first equality comes from (1.11), the sec<strong>on</strong>d equality follows from the fact that<br />

β( T˜<br />

− λI) ≤ β( ˜T ) = 0 by Lemma 1.8.1, and the third equality follows the observati<strong>on</strong><br />

that ˜T is semi-Fredholm. Since the right-hand side of (1.13) is independent of λ,<br />

α(T − λI) is c<strong>on</strong>stant and hence also is β(T − λI).<br />

If instead β(T ) < ∞, apply the above argument with T ∗ .<br />

Theorem 1.8.3. Define<br />

Then<br />

(i) U is an open set;<br />

U :=<br />

{<br />

}<br />

λ ∈ C : T − λI is semi-Fredholm .<br />

(ii) If C is a comp<strong>on</strong>ent of U then <strong>on</strong> C, with the possible excepti<strong>on</strong> of isolated<br />

points,<br />

α(T − λI) and β(T − λI) have c<strong>on</strong>stant values n 1 and n 2 , respectively.<br />

At the isolated points,<br />

α(T − λI) > n 1 and β(T − λI) > n 2 .<br />

Proof. (i) For λ ∈ U apply Lemma 1.8.1 to T − λI in place of T .<br />

(ii) The comp<strong>on</strong>ent C is open since any comp<strong>on</strong>ent of an open set in C is open.<br />

Let α(λ 0 ) = n 1 be the smallest integer which is attained by<br />

α(λ) = α(T − λI) <strong>on</strong> C.<br />

24


CHAPTER 1.<br />

FREDHOLM THEORY<br />

Suppose α(λ ′ ) ≠ n 1 . Since C is c<strong>on</strong>nected there exists an arc Γ lying in C with<br />

endpoints λ 0 and λ ′ . It follows from Theorem 1.8.2 and the fact that C is open that<br />

for each µ ∈ Γ, there exists an open ball S(µ) in C such that<br />

α(λ) is c<strong>on</strong>stant <strong>on</strong> the set S(µ) with the point µ deleted.<br />

Since Γ is compact and c<strong>on</strong>nected there exist points λ 1 , λ 2 , · · · , λ n = λ ′ <strong>on</strong> Γ such<br />

that<br />

S(λ 0 ), S(λ 1 ), . . . , S(λ n ) cover Γ and S(λ i ) ∩ S(λ i+1 ) ≠ ∅ (0 ≤ i ≤ n − 1) (1.14)<br />

We claim that α(λ) = α(λ 0 ) <strong>on</strong> all of S(λ 0 ). Indeed it follows from the Lemma 1.8.1<br />

that<br />

α(λ) ≤ α(λ 0 ) for λ sufficiently close to λ 0 .<br />

Therefore, since α(λ 0 ) is the minimum of α(λ) <strong>on</strong> C,<br />

α(λ) = α(λ 0 ) for λ sufficiently close to λ 0 .<br />

Since α(λ) is c<strong>on</strong>stant for all λ ≠ λ 0 in S(λ 0 ), which is α(λ 0 ). Now α(λ) is c<strong>on</strong>stant<br />

<strong>on</strong> the set S(λ i ) with the point λ i deleted (1 ≤ i ≤ n). Hence it follows from (1.14)<br />

and the observati<strong>on</strong> α(λ) = α(λ 0 ) for all λ ∈ S(λ 0 ) that α(λ) = α(λ 0 ) for all λ ≠ λ ′<br />

in S(λ ′ ) and α(λ ′ ) > n 1 . The result just obtained can be applied to the adjoint. This<br />

completes the proof.<br />

25


CHAPTER 1.<br />

FREDHOLM THEORY<br />

1.9 The Riesz-Schauder (or Browder) <strong>Theory</strong><br />

An operator T ∈ B(X) is said to be quasinilpotent if<br />

and is said to be nilpotent if<br />

∥T n ∥ 1 n −→ 0<br />

T n = 0 for some n.<br />

An example for quasinilpotent but not nilpotent:<br />

T : l 2 → l 2<br />

T (x 1 , x 2 , x 3 , . . .) ↦−→ (0, x 1 , x 2<br />

2 , x 3<br />

3 , . . .).<br />

An example for quasinilpotent but neither nilpotent nor compact:<br />

T = T 1 ⊕ T 2 : l 2 ⊕ l 2 −→ l 2 ⊕ l 2 ,<br />

where<br />

T 1 : (x 1 , x 2 , x 3 , . . .) ↦−→ (0, x 1 , 0, x 3 , 0, x 5 , . . .)<br />

T 2 : (x 1 , x 2 , x 3 , . . .) ↦−→ (0, x 1 , x 2<br />

2 , x 3<br />

3 , . . .).<br />

Remember that if T ∈ B(X) we define L T , R T ∈ B(B(X)) by<br />

L T (S) := T S and R T (S) := ST for S ∈ B(X).<br />

Lemma 1.9.1. We have:<br />

(a) L T is 1-1 ⇐⇒ T is 1-1;<br />

(b) R T is 1-1 ⇐⇒ T is dense;<br />

(c) L T is bounded below ⇐⇒ T is bounded below;<br />

(d) R T is bounded below ⇐⇒ T is open.<br />

Proof. See [Be3].<br />

Theorem 1.9.2. If T ∈ B(X), then<br />

(a) T is nilpotent =⇒ T is neither 1-1 nor dense;<br />

(b) T is quasinilpotent =⇒ T is neither bounded below nor open.<br />

26


CHAPTER 1.<br />

FREDHOLM THEORY<br />

Proof. By Lemma 1.9.1,<br />

(a) T is nilpotent =⇒ T n+1 = 0 ≠ T n<br />

=⇒ L T (T n ) = R T (T n ) = 0 ≠ T n<br />

=⇒ L T and R T are not 1-1<br />

=⇒ T is not 1-1 and not dense.<br />

(b) T is quasinilpotent =⇒ ∀ ε > 0, ∃ n ∈ N such that ∥T n ∥ 1 n ≥ ε > ∥T n+1 ∥ 1<br />

n+1<br />

=⇒ ∥L T (T n )∥ = ∥R T (T n )∥ < ε∥T n ∥<br />

=⇒ L T and R T are not bounded below<br />

=⇒ T is not bounded below and not open.<br />

We would remark that<br />

{quasinilpotents} ⊆ ∂B −1 (X).<br />

Observe that quasinilpotents of finite rank or cofinite rank are nilpotents.<br />

Definiti<strong>on</strong> 1.9.3. An operator T ∈ B(X) is said to be quasipolar [polar, resp.] if<br />

there is a projecti<strong>on</strong> P commuting with T for which T has a matrix representati<strong>on</strong><br />

[ ] [ ] [ ]<br />

T1 0 P (X) P (X)<br />

T = :<br />

0 T 2 P −1 →<br />

(0) P −1 ,<br />

(0)<br />

where T 1 is invertible and T 2 is quasinilpotent [nilpotent, resp.]<br />

Definiti<strong>on</strong> 1.9.4. An operator T ∈ B(X) is said to be simply polar if there is<br />

T ′ ∈ B(X) for which<br />

T = T T ′ T with T T ′ = T ′ T<br />

Propositi<strong>on</strong> 1.9.5. Simply polar operators are decomposably regular.<br />

Proof. Assume T = T T ′ T with T T ′ = T ′ T . Then<br />

T ′′ = T ′ + (1 − T ′ T ) =⇒<br />

{ T = T T ′′ T<br />

(T ′′ ) −1 = T + (1 − T ′ T )<br />

.<br />

Theorem 1.9.6. If T ∈ B(X) then<br />

T is quasipolar but not invertible ⇐⇒ 0 ∈ iso σ(T )<br />

27


CHAPTER 1.<br />

FREDHOLM THEORY<br />

Proof. (⇒) If T is quasipolar we may write<br />

[ ] [ ]<br />

T1 0 P (X)<br />

T = :<br />

0 T 2 P −1 (0)<br />

→<br />

[ ] P (X)<br />

P −1 ,<br />

(0)<br />

where T 1 is invertible and T 2 is quasinilpotent. Thus for sufficiently small λ ≠ 0,<br />

T 1 − λI and T 2 − λI are both invertible, which implies that 0 ∈ iso σ(T )<br />

(⇐) If 0 ∈ iso σ(T ), c<strong>on</strong>struct open discs D 1 and D 2 such that D 1 c<strong>on</strong>tains 0, D 2<br />

c<strong>on</strong>tains the spectrum σ(T ) and D 1 ∩ D 2 = ∅. If we define f : D 1 ∪ D 2 −→ C by<br />

setting<br />

{ 0 <strong>on</strong> D1<br />

f(λ) =<br />

1 <strong>on</strong> D 2<br />

then f is analytic <strong>on</strong> D 1 ∪ D 2 and f(λ) 2 = f(λ). Observe that<br />

P = P D2 = f(T ) = 1 ∫<br />

(λ − T ) −1 dλ<br />

2πi ∂D 2<br />

and P T = T P . Thus we may write<br />

[ ]<br />

T1 0<br />

T = : P (X) ⊕ P −1 (0) −→ P (X) ⊕ P −1 (0),<br />

0 T 2<br />

where σ(T 1 ) = σ(T )\{0} and σ(T 2 ) = {0}. Therefore T 1 is invertible and T 2 is<br />

quasinilpotent; so that T is quasipolar.<br />

Theorem 1.9.7. If T ∈ B(X) then<br />

T is simply polar ⇐⇒ T (X) = T 2 (X), T −1 (0) = T −2 (0)<br />

Proof. (⇒) Observe<br />

T (X) = T T ′ T (X) = T 2 T ′ (X) ⊆ T 2 (X) ⊆ T X;<br />

T −1 (0) = (T T ′ T ) −1 (0) = (T ′ T 2 ) −1 (0) ⊇ T −2 (0) ⊇ T −1 (0).<br />

(⇐) (i) x ∈ T X ∩ T −1 (0) ⇒ x = T y for some y ∈ X and T x = 0<br />

⇒ T 2 y = 0 ⇒ y ∈ T −2 (0) = T −1 (0)<br />

⇒ T y = 0 ⇒ x = 0,<br />

which gives T X ∩ T −1 (0) = {0}.<br />

(ii) By assumpti<strong>on</strong>, T (T (X)) = T (X). Let T 1 := T | T (X) , so that T 1 (X) =<br />

T 2 (X) = T (X). Thus for all x ∈ X,<br />

∃ y ∈ T (X) such that T x = T 1 y = T y.<br />

28


CHAPTER 1.<br />

FREDHOLM THEORY<br />

Define z = x − y, and hence z ∈ T −1 (0). Thus X = T (X) + T −1 (0). In particular,<br />

T (X) is closed by Theorem 1.4.2, so that<br />

X = T (X) ⊕ T −1 (0).<br />

Therefore we can find a projecti<strong>on</strong> P ∈ B(X) for which<br />

We thus write<br />

P (X) = T (X) and P −1 (0) = T −1 (0).<br />

T =<br />

[ ] [ ]<br />

T1 0 P (X)<br />

:<br />

0 0 P −1 (0)<br />

→<br />

[ ] P (X)<br />

P −1 ,<br />

(0)<br />

where T 1 is invertible because T 1 := T | T X is 1-1 and <strong>on</strong>to since T (X) = T 2 (X). If<br />

we put<br />

[ ]<br />

T ′ T<br />

−1<br />

=<br />

1 0<br />

,<br />

0 0<br />

then T T ′ T = T and<br />

[ ] [ ]<br />

T T ′ = T ′ T<br />

−1<br />

T =<br />

1 0 T1 0<br />

=<br />

0 0 0 0<br />

which says that T is simply polar.<br />

[ ] I 0<br />

= P,<br />

0 0<br />

Theorem 1.9.8. If T ∈ B(X) then<br />

T is polar ⇐⇒ T n is simply polar for some n ∈ N<br />

[ ]<br />

T1 0<br />

Proof. (⇒) If T is polar then we can write T =<br />

with T<br />

0 T 1 invertible and<br />

[ ]<br />

2<br />

T<br />

T 2 nilpotent. So T n n<br />

= 1 0<br />

, where n is the nilpotency of T<br />

0 0<br />

2 . If we put S =<br />

[ ]<br />

T<br />

−n<br />

1 0<br />

, then T<br />

0 I<br />

n ST n = T n and ST n = T n S.<br />

(⇐) If T n is simply polar then X = T n (X) ⊕ T −n (0). Observe that since T n is<br />

simply polar we have<br />

T (T n X) = T n+1 (X) ⊇ T 2n (X) = T n (X)<br />

T (T −n (0)) ⊆ T −n+1 (0) ⊆ T −n (0)<br />

Thus we see that T | T n (X) is 1-1 and <strong>on</strong>to, so that invertible. Thus we may write<br />

( )<br />

T1 0<br />

T =<br />

: T n (X) ⊕ T −n (0) −→ T n (X) ⊕ T −n (0),<br />

0 T 2<br />

where T 1 = T | T n (X) is invertible and T 2 = T | T −n (0) is nilpotent with nilpotency n.<br />

Therefore T is polar.<br />

29


CHAPTER 1.<br />

FREDHOLM THEORY<br />

The following is an immediate result of Theorem 1.9.8 :<br />

Corollary 1.9.9. If T ∈ B(X) then<br />

T is polar ⇐⇒ ascent (T ) = descent (T ) < ∞<br />

Corollary 1.9.10. If S, T ∈ B(X) with ST = T S, then<br />

S and T are polar =⇒ ST is polar.<br />

Proof. Suppose S n (X) = S n+1 (X) and T n (X) = T n+1 (X). Then<br />

(ST ) mn+1 (X) = S mn+1 T mn+1 (X) = S mn+1 T mn (X) = T mn S mn+1 (X)<br />

= T mn S mn (X) = (ST ) mn (X)<br />

Similarly,<br />

(ST ) −p−1 (0) = (ST ) −p (0).<br />

Definiti<strong>on</strong> 1.9.11. An operator T ∈ B(X) is called a Browder (or Riesz-Schauder)<br />

operator if T is Fredholm and quasipolar.<br />

If T is Fredholm then by the remark above Definiti<strong>on</strong> 1.9.3,<br />

T is quasipolar ⇐⇒ T is polar.<br />

Thus we have<br />

T is Browder ⇐⇒ T is Fredholm and polar.<br />

Theorem 1.9.12. If T ∈ B(X), the following are equivalent:<br />

(a) T is Browder, but not invertible;<br />

(b) T is Fredholm and 0 ∈ iso σ(T );<br />

(c) T is Weyl and 0 ∈ iso σ(T );<br />

(d) T is Fredholm and ascent (T ) = descent (T ) < ∞.<br />

Proof. (a) ⇔ (b) : Theorem 1.9.6<br />

(b) ⇔ (c) : From the c<strong>on</strong>tinuity of the index<br />

(b) ⇔ (d) : From Corollary 1.9.9.<br />

Theorem 1.9.13. If K ∈ B(X), then<br />

K is compact =⇒ I + K is Browder.<br />

30


CHAPTER 1.<br />

FREDHOLM THEORY<br />

Proof. From the spectral theory of the compact operators,<br />

−1 ∈ iso σ(K)<br />

(in fact, λ ≠ 0 ⇒ λ /∈ acc σ(K)),<br />

which gives<br />

0 ∈ iso σ(I + K).<br />

From Corollary 1.4.6, I + K is Fredholm. Now Theorem 1.9.12 says that I + K is<br />

Browder.<br />

Theorem 1.9.14. (Riesz-Schauder Theorem). If T ∈ B(X) then<br />

T is Browder ⇐⇒ T = S+K, where S is invertible and K is compact with SK = KS.<br />

Proof. (⇒) If T is Browder then it is polar, so that we can write<br />

[ ]<br />

T1 0<br />

T = ,<br />

0 T 2<br />

where T 1 is invertible and T 2 is nilpotent. Since T is Fredholm, T 2 is also Fredholm.<br />

If we put<br />

[ ]<br />

[ ]<br />

T1 0<br />

0 0<br />

S =<br />

and K =<br />

,<br />

0 I<br />

0 T 1 − I<br />

then evidently T 2 − I is of finite rank. Thus S is invertible and K is of finite rank.<br />

Further,<br />

T = S + K and SK = KS.<br />

(⇐) Suppose T = S + K and SK = KS. Since, by Theorem 1.9.13, I + S −1 K<br />

is Browder, so that I + S −1 K is Fredholm and polar. Therefore, by Theorem 1.5.2<br />

and Corollary 1.9.10, T = S(I + S −1 K) is Fredholm and polar, and hence Browder.<br />

Here, note that S and I + S −1 K commutes.<br />

Remark 1.9.15. If S, T ∈ B(X) and ST = T S then<br />

(a) S, T are Browder ⇐⇒ ST is Browder;<br />

(b) S is Browder and T is compact =⇒ S + T is Browder.<br />

Example 1.9.16. There exists a Weyl operator which is not Browder.<br />

[ ] U 0<br />

Proof. Put T =<br />

0 U ∗ : l 2 ⊕l 2 → l 2 ⊕l 2 , where U is the unilateral shift. Evidently,<br />

T is Fredholm and index T = index U + index U ∗ = 0, which says that T is Weyl.<br />

However, σ(T ) = {λ ∈ C : |λ| ≤ 1} ; so that 0 /∈ iso σ(T ), which implies that T is not<br />

Browder.<br />

31


CHAPTER 1.<br />

FREDHOLM THEORY<br />

1.10 Essential Spectra<br />

If T ∈ B(X) we define:<br />

(a) The essential spectrum of T := σ e (T ) = {λ ∈ C : T − λI is not Fredolm}<br />

(b) The Weyl spectrum of T := ω(T ) = {λ ∈ C : T − λI is not Weyl}<br />

(c) The Browder spectrum of T := σ b (T ) = {λ ∈ C : T − λI is not Browder}<br />

Evidently, σ e (T ), ω(T ) and σ b (T ) are all compact;<br />

these are n<strong>on</strong>empty if dim X = ∞.<br />

σ e (T ) ⊂ ω(T ) ⊂ σ b (T );<br />

Theorem 1.10.1. If T ∈ B(X) then<br />

(a) σ(T ) = σ e (T ) ∪ σ p (T ) ∪ σ com (T );<br />

(b) σ(T ) = ω(T ) ∪ ( σ p (T ) ∩ σ com (T ) ) ;<br />

(c) σ b (T ) = σ e (T ) ∪ acc σ(T ),<br />

where σ com (T ) := {λ ∈ C : T − λI does not have dense range}.<br />

Proof. Immediate follow from definiti<strong>on</strong>s.<br />

Definiti<strong>on</strong> 1.10.2. We shall write<br />

P 00 (T ) = iso σ(T )\σ e (T )<br />

for the Riesz points of σ(T ). Evidently, λ ∈ P 00 (T ) means that T − λI is Browder,<br />

but not invertible.<br />

Lemma 1.10.3. If Ω is locally c<strong>on</strong>nected and H, K ⊂ Ω, then<br />

Proof. See [Har4].<br />

∂K ⊆ H ∪ iso K =⇒ K ⊂ ηH ∪ iso K<br />

Theorem 1.10.4. If T ∈ B(X) then<br />

(a) ∂σ(T )\σ e (T ) ⊆ iso σ(T );<br />

(b) σ(T ) ⊆ ησ e (T ) ∪ P 00 (T )<br />

Proof. (a) This is an immediate c<strong>on</strong>sequence of the Punctured Neighborhood Theorem.<br />

(b) From (a) and Lemma 1.10.3,<br />

σ(T ) ⊆ ησ e (T ) ∪ iso σ(T )<br />

= ησ e (T ) ∪ P 00 (T )<br />

by the fact that if λ /∈ ησ e (T ) and λ ∈ iso σ(T ), then T − λI is Fredholm and<br />

λ ∈ iso σ(T ) thus T − λI is Browder.<br />

32


CHAPTER 1.<br />

FREDHOLM THEORY<br />

1.11 Spectral Mapping Theorems<br />

Recall the Calkin algebra B(X)/K(X). The Calkin homomorphism π is defined by<br />

π : B(X) −→ B(X)/K(X)<br />

π(T ) = T + K(X).<br />

Evidently, by the Atkins<strong>on</strong>’s Theorem,<br />

T is Fredholm ⇐⇒ π(T ) is invertible.<br />

Theorem 1.11.1. If T ∈ B(X) and f is analytic in a neighborhood of σ(T ), then<br />

f(σ e (T )) = σ e (f(T ))<br />

Proof. Since f(π(T )) = f(T + K(X)) = f(T ) + K(X) = π(f(T )) it follows that<br />

f(σ e (T )) = f(σ(π(T ))) = σ(f(π(T ))) = σ(π(f(T ))) = σ e (f(T )).<br />

Theorem 1.11.2. If T ∈ B(X) and f is analytic in a neighborhood of σ(T ), then<br />

f(σ b (T )) = σ b (f(T ))<br />

Proof. Since by the analyticity of f, f(acc K) = acc f(K), it follows that<br />

f(σ b (T )) = f(σ e (T ) ∪ acc σ(T ))<br />

= f(σ e (T )) ∪ f(acc σ(T ))<br />

= σ e (f(T )) ∪ acc σ(f(T ))<br />

= σ b (f(T )).<br />

Theorem 1.11.3. If T ∈ B(X) and p is a polynomial then<br />

ω(p(T )) ⊆ p(ω(T )).<br />

Proof. Let p(z) = a 0 + a 1 z + · · · + a n z n ; thus p(z) = c 0 (z − α 1 ) · · · (z − α n ). Then<br />

p(T ) = c 0 (T − α 1 I) · · · (T − α n I).<br />

We now claim that<br />

0 /∈ p(ω(T )) =⇒ c 0 (z − α 1 ) · · · (z − α n ) ≠ 0 for each λ ∈ ω(T )<br />

=⇒ λ ≠ α i for each λ ∈ ω(T )<br />

=⇒ T − α i I is Weyl for each i = 1, 2, . . . n<br />

=⇒ c 0 (T − α 1 I) · · · (T − α n I) is Weyl<br />

=⇒ 0 /∈ ω(p(T ))<br />

33


CHAPTER 1.<br />

FREDHOLM THEORY<br />

In fact, we can show that<br />

neighborhood of σ(T ).<br />

ω(f(T )) ⊆ f(ω(T )) for any analytic functi<strong>on</strong> f in a<br />

The inclusi<strong>on</strong> of Theorem 1.11.3 may be proper. For example, if U is the unilateral<br />

shift, c<strong>on</strong>sider [ ]<br />

U + I 0<br />

T =<br />

0 U ∗ : l 2 ⊕ l 2 −→ l 2 ⊕ l 2 .<br />

− I<br />

Then<br />

ω(T ) = σ(T ) = { z ∈ C : |1 + z| ≤ 1 } ∪ { z ∈ C : |1 − z| ≤ 1 } .<br />

Let p(z) = (z + 1)(z − 1). Then<br />

Therefore 0 ∈ p(ω(T )). However<br />

p(ω(T )) is a cardioid c<strong>on</strong>taining 0.<br />

p(T ) = (T + I)(T − I) =<br />

[ ] [ ]<br />

U + 2I 0 U 0<br />

0 U ∗ 0 U ∗ ,<br />

− 2I<br />

so that index (p(T )) = index U ∗ + index U = 0, which implies 0 /∈ ω(p(T )). Therefore<br />

p(ω(T )) ω(p(T )).<br />

34


CHAPTER 1.<br />

FREDHOLM THEORY<br />

1.12 The C<strong>on</strong>tinuity of Spectra<br />

Let σ n be a sequence of compact subsets of C.<br />

(a) The limit inferior, lim inf σ n , is the set of all λ ∈ C such that every neighborhood<br />

of λ has a n<strong>on</strong>empty intersecti<strong>on</strong> with all but finitely many σ n .<br />

(b) The limit superior, lim sup σ n , is the set of all λ ∈ C such that every neighborhood<br />

of λ intersects infinitely many σ n .<br />

(c) If lim inf σ n = lim sup σ n then lim σ n is said to be exist and is the comm<strong>on</strong> limit.<br />

A mapping T <strong>on</strong> B(X) whose values are compact subsets of C is said to be upper<br />

semi-c<strong>on</strong>tinuous at T when<br />

T n −→ T =⇒ lim sup T (T n ) ⊂ T (T )<br />

and to be lower semi-c<strong>on</strong>tinuous at T when<br />

T n −→ T =⇒ T (T ) ⊂ lim inf T (T n ).<br />

If T is both upper and lower semi-c<strong>on</strong>tinuous, then it is said to be c<strong>on</strong>tinuous.<br />

Example 1.12.1. The spectrum σ : T ↦−→ σ(T ) is not c<strong>on</strong>tinuous in general: for<br />

example, if<br />

[<br />

U<br />

1<br />

T n := n (I − ]<br />

[ ]<br />

UU∗ )<br />

U 0<br />

0 U ∗ and T :=<br />

0 U ∗<br />

then σ(T n ) = ∂D, σ(T ) = D, and T n −→ T .<br />

Propositi<strong>on</strong> 1.12.2. σ is upper semi-c<strong>on</strong>tinuous.<br />

Proof. Suppose T n → T and λ ∈ lim sup σ(T n ). Then there exists λ n ∈ lim sup σ(T n )<br />

so that λ nk → λ. Since T nk − λ nk I is singular and T nk − λ nk I −→ T − λI, it follows<br />

that T − λI is singular; therefore λ ∈ σ(T ).<br />

Theorem 1.12.3. σ is c<strong>on</strong>tinuous <strong>on</strong> the set of all hyp<strong>on</strong>ormal operators.<br />

Proof. Let T n , T be hyp<strong>on</strong>ormal operators such that T n → T in norm. We want to<br />

prove that<br />

σ(T ) ⊂ lim inf σ(T n ).<br />

Assume λ /∈ lim inf σ(T n ). Then there exists a neighborhood N(λ) of λ such that it<br />

does not intersect infinitely many σ(T n ). Thus we can choose a subsequence {T nk } of<br />

{T n } such that for some ε > 0,<br />

( )<br />

dist λ, σ(T nk ) > ε.<br />

35


CHAPTER 1.<br />

FREDHOLM THEORY<br />

Since T nk is hyp<strong>on</strong>ormal, it follows that<br />

dist(λ, σ(T nk )) =<br />

1<br />

min |µ| =<br />

µ∈σ(Tn −λ) k max |µ| = 1<br />

∥(T nk − λ) −1 ∥ ,<br />

µ∈σ((Tn −λ) −1 )<br />

k<br />

where the sec<strong>on</strong>d equality follows from the observati<strong>on</strong><br />

{ }<br />

1<br />

σ(T −1 ) =<br />

z : z ∈ σ(T )<br />

because if f(z) = 1 z then σ(T −1 ) = σ(f(T )) = f(σ(T )) = { 1 z<br />

: z ∈ σ(T )} and the last<br />

equality uses the fact that (T nk − λI) −1 is normaloid. So ∥(T nk − λI) −1 ∥ < 1 ε . We<br />

thus have<br />

}<br />

∥(T nk − λI) −1 − (T nl − λI) −1 ∥ = ∥(T nk − λI)<br />

{(T −1 nk − λI) − (T nl − λI) − (T nl − λI) −1 ∥<br />

≤ ∥(T nk − λI) −1 ∥ · ∥T nl − T nk ∥ · ∥(T nl − λI) −1 ∥<br />

< 1 ε 2 ∥T n l<br />

− T nk ∥.<br />

Since T nk → T , it follows that {(T nk − λI) −1 } c<strong>on</strong>verges, to some operator B, say.<br />

Therefore<br />

(T − λI)B = lim(T nk − λI) · lim(T nk − λI) −1<br />

Similarly, B(T − λI) = 1 and hence λ /∈ σ(T ).<br />

= lim(T nk − λI)(T nk − λI) −1 = 1.<br />

Lemma 1.12.4. Let A be a commutative Banach algebra. If x ∈ A is not invertible<br />

and ∥y − x∥ < ε, then there exists λ such that y − λ is not invertible and |λ| < ε.<br />

Proof. Since x is not invertible, it generates an ideal ≠ A. Thus there exists a maximal<br />

ideal M c<strong>on</strong>taining x. So z ∈ M =⇒ z is not invertible. Since A/M ∼ = F, λ·1 ∈ y+M<br />

for some y. Thus y − λ · 1 ∈ M. Since x ∈ M we have y − x − λ · 1 ∈ M, so that<br />

λ ∈ σ(y − x). Finally, |λ| ≤ ∥y − x∥ < ε.<br />

Theorem 1.12.5. If in a Banach algebra A, x i → x and x i x = xx i for all i, then<br />

lim σ(x i ) = σ(x).<br />

Proof. Let B be the algebra generated by 1, x, and x i . Then (x−µ) −1 and (x i −µ) −1<br />

are commutative whenever they exist. Let λ ∈ σ(x), i.e., x − λ is not invertible. By<br />

Lemma 1.12.4, there exists N such that<br />

i > N =⇒ σ(x i − λ) ∩ N ε (0) ≠ ∅.<br />

So 0 ∈ lim inf σ(x i − λ), or λ ∈ lim inf σ(x i ), so that<br />

σ(x) ⊆ lim inf σ(x i ) ⊆ lim sup σ(x i ) ⊆ σ(x).<br />

36


CHAPTER 1.<br />

FREDHOLM THEORY<br />

Theorem 1.12.6. ω is upper semi-c<strong>on</strong>tinuous.<br />

Proof. We want to prove that<br />

lim sup ω(T n ) ⊂ ω(T ) if T n → T.<br />

Let λ /∈ ω(T ), so T − λI is Weyl. Since the set of Weyl operators forms an open set,<br />

∃ η > 0 such that ∥T − λI − S∥ < η =⇒ S is Weyl.<br />

Let N be such that<br />

∥(T − λI) − (T n − λI)∥ < η 2<br />

for n ≥ N.<br />

Let V = B(λ ; η 2<br />

). Then for µ ∈ V, n ≥ N,<br />

∥(T − λI) − (T n − µI)∥ < η,<br />

so that T n − µI is Weyl, which implies that λ /∈ lim sup ω(T n ).<br />

Theorem 1.12.7. Let T n → T . If T n T = T T n for all n, then lim ω(T n ) = ω(T ).<br />

Proof. In view of Theorem 1.12.6, it suffices to show that<br />

ω(T ) ⊆ lim inf ω(T n ) (1.15)<br />

Observe that π(T n )π(T ) = π(T )π(T n ) and hence by Theorem 1.12.5, lim σ e (T n ) =<br />

σ e (T ). Towards (1.15), suppose λ /∈ lim inf ω(T n ). So there exists a neighborhood<br />

V (x) which does not intersect infinitely many ω(T n ). Since σ e (T n ) ⊂ ω(T n ), V does<br />

not intersect infinitely many σ e (T n ), i.e., λ /∈ lim σ e (T n ) = σ e (T ). This shows that<br />

T − λI is Fredholm. By the c<strong>on</strong>tinuity of index, T − λI is Weyl, i.e., λ /∈ ω(T ).<br />

Theorem 1.12.8. If S and T are commuting hyp<strong>on</strong>ormal operators then<br />

S, T are Weyl ⇐⇒ ST is Weyl.<br />

Hence if f is analytic in a neighborhood of σ(T ), then<br />

ω(f(T )) = f(ω(T )).<br />

Proof. See [LeL2].<br />

37


CHAPTER 1.<br />

FREDHOLM THEORY<br />

1.13 Comments and Problems<br />

Let H be an infinite dimensi<strong>on</strong>al separable Hilbert space. An operator T ∈ B(H)<br />

is called a Riesz operator if σ e (T ) = 0. If T ∈ B(H) then the West decompositi<strong>on</strong><br />

theorem [Wes] says that<br />

T is Riesz ⇐⇒ T = K + Q with compact K and quasinilpotent Q:<br />

this is equivalent to the following: if Q B(H) and Q C(H) denote the sets of quasinilpotents<br />

of B(H) and C(H), respectively, then<br />

π ( Q B(H)<br />

)<br />

= QC(H) , (1.16)<br />

where C(H) = B(H)/K(H) is the Calkin algebra and π denotes the Calkin homomorphism.<br />

It remains still open whether the West decompositi<strong>on</strong> theorem survives<br />

in the Banach space setting.<br />

Problem 1.1. Is the equality (1.16) true if H is a Banach space <br />

Suppose A is a Banach algebra with identity 1: we shall write A −1 for the invertible<br />

group of A and A −1<br />

0 for the c<strong>on</strong>nected comp<strong>on</strong>ents of the identity in A −1 . It was [Har3]<br />

known that<br />

A −1<br />

0 := Exp(A) = {e c1 e c2 · · · e c k<br />

: k ∈ N, c i ∈ A} .<br />

Evidently, Exp (A) is open, relatively closed in A −1 , c<strong>on</strong>nected and a normal subgroup.<br />

Write<br />

κ(A) := A −1 /Exp (A)<br />

for the abstract index group. The exp<strong>on</strong>ential spectrum ϵ(a) of a ∈ A is defined by<br />

Clearly,<br />

ϵ(a) := {λ ∈ C : a − λ /∈ Exp (A)}.<br />

∂ϵ(a) ⊂ σ(a) ⊂ ϵ(a).<br />

If A = B(H) then ϵ(a) = σ(a). We have known that σ(ab) \ {0} = σ(ba) \ {0}.<br />

However we were not able to answer to the following:<br />

Problem 1.2. If A is a Banach algebra and a, b ∈ A, does it follow that<br />

ϵ(ab) \ {0} = ϵ(ba) \ {0} <br />

38


Chapter 2<br />

Weyl <strong>Theory</strong><br />

2.1 Introducti<strong>on</strong><br />

In 1909, writing about differential equati<strong>on</strong>s, Hermann Weyl noticed something about<br />

the essential spectrum of a self adjoint operator <strong>on</strong> Hilbert space: when you take it<br />

away from the spectrum, you are left with the isolated eigenvalues of finite multiplicity.<br />

This was so<strong>on</strong> generalized to normal operators, and then to more and more classes of<br />

operators, bounded and unbounded, <strong>on</strong> Hilbert and <strong>on</strong> Banach spaces.<br />

The spectrum σ(T ) of a bounded linear operator T <strong>on</strong> a complex Banach space<br />

X is of course the set of those complex numbers for which T − λI does not have an<br />

everywhere defined two-sided inverse: this c<strong>on</strong>cept extends at <strong>on</strong>ce to the spectrum<br />

σ A (a) of a Banach algebra element a ∈ A. Thus the Fredholm essential spectrum<br />

σ e (T ) is the spectrum of the coset T + K(X) of the operator T ∈ B(X) in the Calkin<br />

algebra B(X)/K(X). Equivalently λ ∈ C is excluded from the spectrum σ(T ) if<br />

and <strong>on</strong>ly if operator T − λI is <strong>on</strong>e <strong>on</strong>e and <strong>on</strong>to, and is excluded from the essential<br />

spectrum σ e (T ) if and <strong>on</strong>ly if the operator T − λI has finite dimensi<strong>on</strong>al null space<br />

and range of finite co dimensi<strong>on</strong>.<br />

The Fredholm essential spectrum is c<strong>on</strong>tained in the larger Weyl spectrum, which<br />

also includes points λ ∈ C for which T − λI is Fredholm but with n<strong>on</strong> zero index: the<br />

two finite dimensi<strong>on</strong>s involved are unequal. Equivalently, T − λI /∈ B(X) −1 + K(X)<br />

cannot be expressed as the sum of an invertible and a compact operator. What is<br />

relevant here is that for self adjoint and more general normal operators the Weyl<br />

and the Fredholm spectra coincide: every normal Fredholm operator has index zero.<br />

Thus while the original Weyl observati<strong>on</strong> of 1909 may have seemed to subtract the<br />

Fredholm essential spectrum from the spectrum, it can equally be interpreted as<br />

subtracting the Weyl essential spectrum. For n<strong>on</strong> normal operators it is this modified<br />

versi<strong>on</strong> that seems to be the property that is of interest. For a linear operator <strong>on</strong><br />

a Banach space the most obvious points of its spectrum are the eigenvalues π 0 (T ),<br />

collecting λ ∈ C for which T − λI fails to be <strong>on</strong>e-<strong>on</strong>e. As is familiar from matrix<br />

theory, in finite dimensi<strong>on</strong>s this is all of the spectrum. In a sense therefore Weyl’s<br />

theorem seems to be suggesting that for nice operators the spectrum splits into a<br />

39


CHAPTER 2.<br />

WEYL THEORY<br />

finite dimensi<strong>on</strong>al comp<strong>on</strong>ent and a comp<strong>on</strong>ent modulo finite dimensi<strong>on</strong>s. Weyl’s<br />

theorem asks not just that the spectrum split into Fredholm spectra and eigenvalues:<br />

it wants the spectrum to divide into Weyl spectrum and eigenvalues which are both<br />

topologically isolated in the spectrum, and geometrically of finite multiplicity, with<br />

finite dimensi<strong>on</strong>al eigenspaces.<br />

40


CHAPTER 2.<br />

WEYL THEORY<br />

2.2 Weyl’s Theorem<br />

If T ∈ B(X) write π 0f (T ) for the eigenvalues of finite multiplicity; π 0i (T ) for the<br />

eigenvalues of infinite multiplicity; N(T ) and R(T ) for the null space and the range of<br />

T , respectively. If we write iso K = K \ acc K, and ∂ K for the topological boundary<br />

of K, and<br />

π 00 (T ) := {λ ∈ iso σ(T ) : 0 < dim N(T − λI) < ∞} (2.1)<br />

for the isolated eigenvalues of finite multiplicity, and ([Har4])<br />

p 00 (T ) := σ(T ) \ σ b (T ) (2.2)<br />

for the Riesz points of σ(T ), then by the punctured neighborhood theorem, i.e.,<br />

∂ σ(T ) \ σ e (T ) ⊆ iso σ(T ) (cf. [Har4], [HaL1]),<br />

iso σ(T ) \ σ e (T ) = iso σ(T ) \ ω(T ) = p 00 (T ) ⊆ π 00 (T ). (2.3)<br />

H. Weyl [We] examined the spectra of all compact perturbati<strong>on</strong>s T + K of a single<br />

hermitian operator T and discovered that λ ∈ σ(T + K) for every compact operator<br />

K if and <strong>on</strong>ly if λ is not an isolated eigenvalue of finite multiplicity in σ(T ). Today<br />

this result is known as Weyl’s theorem: that is, we say that Weyl’s theorem holds for<br />

T ∈ B(X) if there is equality<br />

σ(T ) \ ω(T ) = π 00 (T ). (2.4)<br />

In this secti<strong>on</strong> we explore the class of operators satisfying Weyl’s theorem.<br />

If T ∈ B(X), write r(T ) for the spectral radius of T . It is familiar that r(T ) ≤ ||T ||.<br />

An operator T is called normaloid if r(T ) = ||T || and isoloid if iso σ(T ) ⊆ π 0 (T ). If X<br />

is a Hilbert space, an operator T ∈ B(X) is called reducti<strong>on</strong>-isoloid if the restricti<strong>on</strong><br />

of T to any reducing subspace is isoloid.<br />

Let X be a Hilbert space and suppose that T ∈ B(X) is reduced by each of its<br />

finite-dimensi<strong>on</strong>al eigenspaces. If<br />

M := ∨ {N(T − λI) : λ ∈ π 0f (T )},<br />

then M reduces T . Let T 1 := T |M and T 2 := T |M ⊥ . Then we have ([Be2, Propositi<strong>on</strong><br />

4.1]) that<br />

(i) T 1 is a normal operator with pure point spectrum;<br />

(ii) π 0 (T 1 ) = π 0f (T );<br />

(iii) σ(T 1 ) = cl π 0 (T 1 );<br />

(iv) π 0 (T 2 ) = π 0 (T ) \ π 0f (T ) = π 0i (T ).<br />

In this case, S.Berberian ([Be2, Definiti<strong>on</strong> 5.4]) defined<br />

τ(T ) := σ(T 2 ) ∪ acc π 0f (T ). (2.5)<br />

41


CHAPTER 2.<br />

WEYL THEORY<br />

We shall call τ(T ) the Berberian spectrum of T . S. Berberian has also shown that τ(T )<br />

is a n<strong>on</strong>empty compact subset of σ(T ). We can, however, show that Weyl spectra,<br />

Browder spectra, and Berberian spectra all coincide for operators reduced by each of<br />

its finite-dimensi<strong>on</strong>al eigenspaces:<br />

Theorem 2.2.1. If X is a Hilbert space and T ∈ B(X) is reduced by each of its<br />

finite-dimensi<strong>on</strong>al eigenspaces then<br />

τ(T ) = ω(T ) = σ b (T ). (2.6)<br />

Proof. Let M be the closed linear span of the eigenspaces N(T − λI) (λ ∈ π 0f (T ))<br />

and write<br />

T 1 := T |M and T 2 := T |M ⊥ .<br />

From the preceding arguments it follows that T 1 is normal, π 0 (T 1 ) = π 0f (T ) and<br />

π 0f (T 2 ) = ∅. For (2.6) it will be shown that<br />

and<br />

ω(T ) ⊆ τ(T ) ⊆ σ b (T ) (2.7)<br />

σ b (T ) ⊆ ω(T ). (2.8)<br />

For the first inclusi<strong>on</strong> of (2.7) suppose λ ∈ σ(T ) \ τ(T ). Then T 2 − λI is invertible<br />

and λ ∈ iso π 0 (T 1 ). Since also π 0 (T 1 ) = π 0f (T 1 ), we have that λ ∈ π 00 (T 1 ). But since<br />

T 1 is normal, it follows that T 1 − λI is Weyl and hence so is T − λI. This proves<br />

the first inclusi<strong>on</strong>. For the sec<strong>on</strong>d inclusi<strong>on</strong> of (2.7) suppose λ ∈ σ(T ) \ σ b (T ). Thus<br />

T − λI is Browder but not invertible. Observe that the following equality holds with<br />

no other restricti<strong>on</strong> <strong>on</strong> either R or S:<br />

σ b (R ⊕ S) = σ b (R) ∪ σ b (S) for each R ∈ B(X 1 ) and S ∈ B(X 2 ). (2.9)<br />

Indeed if λ ∈ iso σ(R ⊕ S) then λ is either an isolated point of the spectra of direct<br />

summands or a resolvent element of direct summands, so that if R − λI and S − λI<br />

are Fredholm then by (2.3), λ is either a Riesz point or a resolvent element of direct<br />

summands, which implies that σ b (R) ∪ σ b (S) ⊆ σ b (R ⊕ S), and the reverse inclusi<strong>on</strong><br />

is evident. From this we can see that T 1 − λI and T 2 − λI are both Browder. But<br />

since π 0f (T 2 ) = ∅, it follows that T 2 − λI is <strong>on</strong>e-<strong>on</strong>e and hence invertible. Therefore<br />

λ ∈ π 00 (T 1 ) \ σ(T 2 ), which implies that λ /∈ τ(T ). This proves the sec<strong>on</strong>d inclusi<strong>on</strong> of<br />

(2.7). For (2.8) suppose λ ∈ σ(T ) \ ω(T ) and hence T − λI is Weyl but not invertible.<br />

Observe that if X 1 is a Hilbert space and if an operator R ∈ B(X 1 ) satisfies the<br />

equality ω(R) = σ e (R), then<br />

ω(R ⊕ S) = ω(R) ∪ ω(S) for each Hilbert space X 2 and S ∈ B(X 2 ) : (2.10)<br />

this follows from the fact that the index of a direct sum is the sum of the indices<br />

index (R ⊕ S − λ(I ⊕ I)) = index (R − λI) + index (S − λI)<br />

whenever λ /∈ σ e (R ⊕ S) = σ e (R) ∪ σ e (S). Since T 1 is normal, applying the equality<br />

(2.10) to T 1 in place of R gives that T 1 − λI and T 2 − λI are both Weyl. But since<br />

42


CHAPTER 2.<br />

WEYL THEORY<br />

π 0f (T 2 ) = ∅, we must have that T 2 − λI is invertible and therefore λ ∈ σ(T 1 ) \ ω(T 1 ).<br />

Thus from Weyl’s theorem for normal operators we can see that λ ∈ π 00 (T 1 ) and<br />

hence λ ∈ iso σ(T 1 ) ∩ ρ(T 2 ), which by (2.3), implies that λ /∈ σ b (T ). This proves (2.8)<br />

and completes the proof.<br />

As applicati<strong>on</strong>s of Theorem 2.2.1 we will give several corollaries below.<br />

Corollary 2.2.2. If X is a Hilbert space and T ∈ B(X) is reduced by each of its<br />

finite-dimensi<strong>on</strong>al eigenspaces then σ(T ) \ ω(T ) ⊆ π 00 (T ).<br />

Proof. This follows at <strong>on</strong>ce from Theorem 2.2.1.<br />

Weyl’s theorem is not transmitted to dual operators: for example if T : l 2 → l 2 is<br />

the unilateral weighted shift defined by<br />

T e n = 1<br />

n + 1 e n+1 (n ≥ 0), (2.11)<br />

then σ(T ) = ω(T ) = {0} and π 00 (T ) = ∅, and therefore Weyl’s theorem holds for T ,<br />

but fails for its adjoint T ∗ . We however have:<br />

Corollary 2.2.3. Let X be a Hilbert space. If T ∈ B(X) is reduced by each of its<br />

finite-dimensi<strong>on</strong>al eigenspaces and iso σ(T ) = ∅, then Weyl’s theorem holds for T and<br />

T ∗ . In this case, σ(T ) = ω(T ).<br />

Proof. If iso σ(T ) = ∅, then it follows from Corollary 2.2.2 that σ(T ) = ω(T ), which<br />

says that Weyl’s theorem holds for T . The asserti<strong>on</strong> that Weyl’s theorem holds for<br />

T ∗ follows from noting that σ(T ) ∗ = ( σ(T ) ) −<br />

, ω(T ∗ ) = ( ω(T ) ) −<br />

and π00 (T ∗ ) =<br />

(<br />

π00 (T ) ) −<br />

= ∅.<br />

In Corollary 2.2.3, the c<strong>on</strong>diti<strong>on</strong> “iso σ(T ) = ∅” cannot be replaced by the c<strong>on</strong>diti<strong>on</strong><br />

“π 00 (T ) = ∅”: for example c<strong>on</strong>sider the operator T defined by (2.11).<br />

Corollary 2.2.4. ([Be1, Theorem]) If X is a Hilbert space and T ∈ B(X) is reducti<strong>on</strong>isoloid<br />

and is reduced by each of its finite-dimensi<strong>on</strong>al eigenspaces then Weyl’s theorem<br />

holds for T .<br />

Proof. In view of Corollary 2.2.2, it suffices to show that π 00 (T ) ⊆ σ(T ) \ ω(T ).<br />

Suppose λ ∈ π 00 (T ). Then with the preceding notati<strong>on</strong>s, λ ∈ π 00 (T 1 ) ∩ [ iso σ(T 2 ) ∪<br />

ρ(T 2 ) ] . If λ ∈ iso σ(T 2 ), then since by assumpti<strong>on</strong> T 2 is isoloid we have that λ ∈ π 0 (T 2 )<br />

and hence λ ∈ π 0f (T 2 ). But since π 0f (T 2 ) = ∅, we should have that λ /∈ iso σ(T 2 ).<br />

Thus λ ∈ π 00 (T 1 ) ∩ ρ(T 2 ). Since T 1 is normal it follows that T 1 − λI is Weyl and so<br />

is T − λI; therefore λ ∈ σ(T ) \ ω(T ).<br />

Since hyp<strong>on</strong>ormal operators are isoloid and are reduced by each of its eigenspaces,<br />

it follows from Corollary 2.2.4 that Weyl’s theorem holds for hyp<strong>on</strong>ormal operators.<br />

43


CHAPTER 2.<br />

WEYL THEORY<br />

If the c<strong>on</strong>diti<strong>on</strong> “reducti<strong>on</strong>-isoloid” is replaced by “isoloid” then Corollary 2.2.4<br />

may fail: for example, c<strong>on</strong>sider the operator T = T 1 ⊕ T 2 , where T 1 is the <strong>on</strong>edimensi<strong>on</strong>al<br />

zero operator and T 2 is an injective quasinilpotent compact operator.<br />

If X is a Hilbert space, an operator T ∈ B(X) is said to be p-hyp<strong>on</strong>ormal if<br />

(T ∗ T ) p − (T T ∗ ) p ≥ 0 (cf. [Al],[Ch3]). If p = 1, T is hyp<strong>on</strong>ormal and if p = 1 2 , T is<br />

semi-hyp<strong>on</strong>ormal.<br />

Corollary 2.2.5. [CIO] Weyl’s theorem holds for every p-hyp<strong>on</strong>ormal operator.<br />

Proof. This follows from the fact that every p-hyp<strong>on</strong>ormal operator is isoloid and is<br />

reduced by each of its eigenspaces ([Ch3]).<br />

L. Coburn [Co, Corollary 3.2] has shown that if T ∈ B(X) is hyp<strong>on</strong>ormal and<br />

π 00 (T ) = ∅, then T is extremally n<strong>on</strong>compact, in the sense that<br />

||T || = ||π(T )||,<br />

where π is the can<strong>on</strong>ical map of B(X) <strong>on</strong>to the Calkin algebra B(X)/K(X). His<br />

proof relies up<strong>on</strong> the fact that Weyl’s theorem holds for hyp<strong>on</strong>ormal operators, and<br />

hence σ(T ) = ω(T ) since π 00 (T ) = ∅. Now we can strengthen the Coburn’s argument<br />

slightly:<br />

Corollary 2.2.6. If T ∈ B(X) is normaloid and π 00 (T ) = ∅, then T is extremally<br />

n<strong>on</strong>compact.<br />

Proof. Since σ(T ) ⊆ η ω(T )∪p 00 (T ) for any T ∈ B(X), we have that η σ(T )\η ω(T ) ⊆<br />

π 00 (T ). Thus by our assumpti<strong>on</strong>, η σ(T ) = η ω(T ). Therefore we can argue that for<br />

each compact operator K ∈ B(X),<br />

||T || = r(T ) = r ω (T ) = r ω (T + K) ≤ r(T + K) ≤ ||T + K||,<br />

where r ω (T ) denotes the “Weyl spectral radius”. This completes the proof.<br />

Note that if T ∈ B(X) is normaloid and π 00 (T ) = ∅, then Weyl’s theorem may<br />

fail for T ; for example take X = l 2 ⊕ l 2 and T = U ⊕ U ∗ , where U is the unilateral<br />

shift.<br />

We next c<strong>on</strong>sider Weyl’s theorem for Toeplitz operators.<br />

The Hilbert space L 2 (T) has a can<strong>on</strong>ical orth<strong>on</strong>ormal basis given by the trig<strong>on</strong>ometric<br />

functi<strong>on</strong>s e n (z) = z n , for all n ∈ Z, and the Hardy space H 2 (T) is the closed<br />

linear span of {e n : n = 0, 1, . . . }. An element f ∈ L 2 is referred to as analytic if<br />

f ∈ H 2 and coanalytic if f ∈ L 2 ⊖H 2 . If P denotes the projecti<strong>on</strong> operator L 2 → H 2 ,<br />

then for every φ ∈ L ∞ (T), the operator T φ <strong>on</strong> H 2 defined by<br />

is called the Toeplitz operator with symbol φ.<br />

T φ g = P (φg) for all g ∈ H 2 (2.12)<br />

44


CHAPTER 2.<br />

WEYL THEORY<br />

Theorem 2.2.7. [Co] Weyl’s theorem holds for every Toeplitz operator T φ .<br />

Proof. It was known [Wi2] that σ(T φ ) is always c<strong>on</strong>nected. Since there are no<br />

quasinilpotent Toeplitz operators except 0, σ(T φ ) can have no isolated eigenvalues<br />

of finite multiplicity. Thus Weyl’s theorem is equivalent to the fact that<br />

σ(T φ ) = ω(T φ ). (2.13)<br />

Since T φ − λI = T φ−λ , it suffices to show that if T φ is Weyl then T φ is invertible.<br />

If T φ is not invertible, but is Weyl then it is easy to see that both T φ and Tφ ∗ = T φ<br />

must have n<strong>on</strong>trivial kernels. Thus we want to show that this can not happen, unless<br />

φ = 0 and hence T φ is the n<strong>on</strong>-Weyl operator.<br />

Suppose that there exist n<strong>on</strong>zero functi<strong>on</strong>s φ, f, and g (φ ∈ L ∞ and f, g ∈ H 2 )<br />

such that T φ f = 0 and T φ g = 0. Then P (φf) = 0 and P (φg) = 0, so that there exist<br />

functi<strong>on</strong>s h, k ∈ H 2 such that<br />

∫ ∫<br />

h dθ = k dθ = 0 and φf = h, φg = k.<br />

Thus by the F. and M. Riesz’s theorem, φ, f, g, h, k are all n<strong>on</strong>zero except <strong>on</strong> a set<br />

of measure zero. We thus have that f/g = h/k pointwise a.e., so that fk = gh a.e.,<br />

which implies gh = 0 a.e. Again by the F. and M. Riesz’s theorem, we can c<strong>on</strong>clude<br />

that either g = 0 a.e. or h = 0 a.e. This c<strong>on</strong>tradicti<strong>on</strong> completes the proof.<br />

We review here a few essential facts c<strong>on</strong>cerning Toeplitz operators with c<strong>on</strong>tinuous<br />

symbols, using [Do1] as a general reference. The sets C(T) of all c<strong>on</strong>tinuous complexvalued<br />

functi<strong>on</strong>s <strong>on</strong> the unit circle T and H ∞ (T) = L ∞ ∩H 2 are Banach algebras, and<br />

it is well-known that every Toeplitz operator with symbol φ ∈ H ∞ is subnormal. The<br />

C ∗ -algebra A generated by all Toeplitz operators T φ with φ ∈ C(T) has an important<br />

property which is very useful for spectral theory: the commutator ideal of A is the<br />

ideal K(H 2 ) of compact operators <strong>on</strong> H 2 . As C(T) and A/K(H 2 ) are ∗-isomorphic<br />

C ∗ -algebras, then for every φ ∈ C(T),<br />

T φ is a Fredholm operator if and <strong>on</strong>ly if φ is invertible (2.14)<br />

index T φ = −wn(φ) , (2.15)<br />

σ e (T φ ) = φ(T) , (2.16)<br />

where wn(φ) denotes the winding number of φ with respect to the origin. Finally,<br />

we make note that if φ ∈ C(T) and if f is an analytic functi<strong>on</strong> defined <strong>on</strong> an open<br />

set c<strong>on</strong>taining σ(T φ ), then f ◦ φ ∈ C(T) and f(T φ ) is well-defined by the analytic<br />

functi<strong>on</strong>al calculus.<br />

We require the use of certain closed subspaces and subalgebras of L ∞ (T), which<br />

are described in further detail in [Do2] and Appendix 4 of [Ni]. Recall that the<br />

subspace H ∞ (T) + C(T) is a closed subalgebra of L ∞ . The elements of the closed<br />

selfadjoint subalgebra QC, which is defined to be<br />

QC = ( H ∞ (T) + C(T) ) ∩ ( H ∞ (T) + C(T) ) ,<br />

45


CHAPTER 2.<br />

WEYL THEORY<br />

are called quasic<strong>on</strong>tinuous functi<strong>on</strong>s. The subspace P C is the closure in L ∞ (T) of<br />

the set of all piecewise c<strong>on</strong>tinuous functi<strong>on</strong>s <strong>on</strong> T. Thus φ ∈ P C if and <strong>on</strong>ly if it is<br />

right c<strong>on</strong>tinuous and has both a left- and right-hand limit at every point. There are<br />

certain algebraic relati<strong>on</strong>s am<strong>on</strong>g Toeplitz operators whose symbols come from these<br />

classes, including<br />

and<br />

T ψ T φ − T ψφ ∈ K(H 2 ) for every φ ∈ H ∞ (T) + C(T) and ψ ∈ L ∞ (T) , (2.17)<br />

the commutator [T φ , T ψ ] is compact for every φ, ψ ∈ P C . (2.18)<br />

We add to these relati<strong>on</strong>s the following <strong>on</strong>e.<br />

Lemma 2.2.8. If T φ is a Toeplitz operator with quasic<strong>on</strong>tinuous symbol φ, and if<br />

f ∈ H(σ(T φ )), then T f◦φ − f(T φ ) is a compact operator.<br />

Proof. Assume that φ ∈ QC. Recall from [Do1, p.188] that if ψ ∈ H ∞ + C(T),<br />

then T ψ is Fredholm if and <strong>on</strong>ly if ψ is invertible in H ∞ + C(T). Therefore for every<br />

λ ∉ σ(T φ ), both φ−λ and φ − λ are invertible in H ∞ +C(T); hence, (φ−λ) −1 ∈ QC.<br />

Using this fact together with (2.17) we have that, for ψ ∈ L ∞ and λ, µ ∈ C,<br />

T φ−µ T ψ T (φ−λ) −1 − T (φ−µ)ψ(φ−λ) −1 ∈ K(H 2 ) whenever λ /∈ σ(T φ ) .<br />

The arguments above extend to rati<strong>on</strong>al functi<strong>on</strong>s to yield: if r is any rati<strong>on</strong>al functi<strong>on</strong><br />

with all of its poles outside of σ(T φ ), then r(T φ ) − T r◦φ ∈ K(H 2 ). Suppose that f<br />

is an analytic functi<strong>on</strong> <strong>on</strong> an open set c<strong>on</strong>taining σ(T φ ). By Runge’s theorem there<br />

exists a sequence of rati<strong>on</strong>al functi<strong>on</strong>s r n such that the poles of each r n lie outside of<br />

σ(T φ ) and r n → f uniformly <strong>on</strong> σ(T φ ). Thus r n (T φ ) → f(T φ ) in the norm-topology of<br />

L(H 2 ). Furthermore, because r n ◦ φ → f ◦ φ uniformly, we have T rn◦φ → T f◦φ in the<br />

norm-topology. Hence, T f◦φ − f(T φ ) = lim ( T rn ◦φ − r n (T φ ) ) , which is compact.<br />

Lemma 2.2.8 does not extend to piecewise c<strong>on</strong>tinuous symbols φ ∈ P C, as we<br />

cannot guarantee that T n φ − T φ n ∈ K(H 2 ) for each n ∈ Z + . For example, if φ(e iθ ) =<br />

χ T+ − χ T− , where χ T+ and χ T− are characteristic functi<strong>on</strong>s of, respectively, the upper<br />

semicircle and the lower semicircle, then T 2 φ − I is not compact.<br />

Corollary 2.2.9. If T φ is a Toeplitz operator with quasic<strong>on</strong>tinuous symbol φ, then<br />

for every f ∈ H(σ(T φ )),<br />

1. ω(f(T φ )) = σ(T f◦φ ), and<br />

2. f(T φ ) is essentially normal and is unitarily equivalent to a compact perturbati<strong>on</strong><br />

of f(T φ )⊕M f◦φ , where M f◦φ is the operator of multiplicati<strong>on</strong> by f ◦φ <strong>on</strong> L 2 (T).<br />

Proof. Because the Weyl spectrum is stable under the compact perturbati<strong>on</strong>s, it follows<br />

from Lemma 2.2.8 that ω(f(T φ )) = ω(T f◦φ ) = σ(T f◦φ ), which proves statement<br />

(1). To prove (2), observe that because QC is a closed algebra, the compositi<strong>on</strong> of the<br />

analytic functi<strong>on</strong> f with φ ∈ QC produces a quasic<strong>on</strong>tinuous functi<strong>on</strong> f ◦ φ ∈ QC.<br />

46


CHAPTER 2.<br />

WEYL THEORY<br />

Moreover, by (2.17), every Toeplitz operator with quasic<strong>on</strong>tinuous symbol is essentially<br />

normal. The (normal) Laurent operator M f◦φ <strong>on</strong> L 2 (T) has its spectrum c<strong>on</strong>tained<br />

within the spectrum of the (essentially normal) Toeplitz operator T f◦φ . Thus<br />

there is the following relati<strong>on</strong>ship involving the essentially normal operators f(T φ )<br />

and M f◦φ ⊕ f(T φ ):<br />

σ e<br />

(<br />

f(Tφ ) ⊕ M f◦φ<br />

)<br />

= σe (f(T φ )) and SP(f(T φ )) = SP ( f(T φ ) ⊕ M f◦φ<br />

)<br />

,<br />

where SP(T ) denotes the spectral picture of an operator T . (The spectral picture<br />

SP(T ) is the structure c<strong>on</strong>sisting of the set σ e (T ), the collecti<strong>on</strong> of holes and pseudoholes<br />

in σ e (T ), and the Fredholm indices associated with these holes and pseudoholes.)<br />

Thus it follows from the Brown-Douglas-Fillmore theorem [Pe] that f(T φ ) is compalent<br />

to f(T φ ) ⊕ M f◦φ , in the sense that there exists a unitary operator W and a<br />

compact operator K such that W ( f(T φ ) ⊕ M f◦φ<br />

)<br />

W ∗ + K = f(T φ ).<br />

Corollary 2.2.9 (1) can be viewed as saying that σ(f(T φ )) \ σ(T f◦φ ) c<strong>on</strong>sists of<br />

holes with winding number zero.<br />

We c<strong>on</strong>sider the following questi<strong>on</strong> ([Ob2]):<br />

if T φ is a Toeplitz operator, then does Weyl’s theorem hold for T 2 φ (2.19)<br />

To answer the above questi<strong>on</strong>, we need a spectral property of Toeplitz operators with<br />

c<strong>on</strong>tinuous symbols.<br />

Lemma 2.2.10. Suppose that φ is c<strong>on</strong>tinuous and that f ∈ H(σ(T φ )). Then<br />

σ(T f◦φ ) ⊆ f(σ(T φ )) , (2.20)<br />

and equality occurs if and <strong>on</strong>ly if Weyl’s theorem holds for f(T φ ).<br />

Proof. By Corollary 2.2.9, σ(T f◦φ ) = ω(f(T φ )) ⊆ σ(f(T φ )) = f(σ(T φ )). Because<br />

σ(T φ ) is c<strong>on</strong>nected, so is f(σ(T φ )) = σ(f(T φ )); therefore the set π 00 (f(T φ )) is<br />

empty. Again by Corollary 2.2.9, ω(f(T φ )) = σ(T f◦φ ) and so ω(f(T φ )) = σ(f(T φ )) \<br />

π 00 (f(T φ )) if and <strong>on</strong>ly if σ(T f◦φ ) = f(σ(T φ )).<br />

If φ is not c<strong>on</strong>tinuous, it is possible for Weyl’s theorem to hold for some f(T φ )<br />

without σ(T f◦φ ) being equal to f(σ(T φ )). One example is as follows. Let φ(e i θ ) =<br />

e i θ<br />

3 (0 ≤ θ < 2π), a piecewise c<strong>on</strong>tinuous functi<strong>on</strong>. The operator T φ is invertible but<br />

T φ 2 is not; hence 0 ∈ σ(T φ 2) \ {σ(T φ )} 2 . However ω(Tφ) 2 = σ(Tφ), 2 and π 00 (Tφ) 2 is<br />

empty (see Figure 2); therefore Weyl’s theorem holds for Tφ.<br />

2<br />

We can now answer the questi<strong>on</strong> (2.19): the answer is no.<br />

Example 2.2.11. There exists a c<strong>on</strong>tinuous functi<strong>on</strong> φ ∈ C(T) such that σ(T φ 2) ≠<br />

{σ(T φ )} 2 .<br />

47


CHAPTER 2.<br />

WEYL THEORY<br />

Proof. Let φ be defined by<br />

φ(e i θ ) =<br />

{<br />

−e 2i θ + 1 (0 ≤ θ ≤ π)<br />

e −2i θ − 1 (π ≤ θ ≤ 2π) .<br />

The orientati<strong>on</strong> of the graph of φ is shown in Figure 3. Evidently, φ is c<strong>on</strong>tinuous and,<br />

in Figure 3, φ has winding number +1 with respect to the hole of C 1 ; the hole of C 2<br />

has winding number −1. Thus we have σ e (T φ ) = φ(T) and σ(T φ ) = c<strong>on</strong>v φ(T). On the<br />

other hand, a straightforward calculati<strong>on</strong> shows that φ 2 (T) is the Cardioid r = 2(1 +<br />

cos θ). In particular, φ 2 (T) traverses the Cardioid <strong>on</strong>ce in a counterclockwise directi<strong>on</strong><br />

and then traverses the Cardioid <strong>on</strong>ce in a clockwise directi<strong>on</strong>. Thus wn(φ 2 − λ) = 0<br />

for each λ in the hole of φ 2 (T). Hence T φ2 −λ is a Weyl operator and is, therefore,<br />

invertible for each λ in the hole of φ 2 (T). This implies that σ(T φ 2) is the Cardioid<br />

r = 2(1 + cos θ). But because {σ(T φ )} 2 = {c<strong>on</strong>v φ(T)} 2 = {(r, θ) : r ≤ 2(1 + cos θ)},<br />

it follows that σ(T φ 2) ≠ {σ(T φ )} 2 .<br />

We next c<strong>on</strong>sider Weyl’s theorem through the local spectral theory. Local spectral<br />

theory is based <strong>on</strong> the existence of analytic soluti<strong>on</strong>s f : U → X to the equati<strong>on</strong><br />

(T − λI)f(λ) = x <strong>on</strong> an open subset U ⊂ C, for a given operator T ∈ B(X) and a<br />

given element x ∈ X. We define the spectral subspace as follows: for a closed set<br />

F ⊂ C, let<br />

X T (F ) := {x ∈ X : (T − λI)f(λ) = x has an analytic soluti<strong>on</strong> f : C \ F → X}.<br />

We say that T ∈ B(X) has the single valued extensi<strong>on</strong> property (SVEP) at λ 0 ∈ C<br />

if for every neighborhood U of λ 0 , f = 0 is the <strong>on</strong>ly analytic soluti<strong>on</strong> f : U → X<br />

satisfying (T − λI)f(λ) = 0. We also say that T has the SVEP if T has this property<br />

at every λ ∈ C. The local spectrum of T at x is defined by<br />

σ T (x) := C\ ∪{ }<br />

(T −λI)f(λ) = x has an analytic soluti<strong>on</strong> f : U → X <strong>on</strong> the open subset U ⊂ C .<br />

If T has the SVEP then X T (F ) = {x ∈ X : σ T (x) ⊂ F }.<br />

The following lemma gives a c<strong>on</strong>necti<strong>on</strong> of the SVEP with a finite ascent property.<br />

Lemma 2.2.12. [Fin] If T ∈ B(X) is semi-Fredholm then<br />

T has the SVEP at 0 ⇐⇒ T has a finite ascent at 0.<br />

The finite dimensi<strong>on</strong>ality of X T ({λ}) is necessary ad sufficient for T − λI to be<br />

Fredholm whenever λ is an isolated point of the spectrum.<br />

Lemma 2.2.13. [Ai] Let T ∈ B(X). If λ ∈ iso σ(T ) then<br />

λ /∈ σ e (T ) ⇐⇒ X T ({λ}) is finite dimensi<strong>on</strong>al.<br />

48


CHAPTER 2.<br />

WEYL THEORY<br />

Theorem 2.2.14. If T ∈ B(X) has the SVEP then the following are equivalent:<br />

(a) Weyl’s theorem holds for T ;<br />

(b) R(T − λI) is closed for every λ ∈ π 00 (T );<br />

(c) X T ({λ}) is finite dimensi<strong>on</strong>al for every λ ∈ π 00 (T ).<br />

Proof. (a) ⇒ (b): Evident.<br />

(b) ⇒ (a): If λ ∈ σ(T ) \ ω(T ) then by Lemma 2.2.12, T − λI has a finite ascent.<br />

Thus T − λI is Browder and hence λ ∈ π 00 (T ). C<strong>on</strong>versely, if λ ∈ π 00 (T ) then by<br />

assumpti<strong>on</strong> T − λI is Browder, so λ ∈ σ(T ) \ ω(T ).<br />

(b) ⇔ (c): Immediate from Lemma 2.2.13.<br />

An operator T ∈ B(X) is called reguloid if each isolated point of spectrum is a<br />

regular point, in the sense that there is a generalized inverse:<br />

λ ∈ iso σ(T ) =⇒ T − λI = (T − λI)S λ (T − λI) with S λ ∈ B(X).<br />

It was known [Har4] that if T is reguloid then R(T −λI) is closed for each λ ∈ iso σ(T ).<br />

Also an operator T ∈ B(X) is said to satisfy the growth c<strong>on</strong>diti<strong>on</strong> (G 1 ), if for all<br />

λ ∈ C \ σ(T )<br />

||(T − λI) −1 ||dist(λ, σ(T )) ≤ 1.<br />

Lemma 2.2.15. If T ∈ B(X) then<br />

(G 1 ) =⇒ reguloid =⇒ isoloid. (2.21)<br />

Proof. Recall ([Har4, Theorem 7.3.4]) that if T − λI has a generalized inverse and<br />

if λ ∈ ∂σ(T ) is in the boundary of the spectrum then T − λI has an invertible<br />

generalized inverse. If therefore T is reguloid and λ ∈ iso σ(T ) then T − λI has an<br />

invertible generalized inverse, and hence ([Har4, (3.8.6.1)])<br />

N(T − λI) ∼ = X/R(T − λI).<br />

Thus if N(T − λI) = {0} then T − λI is invertible, a c<strong>on</strong>tradicti<strong>on</strong>. Therefore λ is<br />

an eigenvalue of T , which proves the sec<strong>on</strong>d implicati<strong>on</strong> of (2.21). Towards the first<br />

implicati<strong>on</strong> we may write T in place of T − λI and hence assume λ = 0: then using<br />

the spectral projecti<strong>on</strong> at 0 ∈ C we can represent T as a 2 × 2 operator matrix:<br />

[ ]<br />

T0 0<br />

T = ,<br />

0 T 1<br />

where σ(T 0 ) = {0} and σ(T 1 ) = σ(T ) \ {0}. Now we can borrow an argument of J.<br />

Stampfli ([Sta1, Theorem C]): take 0 < ϵ ≤ 1 2dist(0, σ(T ) \ {0}) and argue<br />

T 0 = 1 ∫<br />

z(T − zI) −1 dz,<br />

2πi<br />

|z|=ϵ<br />

using the growth c<strong>on</strong>diti<strong>on</strong> (G 1 ) to see that<br />

||T 0 || ≤ 1 ∫<br />

|z| ||(T − zI) −1 || |dz| ≤ 1<br />

2π<br />

2π ϵ1 2πϵ = ϵ, (2.22)<br />

ϵ<br />

|z|=ϵ<br />

49


CHAPTER 2.<br />

WEYL THEORY<br />

which tends to 0 with ϵ. It follows that T 0 = 0 and hence that<br />

[ ]<br />

[ ]<br />

0 0<br />

0 0<br />

T = = T ST with S =<br />

0 T 1 0 T1<br />

−1<br />

has a generalized inverse.<br />

Corollary 2.2.16. If T ∈ B(X) is reguloid and has the SVEP then Weyl’s theorem<br />

holds for T .<br />

Proof. Immediate from Theorem 2.2.14.<br />

Lemma 2.2.17. Let T ∈ B(X). If for any λ ∈ C, X T ({λ}) is closed then T has the<br />

SVEP.<br />

Proof. This follows from [Ai, Theorem 2.31] together with the fact that<br />

X T ({λ}) = {x ∈ X : lim<br />

n→∞ ||(T − λI)n x|| 1 n = 0}.<br />

Corollary 2.2.18. If T ∈ B(X) satisfies<br />

X T ({λ}) = N(T − λI) for every λ ∈ C, (2.23)<br />

then T has the SVEP and both T and T ∗ are reguloid. Thus in particular if T satisfies<br />

(2.23) then Weyl’s theorem holds for T .<br />

Proof. If T satisfies the c<strong>on</strong>diti<strong>on</strong> (2.23) then by Lemma 2.2.17, T has the SVEP. The<br />

sec<strong>on</strong>d asserti<strong>on</strong> follows from [Ai, Theorem 3.96]. The last asserti<strong>on</strong> follows at <strong>on</strong>ce<br />

from Corollary 2.2.16.<br />

An operator T ∈ B(X) is said to be paranormal if<br />

||T x|| 2 ≤ ||T 2 x|| ||x|| for every x ∈ X.<br />

It was well known that if T ∈ B(X) is paranormal then the following hold:<br />

(a) T is normaloid;<br />

(b) T has finite ascent;<br />

(c) if x and y are n<strong>on</strong>zero eigenvectors corresp<strong>on</strong>ding to, respectively, distinct<br />

n<strong>on</strong>zero eigenvalues of T , then ||x|| ≤ ||x + y|| ([ChR, Theorem 2,6])<br />

In particular, p-hyp<strong>on</strong>ormal operators are paranormal (cf. [FIY]). An operator T ∈<br />

B(X) is said to be totally paranormal if T − λI is paranormal for every λ ∈ C.<br />

Evidently, every hyp<strong>on</strong>ormal operator is totally paranormal. On the other hand,<br />

every totally paranormal operator satisfies (2.23): indeed, for every x ∈ X and λ ∈ C,<br />

||(T − λI) n x|| 1 n ≥ ||(T − λI)x|| for every n ∈ N.<br />

50


CHAPTER 2.<br />

WEYL THEORY<br />

So if x ∈ X T ({λ}) then ||(T − λI) n x|| 1 n → 0 as n → ∞, so that x ∈ N(T − λI),<br />

which gives X T ({λ}) ⊂ N(T − λI). The reverse inclusi<strong>on</strong> is true for every operator.<br />

Therefore by Corollary 2.2.18 we can c<strong>on</strong>clude that Weyl’s theorem holds for totally<br />

paranormal operators. We can prove more:<br />

Theorem 2.2.19. Weyl’s theorem holds for paranormal operators <strong>on</strong> a separable<br />

Banach space.<br />

Proof. It was known [ChR] that paranormal operators <strong>on</strong> a separable Banach space<br />

have the SVEP. So in view of Theorem 2.2.14 it suffices to show that R(T − λI)<br />

is closed ∫for each λ ∈ π 00 (T ). Suppose λ ∈ π 00 (T ). Using the spectral projecti<strong>on</strong><br />

P = 1<br />

2πi ∂B (λI − T )−1 dλ, where B is an open disk of center λ which c<strong>on</strong>tains no<br />

other points of σ(T ), we can represent T as the direct sum<br />

T = T 1 ⊕ T 2 ,<br />

where σ(T 1 ) = {λ} and σ(T 2 ) = σ(T ) \ {λ}.<br />

If λ = 0 then T 1 is a quasinilpotent paranormal operator, so that T 1 = 0. If λ ≠ 0<br />

write T A = 1 λ T 1. Then T A is paranormal and σ(T A ) = {1}. Since T A is invertible we<br />

have that T A and T −1<br />

A<br />

are paranormal, and hence normaloid. So ||T A|| = ||T −1<br />

A || = 1<br />

and hence<br />

||x|| = ||T −1<br />

A<br />

T Ax|| ≤ ||T A x|| ≤ ||x||,<br />

which implies that T A and T −1<br />

A<br />

are isometries. Also since T A − 1 is a quasinilpotent<br />

operator it follows that T A = I, and hence T 1 = λI. Thus we have that T − λI =<br />

0 ⊕ (T 2 − λI) has closed range. This completes the proof.<br />

Does Weyl’s theorem hold for paranormal operators <strong>on</strong> an arbitrary Banach space<br />

Paranormal operators <strong>on</strong> an arbitrary Banach space may not have the SVEP. So the<br />

proof of Theorem 2.2.19 does not work for arbitrary Banach spaces. In spite of it<br />

Weyl’s theorem holds for paranormal operators <strong>on</strong> an arbitrary Banach space. To see<br />

this recall the reduced minimum modulus of T is defined by<br />

γ(T ) := inf<br />

||T x||<br />

dist (x, N(T )<br />

(x /∈ N(T )).<br />

It was known [Go] that γ(T ) > 0 if and <strong>on</strong>ly if T has closed range.<br />

Theorem 2.2.20. Weyl’s theorem holds for paranormal operators <strong>on</strong> a Banach space.<br />

Proof. The proof of Theorem 2.2.19 shows that with no restricti<strong>on</strong> <strong>on</strong> X, π 00 (T ) ⊂<br />

σ(T ) \ ω(T ) for every paranormal operator T ∈ B(X). Thus we must show that<br />

σ(T ) \ ω(T ) ⊂ iso σ(T ). Suppose λ ∈ σ(T ) \ ω(T ). If λ = 0 then T is Weyl and has<br />

finite ascent. Thus T is Browder, and hence 0 ∈ iso σ(T ). If λ ≠ 0 and λ /∈ iso σ(T )<br />

then we can find a sequence {λ n } of n<strong>on</strong>zero eigenvalues such that λ n → λ. By the<br />

property (c) above Theorem 2.2.19,<br />

(<br />

)<br />

dist x λn , N(T − λI) ≥ 1 for each unit vector x λn ∈ N(T − λ n I).<br />

51


CHAPTER 2.<br />

WEYL THEORY<br />

We thus have<br />

||(T − λI)x n ||<br />

dist (x λn , N(T − λI)) = |λ n − λ|<br />

dist (x λn , N(T − λI)) → 0,<br />

which shows that γ(T − λI) = 0 and hence T − λI does not have closed range, a<br />

c<strong>on</strong>tradicti<strong>on</strong>. Therefore λ ∈ iso σ(T ). This completes the proof.<br />

52


CHAPTER 2.<br />

WEYL THEORY<br />

2.3 Spectral Mapping Theorem for the Weyl spectrum<br />

Let S denote the set, equipped with the Hausdorff metric, of all compact subsets<br />

of C. If A is a unital Banach algebra then the spectrum can be viewed as a functi<strong>on</strong><br />

σ : A → S, mapping each T ∈ A to its spectrum σ(T ). It is well-known that<br />

the functi<strong>on</strong> σ is upper semic<strong>on</strong>tinuous, i.e., if T n → T then lim sup σ(T n ) ⊂ σ(T )<br />

and that in n<strong>on</strong>commutative algebras, σ does have points of disc<strong>on</strong>tinuity. The work<br />

of J. Newburgh [Ne] c<strong>on</strong>tains the fundamental results <strong>on</strong> spectral c<strong>on</strong>tinuity in general<br />

Banach algebras. J. C<strong>on</strong>way and B. Morrel [CoM] have undertaken a detailed<br />

study of spectral c<strong>on</strong>tinuity in the case where the Banach algebra is the C ∗ -algebra<br />

of all operators acting <strong>on</strong> a complex separable Hilbert space. Of interest is the identificati<strong>on</strong><br />

of points of spectral c<strong>on</strong>tinuity, and of classes C of operators for which σ<br />

becomes c<strong>on</strong>tinuous when restricted to C. In [BGS], the c<strong>on</strong>tinuity of the spectrum<br />

was c<strong>on</strong>sidered when restricted to certain subsets of the entire manifold of Toeplitz<br />

operators. The set of normal operators is perhaps the most immediate in the latter<br />

directi<strong>on</strong>: σ is c<strong>on</strong>tinuous <strong>on</strong> the set of normal operators. As noted in Soluti<strong>on</strong> 104 of<br />

[Ha3], Newburgh’s argument uses the fact that the inverses of normal resolvents are<br />

normaloid. This argument can be easily extended to the set of hyp<strong>on</strong>ormal operators<br />

because the inverses of hyp<strong>on</strong>ormal resolvents are also hyp<strong>on</strong>ormal and hence normaloid.<br />

Although p-hyp<strong>on</strong>ormal operators are normaloid, it was shown [HwL1] that<br />

σ is c<strong>on</strong>tinuous <strong>on</strong> the set of all p-hyp<strong>on</strong>ormal operators.<br />

We now examine the c<strong>on</strong>tinuity of the Weyl spectrum in pace of the spectrum.<br />

In general the Weyl spectrum is not c<strong>on</strong>tinuous: indeed, it was in [BGS] that the<br />

spectrum is disc<strong>on</strong>tinuous <strong>on</strong> the entire manifold of Toeplitz operators. Since the<br />

spectra and the Weyl spectra coincide for Toeplitz operators, it follows at <strong>on</strong>ce that<br />

the weyl spectrum is disc<strong>on</strong>tinuous.<br />

However the Weyl spectrum is upper semic<strong>on</strong>tinuous.<br />

Lemma 2.3.1. The map T → ω(T ) is upper semic<strong>on</strong>tinuous.<br />

Proof. Let λ ∈ ω(T ). Since the set of Weyl operators forms an open set, there exists<br />

δ > 0 such that if S ∈ B(X) and ||T − λI − S|| < δ then S is Weyl. So there exists<br />

an integer N such that ||T − λI − (T n − λI)|| < δ 2<br />

for n ≥ N. Let V be an open<br />

(δ/2)-neighborhhod of λ. We have, for µ ∈ V and n ≥ N,<br />

||T − λI − (T n − µI)|| < δ,<br />

so that T n − µI is Weyl. This shows that λ /∈ lim sup ω(T n ). Thus lim sup ω(T n ) ⊂<br />

ω(T ).<br />

Lemma 2.3.2. [Ne, Theorem 4] If {T n } n is a sequence of operators c<strong>on</strong>verging to an<br />

operator T and such that [T n , T ] is compact for each n, then lim σ e (T n ) = σ e (T ).<br />

53


CHAPTER 2.<br />

WEYL THEORY<br />

Proof. Newburgh’s theorem is stated as follows: if in a Banach algebra A, {a i } i is a<br />

sequence of elements commuting with a ∈ A and such that a i → a, then lim σ(a i ) =<br />

σ(a). If π denotes the can<strong>on</strong>ical homomorphism of B(X) <strong>on</strong>to the Calkin algebra<br />

B(X)/K(X), then the assumpti<strong>on</strong>s give that π(T n ) → π(T ) and [π(T n ), π(T )] = 0<br />

for each n. Hence, lim σ(π(T n )) = σ(π(T )); that is, lim σ e (T n ) = σ e (T ).<br />

Theorem 2.3.3. Suppose that T, T n ∈ B(X), for n ∈ Z + , are such that T n c<strong>on</strong>verges<br />

to T . If [T n , T ] ∈ K(X) for each n, then<br />

lim ω(f(T n )) = ω(f(T )) for every f ∈ H(σ(T )). (2.24)<br />

Remark. Because T n → T , by the upper-semic<strong>on</strong>tinuity of the spectrum, there is<br />

a positive integer N such that σ(T n ) ⊆ V whenever n > N. Thus, in the left-hand<br />

side of (2.24) it is to be understood that n > N.<br />

Proof. If T n and T commute modulo the compact operators then, whenever Tn<br />

−1 and<br />

T −1 exist, T n , T, Tn<br />

−1 and T −1 all commute modulo the compact operators. Therefore<br />

r(T n ) and r(T ) also commute modulo K(X) whenever r is a rati<strong>on</strong>al functi<strong>on</strong> with no<br />

poles in σ(T ) and n is sufficiently large. Using Runge’s theorem we can approximate<br />

f <strong>on</strong> compact subsets of V by rati<strong>on</strong>al functi<strong>on</strong>s r who poles lie off of V . So there<br />

exists a sequence of rati<strong>on</strong>al functi<strong>on</strong>s r i whose poles lie outside of V and r i → f<br />

uniformly <strong>on</strong> compact subsets of V . If n > N, then by the functi<strong>on</strong>al calculus,<br />

f(T n )f(T ) − f(T )f(T n ) = lim<br />

i<br />

(<br />

ri (T n )r i (T ) − r i (T )r i (T n ) ) ,<br />

which is compact for each n. Furthermore,<br />

∫<br />

1<br />

||f(T n ) − f(T )|| = || f(λ) ( (λ − T n ) −1 − (λ − T ) −1) dλ||<br />

2πi Γ<br />

≤ 1 l(Γ) max |f(λ)| · max<br />

2πi ||(λ − T n) −1 − (λ − T ) −1 || ,<br />

λ∈Γ λ∈Γ<br />

where Γ is the boundary of V and l(Γ) denotes the arc length of Γ. The righthand<br />

side of the above inequality c<strong>on</strong>verges to 0, and so f(T n ) → f(T ). By Lemma<br />

2.25, lim σ e (f(T n )) = σ e (f(T )). The arguments used by J.B. C<strong>on</strong>way and B.B.<br />

Morrel in Propositi<strong>on</strong> 3.11 of [CoM] can now be used here to obtain the c<strong>on</strong>clusi<strong>on</strong><br />

lim ω(f(T n )) = ω(f(T )).<br />

In general there is <strong>on</strong>ly inclusi<strong>on</strong> for the Weyl spectrum:<br />

Theorem 2.3.4. If T ∈ B(X) then<br />

ω(p(T )) ⊆ p(ω(T )) for every polynomial p.<br />

Proof. We can suppose p is n<strong>on</strong>c<strong>on</strong>stant. Suppose λ /∈ pω(T ). Writing p(µ) − λ =<br />

a(µ − µ 1 )(µ − µ 2 ) · · · (µ − µ n ), we have<br />

p(T ) − λI = a(T − µ 1 I) · · · (T − µ n I). (2.25)<br />

For each i, p(µ i ) = λ /∈ pω(T ), so that µ i /∈ ω(T ), i.e., T − µ i I is weyl. It thus follows<br />

from (2.25) that p(T ) − λI is Weyl since the product of Weyl operators is Weyl.<br />

54


CHAPTER 2.<br />

WEYL THEORY<br />

In general the spectral mapping theorem is liable to fail for the Weyl spectrum:<br />

Example 2.3.5. Let T = U ⊕ (U ∗ + 2I), where U is the unilateral shift <strong>on</strong> l 2 , and<br />

let p(λ) := λ(λ − 2). Then 0 ∈ p(ω(T )) but 0 /∈ ω(p(T )).<br />

Proof. Observe p(T ) = T (T − 2I) = [U ⊕ (U ∗ + 2I)][(U − 2I) ⊕ U ∗ ]. Since U is<br />

Fredholm of index −1, and since U ∗ + 2I and U − 2I are invertible it follows that T<br />

and T − 2I are Fredholm of indices −1 and +1, respectively. Therefore p(T ) is Weyl,<br />

so that 0 /∈ ω(p(T )), while 0 = p(0) ∈ p(ω(T )).<br />

Lemma 2.3.6. If T ∈ B(X) is isoloid then for every polynomial p,<br />

p(σ(T ) \ π 00 (T )) = σ(p(T )) \ π 00 (p(T )).<br />

Proof. We first claim that with no restricti<strong>on</strong> <strong>on</strong> T ,<br />

σ(p(T )) \ π 00 (p(T )) ⊂ p(σ(T ) \ π 00 (T )). (2.26)<br />

Let λ ∈ σ(p(T )) \ π 00 (p(T )) = p(σ(T )) \ π 00 (p(T )). There are two cases to c<strong>on</strong>sider.<br />

Case 1. λ /∈ iso p(σ(T )). In this case, there exists a sequence (λ n ) in p(σ(T )) such<br />

that λ n → λ. So there exists a sequence (µ n ) in σ(T ) such that p(µ n ) = λ n → λ.<br />

This implies that (µ n ) c<strong>on</strong>tains a c<strong>on</strong>vergent subsequence and we may assume that<br />

lim µ n = µ 0 . Thus λ = lim p(µ n ) = p(µ 0 ). Since µ 0 ∈ σ(T ) \ π 00 (T ), it follows that<br />

λ ∈ p(σ(T ) \ π 00 (T )).<br />

Case 2. λ ∈ iso p(σ(T )). In this case either λ is not an eigenvalue of p(T ) or it<br />

is an eigenvalue of infinite multiplicity. Let p(T ) − λI = a 0 (T − µ 1 I) · · · (T − µ n I).<br />

If λ is not an eigenvalue of p(T ) then n<strong>on</strong>e of µ 1 , · · · , µ n can be an eigenvalue of T<br />

and at least <strong>on</strong>e of µ 1 , · · · , µ n is in σ(T ). Therefore λ ∈ p(σ(T ) \ π 00 (T )). If λ is an<br />

eigenvalue of p(T ) of infinite multiplicity then at least <strong>on</strong>e of µ 1 , · · · , µ n , say µ 1 , is<br />

an eigenvalue of T of infinite multiplicity. Then µ 1 ∈ σ(T ) \ π 00 (T ) and p(µ 1 ) = λ,<br />

so that λ ∈ p(σ(T ) \ π 00 (T )). This proves (2.26). For the reverse inclusi<strong>on</strong> of (2.26),<br />

we assume λ ∈ p(σ(T ) \ π 00 (T )). Since p(σ(T )) = σ(p(T )), we have λ ∈ σ(p(T )). If<br />

possible let λ ∈ π 00 (p(T )). So λ ∈ iso σ(p(T )). Let<br />

p(T ) − λI = a 0 (T − µ 1 I) · · · (T − µ n I). (2.27)<br />

The equality (2.27) shows that if any of µ 1 , · · · , µ n is in σ(T ) then it must be an<br />

isolated point of σ(T ) and hence an eigenvalue since T is isoloid. Since λ is an<br />

eigenvalue of finite multiplicity, any such µ must be an eigenvalue of finite multiplicity<br />

and hence bel<strong>on</strong>gs to π 00 (T ). This c<strong>on</strong>tradicts the fact that λ ∈ p(σ(T ) \ π 00 (T )).<br />

Therefore λ /∈ π 00 (T ) and<br />

p(σ(T ) \ π 00 (T )) ⊂ σ(p(T )) \ π 00 (p(T )).<br />

Theorem 2.3.7. If T ∈ B(X) is isoloid and Weyl’s theorem holds for T then for<br />

every polynomial p, Weyl’s theorem holds for p(T ) if and <strong>on</strong>ly if p(ω(T )) = ω(p(T )).<br />

55


CHAPTER 2.<br />

WEYL THEORY<br />

Proof. By Lemma 2.3.6, p(σ(T ) \ π 00 (T )) = σ(p(T )) \ π 00 (p(T )). If Weyl’s theorem<br />

holds for T then ω(T ) = σ(T ) \ π 00 (T ), so that<br />

p(ω(T )) = p(σ(T ) \ π 00 (T )) = σ(p(T )) \ π 00 (p(T )).<br />

The result follows at <strong>on</strong>ce from this relati<strong>on</strong>ship.<br />

Example 2.3.8. Theorem 2.3.7 may fail if T is not isoloid. To see this define T 1 and<br />

T 2 <strong>on</strong> l 2 by<br />

T 1 (x 1 , x 2 , · · · ) = (x 1 , 0, x 2 /2, x 3 /2, · · · )<br />

and<br />

T 2 (x 1 , x 2 , · · · ) = (0, x 1 /2, x 2 /3, x 3 /4, · · · ).<br />

Let T := T 1 ⊕ (T 2 − I) <strong>on</strong> X = l 2 ⊕ l 2 . Then<br />

and<br />

σ(T ) = {1} ∪ {z : |z| ≤ 1/2} ∪ {−1}, π 00 (T ) = {1},<br />

ω(T ) = {z : |z| ≤ 1/2} ∪ {−1},<br />

which shows that Weyl’s theorem holds for T . Let p(t) = t 2 . Then<br />

and<br />

σ(p(T )) = {z : |z| ≤ 1/4} ∪ {1}, π 00 (p(T )) = {1}<br />

ω(p(T )) = {z : |z| ≤ 1/4} ∪ {1}.<br />

Thus 1 ∈ p(σ(T ) \ π 00 (T )), but 1 /∈ σ(p(T )) \ π 00 (p(T )). Also ω(p(T )) = p(ω(T )) but<br />

Weyl’s theorem does not hold for p(T ).<br />

Theorem 2.3.9. If p(ω(T )) = ω(p(T )) for every polynomial p, then f(ω(T )) =<br />

ω(f(T )) for every f ∈ H(σ(T )).<br />

Proof. Let (p n (T )) be a sequence of polynomials c<strong>on</strong>verging uniformly in a neighborhood<br />

of σ(T ) to f(t) so that p n (T ) → f(T ). Since f(T ) commutes with each p n (T ),<br />

it follows from Theorem 2.3.3 that<br />

ω(f(T )) = lim ω(p n (T )) = lim p n (ω(T )) = f(ω(T )).<br />

Theorem 2.3.10. If T ∈ B(X) then the following are equivalent:<br />

index(T − λI) index(T − µI) ≥ 0 for each pair λ, µ ∈ C \ σ e (T ); (2.28)<br />

f(ω(T )) = ω(f(T )) for every f ∈ H(σ(T )). (2.29)<br />

56


CHAPTER 2.<br />

WEYL THEORY<br />

Proof. The spectral mapping theorem for the Weyl spectrum may be rewritten as<br />

implicati<strong>on</strong>, for arbitrary n ∈ N and λ ∈ C n ,<br />

(T − λ 1 I)(T − λ 2 I) · · · (T − λ n I) Weyl =⇒ T − λ j I Weyl for each j = 1, 2, · · · , n.<br />

(2.30)<br />

Now if index(T − zI) ≥ 0 <strong>on</strong> C \ σ e (T ) then we have<br />

n∑<br />

n∏<br />

index(T −λ j I) = index (T −λ j I) = 0 =⇒ index(T −λ j I) = 0 (j = 1, 2, · · · , n),<br />

j=1<br />

j=1<br />

and similarly if index (T − zI) ≤ 0 off σ e (T ). If c<strong>on</strong>versely there exist λ, µ for which<br />

then<br />

index(T − λI) = −m < 0 < k = index(T − µI) (2.31)<br />

(T − λI) k (T − µI) m (2.32)<br />

is a Weyl operator whose factors are not Weyl. This together with Theorem 2.3.9<br />

proves the equivalence of the c<strong>on</strong>diti<strong>on</strong>s (2.28) and (2.29).<br />

Corollary 2.3.11. If X is a Hilbert space and T ∈ B(X) is hyp<strong>on</strong>ormal then<br />

f(ω(T )) = ω(f(T )) for every f ∈ H(σ(T )). (2.33)<br />

Proof. Immediate from Theorem 2.3.10 together with the fact that if T is hyp<strong>on</strong>ormal<br />

then index (T − λI) ≤ 0 for every λ ∈ C \ σ e (T ).<br />

Corollary 2.3.12. Let T ∈ B(X). If<br />

(i) Weyl’s theorem holds for T ;<br />

(ii) T is isoloid;<br />

(iii) T satisfies the spectral mapping theorem for the Weyl spectrum,<br />

then Weyl’s theorem holds for f(T ) for every f ∈ H(σ(T )).<br />

Proof. A slight modificati<strong>on</strong> of the proof of Lemma 2.3.6 shows that if T is isoloid<br />

then<br />

f ( σ(T ) \ π 00 (T ) ) = σ(f(T )) \ π 00 (f(T )) for every f ∈ H(σ(T )).<br />

It thus follows from Theorem 1.7.8 and Corollary 2.3.11 that<br />

σ(f(T )) \ π 00 (f(T )) = f ( σ(T ) \ π 00 (T ) ) = f(ω(T )) = ω(f(T )),<br />

which implies that Weyl’s theorem holds for f(T ).<br />

Corollary 2.3.13. If T ∈ B(X) has the SVEP then<br />

ω(f(T )) = f(ω(T )) for every f ∈ H(σ(T )).<br />

57


CHAPTER 2.<br />

WEYL THEORY<br />

Proof. If λ /∈ σ e (T ) then by Lemma 2.2.12, T − λI has a finite ascent. Since if<br />

S ∈ B(X) is Fredholm of finite ascent then index (S) ≤ 0: indeed, either if S has<br />

finite descent then S is Browder and hence index (S) = 0, or if S does not have finite<br />

descent then<br />

n index (S) = dim N(S n ) − dim R(S n ) ⊥ → −∞ as n → ∞,<br />

which implies that index (S) < 0. Thus we have that index (T − λI) ≤ 0. Thus T<br />

satisfies the c<strong>on</strong>diti<strong>on</strong> (2.28), which gives the result.<br />

Theorem 2.3.14. If T ∈ B(X) satisfies<br />

X T ({λ}) = N(T − λI) for every λ ∈ C,<br />

then Weyl’s theorem holds for f(T ) for every f ∈ H(σ(T )).<br />

Proof. By Corollary 2.2.18, Weyl’s theorem holds for T , T is isoloid, and T has the<br />

SVEP. In particular by Corollary 2.3.13, T satisfies the spectral mapping theorem for<br />

the Weyl spectrum. Thus the result follows from Corollary 2.3.12.<br />

58


CHAPTER 2.<br />

WEYL THEORY<br />

2.4 Perturbati<strong>on</strong> Theorems<br />

In this secti<strong>on</strong> we c<strong>on</strong>sider how Weyl’s theorem survives under “small” perturbati<strong>on</strong>s.<br />

Weyl’s theorem is transmitted from T ∈ B(X) to T − K for commuting nilpotents<br />

K ∈ B(X) To see this we need:<br />

Lemma 2.4.1. If T ∈ B(X) and if N is a quasinilpotent operator commuting with<br />

T then ω(T + N) = ω(T ).<br />

Proof. It suffices to show that if 0 /∈ ω(T ) then 0 /∈ ω(T + N). Let 0 /∈ ω(T ) so that<br />

0 /∈ σ(π(T )). For all λ ∈ C we have σ(π(T +λN)) = σ(π(T )). Thus 0 /∈ σ(π(T +λN))<br />

for all λ ∈ C, which implies T + λN is a Fredholm operator forall λ ∈ C. But since<br />

T is Weyl, it follows that T + N is also Weyl, that is, 0 /∈ ω(T + N).<br />

Theorem 2.4.2. Let T ∈ B(X) and let N be a nilpotent operator commuting with<br />

T . If Weyl’s theorem holds for T then it holds for T + N.<br />

Proof. We first claim that<br />

π 00 (T + N) = π 00 (T ). (2.34)<br />

Let 0 ∈ π 00 (T ) so that ker (T ) is finite dimensi<strong>on</strong>al. Let (T +N)x = 0 for some x ≠ 0.<br />

Then T x = −Nx. Since T commutes with N it follows that<br />

T m x = (−1) m N m x for every m ∈ N. (2.35)<br />

Let n be the nilpotency of N, i.e., n be the smallest positive integer such that N n = 0.<br />

Then by (2.35) we have that for some r with 1 ≤ r ≤ n, T r x = 0 and then T r−1 x ∈<br />

N(T ). Thus N(T + N) ⊂ N(T n−1 ). Therefore N(T + N) is finite dimensi<strong>on</strong>al. Also<br />

if for some x (≠ 0) T x = 0 then (T + N) n x = 0, and hence 0 is an eigenvalue of<br />

T + N. Again since σ(T + N) = σ(T ) it follows that 0 ∈ π 00 (T + N). By symmetry<br />

0 ∈ π 00 (T + N) implies 0 ∈ π 00 (T ), which proves (2.34). Thus we have<br />

ω(T + N) = ω(T ) (by Lemma 2.4.1)<br />

= σ(T ) \ π 00 (T ) (since Weyl’s theorem holds for T )<br />

= σ(T + N) \ π 00 (T + N),<br />

which shows that Weyl’s theorem holds for T + N.<br />

Theorem 2.4.2 however does not extend to quasinilpotents: let<br />

Q : (x 1 , x 2 , x 3 , · · · ) ↦→ ( 1 2 x 2, 1 3 x 3, 1 4 x 4, · · · ) <strong>on</strong> l 2<br />

and set <strong>on</strong> l 2 ⊕ l 2 ,<br />

T =<br />

[ ] 1 0<br />

0 0<br />

and K =<br />

[ ] 0 0<br />

. (2.36)<br />

0 Q<br />

59


CHAPTER 2.<br />

WEYL THEORY<br />

Evidently K is quasinilpotent commutes with T : but Weyl’s Theorem holds for T<br />

because<br />

σ(T ) = ω(T ) = {0, 1} and π 00 (T ) = ∅, (2.37)<br />

while Weyl’s Theorem does not hold for T + K because<br />

σ(T + K) = ω(T + K) = {0, 1} and π 00 (T + K) = {0}. (2.38)<br />

But if K is an injective quasinilpotent operator commuting with T then Weyl’s<br />

theorem is transmitted from T to T + K.<br />

Theorem 2.4.3. If Weyl’s theorem holds for T ∈ B(X) then Weyl’s theorem holds<br />

for T + K if K ∈ B(X) is an injective quasinilpotent operator commuting with T .<br />

Proof. First of all we prove that if there exists an injective quasinilpotent operator<br />

commuting with T , then<br />

T is Weyl =⇒ T is injective. (2.39)<br />

To show this suppose K is an injective quasinilpotent operator commuting with T .<br />

Assume to the c<strong>on</strong>trary that T is Weyl but not injective. Then there exists a n<strong>on</strong>zero<br />

vector x ∈ X such that T x = 0. Then by the commutativity assumpti<strong>on</strong>, T K n x =<br />

K n T x = 0 for every n = 0, 1, 2, · · · , so that K n x ∈ N(T ) for every n = 0, 1, 2, · · · .<br />

We now claim that {K n x} ∞ n=0 is a sequence of linearly independent vectors in X.<br />

To see this suppose c 0 x + c 1 Kx + · · · + c n K n x = 0. We may then write c n (K −<br />

λ 1 I) · · · (K − λ n I)x = 0. Since K is an injective quasinilpotent operator it follows<br />

that (K − λ 1 I) · · · (K − λ n I) is injective. But since x ≠ 0 we have that c n = 0. By an<br />

inducti<strong>on</strong> we also have that c n−1 = · · · = c 1 = c 0 = 0. This shows that {K n x} ∞ n=0 is a<br />

sequence of linearly independent vectors in X. From this we can see N(T ) is infinitedimensi<strong>on</strong>al,<br />

which c<strong>on</strong>tradicts to the fact that T is Weyl. This proves (2.39). From<br />

(2.39) we can see that if Weyl’s theorem holds for T then π 00 (T ) = ∅. We now claim<br />

that π 00 (T + K) = ∅. Indeed if λ ∈ π 00 (T + K), then 0 < dim N(T + K − λI) < ∞,<br />

so that there exists a n<strong>on</strong>zero vector x ∈ X such that (T + K − λI)x = 0. But<br />

since K commutes with T + K − λI, the same argument as in the proof of (2.39)<br />

with T + K − λI in place of T shows that N(T + K − λI) is infinite-dimensi<strong>on</strong>al, a<br />

c<strong>on</strong>tradicti<strong>on</strong>. Therefore π 00 (T + K) = ∅ and hence Weyl’s theorem holds for T + K<br />

because ϖ(T ) = ϖ(T + K) with ϖ = σ, ω.<br />

In Theorem 2.4.3, “quasinilpotent” cannot be replaced by “compact”. For example<br />

c<strong>on</strong>sider the following operators <strong>on</strong> l 2 ⊕ l 2 :<br />

T =<br />

⎛<br />

⎜<br />

⎝<br />

0 1<br />

2 0<br />

1<br />

3<br />

1 0<br />

⎞<br />

⎛<br />

⎟<br />

4<br />

⎠ ⊕ I and K = ⎜<br />

⎝<br />

...<br />

1<br />

− 1 2 0<br />

− 1 3<br />

⎞<br />

⎟<br />

0 − 1 4<br />

⎠ ⊕ Q,<br />

...<br />

60


CHAPTER 2.<br />

WEYL THEORY<br />

where Q is an injective compact quasinilpotent operator <strong>on</strong> l 2 . Observe that Weyl’s<br />

theorem holds for T , K is an injective compact operator, and T K = KT . But<br />

σ(T + K) = {0, 1} = ω(T + K) and π 00 (T + K) = {1},<br />

which says that Weyl’s theorem does not hold for T + K.<br />

On the other hand, Weyl’s theorem for T is not sufficient for Weyl’s theorem for<br />

T + F with finite rank F . To see this, let X = l 2 and let T, F ∈ B(X) be defined by<br />

and<br />

T (x 1 , x 2 , x 3 , · · · ) = (0, x 1 /2, x 2 /3, · · · )<br />

F (x 1 , x 2 , x 3 , · · · ) = (0, −x 1 /2, 0, 0, · · · ).<br />

since the point spectrum of T is empty it follows Weyl’s theorem holds for T . Also<br />

F is a nilpotent operator. Since 0 ∈ π 00 (T + F ) ∩ ω(T + F ), it follows that Weyl’s<br />

theorem fails for T + F .<br />

Lemma 2.4.4. Let T ∈ B(X). If F ∈ B(X) is a finite rank operator then<br />

Further if T F = F T then<br />

dim N(T ) < ∞ ⇐⇒ dim N(T + F ) < ∞.<br />

acc σ(T ) = acc σ(T + F ).<br />

Proof. This follows from a straightforward calculati<strong>on</strong>.<br />

Theorem 2.4.5. Let T ∈ B(X) be an isoloid operator and let F ∈ B(X) be a finite<br />

rank operator commuting with T . If Weyl’s theorem holds for T then it holds for<br />

T + F .<br />

Proof. We have to show that λ ∈ σ(T + F ) \ ω(T + F ) if and <strong>on</strong>ly if λ ∈ π 00 (T + F ).<br />

Without loss of generality we may assume that λ = 0. We first suppose that 0 ∈<br />

σ(T + F ) \ ω(T + F ) and thus T + F is Weyl but not invertible. It suffice to show<br />

that 0 ∈ iso σ(T + F ). Since T is Weyl and Weyl’s theorem holds for T , it follows<br />

that 0 ∈ ρ(T ) or 0 ∈ iso σ(T ). Thus by Lemma 2.4.4, 0 /∈ acc σ(T + F ). But since<br />

T + F is not invertible we have that 0 ∈ iso σ(T + F ).<br />

C<strong>on</strong>versely, suppose that 0 ∈ π 00 (T + F ). We want to show that T + F is Weyl.<br />

By our assumpti<strong>on</strong>, 0 ∈ iso σ(T + F ) and 0 < dim N(T + F ) < ∞. By Lemma 2.4.4,<br />

we have<br />

0 /∈ acc σ(T ) and dim N(T ) < ∞. (2.40)<br />

If T is invertible then it is evident that T + F is Weyl. If T is not invertible then by<br />

the first part of (2.40) we have 0 ∈ iso σ(T ). But since T is isoloid it follows that T is<br />

not <strong>on</strong>e-<strong>on</strong>e, which together with the sec<strong>on</strong>d part of (2.40) gives 0 < dim N(T ) < ∞.<br />

Since Weyl’s theorem holds for T it follows that T is Weyl and so is T + F .<br />

61


CHAPTER 2.<br />

WEYL THEORY<br />

Example 2.4.6. There exists an operator T ∈ B(X) and a finite rank operator<br />

F ∈ B(X) commuting with T such that Weyl’s theorem holds for T but it does not<br />

hold for T + F .<br />

Proof. Define <strong>on</strong> l 2 ⊕ l 2 , T := I ⊕ S and F = K ⊕ 0, where S : l 2 → l 2 is an injective<br />

quasinilpotent operator and F : l 2 → l 2 is defined by<br />

F (x 1 , x 2 , x 3 , · · · ) = (−x 1 , 0, 0, · · · ).<br />

Then F is of finite rank and commutes with T . It is easy to see that<br />

σ(T ) = ω(T ) = {0, 1} and π 00 (T ) = ∅,<br />

which implies that Weyl’s theorem holds for T . We however have<br />

σ(T + F ) = ω(T + F ) = {0, 1} and π 00 (T + F ) = {0},<br />

which implies that Weyl’s theorem fails for T + F .<br />

Theorem 2.4.5 may fail if “finite rank” is replaced by “compact”. In fact Weyl’s<br />

theorem may fail even if K is both compact and quasinilpotent: for example, take<br />

T = 0 and K the operator <strong>on</strong> l 2 defined by K(x 1 , x 2 , · · · ) = ( x 2<br />

2<br />

, x 3<br />

3<br />

, x 4<br />

4<br />

, · · · ). We will<br />

however show that if “isoloid” c<strong>on</strong>diti<strong>on</strong> is strengthened slightly then Weyl’s theorem<br />

is transmitted from T to T + K if K is either a compact or a quasinilpotent operator<br />

commuting with T . To see this we observe:<br />

Lemma 2.4.7. If K ∈ B(X) is a compact operator commuting with T ∈ B(X) then<br />

Proof. See [HanL2].<br />

π 00 (T + K) ⊆ iso σ(T ) ∪ ρ(T ).<br />

An operator T ∈ B(X) will be said to be finite-isoloid if iso σ(T ) ⊆ π 0f (T ).<br />

Evidently finite-isoloid ⇒ isoloid. The c<strong>on</strong>verse is not true in general: for example,<br />

take T = 0. In particular if σ(T ) has no isolated points then T is finite-isoloid. We<br />

now have:<br />

Theorem 2.4.8. Suppose T ∈ B(X) is finite-isoloid. If Weyl’s theorem holds for T<br />

then Weyl’s theorem holds for T + K if K ∈ B(X) commutes with T and is either<br />

compact or quasinilpotent.<br />

Proof. First we assume that K is a compact operator commuting with T . Suppose<br />

Weyl’s theorem holds for T . We first claim that with no restricti<strong>on</strong> <strong>on</strong> T ,<br />

σ(T + K) \ ω(T + K) ⊆ π 00 (T + K). (2.41)<br />

For (2.41), it suffices to show that if λ ∈ σ(T + K) \ ω(T + K) then λ ∈ iso σ(T + K).<br />

Assume to the c<strong>on</strong>trary that λ ∈ acc σ(T + K). Then we have that λ ∈ σ b (T + K) =<br />

σ b (T ), so that λ ∈ σ e (T ) or λ ∈ acc σ(T ). Remember that the essential spectrum and<br />

62


CHAPTER 2.<br />

WEYL THEORY<br />

the Weyl spectrum are invariant under compact perturbati<strong>on</strong>s. Thus if λ ∈ σ e (T )<br />

then λ ∈ σ e (T + K) ⊆ ω(T + K), a c<strong>on</strong>tradicti<strong>on</strong>. Therefore we should have that<br />

λ ∈ acc σ(T ). But since Weyl’s theorem holds for T and λ /∈ ω(T + K) = ω(T ), it<br />

follows that λ ∈ π 00 (T ), a c<strong>on</strong>tradicti<strong>on</strong>. This proves (2.41). For the reverse inclusi<strong>on</strong><br />

suppose λ ∈ π 00 (T + K). Then by Lemma 2.4.7, either λ ∈ iso σ(T ) or λ ∈ ρ(T ). If<br />

λ ∈ ρ(T ) then evidently T +K −λI is Weyl, i.e., λ /∈ ω(T +K). If instead λ ∈ iso σ(T )<br />

then λ ∈ π 00 (T ) whenever T is finite-isoloid. Since Weyl’s theorem holds for T , it<br />

follows that λ /∈ ω(T ) and hence λ /∈ ω(T + K). Therefore Weyl’s theorem holds for<br />

T + K.<br />

Next we assume that K is a quasinilpotent operator commuting with T . Then by<br />

Lemma 2.4.1, ϖ(T ) = ϖ(T + Q) with ϖ = σ, ω. Suppose Weyl’s theorem holds for<br />

T . Then<br />

σ(T + K) \ ω(T + K) = σ(T ) \ ω(T ) = π 00 (T ) ⊆ iso σ(T ) = iso σ(T + K),<br />

which implies that σ(T + K) \ ω(T + K) ⊆ π 00 (T + K). C<strong>on</strong>versely, suppose λ ∈<br />

π 00 (T + K). If T is finite-isoloid then λ ∈ iso σ(T + K) = iso σ(T ) ⊆ π 0f (T ). Thus<br />

λ ∈ π 00 (T ) = σ(T ) \ ω(T ) = σ(T + K) \ ω(T + K). This completes the proof.<br />

Corollary 2.4.9. Suppose X is a Hilbert space and T ∈ B(X) is p-hyp<strong>on</strong>ormal. If<br />

T satisfies <strong>on</strong>e of the following:<br />

(i) iso σ(T ) = ∅;<br />

(ii) T has finite-dimensi<strong>on</strong>al eigenspaces,<br />

then Weyl’s theorem holds for T + K if K ∈ B(X) is either compact or quasinilpotent<br />

and commutes with T .<br />

Proof. Observe that each of the c<strong>on</strong>diti<strong>on</strong>s (i) and (ii) forces p-hyp<strong>on</strong>ormal operators<br />

to be finite-isoloid. Since by Corollary 2.2.5 Weyl’s theorem holds for p-hyp<strong>on</strong>ormal<br />

operators, the result follows at <strong>on</strong>ce from Theorem 2.4.8.<br />

In the perturbati<strong>on</strong> theory the “commutative” c<strong>on</strong>diti<strong>on</strong> looks so rigid. Without<br />

the commutativity, the spectrum can however undergo a large change under even rank<br />

<strong>on</strong>e perturbati<strong>on</strong>s. In spite of it, Weyl’s theorem may hold for (n<strong>on</strong>-commutative)<br />

compact perturbati<strong>on</strong>s of “good” operators. We now give such a perturbati<strong>on</strong> theorem.<br />

To do this we need:<br />

Lemma 2.4.10. If N ∈ B(X) is a quasinilpotent operator commuting with T ∈ B(X)<br />

modulo compact operators (i.e., T N − NT ∈ K(X)) then σ e (T + N) = σ e (T ) and<br />

ω(T + N) = ω(T ).<br />

Proof. Immediate from Lemma 2.4.1.<br />

Theorem 2.4.11. Suppose T ∈ B(X) satisfies the following:<br />

(i) T is finite-isoloid;<br />

63


CHAPTER 2.<br />

WEYL THEORY<br />

(ii) σ(T ) has no “holes” (bounded comp<strong>on</strong>ents of the complement), i.e., σ(T ) =<br />

η σ(T );<br />

(iii) σ(T ) has at most finitely many isolated points;<br />

(iv) Weyl’s theorem holds for T .<br />

If K ∈ B(X) is either compact or quasinilpotent and commutes with T modulo compact<br />

operators then Weyl’s theorem holds for T + K.<br />

Proof. By Lemma 2.4.10, we have that σ e (T + K) = σ e (T ) and ω(T + K) = ω(T ).<br />

Suppose Weyl’s theorem holds for T and λ ∈ σ(T + K) \ ω(T + K). We now claim<br />

that λ ∈ iso σ(T + K). Assume to the c<strong>on</strong>trary that λ ∈ acc σ(T + K). Since<br />

λ /∈ ω(T + K) = ω(T ), it follows from the punctured neighborhood theorem that<br />

λ /∈ ∂ σ(T + K). Also since the set of all Weyl operators forms an open subset of<br />

B(X), we have that λ ∈ int ( σ(T + K) \ ω(T + K) ) . Then there exists ϵ > 0 such<br />

that {µ ∈ C : |µ − λ| < ϵ} ⊆ int ( σ(T + K) \ ω(T + K) ) , and hence {µ ∈ C : |µ − λ| <<br />

ϵ} ∩ ω(T ) = ∅. But since<br />

∂ σ(T + K) \ iso σ(T + K) ⊆ σ e (T + K) = σ e (T ),<br />

it follows from our assumpti<strong>on</strong> that<br />

{µ ∈ C : |µ − λ| < ϵ} ⊆ int ( σ(T + K) \ ω(T + K) )<br />

⊆ η ( ∂ σ(T + K) \ iso σ(T + K) )<br />

⊆ η σ e (T ) ⊆ η σ(T ) = σ(T ),<br />

which implies that {µ ∈ C : |µ − λ| < ϵ} ⊆ σ(T ) \ ω(T ). This c<strong>on</strong>tradicts to Weyl’s<br />

theorem for T . Therefore λ ∈ iso σ(T + K) and hence σ(T + K) \ ω(T + K) ⊆<br />

π 00 (T + K). For the reverse inclusi<strong>on</strong> suppose λ ∈ π 00 (T + K). Assume to the<br />

c<strong>on</strong>trary that λ ∈ ω(T + K) and hence λ ∈ ω(T ). Then we claim λ /∈ ∂ σ(T ). Indeed<br />

if λ ∈ iso σ(T ) then by assumpti<strong>on</strong> λ ∈ π 00 (T ), which c<strong>on</strong>tradicts to Weyl’s theorem<br />

for T . If instead λ ∈ acc σ(T ) ∩ ∂ σ(T ) then since iso σ(T ) is finite it follows that<br />

λ ∈ acc ( ∂ σ(T ) ) ⊆ acc σ e (T ) = acc σ e (T + K),<br />

which c<strong>on</strong>tradicts to the fact that λ ∈ iso σ(T + K). Therefore λ /∈ ∂ σ(T ). Also since<br />

λ ∈ iso σ(T + K), there exists ϵ > 0 such that<br />

{µ ∈ C : 0 < |µ − λ| < ϵ} ⊆ σ(T ) ∩ ρ(T + K),<br />

so that {µ ∈ C : 0 < |µ − λ| < ϵ} ∩ ω(T ) = ∅, which c<strong>on</strong>tradicts to Weyl’s theorem for<br />

T . Thus λ ∈ σ(T + K) \ ω(T + K) and therefore Weyl’s theorem holds for T + K.<br />

If, in Theorem 2.4.11, the c<strong>on</strong>diti<strong>on</strong> “σ(T ) has no holes” is dropped then Theorem<br />

2.4.11 may fail even though T is normal. For example, if <strong>on</strong> l 2 ⊕ l 2<br />

T = ( )<br />

U I−UU ∗<br />

and K = ( )<br />

0 I−UU ∗<br />

0 U ∗ 0 0 ,<br />

where U is the unilateral shift <strong>on</strong> l 2 , then T is unitary (essentially the bilateral shift)<br />

with σ(T ) = T, K is a rank <strong>on</strong>e nilpotent, and Weyl’s theorem does not hold for<br />

T − K.<br />

64


CHAPTER 2.<br />

WEYL THEORY<br />

Also in Theorem 2.4.11, the c<strong>on</strong>diti<strong>on</strong> “iso σ(T ) is finite” is essential in the cases<br />

where K is compact. For example, if <strong>on</strong> l 2<br />

T (x 1 , x 2 , · · · ) = (x 1 , x 2<br />

2 , x 3<br />

3 , · · · ) and Q(x 1, x 2 , · · · ) = ( x 2<br />

2 , x 3<br />

3 , x 4<br />

4 , · · · ),<br />

we define K := −(T + Q). Then we have that (i) T is finite-isoloid; (ii) σ(T ) has<br />

no holes; (iii) Weyl’s theorem holds for T ; (iv) iso σ(T ) is infinite; (v) K is compact<br />

because T and Q are both compact; (vi) Weyl’s theorem does not hold for T + K<br />

(= −Q).<br />

Corollary 2.4.12. If σ(T ) has no holes and at most finitely many isolated points<br />

and if K is a compact operator then Weyl’s theorem is transmitted from T to T + K.<br />

Proof. Straightforward from Theorem 2.4.11.<br />

Corollary 2.4.12 shows that if Weyl’s theorem holds for T whose spectrum has no<br />

holes and at most finitely many isolated points then for every compact operator K,<br />

the passage from σ(T ) to σ(T +K) is putting at most countably many isolated points<br />

outside σ(T ) which are Riesz points of σ(T + K). Here we should note that this<br />

holds even if T is quasinilpotent because for every quasinilpotent operator T (more<br />

generally, “Riesz operators”), we have<br />

σ(T + K) ⊆ η σ e (T + K) ∪ p 00 (T + K) = η σ e (T ) ∪ p 00 (T + K) = {0} ∪ p 00 (T + K).<br />

65


CHAPTER 2.<br />

WEYL THEORY<br />

2.5 Weyl’s theorem in several variables<br />

In this secti<strong>on</strong> we c<strong>on</strong>sider Weyl’s theorem from multivariable operator theory. Let<br />

H be a complex Hilbert space and write B(H) for the set of bounded linear operators<br />

acting <strong>on</strong> H. Let T = (T 1 , · · · , T n ) be a commuting n-tuple of operators in B(H), let<br />

Λ[e] ≡ {Λ k [e 1 , · · · , e n ]} n k=0 be the exterior algebra <strong>on</strong> n generators (e i ∧ e j = −e j ∧ e i<br />

for all i, j = 1, · · · , n) and write Λ(H) := Λ[e] ⊗ H. Let Λ(T ) : Λ(H) → Λ(H) be<br />

defined by (cf. [Cu1], [Har1], [Har4], [Ta1])<br />

Λ(T )(ω ⊗ x) =<br />

n∑<br />

(e i ∧ ω) ⊗ T i x. (2.42)<br />

The operator Λ(T ) in (2.42) can be represented by the Koszul complex for T :<br />

i=1<br />

0 Λ 0 (H) Λ0 (T ) Λ 1 (H) Λ1 (T ) · · · Λn−1 (T ) Λ(H) 0 , (2.43)<br />

where Λ k (H) is the collecti<strong>on</strong> of k-forms and Λ k (T ) = Λ(T )| Λk (H). For n = 2, the<br />

Koszul complex for T = (T 1 , T 2 ) is given by<br />

0 H<br />

⎡<br />

⎣ T ⎤<br />

1⎦<br />

T 2<br />

[ H<br />

H] [ −T 2 T 1<br />

]<br />

H 0<br />

Evidently, Λ(T ) 2 = 0, so that ran Λ(T ) ⊆ ker Λ(T ), or equivalently, ran Λ k−1 (T ) ⊆<br />

ker Λ k (T ) for every k = 0, · · · , n, where, for notati<strong>on</strong>al c<strong>on</strong>venience, Λ −1 (T ) := 0 and<br />

Λ n (T ) = 0. For the representati<strong>on</strong> of Λ(T ), we may put together its odd and even<br />

parts, writing<br />

where<br />

Write<br />

[<br />

Λ(T ) =<br />

Λ ∗ (H) = ⊕<br />

p is ∗<br />

0 Λ odd (T )<br />

Λ even (T ) 0<br />

Λ p (H),<br />

]<br />

:<br />

Λ ∗ (T ) = ⊕<br />

p is ∗<br />

[ ]<br />

Λ odd (H)<br />

Λ even (H)<br />

Λ p (T )<br />

H k (T ) := ker Λ k (T )/ran Λ k−1 (T )<br />

→<br />

[ ]<br />

Λ odd (H)<br />

Λ even ,<br />

(H)<br />

with ∗ = even, odd.<br />

(k = 0, · · · , n),<br />

which is called the k-th cohomology for the Koszul complex Λ(T ). We recall ([Cu1],<br />

[Har4], [Ta1]) that T is said to be Taylor invertible if ker Λ(T ) = ran Λ(T ) (in other<br />

words, the Koszul complex (2.43) is exact at every stage, i.e., H k (T ) = {0} for<br />

every k = 0, · · · , n) and is said to be Taylor Fredholm if ker Λ(T )/ran Λ(T ) is finite<br />

dimensi<strong>on</strong>al (in other words, all cohomologies of (2.43) are finite dimensi<strong>on</strong>al). If<br />

T = (T 1 , · · · , T n ) is Taylor Fredholm, define the index of T by<br />

index (T ) ≡ Euler(0, Λ n−1 (T ), · · · , Λ 0 (T ), 0) :=<br />

66<br />

n∑<br />

(−1) k dim H k (T ),<br />

k=0


CHAPTER 2.<br />

WEYL THEORY<br />

where Euler(·) is the Euler characteristic of the Koszul complex for T . We shall<br />

write σ T (T ) and σ Te (T ) for the Taylor spectrum and Taylor essential spectrum of T ,<br />

respectively: namely,<br />

σ T (T ) = {λ ∈ C n : T − λ is not Taylor invertible};<br />

σ Te (T ) = {λ ∈ C n : T − λ is not Taylor Fredholm}.<br />

Following to R. Harte [Har4, Definiti<strong>on</strong> 11.10.5], we shall say that T = (T 1 , · · · , T n )<br />

is Taylor Weyl if T is Taylor Fredholm and index(T ) = 0. The Taylor Weyl spectrum,<br />

σ Tw (T ), of T is defined by<br />

σ Tw (T ) = {λ ∈ C n : T − λ is not Taylor Weyl}.<br />

It is known ([Har4, Theorem 10.6.4]) that σ Tw (T ) is compact and evidently,<br />

σ Te (T ) ⊂ σ Tw (T ) ⊂ σ T (T ).<br />

On the other hand, “Weyl’s theorem” for an operator <strong>on</strong> a Hilbert space is the statement<br />

that the complement in the spectrum of the Weyl spectrum coincides with the<br />

isolated eigenvalues of finite multiplicity. In this note we introduce the joint versi<strong>on</strong><br />

of Weyl’s theorem and then examine the classes of n-tuples of operators satisfying<br />

Weyl’s theorem.<br />

The spectral mapping theorem is liable to fail for σ Tw (T ) even though T =<br />

(T 1 , · · · , T n ) is a commuting n-tuple of hyp<strong>on</strong>ormal operators (remember [LeL] that<br />

if n = 1 then every hyp<strong>on</strong>ormal operator enjoys the spectral mapping theorem for the<br />

Weyl spectrum). For example, let U be the unilateral shift <strong>on</strong> l 2 and T := (U, U).<br />

Then a straightforward calculati<strong>on</strong> shows that σ Tw (T ) = {(λ, λ) : |λ| = 1}. If<br />

f : C 2 → C 1 is the map f(z 1 , z 2 ) = z 1 + z 2 then σ Tw f(T ) = σ Tw (2U) = {2λ :<br />

|λ| ≤ 1} fσ Tw (T ) = {2λ : |λ| = 1}. If instead f : C 1 → C 2 is the map f(z) = (z, z)<br />

then σ Tw f(U) = {(λ, λ) : |λ| = 1} fσ Tw (U) = {(λ, λ) : |λ| ≤ 1}. Therefore σ Tw (T )<br />

satisfies no way spectral mapping theorem in general.<br />

The Taylor Weyl spectrum however satisfies a “subprojective” property.:<br />

Lemma 2.5.1. If T = (T 1 , · · · , T n ) is a commuting n-tuple then σ Tw (T ) ⊂ ∏ n<br />

j=1 σ T e<br />

(T j ).<br />

Proof. This follows at <strong>on</strong>ce from the fact (cf. [Cu1, p.144]) that every commuting<br />

n-tuple having a Fredholm coordinate has index zero.<br />

□<br />

On the other hand, M. Cho and M. Takaguchi [ChT] have defined the joint Weyl<br />

spectrum, ω(T ), of a commuting n-tuple T = (T 1 , · · · , T n ) by<br />

ω(T ) = ∩ {<br />

σT (T + K) : K = (K 1 , · · · , K n ) is an n-tuple of compact operators<br />

and T + K = (T 1 + K 1 , · · · , T n + K n ) is commutative. }<br />

67


CHAPTER 2.<br />

WEYL THEORY<br />

A questi<strong>on</strong> arises naturally: For a commuting n-tuple T , does it follow that σ Tw (T ) =<br />

ω(T ) If n = 1 then σ Tw (T ) and ω(T ) coalesce: indeed, T is Weyl if and <strong>on</strong>ly if T is<br />

a sum of an invertible operator and a compact operator.<br />

We first observe:<br />

Lemma 2.5.2. If T = (T 1 , · · · , T n ) is a commuting n-tuple then<br />

σ Tw (T ) ⊂ ω(T ). (2.44)<br />

Proof. Write K 0 (T ) := Λ odd (T ) + Λ even (T ) ∗ . Then it was known that (cf. [Cu1],<br />

[Har4], [Va])<br />

T is Taylor invertible [Taylor Fredholm] ⇐⇒ K 0 (T ) is invertible [Fredholm] (2.45)<br />

and moreover index(T ) = index(K 0 (T )). If λ = (λ 1 , · · · , λ n ) /∈ ω(T ) then there<br />

exists an n-tuple of compact operators K = (K 1 , · · · , K n ) such that T + K − λ is<br />

commutative and Taylor invertible. By (2.45), K 0 (T + K − λ) is invertible. But since<br />

K 0 (T + K − λ) − K 0 (T − λ) is a compact operator it follows that K 0 (T − λ) is Weyl,<br />

and hence, by (2.45), T − λ is Taylor Weyl, i.e., λ /∈ σ Tw (T ).<br />

□<br />

The inclusi<strong>on</strong> (2.44) cannot be strengthened by the equality. R. Gelca [Ge] showed<br />

that if S is a Fredholm operator with index(S) ≠ 0 then there do not exist compact<br />

operators K 1 and K 2 such that (T + K 1 , K 2 ) is commutative and Taylor invertible.<br />

Thus for instance, if U is the unilateral shift then ω(U, 0) σ Tw (U, 0).<br />

We introduce an interesting noti<strong>on</strong> which commuting n-tuples may enjoy.<br />

A commuting n-tuple T = (T 1 , · · · , T n ) is said to have the quasitriangular property<br />

if the dimensi<strong>on</strong> of the left cohomology for the Koszul complex Λ(T − λ) is greater<br />

than or equal to the dimensi<strong>on</strong> of the right cohomology for Λ(T − λ) for all λ =<br />

(λ 1 , · · · , λ n ) ∈ C n , i.e.,<br />

dim H n (T − λ) ≤ dim H 0 (T − λ) for all λ = (λ 1 , · · · , λ n ) ∈ C n . (2.46)<br />

Since H 0 (T − λ) = kerΛ 0 (T − λ) = ∩ n<br />

i=1 ker (T i − λ i ) and H n (T − λ) = kerΛ n (T −<br />

λ)/ranΛ n−1 (T − λ) ∼ = ( ranΛ n−1 (T − λ) )⊥ ∼ =<br />

∩ n<br />

i=1 ker (T i − λ i ) ∗ , the c<strong>on</strong>diti<strong>on</strong> (2.46)<br />

is equivalent to the c<strong>on</strong>diti<strong>on</strong><br />

n∩<br />

n∩<br />

dim ker (T i − λ i ) ∗ ≤ dim ker (T i − λ i ).<br />

i=1<br />

If n = 1, the c<strong>on</strong>diti<strong>on</strong> (2.46) is equivalent to the c<strong>on</strong>diti<strong>on</strong> dim (T − λ) ∗−1 (0) ≤<br />

dim (T − λ) −1 (0) for all λ ∈ C, or equivalently, the spectral picture of T c<strong>on</strong>tains<br />

no holes or pseudoholes associated with a negative index, which, by the celebrated<br />

i=1<br />

68


CHAPTER 2.<br />

WEYL THEORY<br />

theorem due to Apostol, Foias and Voiculescu, is equivalent to the fact that T is<br />

quasitriangular (cf. [Pe, Theorem 1.31]). Evidently, every commuting n-tuple of<br />

quasitriangular operators has the quasitriangular property. Also if a commuting n-<br />

tuple T = (T 1 , · · · , T n ) has a coordinate whose adjoint has no eigenvalues then T has<br />

the quasitriangular property.<br />

As we have seen in the above, the inclusi<strong>on</strong> (2.44) cannot be reversible even though<br />

T = (T 1 , · · · , T n ) is a doubly commuting n-tuple (i.e., [T i , T ∗ j ] ≡ T iT ∗ j − T ∗ j T i = 0<br />

for all i ≠ j) of hyp<strong>on</strong>ormal operators. On the other hand, R. Curto [Cu1, Corollary<br />

3.8] showed that if T = (T 1 , · · · , T n ) is a doubly commuting n-tuple of hyp<strong>on</strong>ormal<br />

operators then<br />

T is Taylor invertible [Taylor Fredholm] ⇐⇒<br />

n∑<br />

T i Ti<br />

∗<br />

i=1<br />

is invertible [Fredholm].<br />

(2.47)<br />

On the other hand, many authors have c<strong>on</strong>sidered the joint versi<strong>on</strong> of the Browder<br />

spectrum. We recall ([BDW], [CuD], [Da1], [Da2], [Har4], [JeL], [Sn]) that a commuting<br />

n-tuple T = (T 1 , · · · , T n ) is called Taylor Browder if T is Taylor Fredholm<br />

and there exists a deleted open neighborhood N 0 of 0 ∈ C n such that T − λ is Taylor<br />

invertible for all λ ∈ N 0 . The Taylor Browder spectrum, σ Tb (T ), is defined by<br />

σ Tb (T ) = {λ ∈ C n : T − λ is not Taylor Browder}.<br />

Note that σ Tb (T ) = σ Te (T ) ∪ acc σ T (T ), where acc(·) denotes the set of accumulati<strong>on</strong><br />

points. We can easily show that<br />

σ Tw (T ) ⊂ σ Tb (T ). (2.48)<br />

Indeed, if λ /∈ σ Tb (T ) then T − λ is Taylor Fredholm and there there exists δ > 0 such<br />

that T − λ − µ is Taylor invertible for 0 < |µ| < δ. Since the index is c<strong>on</strong>tinuous it<br />

follows that index(T − λ) = 0, which says that λ /∈ σ Tw (T ), giving (2.48).<br />

If T = (T 1 , · · · , T n ) is a commuting n-tuple, we write π 00 (T ) for the set of all<br />

isolated points of σ T (T ) which are joint eigenvalues of finite multiplicity and write<br />

R(T ) ≡ iso σ T (T ) \ σ Te (T ) for the Riesz points of σ T (T ). By the c<strong>on</strong>tinuity of the<br />

index, we can see that R(T ) = iso σ T (T ) \ σ Tw (T ).<br />

Lemma 2.5.3. If T = (T 1 , · · · , T n ) is a commuting n-tuple then ω(T ) ⊂ σ Tb (T ).<br />

Proof. Suppose without loss of generality that 0 /∈ σ Tb (T ). Then T is Taylor invertible<br />

and 0 ∈ isoσ T (T ). So there exists a projecti<strong>on</strong> P ∈ B(H) satisfying that<br />

(i) P commutes with T i (i = 1, · · · , n);<br />

(ii) σ T (T | P (H) ) = {0} and σ T (T | (I−P )(H) ) = σ T (T ) \ {0};<br />

(iii) P is of finite rank<br />

69


CHAPTER 2.<br />

WEYL THEORY<br />

(see [Ta2, Theorem 4.9]). Put Q = (P, · · · , P ). Evidently, 0 /∈ σ T ((T + Q)| (I−P )(H) ).<br />

Since a commuting quasinilpotent perturbati<strong>on</strong> of an invertible operator is also invertible,<br />

it follows that 0 /∈ σ T ((T +Q)| P (H) ). But since σ T (T ) = σ T ((T +Q)| (I−P )(H) ) ∪ σ T ((T +<br />

Q)| P (H) ), we can c<strong>on</strong>clude that T + Q is Taylor invertible. So 0 /∈ ω(T ). □<br />

“Weyl’s theorem” for an operator <strong>on</strong> a Hilbert space is the statement that the complement<br />

in the spectrum of the Weyl spectrum coincides with the isolated eigenvalues<br />

of finite multiplicity. There are two versi<strong>on</strong>s of Weyl’s theorem in several variables.<br />

If T = (T 1 , · · · , T n ) is a commuting n-tuple then we say that Weyl’s theorem (I)<br />

holds for T if<br />

σ T (T ) \ π 00 (T ) = σ Tw (T ) (2.49)<br />

and that Weyl’s theorem (II) holds for T if<br />

σ T (T ) \ π 00 (T ) = ω(T ). (2.50)<br />

The noti<strong>on</strong> of Weyl’s theorem (II) was first introduced by M. Cho and M. Takaguchi<br />

[ChT]. We note that<br />

Weyl’s theorem (I) =⇒ Weyl’s theorem (II). (2.51)<br />

Indeed, since σ Tw (T ) ⊂ ω(T ), it follows that if σ T (T ) \ π 00 (T ) ⊂ σ Tw (T ), then<br />

σ T (T ) \ π 00 (T ) ⊂ ω(T ). Now suppose σ Tw (T ) ⊂ σ T (T ) \ π 00 (T ). So if λ ∈ π 00 (T )<br />

then T − λ is Taylor Weyl, and hence Taylor Browder. By Lemma 4, λ /∈ ω(T ).<br />

Therefore ω(T ) ⊂ σ T (T )\π 00 (T ), and so Weyl’s theorem (II) holds for T , which gives<br />

(2.51).<br />

But the c<strong>on</strong>verse of (2.51) is not true in general. To see this, let T := (U, 0), where<br />

U is the unilateral shift <strong>on</strong> l 2 . Then<br />

(a) σ T (T ) = cl D × {0};<br />

(b) σ T w (T ) = ∂ D × {0};<br />

(c) ω(T ) = cl D × {0};<br />

(d) π 00 (T ) = ∅,<br />

where D is the open unit disk. So Weyl’s theorem (II) holds for T while Weyl’s theorem<br />

(I) fails even though T is a doubly commuting n-tuple of hyp<strong>on</strong>ormal operators.<br />

M. Cho [Ch2] showed that Weyl’s theorem (II) holds for a commuting n-tuple of<br />

normal operators. The following theorem is an extensi<strong>on</strong> of this result.<br />

Theorem 2.5.4. Let T = (T 1 , · · · , T n ) be a doubly commuting n-tuple of hyp<strong>on</strong>ormal<br />

operators. If T has the quasitriangular property then Weyl’s theorem (I) holds for T .<br />

70


CHAPTER 2.<br />

WEYL THEORY<br />

Proof. In [Ch2] it was shown that if T is a doubly commuting n-tuple of hyp<strong>on</strong>ormal<br />

operators then ω(T ) ⊂ σ T (T ) \ π 00 (T ). Then by Lemma 2, σ T w (T ) ⊂ σ T (T ) \ π 00 (T ).<br />

For the reverse inclusi<strong>on</strong>, we first claim that<br />

σ T e (T ) = σ T w (T ) = ω(T ). (2.52)<br />

In view of 2.5.2, we need to show that ω(T ) ⊂ σ T e (T ). Suppose without loss of<br />

generality that 0 /∈ σ T e (T ). Thus by (2.47) we have that ∑ n<br />

i=1 T iTi ∗ is Fredholm<br />

(and hence Weyl since it is self-adjoint). Let P denote the orthog<strong>on</strong>al projecti<strong>on</strong><br />

<strong>on</strong>to ker ∑ n<br />

i=1 T iTi ∗ . Since P is of finite rank and Weyl-ness is stable under compact<br />

perturbati<strong>on</strong>s, we have that ∑ n<br />

i=1 T iTi ∗ +nP is Weyl. In particular, a straightforward<br />

calculati<strong>on</strong> shows that ∑ n<br />

i=1 T iTi ∗ + nP is <strong>on</strong>e-<strong>on</strong>e and therefore ∑ n<br />

i=1 T iTi ∗ + nP is<br />

invertible. Since each T i is a hyp<strong>on</strong>ormal operator, we have that<br />

⎡ ⎤<br />

ran P = ker [ ] ⎢<br />

T 1 , · · · , T n ⎣T1. ∗ ⎥<br />

n∩<br />

n∩<br />

. ⎦ = ker Ti<br />

∗ ⊃ ker T i .<br />

Tn<br />

∗ i=1<br />

i=1<br />

So if T has the quasitriangular property then since ran P is finite dimensi<strong>on</strong>al, it<br />

follows that<br />

n∩<br />

n∩<br />

ran P = ker T i = ker Ti ∗ .<br />

i=1<br />

So T i P = P T i = 0 for all i = 1, · · · , n. Hence we can see that (T 1 + P, · · · , T n + P ) is<br />

a doubly commuting n-tuple of hyp<strong>on</strong>ormal operators. Thus (T 1 + P, · · · , T n + P ) is<br />

Taylor invertible if and <strong>on</strong>ly if ∑ T i T ∗ i + nP is invertible. Therefore (T 1, · · · , T n ) +<br />

(P, · · · , P ) is Taylor invertible, and hence 0 /∈ ω(T ), which proves (2.52). So in view<br />

of (2.52), it now suffices to show that σ T (T ) \ π 00 (T ) ⊂ σ T e (T ). To see this we need<br />

to prove that<br />

acc σ T (T ) ⊂ σ T e (T ). (2.53)<br />

Suppose λ = lim λ k with distinct λ k ∈ σ T (T ). Write λ := (λ 1 , · · · , λ n ) and λ k :=<br />

(λ k1 , · · · , λ kn ). If λ k ∈ σ T e (T ) then clearly, λ ∈ σ T e (T ) since σ T e (T ) is a closed set. So<br />

we assume λ k ∈ σ T (T )\σ T e (T ). Then by (2.47), ∑ n<br />

i=1 (T i−λ ki )(T i −λ ki ) ∗ is Fredholm<br />

but not invertible. So there exists a unit vector x k such that (T i − λ ki ) ∗ x k = 0 for all<br />

i = 1, · · · , n. If T has the quasitriangular property, it follows that (T i − λ ki )x k = 0.<br />

In particular, since the T i are hyp<strong>on</strong>ormal, {x k } forms an orth<strong>on</strong>ormal sequence.<br />

Further, we have<br />

n∑<br />

||(T i − λ i )x k || ≤<br />

i=1<br />

=<br />

i=1<br />

n∑<br />

(||(T i − λ ki )x k || + ||(λ ki − λ k )x k ||)<br />

i=1<br />

n∑<br />

|λ ki − λ i | −→ 0 as k → ∞.<br />

i=1<br />

Therefore λ ∈ σ T e (T ) (see [Da1, Theorem 2.6] or [Ch2, Theorem 1]), which proves<br />

(2.53) and completes the proof. □<br />

71


CHAPTER 2.<br />

WEYL THEORY<br />

Corollary 2.5.5. A commuting n-tuple of normal operators satisfies Weyl’s theorem<br />

(I) and hence Weyl’s theorem (II).<br />

Proof. Immediate from (2.51) and 2.5.4.<br />

□<br />

Corollary 2.5.6. (Riesz-Schauder theorem in several variables) Let T = (T 1 , · · · , T n )<br />

be a doubly commuting n-tuple of hyp<strong>on</strong>ormal operators. If T has the quasitriangular<br />

property then<br />

ω(T ) = σ Tb (T ).<br />

Proof. In view of refthm5.63, we need to show that σ Tb (T ) ⊂ ω(T ). Indeed if<br />

λ ∈ σ T (T ) \ ω(T ) then by (2.53), λ ∈ iso σ T (T ), and hence T − λ is Taylor-Browder.<br />

□<br />

72


CHAPTER 2.<br />

WEYL THEORY<br />

2.6 Comments and Problems<br />

(a) Transaloid and SVEP. For an operator T ∈ B(X) for a Hilbert space X,<br />

denote W (T ) = {(T x, x) : ||x|| = 1} for the numerical range of T and w(T ) =<br />

sup {|λ| : λ ∈ W (T )} for the numerical radius of T . An operator T is called c<strong>on</strong>vexoid<br />

if c<strong>on</strong>v σ(T ) = cl W (T ) and is called spectraloid if w(T ) = r(T ) = the spectral radius.<br />

We call an operator T ∈ B(X) transaloid if T − λI is normaloid for all λ ∈ C. It was<br />

well known that<br />

transaloid =⇒ c<strong>on</strong>vexoid =⇒ spectraloid,<br />

(G 1 ) =⇒ c<strong>on</strong>vexoid and (G 1 ) =⇒ reguloid.<br />

We would like to expect that Corollary 2.2.16 remains still true if “reguloid” is<br />

replaced by “transaloid”<br />

Problem 2.1. If T ∈ B(X) is transaloid and has the SVEP, does Weyl’s theorem<br />

hold for T <br />

The following questi<strong>on</strong> is a strategy to answer Problem 2.1.<br />

Problem 2.2. Does it follow that<br />

transaloid =⇒ reguloid <br />

If the answer to Problem 2.2 is affirmative then the answer to Problem 2.1 is affirmative<br />

by Corollary 2.2.16.<br />

(b) ∗-paranormal operators.<br />

said to be ∗-paranormal if<br />

An operator T ∈ B(X) for a Hilbert space X is<br />

||T ∗ x|| 2 ≤ ||T 2 x|| ||x|| for every x ∈ X.<br />

It was [AT] known that if T ∈ B(X) is ∗-paranormal then the following hold:<br />

T is normaloid; (2.54)<br />

N(T − λI) ⊂ N((T − λI) ∗ ). (2.55)<br />

So if T ∈ B(X) is ∗-paranormal then by (2.55), T − λI has finite ascent for every<br />

λ ∈ C. Thus ∗-paranormal operators have the SVEP ([La]). On the other hand,<br />

by the same argument as the proof of Corollary we can see that if T ∈ B(X) is<br />

∗-paranormal then<br />

σ(T ) \ ω(T ) ⊂ π 00 (T ). (2.56)<br />

However we were unable to decide:<br />

Problem 2.3. Does Weyl’s theorem hold for ∗-paranormal operators <br />

The following questi<strong>on</strong> is a strategy to answer Problem 2.3.<br />

73


CHAPTER 2.<br />

WEYL THEORY<br />

Problem 2.4. Is every ∗-paranormal operator isoloid <br />

If the answer to Problem 2.4 is affirmative then the answer to Problem 2.3 is<br />

affirmative. To see this suppose T ∈ B(X) is ∗-paranormal. In view of (2.56), it<br />

suffices to show that π 00 (T ) ⊆ σ(T ) \ ω(T ). Assume λ ∈ π 00 (T ). By (2.43), T − λI<br />

is reduced by its eigenspaces. Thus we can write<br />

T − λI =<br />

[ ] 0 0<br />

0 S<br />

:<br />

[ ]<br />

N(T − λI)<br />

N(T − λI) ⊥ −→<br />

[ ]<br />

N(T − λI)<br />

N(T − λI) ⊥ .<br />

Thus T = ( λI 0<br />

0 S+λI<br />

)<br />

. We now claim that S is invertible. Assume to the c<strong>on</strong>trary<br />

that S is not invertible. Then 0 ∈ iso σ(S) since λ ∈ iso σ(T ). Thus λ ∈ iso σ(S + λI).<br />

But since S + λI is also ∗-paranormal, it follows from our assumpti<strong>on</strong> that λ is an<br />

eigenvalue of S + λI. Thus 0 ∈ π 0 (S), which c<strong>on</strong>tradicts to the fact that S is <strong>on</strong>e-<strong>on</strong>e.<br />

Therefore S should be invertible. Note that N(T − λI) is finite-dimensi<strong>on</strong>al. Thus<br />

evidently T − λI is Weyl, so that λ ∈ σ(T ) \ ω(T ). This gives a proof.<br />

(c) Subclasses of paranormal operators. An operator T ∈ B(X) for a Hilbert<br />

space X is said to be quasihyp<strong>on</strong>ormal if T ∗ (T ∗ T − T T ∗ )T ≥ 0 and is said to be class<br />

A-operator if |T 2 | ≥ |T | 2 (cf. [FIY]). Let T = U |T | be the polar decompositi<strong>on</strong> of<br />

T and ˜T := |T | 1 2 U|T | 1 2 be the Aluthge transformati<strong>on</strong> of T (cf. [Al]). An operator<br />

T ∈ B(X) for a Hilbert space X is called w-hyp<strong>on</strong>ormal if | ˜T | ≥ |T | ≥ | ˜T ∗ |. It was<br />

well known that<br />

hyp<strong>on</strong>ormal =⇒ quasihyp<strong>on</strong>ormal =⇒ class A =⇒ paranormal (2.57)<br />

hyp<strong>on</strong>ormal =⇒ p-hyp<strong>on</strong>ormal =⇒ w-hyp<strong>on</strong>ormal =⇒ paranormal. (2.58)<br />

Since by Theorem 2.2.20, Weyl’s theorem holds for paranormal operators <strong>on</strong> an arbitrary<br />

Banach space, all classes of operators in (2.57) and (2.58) enjoy Weyl’s theorem.<br />

(d) Open problems in multivariable operator theory.<br />

It was known that<br />

(i) If (A 1 , · · · , A n ) is invertible and (( A 1 B 1<br />

) (<br />

0 C 1<br />

, · · · ,<br />

An B n<br />

))<br />

0 C n<br />

is invertible then (C1 , · · · , C n )<br />

is invertible.<br />

(ii) If (A 1 , · · · , A n ) is invertible and (C 1 , · · · , C n ) is invertible then (( A 1 B 1<br />

) (<br />

0 C 1<br />

, · · · ,<br />

An B n<br />

))<br />

0 C n<br />

is invertible.<br />

Problem 2.5. If ( A 0 B<br />

C ) ≡ (( A 1 B 1<br />

) (<br />

0 C 1<br />

, · · · ,<br />

An B n<br />

))<br />

0 C n<br />

, find a necessary and sufficient<br />

c<strong>on</strong>diti<strong>on</strong> for ( A 0 B<br />

C<br />

) to be invertible for some B.<br />

If n = 1 then it was known that ( A 0 B<br />

C<br />

) is invertible for some B if and <strong>on</strong>ly if<br />

(i) A is left invertible;<br />

(ii) C is right invertible;<br />

74


CHAPTER 2.<br />

WEYL THEORY<br />

(iii) ran(A) ⊥ ∼ = ker (C).<br />

Problem 2.6. What is a kind of several variable versi<strong>on</strong> of the Punctured Neighborhood<br />

Theorem <br />

The Punctured Neighborhood Theorem says that ∂σ(T ) \ σ e (T ) ⊂ iso σ(T ). Our<br />

questi<strong>on</strong> is that if T = (T 1 , · · · , T n ) then<br />

∂σ T (T ) \ σ T e (T ) ⊂ ( ) of σ T (T ).<br />

Problem 2.7. (Deformati<strong>on</strong> Problem) Given two Fredholm n-tuples A = (A 1 , · · · , A n )<br />

and B = (B 1 , · · · , B n ) ∈ F with the same index, is it always possible to find a c<strong>on</strong>tinuous<br />

path γ : [0, 1] → F such that γ(0) = A and γ(1) = B <br />

The answer for n = 1 is yes. Also if dimH < ∞ then the answer is yes.<br />

Problem 2.8. If (A 1 , · · · , A n ) and (A k 1<br />

1 , · · · , Ak n n ) are Fredholm, does it follow<br />

index (A k1<br />

1 , · · · , Ak n<br />

n ) = k 1 · · · k n · index(A 1 , · · · , A n ) <br />

Problem 2.9. If S = (S 1 , · · · , S n ) is subnormal (i.e., there exists a commuting n-<br />

tuple N = (N 1 , · · · , N n ) such that N j = mne(S j )) and N = (N 1 , · · · , N n ) = mne(S),<br />

how σ T (S) can be obtained from σ T (N) <br />

R. Curto and M. Putinar [CP2] showed that<br />

σ T (N) ⊂ σ T (S) ⊂ ησ T (N).<br />

If n = 1 then σ(S) is obtained from σ(N) y “filling in some holes”.<br />

Problem 2.10. If T = (T 1 , · · · , T n ) is commutative then<br />

(i) σ T (T ) ⊂ ∏ n<br />

j=1 σ(T j);<br />

(ii) If p ∈ poly m n then σ T (p(T )) = p(σ T (T )).<br />

Let T = (T 1 , · · · , T n ) be a hyp<strong>on</strong>ormal n-tuple of commuting operators and p ∈ poly m n .<br />

Does it follow<br />

σ T (p(T )) = 0 =⇒ p(T ) = 0 <br />

If n = 1 then the answer is yes: indeed, if σ(p(T )) = 0 and hence p(σ(T )) = 0<br />

then σ(T ) is finite, so that T should be normal, which implies that p(T ) is normal<br />

and quasinilpotent then p(T ) = 0.<br />

75


CHAPTER 2.<br />

WEYL THEORY<br />

76


Chapter 3<br />

Hyp<strong>on</strong>ormal and Subnormal<br />

<strong>Theory</strong><br />

3.1 Hyp<strong>on</strong>ormal <strong>Operator</strong>s<br />

An operator A ∈ B(H) is called hyp<strong>on</strong>ormal if<br />

[A ∗ , A] ≡ A ∗ A − AA ∗ ≥ 0.<br />

Thus if A ∈ B(H) then<br />

A is hyp<strong>on</strong>ormal ⇐⇒ ∥Ah∥ ≥ ∥A ∗ h∥ for all h ∈ H.<br />

If A ∗ A ≤ AA ∗ , or equivalently, ∥A ∗ h∥ ≥ ∥Ah∥ for all h, then A is called a cohyp<strong>on</strong>ormal<br />

operator. <strong>Operator</strong>s that are either hyp<strong>on</strong>ormal or cohyp<strong>on</strong>ormal are<br />

called seminormal.<br />

Propositi<strong>on</strong> 3.1.1. Let A ∈ B(H) be a hyp<strong>on</strong>ormal operator. Then we have:<br />

(a) If A is invertible then A −1 is hyp<strong>on</strong>ormal.<br />

(b) A − λ is hyp<strong>on</strong>ormal for every λ ∈ C.<br />

(c) If λ ∈ π 0 (A) and Af = λf then A ∗ f = λf, i.e., ker (A − λ) ⊆ ker (A − λ) ∗ .<br />

(d) If f and g are eigenvectors corresp<strong>on</strong>ding to distinct eigenvalues of A then<br />

f ⊥ g.<br />

(e) If M ∈ Lat A then A| M is hyp<strong>on</strong>ormal.<br />

Proof. (a) Recall that if T is positive and invertible then<br />

T ≥ 1 =⇒ T −1 ≤ 1 :<br />

77


CHAPTER 3.<br />

HYPONORMAL AND SUBNORMAL THEORY<br />

because if T ∈ C ∗ (T ) ≡ C(X) then T = f ≥ 1 ⇒ T −1 = 1 f ≤ 1. Since A∗ A ≥ AA ∗<br />

and A is invertible,<br />

A −1 (A ∗ A)(A ∗ ) −1 ≥ A −1 (AA ∗ )(A ∗ ) −1 = 1<br />

=⇒ A ∗ A −1 (A ∗ ) −1 A ≤ 1<br />

=⇒ A −1 (A ∗ ) −1 = (A ∗ ) −1 (A ∗ A −1 A ∗−1 A)A −1 ≤ (A ∗ ) −1 A −1<br />

=⇒ A −1 is hyp<strong>on</strong>ormal.<br />

(b) (A−λ)(A ∗ −λ) = AA ∗ −λA ∗ −λA+|λ| 2 ≤ A ∗ A−λA ∗ −λA+|λ| 2 = (A ∗ −λ)(A−λ).<br />

(c) Immediate from the fact that ∥(A ∗ − λ)f∥ ≤ ∥(A − λ)f∥.<br />

(d) Af = λf, Ag = µg ⇒ λ⟨f, g⟩ = ⟨Af, g⟩ = ⟨f, A ∗ g⟩ = ⟨f, µg⟩ = µ⟨f, g⟩.<br />

(e) If M ∈ Lat A then<br />

[ ] B C M<br />

A =<br />

0 D M ⊥ is hyp<strong>on</strong>ormal<br />

[ ]<br />

=⇒ 0 ≤ [A ∗ [B<br />

, A] =<br />

∗ , B] − CC ∗ ∗<br />

∗ ∗<br />

=⇒ [B ∗ , B] ≥ CC ∗ ≥ 0<br />

=⇒ B is hyp<strong>on</strong>ormal.<br />

Corollary 3.1.2. If A is hyp<strong>on</strong>ormal and λ ∈ π 0 (A) then ker (A − λ) reduces A.<br />

Hence if A is a pure hyp<strong>on</strong>ormal then π 0 (A) = ∅.<br />

Proof. From Propositi<strong>on</strong> 3.1.1(c), if f ∈ ker (A − λ) then Af = λf ∈ ker (A − λ) and<br />

A ∗ f = λf ∈ ker (A − λ).<br />

Propositi<strong>on</strong> 3.1.3. [Sta1] If A is hyp<strong>on</strong>ormal then ∥A n ∥ = ∥A∥ n , so<br />

in other words, A is normaloid.<br />

Proof. Observe<br />

∥A∥ = γ(A), where r(·) denoted the spectral radius,<br />

∥A n f∥ 2 =< A n f, A n f >=< A ∗ A n f, A n−1 f >≤ ∥A ∗ A n f∥·∥A n−1 f∥ ≤ ∥A n+1 f∥·∥A n−1 f∥.<br />

Hence ∥A n ∥ 2 ≤ ∥A n+1 ∥ · ∥A n−1 ∥. We use an inducti<strong>on</strong>. Clearly, it is true for n = 1.<br />

Suppose ∥A k ∥ = ∥A∥ k for 1 ≤ k ≤ n. Then ∥A∥ 2n = ∥A n ∥ 2 ≤ ∥A n+1 ∥ · ∥A n−1 ∥ =<br />

∥A n+1 ∥ · ∥A∥ n−1 , so ∥A∥ n+1 ≤ ∥A n+1 ∥. Also r(A) = lim ∥A n ∥ 1 n = ∥A∥.<br />

78


CHAPTER 3.<br />

HYPONORMAL AND SUBNORMAL THEORY<br />

Corollary 3.1.4. If A is hyp<strong>on</strong>ormal and λ /∈ σ(A) then<br />

Proof. Observe<br />

1<br />

∥(λ − A) −1 ∥ = dist ( λ, σ(A) ) .<br />

1<br />

||<br />

(λ − A) −1 || = 1<br />

max µ∈σ(λ−A) −1|µ| = min µ∈σ(λ−A)|µ| = dist ( λ, σ(A) ) .<br />

Propositi<strong>on</strong> 3.1.5. [Sta1] If A is hyp<strong>on</strong>ormal then A is isoloid, i.e., iso σ(A) ⊆<br />

π 0 (A). The pure hyp<strong>on</strong>ormal operators have no isolated points in their spectrum.<br />

Proof. Replacing A by A − λ we may assume that λ = 0. Observe that the <strong>on</strong>ly<br />

quasinilpotent hyp<strong>on</strong>ormal operator is zero. C<strong>on</strong>sider the spectral decompositi<strong>on</strong> of<br />

A:<br />

[ ]<br />

A1 0<br />

A = , where σ(A<br />

0 A 1 ) = {0}, σ(A 2 ) = σ(A)\{0}.<br />

2<br />

Then A 1 = 0, so 0 ∈ π 0 (A).<br />

The sec<strong>on</strong>d asserti<strong>on</strong> comes from the fact that ker (A − λ) is a reducing subspaces<br />

of a hyp<strong>on</strong>ormal operator A.<br />

Corollary 3.1.6. The <strong>on</strong>ly compact hyp<strong>on</strong>ormal operator is normal.<br />

Proof. Recall that if K is compact then every n<strong>on</strong>zero point of σ(K) is isolated. So<br />

if K is hyp<strong>on</strong>ormal then every eigenspaces reduces K and the restricti<strong>on</strong> of K to<br />

each eigenspace is normal. C<strong>on</strong>sider the restricti<strong>on</strong> of K to the orthog<strong>on</strong>al complement<br />

of the span of all the eigenvectors. The resulting operator is hyp<strong>on</strong>ormal and<br />

quasinilpotent, and hence 0. Therefore K is normal.<br />

Propositi<strong>on</strong> 3.1.7. Let A be a hyp<strong>on</strong>ormal operator. Then we have:<br />

(a) A is invertible ⇐⇒ A is right invertible.<br />

(b) A is Fredholm ⇐⇒ A is right Fredholm.<br />

(c) σ(A) = σ r (A) and σ e (A) = σ re (A).<br />

(d) A is pure, λ ∈ σ(A)\σ e (A) =⇒ index (A − λ) ≤ −1.<br />

Proof. (a) Observe that<br />

A is right invertible =⇒ ∃ B such that AB = 1<br />

=⇒ A is <strong>on</strong>to and hence ker A ∗ = (ranA) ⊥ = {0}<br />

=⇒ ker A = {0}<br />

=⇒ A is invertible.<br />

79


CHAPTER 3.<br />

HYPONORMAL AND SUBNORMAL THEORY<br />

(b) Similar to (a).<br />

(c) From (a) and (b).<br />

(d) Observe that<br />

A is pure hyp<strong>on</strong>ormal =⇒ A − λ is pure hyp<strong>on</strong>ormal<br />

=⇒ ker (A − λ) = {0} (by Propositi<strong>on</strong> 3.1.5)<br />

=⇒ A − λ is not <strong>on</strong>to since λ ∈ σ(A)<br />

=⇒ index (A − λ) = dim (ker (A − λ)) − dim ( ran(A − λ) ⊥)<br />

= −dim ( ran(A − λ) ⊥) ≤ −1.<br />

Write F denotes the set of Fredholm operators. We here give a direct proof<br />

showing that Weyl’s theorem holds for hyp<strong>on</strong>ormal operators.<br />

Propositi<strong>on</strong> 3.1.8. If A ∈ B(H) is hyp<strong>on</strong>ormal then<br />

σ(A)\ω(A) = π 00 (A),<br />

where π 00 (A) = the set of isolated eigenvalues of finite multiplicity.<br />

Proof. (⇐) If λ ∈ π 00 (A) then ker (A − λ) reduces A. So<br />

A = λI ⊕ B,<br />

where I is the identity <strong>on</strong> a finite dimensi<strong>on</strong>al space, B is hyp<strong>on</strong>ormal and λ /∈ σ(B).<br />

So λ /∈ ω(A).<br />

(⇒) Suppose λ ∈ σ(A)\ω(A), and so A−λ not invertible, Fredholm with index (A−<br />

λ) = 0. We may assume λ = 0. Since A ∈ F and index A = 0, it follows that 0 is an<br />

eigenvalue of finite multiplicity.<br />

It remains to show that 0 ∈ iso σ(A). Observe that<br />

ker (A) ⊆ ker (A ∗ ) = (ranA) ⊥ and 0 = index(A) = dim (ker (A)) − dim ( ranA) ⊥) ,<br />

so that ker(A) = (ranA) ⊥ . So<br />

where B is invertible.<br />

σ(A).<br />

A = 0 ⊕ B,<br />

Since σ(A) = {0} ∪ σ(B), 0 must be an isolated point of<br />

Corollary 3.1.9. If A ∈ B(H) is a pure hyp<strong>on</strong>ormal then<br />

∥A∥ ≤ ∥A + K∥ for every compact operator K.<br />

Proof. Since A is pure, π 0 (A) = ∅. So σ(A) = ω(A) = ∩ K∈K(H)<br />

σ(A + K). Thus for<br />

every compact operator K, σ(A) ⊆ σ(A + K). Therefore, ∥A∥ = r(A) ≤ r(A + K) ≤<br />

∥A + K∥.<br />

80


CHAPTER 3.<br />

HYPONORMAL AND SUBNORMAL THEORY<br />

3.2 The Berger-Shaw Theorem<br />

If A is a selfadjoint operator then A is said to be absolutely c<strong>on</strong>tinuous if its scalarvalued<br />

spectral measure is absolutely c<strong>on</strong>tinuous with respect to the Lebesgue measure<br />

<strong>on</strong> the line.<br />

Let N = ∫ zdE(z) be the spectral decompositi<strong>on</strong> of N. A scalar-valued spectral<br />

measure for N is a positive Borel measure µ <strong>on</strong> σ(N) such that<br />

µ(△) = 0 ⇐⇒ E(△) = 0.<br />

Since W ∗ (N) is an abelian v<strong>on</strong> Neumann algebra, W ∗ (N) has a separating vector e 0 ,<br />

i.e.,<br />

Ae 0 = 0 =⇒ A = 0 for A ∈ W ∗ (N).<br />

Define µ <strong>on</strong> σ(N) by<br />

µ(△) = ∥E(△)e 0 ∥ 2 .<br />

In fact, this µ is the unique scalar-valued spectral measure for N.<br />

Theorem 3.2.1. (Putnam, 1963) If S is a pure hyp<strong>on</strong>ormal operator and S = A+iB,<br />

where A and B are selfadjoint then A and B are absolutely c<strong>on</strong>tinuous.<br />

Proof. See [C<strong>on</strong>2, p.150].<br />

Definiti<strong>on</strong> 3.2.2. An operator T ∈ B(H) is said to be finitely multicyclic if there<br />

exist a finite number of vectors g 1 , · · · , g m ∈ H such that<br />

H = ∨ {<br />

f(T )gj : 1 ≤ j ≤ m and f ∈ Rat σ(T ) } .<br />

The vectors g 1 , · · · , g m are called generating vectors. If T is finitely multicyclic and<br />

m is the smallest number of generating vectors then T is said to be m-multicyclic.<br />

Theorem 3.2.3. (The Berger-Shaw Theorem) If T is an m-multicyclic hyp<strong>on</strong>ormal<br />

operator then [T ∗ , T ] is a trace class operator and<br />

tr [T ∗ , T ] ≤ m Area (σ(T )).<br />

π<br />

This inequality is sharp: indeed, c<strong>on</strong>sider the unilateral shift T :<br />

⎡ ⎤<br />

1<br />

[T ∗ ⎢<br />

, T ] = ⎣<br />

0 ⎥<br />

⎦ , σ(T ) = cl D, m = 1,<br />

. ..<br />

81


CHAPTER 3.<br />

HYPONORMAL AND SUBNORMAL THEORY<br />

so<br />

tr [T ∗ m<br />

, T ] = 1,<br />

π Area(σ(T )) = 1 π · π = 1.<br />

To prove Theorem 3.2.3 we need auxiliary lemmas. Recall the Hilbert-Schmidt norm<br />

of X:<br />

[∑ ⟨<br />

∥X∥ 2 ≡ |X| 2 ⟩ ] 1 2<br />

e n , e n<br />

[∑<br />

1<br />

= ⟨X ∗ 2<br />

Xe n , e n ⟩]<br />

= [tr (X ∗ X)] 1 2<br />

.<br />

Lemma 3.2.4. If T ∈ B(H) and P is a finite rank projecti<strong>on</strong> then<br />

Proof. Write<br />

Since P =<br />

[ ] 1 0<br />

,<br />

0 0<br />

tr ( P [T ∗ , T ]P ) ≤ ∥P ⊥ T P ∥ 2 2.<br />

[ ] A B<br />

T = :<br />

C P<br />

[ ] [ ]<br />

ranP ranP<br />

ranP ⊥ →<br />

ranP ⊥ .<br />

P [T ∗ , T ]P = [A ∗ , A] + C ∗ C − BB ∗ .<br />

So by the above remark, tr (P [T ∗ , T ]P ) = tr[A ∗ , A] + ∥C∥ 2 2 − ∥B∥ 2 2. But since A is a<br />

finite-dimensi<strong>on</strong>al operator,<br />

tr[A ∗ , A] = 0.<br />

Hence tr (P [T ∗ , T ]P ) ≤ ∥C∥ 2 2 = ||P ⊥ T P || 2 2.<br />

Lemma 3.2.5. If T ∈ B(H) is an m-multicyclic operator then there exists a sequence<br />

{P k } of finite rank projecti<strong>on</strong>s such that P k ↑ 1(SOT) and<br />

rank ( P ⊥ k T P k<br />

)<br />

≤ m for all k ≥ 1.<br />

Proof. Let g 1 , · · · , g m be the generating vectors for T and let {λ j } be a countable<br />

dense subset of C \ σ(T ); for c<strong>on</strong>venience, arrange {λ j } so that each point is repeated<br />

infinitely often. Let P k be the projecti<strong>on</strong> of H <strong>on</strong>to<br />

∨ {<br />

T j (T − λ 1 ) −1 · · · (T − λ k ) −1 g i : 0 ≤ j ≤ 2k, 1 ≤ i ≤ m } .<br />

Thus P k is finite rank, P k ≤ P k+1 , and<br />

rank [P ⊥ k T P k ] ≤ m for all k ≥ 1.<br />

82


CHAPTER 3.<br />

HYPONORMAL AND SUBNORMAL THEORY<br />

We should prove that P k → 1(SOT). Since {P k } is increasing, L = cl ∪ k ranP k is a<br />

closed linear space. To show that P k → 1(SOT) it suffices to show that L = H. To<br />

do this, it suffices to show that f(T )L ⊆ L for all f ∈ Rat(σ(T )). Since {λ j } is dense<br />

in σ(T ) c , it is <strong>on</strong>ly necessary to show that f(T )L ⊆ L when f is a rati<strong>on</strong>al functi<strong>on</strong><br />

with poles in {λ j }. Hence we must show that<br />

T L ⊆ L and (T − λ j ) −1 L ⊆ L.<br />

From the definiti<strong>on</strong> of L we see that these two c<strong>on</strong>diti<strong>on</strong>s are equivalent, respectively,<br />

to show that for all β ≥ 1:<br />

)<br />

T<br />

(T j (T − λ 1 ) −1 · · · (T − λ k ) −1 g i ∈ L for 0 ≤ j ≤ 2k ; (3.1)<br />

(T − λ m ) −1 (<br />

T j (T − λ 1 ) −1 · · · (T − λ k ) −1 g i<br />

)<br />

∈ L for 0 ≤ j ≤ 2k and all m. (3.2)<br />

To prove (3.1) we need <strong>on</strong>ly c<strong>on</strong>sider the case where j = 2k. Now<br />

T 2k+1 (T − λ 1 ) −1 · · · (T − λ 2k ) −1 g i ∈ ranP 2k<br />

and A = (T − λ k+1 ) · · · (T − λ 2k ) is a polynomial in T of degree 2k − k. Hence<br />

T 2k+1 (T −λ 1 ) −1 · · · (T −λ k ) −1 g i = AT 2k+1 (T −λ 1 ) −1 · · · (T −λ 2k ) −1 g i ∈ ranP 2k ⊆ L,<br />

which proves (3.1).<br />

Since (3.1) implies that L is an invariant subspace for T , to show (3.2) it suffices<br />

to show that<br />

(<br />

)<br />

(T − λ m ) −1 (T − λ 1 ) −1 · · · (T − λ k ) −1 g i ∈ L for all m.<br />

Since λ m is repeated infinitely often, we may assume m ≥ k + 2. If B = (T −<br />

λ k+1 ) · · · (T − λ m−1 ), then B is a polynomial in T of degree m + k − 1. Hence<br />

(<br />

)<br />

(T −λ m ) −1 (T −λ 1 ) −1 · · · (T −λ k ) −1 g i = B(T −λ 1 ) −1 · · · (T −λ m ) −1 g i ∈ ranP m ⊆ L,<br />

which proves (3.2).<br />

Lemma 3.2.6. If T ∈ B(H) is an m-multicyclic hyp<strong>on</strong>ormal operator then<br />

tr [T ∗ , T ] ≤ m∥T ∥ 2 .<br />

Proof. By Lemma 3.2.5, there exists an increasing sequence {P k } of finite rank projecti<strong>on</strong>s<br />

such that P k ↑ 1(SOT) and rank [P ⊥ k T P k] ≤ m for all k ≥ 1. Note that<br />

∥P ⊥ k T P k ∥ 2 2 ≤ m∥P ⊥ k T P k ∥ 2 .<br />

83


CHAPTER 3.<br />

HYPONORMAL AND SUBNORMAL THEORY<br />

Since {P k } is an increasing sequence,<br />

tr[T ∗ , T ] = lim k tr ( P k [T ∗ )<br />

, T ]P k .<br />

By Lemma 3.2.4 we get<br />

)<br />

tr [T ∗ , T ] ≤ limsup∥Pk ⊥ T P k ∥ 2 2 ≤ limsup<br />

(m∥Pk ⊥ T P k ∥ 2 ≤ m∥T ∥ 2 .<br />

We are ready for:<br />

Proof of the Berger-Shaw Theorem. Let R = ∥T ∥ and put D = B(0; R). If ε > 0, let<br />

D 1 , · · · , D n be pairwise disjoint closed disks c<strong>on</strong>tained in D \ σ(T ) such that<br />

Area (D) < Area σ(T ) + ∑ j<br />

Area (D j ) + ε.<br />

If D j = B(a j ; r j ), this inequality says<br />

πR 2 − π ∑ j<br />

r 2 j < Area σ(T ) + ε.<br />

If S is the unilateral shift of multiplicity 1, let S j = (a j + r j S) (m) . Now that each S j<br />

is m-multicyclic. Thus<br />

⎡<br />

⎤<br />

T<br />

S 1 0<br />

A = ⎢<br />

⎣<br />

.<br />

0 ..<br />

⎥<br />

⎦<br />

S n<br />

is an m-multicyclic hyp<strong>on</strong>ormal operator since the spectra of the operator summands<br />

are pairwise disjoint. Also ∥A∥ = R. By Lemma 3.2.6, tr [A ∗ , A] ≤ mR 2 . But<br />

Therefore<br />

tr [A ∗ , A] = tr [T ∗ , T ] +<br />

⎛<br />

n∑<br />

n∑<br />

tr [Sj ∗ , S j ] = tr [T ∗ , T ] + m rj 2 .<br />

j=1<br />

π tr [T ∗ , T ] ≤ m ⎝πR 2 − π<br />

n∑<br />

j=1<br />

Since ε was arbitrary, the proof is complete.<br />

r 2 j<br />

⎞<br />

j=1<br />

⎠ ≤ m ( Area σ(T ) + ε ) .<br />

Theorem 3.2.7. (Putnam’s inequality) If S ∈ B(H) is a hyp<strong>on</strong>ormal operator then<br />

∥[S ∗ , S]∥ ≤ 1 Area (σ(S)).<br />

π<br />

84


CHAPTER 3.<br />

HYPONORMAL AND SUBNORMAL THEORY<br />

Proof. Fix ∥f∥ ≤ 1 and let K ≡ ∨ {r(s)f : r ∈ Rat (σ(S))}. If T = S| K then T is an<br />

1-multicyclic hyp<strong>on</strong>ormal operator. By the Berger-Shaw theorem and the fact that<br />

∥T ∗ f∥ ≤ ∥S ∗ f∥, we get<br />

Since f was arbitrary, the result follows.<br />

⟨[S ∗ , S]f, f⟩ = ∥Sf∥ 2 − ∥S ∗ f∥ 2<br />

≤ ∥T f∥ 2 − ∥T ∗ f∥ 2<br />

= ⟨[T ∗ , T ]f, f⟩<br />

≤ tr [T ∗ , T ]<br />

≤ 1 Area(σ(T ))<br />

π<br />

≤ 1 π Area(σ(S)).<br />

Corollary 3.2.8. If S is a hyp<strong>on</strong>ormal operator such that Area (σ(S)) = 0 then S is<br />

normal.<br />

85


CHAPTER 3.<br />

HYPONORMAL AND SUBNORMAL THEORY<br />

3.3 Subnormal <strong>Operator</strong>s<br />

Definiti<strong>on</strong> 3.3.1. An operator S <strong>on</strong> a Hilbert space H is called subnormal if there<br />

exists a Hilbert space K ⊇ H and a normal operator N <strong>on</strong> K such that<br />

NH ⊆ H and N| H = S.<br />

The c<strong>on</strong>cept of subnormality was introduced in P. Halmos in 1950. Loosely speaking,<br />

a subnormal operator is <strong>on</strong>e that has a normal extensi<strong>on</strong>. Every isometry is<br />

subnormal (by Wold-v<strong>on</strong> Neumann decompositi<strong>on</strong>).<br />

Propositi<strong>on</strong> 3.3.2. Every subnormal operator is hyp<strong>on</strong>ormal.<br />

Proof. If S is subnormal then<br />

So<br />

∃ a normal operator N =<br />

0 = N ∗ N − NN ∗ =<br />

which implies that [S ∗ , S] = AA ∗ ≥ 0.<br />

[ ] S A<br />

.<br />

0 B<br />

[ ]<br />

[S ∗ , S] − AA ∗ S ∗ A<br />

A ∗ S A ∗ A + [B ∗ ,<br />

, B]<br />

An example of a hyp<strong>on</strong>ormal operator that is not subnormal:<br />

A ≡ U ∗ + 2U;<br />

then A is hyp<strong>on</strong>ormal, but A 2 is not; so A is not subnormal (To see this use Theorem<br />

3.3.7 below).<br />

Example 3.3.3. Let µ be a compactly supported measure <strong>on</strong> C and define N µ <strong>on</strong><br />

L 2 (µ) by<br />

N µ f = zf.<br />

Then N µ is normal since Nµf ∗ = zf.<br />

polynomials, define S µ <strong>on</strong> P 2 (µ) by<br />

If P 2 (µ) is the closure in L 2 (µ) of analytic<br />

S µ f = zf.<br />

Then S µ is subnormal and N µ is a normal extensi<strong>on</strong> of S µ .<br />

Definiti<strong>on</strong> 3.3.4. An operator S is called quasinormal if S and S ∗ S commute.<br />

86


CHAPTER 3.<br />

HYPONORMAL AND SUBNORMAL THEORY<br />

Propositi<strong>on</strong> 3.3.5. If S = UA is the polar decompositi<strong>on</strong> of S then<br />

S is quasinormal ⇐⇒ UA = AU.<br />

Proof. (⇐) UA = AU =⇒ SA 2 = UA 3 = A 2 UA = A 2 S =⇒ S is quasinormal.<br />

(⇒) If S is quasinormal then SA 2 = A 2 S (A 2 = S ∗ S). Thus SA = AS, so<br />

(UA − AU)A = SA − AS = 0. Thus UA − AU = 0 <strong>on</strong> ran A. But if f ∈ (ran A) ⊥ =<br />

ker A then since ker A = ker U, we have Uf = 0. Therefore UA = AU.<br />

Propositi<strong>on</strong> 3.3.6. Every quasinormal operator is subnormal.<br />

Proof. Suppose S is quasinormal.<br />

(Case 1: ker S = {0}) If S = UA is the polar decompositi<strong>on</strong> of S then U must<br />

be an isometry. If E = UU ∗ then E is the projecti<strong>on</strong> <strong>on</strong>to ranU. Thus (I − E)U =<br />

U ∗ (I − E) = 0. Define V, B ∈ B(H ⊕ H) by<br />

[ ] [ ]<br />

U I − E A 0<br />

V =<br />

0 U ∗ , V = .<br />

0 A<br />

Let N = V B. Since UA = AU and U ∗ A = AU ∗ it follows that N is normal. Since<br />

[ ] [ ]<br />

S (I − E)A S (I − E)A<br />

N =<br />

0 U ∗ =<br />

A 0 S ∗ ,<br />

we have NH ⊆ H and N| H = S.<br />

(Case 2: ker S ≠ {0}) Here ker S = L ⊆ ker S ∗ since S ∗ = AU ∗ = U ∗ A. Let<br />

S 1 := ( ) ⊥.<br />

S| L So S = S1 ⊕ 0 <strong>on</strong> L ⊥ ⊕ L = H. Now S ∗ S = S1S ∗ 1 ⊕ 0. Observe S 1 is<br />

quasinormal. By Case 1, S 1 is subnormal and therefore S is subnormal.<br />

Remember [C<strong>on</strong>2, p.44] that<br />

S is pure quasinormal ⇐⇒ S = U⊗A, where A is a positive operator with ker A = {0}.<br />

If X is a locally compact space, a positive operator-valued measure(POM) <strong>on</strong> X<br />

is defined by a functi<strong>on</strong> Q such that<br />

Q: a Borel set △ ⊆ X ↦→ Q(△), a positive operator, ∈ B(H);<br />

Q(X) = 1;<br />

⟨Q(·)f, f⟩ is a regular Borel measure <strong>on</strong> X.<br />

Every spectral measure is a POM. But the c<strong>on</strong>verse is false. Let E be a spectral<br />

measure <strong>on</strong> X with values in B(K), H be a subspace of K and let P be the orthog<strong>on</strong>al<br />

projecti<strong>on</strong> of K <strong>on</strong>to H. Define<br />

Q(△) := P E(△)| H .<br />

87


CHAPTER 3.<br />

HYPONORMAL AND SUBNORMAL THEORY<br />

Then Q is a POM with ∥Q(△)∥ ≤ 1 for all △. But Q is a spectral measure if and<br />

<strong>on</strong>ly if P commutes with E(△) for any △.<br />

If Q is a POM and ϕ is a bounded Borel functi<strong>on</strong> <strong>on</strong> X then ∫ ϕdQ denotes the<br />

unique operator T defined by the bounded quadratic form<br />

∫<br />

⟨T f, f⟩ = ϕ(x)d⟨Q(x)f, f⟩.<br />

Theorem 3.3.7. If S ∈ B(H), the following are equivalent:<br />

(a) S is subnormal.<br />

(b) (Bram-Halmos, 1955/1950) If f 0 , · · · , f n ∈ H then<br />

∑<br />

⟨S j f k , S k f j ⟩ ≥ 0. (3.3)<br />

j,k<br />

(c) (Embry, 1973) For any f 0 , · · · , f n ∈ H<br />

∑<br />

⟨S j+k f j , S j+k f k ⟩ ≥ 0. (3.4)<br />

j,k<br />

(d) (Bunce and Deddens, 1977) If B 0 , · · · , B n ∈ C ∗ (S) then<br />

∑<br />

Bj ∗ S ∗k S j B k ≥ 0<br />

j,k<br />

(e) (Bram, 1955) There is a POM Q supported <strong>on</strong> a compact subset of C such that<br />

∫<br />

S ∗n S m = z n z m dQ(z) for all m, n ≥ 0. (3.5)<br />

(f) (Embry, 1973) There is a POM Q <strong>on</strong> some interval [0, a] ⊆ R such that<br />

∫<br />

S ∗n S n = t 2n dQ(t) for all n ≥ 0.<br />

Proof. (a) ⇒ (b): Let N =<br />

[ ] S ∗ H<br />

0 ∗ H′<br />

be a normal operator <strong>on</strong> K. If P is the<br />

88


CHAPTER 3.<br />

HYPONORMAL AND SUBNORMAL THEORY<br />

projecti<strong>on</strong> of K <strong>on</strong>to H, then S ∗n f = P N ∗n f, f ∈ H. If f 0 , · · · , f n ∈ H then<br />

∑<br />

⟨S j f k , S k f j ⟩ = ∑ ⟨N j f k , N k f j ⟩<br />

j,k<br />

j,k<br />

= ∑ ⟨N ∗k N j f k , f j ⟩<br />

j,k<br />

= ∑ ⟨N j N ∗k f k , f j ⟩<br />

j,k<br />

= ∑ ⟨N ∗k f k , N ∗j f j ⟩<br />

j,k<br />

∥ ∑ ∥∥∥∥<br />

2<br />

=<br />

N ∗k f<br />

∥<br />

k .<br />

So (3.3) holds.<br />

(b) ⇒ (c): Put g k = S k f k . Then (3.3) implies<br />

∑<br />

⟨S j g k , S k g j ⟩ = ∑ ⟨S j+k f k , S j+k f j ⟩.<br />

j,k<br />

j,k<br />

k<br />

So (3.4) holds.<br />

(c) ⇒ (a): See [C<strong>on</strong>2].<br />

(b) ⇒ (d): If B 0 , · · · , B n ∈ C ∗ (S), let f k = B k f. Then<br />

(3.3) ⇐⇒<br />

⟨ ∑<br />

j,k<br />

B ∗ j S ∗k S j B k f, f<br />

⟩<br />

≥ 0.<br />

(d) ⇒ (b): By Zorn’s lemma,<br />

any operator = ⊕ star-cyclic operator.<br />

So we may assume that S has a star-cyclic vector e 0 , i.e., assume H = cl [C ∗ (S)e 0 ].<br />

If B 0 , · · · , B n ∈ C ∗ (S) then (3.3) holds for f k = B k e 0 . Since (3.3) holds for a dense<br />

set of vector, (3.3) holds for all vectors.<br />

(a) ⇒ (e): Let N = ∫ zdE(z) be the spectral decompositi<strong>on</strong> of N, a normal<br />

extensi<strong>on</strong> of S acting <strong>on</strong> K ⊇ H. Let P be the orthog<strong>on</strong>al projecti<strong>on</strong> of K <strong>on</strong>to H.<br />

Define<br />

Q(△) := P E(△)| H for every Borel subset △ of C.<br />

89


CHAPTER 3.<br />

HYPONORMAL AND SUBNORMAL THEORY<br />

Then Q is a POM and is supported <strong>on</strong> σ(N). Also for all h ∈ H,<br />

⟨S ∗n S m h, h⟩ = ⟨N ∗n N m h, h⟩<br />

∫<br />

= z n z m d⟨E(z)h, h⟩<br />

∫<br />

= z n z m d⟨E(z)h, P h⟩<br />

∫<br />

= z n z m d⟨P E(z)h, h⟩<br />

∫<br />

= z n z m d⟨Q(z)h, h⟩<br />

⟨(∫<br />

) ⟩<br />

= z n z m dQ(z) h, h ,<br />

so that<br />

∫<br />

S ∗n S m =<br />

z n z m dQ(z).<br />

(e) ⇒ (f): Let Q be the POM hypothesized in (i) and K = supp Q. For a Borel<br />

set △ ⊆ [0, ∞), define<br />

Q + (△) := Q{z ∈ C : |z| ∈ △}.<br />

In fact, Q + (△) = Q(τ −1 △), where τ(z) = |z|. Then Q + is a POM whose support<br />

⊆ [0, a] with a = max z∈K |z|. For any f ∈ H,<br />

∫<br />

∫<br />

t 2n d⟨Q + (t)f, f⟩ = |z| 2n d⟨Q(z)f, f⟩.<br />

(f) ⇒ (c): Fix f 0 , · · · , f n ∈ H and define scalar-valued measures µ jk by<br />

µ jk (△) = ⟨Q(△)f j , f k ⟩.<br />

Let µ be a positive measure <strong>on</strong> [0, a] such that µ jk ≪ µ for any j, k. Let h jk = dµ jk<br />

dµ<br />

(Rad<strong>on</strong>-Nikodym derivative). For each u ∈ C[0, a], ρ(u) = ∫ udQ defines a bounded<br />

operator and ρ : C[0, a] → B(H) is a positive linear map. Note that for all u,<br />

⟨ρ(u)f j , f k ⟩ = ∫ udµ jk = ∫ uh j,k dµ. Moreover if λ 0 , · · · , λ n ∈ C and u ≥ 0 then<br />

∑<br />

(∫<br />

j,k<br />

)<br />

uh jk dµ λ j λ k =<br />

∥ ∑ ∥∥∥∥∥<br />

2 ρ(u) 1 2 λj f j ≥ 0.<br />

∥<br />

It follows that (h jk (t)) j,k<br />

is positive (n + 1) × (n + 1) matrix for [µ] almost every t.<br />

This implies that<br />

∑<br />

h jk (t)t 2j t 2k ≥ 0 a.e. [µ].<br />

j,k<br />

90<br />

j


= ∑ j,k<br />

CHAPTER 3.<br />

HYPONORMAL AND SUBNORMAL THEORY<br />

Therefore<br />

∫ ∑<br />

0 ≤<br />

j,k<br />

= ∑ ∫<br />

j,k<br />

h jk (t)t 2(j+k) dµ(t)<br />

t 2(j+k) dµ jk (t)<br />

⟨<br />

S j+k f j , S j+k f k<br />

⟩<br />

,<br />

so (3.4) holds.<br />

Remark. Without loss of generality we may assume that ||S|| < 1. Let K = H ∞ and<br />

let K 0 = the finitely n<strong>on</strong>zero sequences in K. Let<br />

⎡<br />

1 S ∗ S ∗2 ⎤<br />

· · ·<br />

S S ∗ S S ∗2 S · · ·<br />

M =<br />

S 2 S ∗ S 2 S ∗2 S 2 · · ·<br />

⎢<br />

⎣S 3 S ∗ S 3 S ∗2 S 3 · · · ⎥<br />

⎦<br />

. . .<br />

<strong>on</strong> K 0 .<br />

If f = (f 0 , · · · , f n , · · · ) ∈ K 0 then<br />

∑<br />

∥(Mf) j ∥ 2 = ∑<br />

j<br />

j<br />

≤ ∑ j<br />

∥ ∑ ∥∥∥∥<br />

2<br />

S ∗k S j f<br />

∥<br />

k<br />

k<br />

[ ]<br />

∑<br />

∥S∥ k+j ∥f k ∥ 2<br />

k<br />

≤ ∑ j<br />

[ ∑<br />

k<br />

∥S∥ 2k+2j ] [ ∑<br />

k<br />

∥f k ∥<br />

] 2<br />

≤ (1 − ∥S∥ 2 ) −2 ∥f∥ 2 .<br />

Since ∥S∥ < 1, Mf ∈ K and M extends to a bounded operator <strong>on</strong> K. Clearly, M is<br />

hermitian. Note<br />

⟨Mf, f⟩ K = ∑ ⟨S j f k , S k f j ⟩.<br />

j,k<br />

So<br />

(3.3) holds ⇐⇒ M is positive.<br />

[ ] A B<br />

Recall the Smul’jan theorem – if M =<br />

B ∗ (A, C hermitian, A invertible), then<br />

C<br />

M ≥ 0 ⇐⇒ B ∗ A −1 B ≤ C.<br />

91


CHAPTER 3.<br />

HYPONORMAL AND SUBNORMAL THEORY<br />

We thus have that if<br />

then<br />

⎡<br />

1 S ∗ · · · S ∗k ⎤<br />

· · ·<br />

S S ∗ S · · · S ∗k S · · ·<br />

M = ⎢<br />

⎣S 2 S ∗ S 2 · · · S ∗k S 2 · · · ⎥<br />

⎦<br />

. . · · · . · · ·<br />

⎡ ⎤<br />

S<br />

⎡<br />

⎤<br />

S 2<br />

S ∗ S S ∗2 S S ∗3 S · · ·<br />

[<br />

M ≥ 0 ⇐⇒ ⎢<br />

⎣<br />

S 3 ⎥ S<br />

∗<br />

S ∗2 S ∗3 ⎢<br />

· · ·] ≤ ⎣S ∗ S 2 S ∗2 S 2 S ∗3 S 2 · · · ⎥<br />

⎦<br />

⎦<br />

.<br />

.<br />

.<br />

.<br />

.<br />

⎡<br />

⎤<br />

S ∗ S − SS ∗ S ∗2 S − SS ∗2 · · ·<br />

⎢<br />

⇐⇒ ⎣S ∗ S 2 − S 2 S ∗ S ∗2 S 2 − S 2 S ∗2 · · · ⎥<br />

⎦ ≥ 0<br />

.<br />

.<br />

.<br />

⎡<br />

[S ∗ , S] [S ∗2 , S] [S ∗3 ⎤<br />

, S] · · ·<br />

[S ∗ , S 2 ] [S ∗2 , S 2 ] [S ∗3 , S 2 ] · · ·<br />

⇐⇒ ⎢<br />

⎣[S ∗ , S 3 ] [S ∗2 , S 3 ] [S ∗3 , S 3 ] · · · ⎥<br />

⎦ ≥ 0<br />

. . . .<br />

Definiti<strong>on</strong> 3.3.8. If A is a C ∗ −algebra, define s ∈ A to be subnormal if ∑ j,k a∗ j s∗k s j a k ≥<br />

0 for any choice a 0 , · · · , a n ∈ C ∗ (s).<br />

It is easy to see that if A, B are C ∗ −algebras and ρ : A −→ B is a ∗−homomorphism<br />

then ρ maps subnormal elements of A <strong>on</strong>to subnormal elements of B. In particular,<br />

if (ρ, H) is a representati<strong>on</strong> of A, ρ(s) is a subnormal operator <strong>on</strong> H whenever S is<br />

a subnormal element of A.<br />

Remark. (Agler [Ag2, 1985]’s characterizati<strong>on</strong> of subnormal operators) If S is a c<strong>on</strong>tracti<strong>on</strong><br />

then<br />

n∑<br />

( ) n<br />

S is subnormal ⇐⇒ (−1) k S ∗k S k ≥ 0 for all n ≥ 1.<br />

k<br />

k=0<br />

Note that if N is a normal extensi<strong>on</strong> of S ∈ B(H) to K then<br />

K ⊇ ∨ }<br />

{N ∗k h : h ∈ H, k = 0, 1, 2, · · · .<br />

If L := ∨ { }<br />

N ∗k h : h ∈ H, k = 0, 1, 2, · · · then<br />

L is a reducing subspace for N that c<strong>on</strong>tains H.<br />

Thus N| L is also a normal extensi<strong>on</strong> of S. Moreover if R is any reducing subspace<br />

for N that c<strong>on</strong>tains H then R must c<strong>on</strong>tain L.<br />

92


CHAPTER 3.<br />

HYPONORMAL AND SUBNORMAL THEORY<br />

Definiti<strong>on</strong> 3.3.9. If S is a subnormal operator <strong>on</strong> H and N is a normal extensi<strong>on</strong><br />

of S to K then N is called a minimal normal extensi<strong>on</strong> of S if<br />

K = ∨ }<br />

{N ∗k h : h ∈ H, k ≥ 0 .<br />

Propositi<strong>on</strong> 3.3.10. If S is a subnormal operator then any two minimal normal<br />

extensi<strong>on</strong>s are unitarily equivalent.<br />

Proof. For p = 1, 2 let N p be a minimal normal extensi<strong>on</strong> of S acting <strong>on</strong> K p ⊇ H.<br />

Define U : K 1 −→ K 2 by<br />

U (N ∗ 1 n h) = N ∗ 2 n h (h ∈ H).<br />

We want to show that U is an isomorphism. If h 1 , · · · , h m ∈ H and n 1 , · · · , n m ≥ 0<br />

then<br />

∥ ∑ ∥∥∥∥<br />

2 ⟨ N2<br />

∗ n ∑<br />

k<br />

h<br />

∥<br />

k = N ∗ n k<br />

2 h k , ∑ ⟩<br />

N ∗ n j<br />

2 h j<br />

k<br />

k<br />

j<br />

= ∑ j,k<br />

⟨N 2<br />

n j<br />

h k , N 2<br />

n k<br />

h j ⟩<br />

= ∑ j,k<br />

= ∑ j,k<br />

⟨S n j<br />

h k , S n k<br />

h j ⟩<br />

⟨N 1<br />

n j<br />

h k , N 1<br />

n k<br />

h j ⟩<br />

∥ ∑ ∥∥∥∥<br />

2<br />

=<br />

N1<br />

∗ n k<br />

h<br />

∥<br />

k ,<br />

k<br />

which shows that<br />

U<br />

]<br />

N1<br />

∗ n k<br />

h k = ∑ k<br />

N ∗ 2<br />

n k<br />

h k<br />

[ ∑<br />

k<br />

is a well defined linear operator from a dense linear manifold in K 1 <strong>on</strong>to a dense linear<br />

manifold in K 2 and U is an isometry. Also for all h ∈ H, Uh = h. Thus for h ∈ H<br />

and n ≥ 0,<br />

UN 1 N ∗ 1 n h = UN ∗ 1 n Sh = N ∗ 2 n Sh = N 2 N ∗ 2 n h = N 2 UN ∗ 1 n h,<br />

i.e., UN 1 = N 2 U, so that N 1 and N 2 are unitarily equivalent.<br />

Now it is legitimate to speak of the minimal normal extensi<strong>on</strong> of a subnormal<br />

operator. Therefore it is unambiguous to define the normal spectrum of a subnormal<br />

operator S, σ n (S), as the spectrum of its minimal normal extensi<strong>on</strong>.<br />

93


CHAPTER 3.<br />

HYPONORMAL AND SUBNORMAL THEORY<br />

Propositi<strong>on</strong> 3.3.11. If S is a subnormal operator then the following hold:<br />

(a) (Halmos, 1952) σ n (S) ⊆ σ(S).<br />

(b) σ ap (S) ⊆ σ n (S) and ∂σ(S) ⊆ ∂σ n (S).<br />

(c) (Bram, 1955) If U is a bounded comp<strong>on</strong>ent of C\σ n (S), then either U ∩σ(S) = ∅<br />

or U ⊆ σ(S).<br />

Proof. (a) We want to show that S is invertible ⇒ N is invertible.<br />

If N = ∫ zdE(z) is the spectral decompositi<strong>on</strong> of N, ε > 0, and M = E (B(0; ε)) K<br />

then we claim that<br />

||N k f|| ≤ ε k ∥f∥ for k = 1, 2, 3, · · · and f ∈ M.<br />

To see this let △ := B(0; ε). Then<br />

∫<br />

NE(△) = zχ △ (z)dE(z) = ϕ(N), where ϕ = zχ △ .<br />

We thus have<br />

∥NE(△)∥ = ∥ϕ(N)∥ ≤ ∥ϕ∥ = sup|ϕ(z)| = sup{|z| : z ∈ △} ≤ ε.<br />

So if f ∈ M then E(△)f = f. Therefore<br />

∥Nf∥ = ∥NE(△)f∥ ≤ ∥NE(△)∥∥f∥ ≤ ε∥f∥.<br />

So if f ∈ M and h ∈ H,<br />

|⟨f, h⟩| = ∣ ∣⟨f, S k S −k h⟩ ∣ = ∣ ∣⟨f, N k S −k h⟩ ∣ ∣<br />

= ∣⟨N ∗k f, S −k h⟩ ∣<br />

Letting k → ∞ shows that<br />

≤ ∥ ∥N ∗k f ∥ ∥ · ∥∥S −k h ∥ ∥ ≤ ε k ∥f∥ ∥ ∥S −k∥ ∥ ≤ ε k ∥ ∥S −1∥ ∥ k ∥f∥ ∥h∥.<br />

ε < 1<br />

∥S −1 ∥<br />

=⇒ ⟨f, h⟩ = 0,<br />

so that H ⊆ M ⊥ . Since M is a reducing subspace for N, N| M ⊥ is a normal extensi<strong>on</strong><br />

of S. By the minimality of N, M = {0} and so N is invertible because N = N φ and<br />

|φ(x) ≥ ε a.e.<br />

(b) Observe that<br />

λ ∈ σ ap (S) =⇒ ∃ unit vectors h n ∈ H such that ∥(λ − S)h n ∥ −→ 0.<br />

But (λ − S)h n = (λ − N)h n .<br />

=⇒ σ ap (S) ⊆ σ ap (N) = σ(N) = σ n (S).<br />

λ ∈ ∂σ(S) =⇒ λ ∈ σ ap (S) =⇒ λ ∈ σ n (S) =⇒ λ /∈ int σ n (S) =⇒ λ ∈ ∂σ n (S).<br />

(c) (Due to S. Parrot) Let U be a bounded comp<strong>on</strong>ent of σ n (S) c and put<br />

U + = U \ σ(S) and U − = U ∩ σ(S).<br />

So U = U − ∪ U + , U + ∩ U − = ∅ and U + is open. By (b), U − = U ∩ int σ(S), so that<br />

U − is open. By the c<strong>on</strong>nectedness of U, either U + = ∅ or U − = ∅.<br />

94


CHAPTER 3.<br />

HYPONORMAL AND SUBNORMAL THEORY<br />

Corollary 3.3.12. If S is a subnormal operator whose minimal normal extensi<strong>on</strong> is<br />

N then<br />

r(S) = ∥S∥ = ∥N∥ = r(N).<br />

Proof. Since r(S) ≤ ∥S∥ ≤ ∥N∥ = r(N), the result follows from Propositi<strong>on</strong> 3.3.11.<br />

Definiti<strong>on</strong> 3.3.13. If A ∈ B(H), e 0 ∈ H, K is a compact subset of C c<strong>on</strong>taining<br />

σ(A) then e 0 is called a Rat(K) cyclic vector for A if<br />

{<br />

}<br />

u(A)e 0 : u ∈ Rat (K) is dense in H.<br />

An operator is called Rat (K) cyclic if it has a Rat (K) cyclic vector. In the cases<br />

that K = σ(S), A is called a rati<strong>on</strong>ally cyclic operator.<br />

Recall that e 0 is a cyclic vector for A (A is a cyclic operator) if {p(A)e 0 :<br />

p is a polynomial} is dense in H. By Runge’s theorem, e 0 is cyclic for A ⇔ e 0 is<br />

Rat̂σ(A) cyclic for A.<br />

Note that if S is subnormal and N = mme (S) then since σ(N) ⊆ σ(S), it follows<br />

that if K c<strong>on</strong>tains σ(S) then u(N) is well defined for any U ∈ Rat (K).<br />

Theorem 3.3.14. If S is subnormal and has a Rat (K) cyclic vector e 0 then there<br />

exists a unique compactly supported measure µ <strong>on</strong> K and an isomorphism U : K →<br />

L 2 (µ) such that<br />

(a) UH = R 2 (K, µ);<br />

(b) Ue 0 = 1;<br />

(c) UNU −1 = N µ ;<br />

(d) if V = U| H , then V is an isomorphism of H <strong>on</strong>to R 2 (K, µ) and V SV −1 =<br />

N µ | R2 (K,µ).<br />

Proof. If N = mme (S) then K = ∨ {N ∗n u(N)e 0 : n ≥ 0, u ∈ Rat (K)}. We claim<br />

that e 0 is a ∗−cyclic vector for N. Indeed, let L = ∨ {N ∗n u(N)e 0 : n, k ≥ 0}. Evidently,<br />

L is a reducing subspace for N. By the St<strong>on</strong>e-Weierstrass theorem, C(K)<br />

is the uniformly closed linear span of { z n z k : n, k ≥ 0 } . Since Rat (K) ⊆ C(K), we<br />

have that u(N)e 0 ∈ L for every U ∈ Rat (K). Thus H ⊆ L. By the minimality of<br />

N we have H = K. Hence e 0 is a ∗−cyclic vector for N. Therefore there exists a<br />

compactly supported measure µ and an isomorphism<br />

U : K → L 2 (µ) such that Ue 0 = 1 and UNU −1 = N µ .<br />

So (b) and (c) hold. Observe Uϕ(N) = ϕ(N µ )U for every bounded Borel functi<strong>on</strong><br />

ϕ. In particular, for u ∈ Rat (K), Uu(S)e 0 = Uu(N)e 0 = u(N µ )Ue 0 = u(N µ )1 = u.<br />

Taking limits gives (a). The asserti<strong>on</strong> (d) is immediate. The proof of the uniqueness<br />

of µ comes from the St<strong>on</strong>e-Weierstrass theorem.<br />

95


CHAPTER 3.<br />

HYPONORMAL AND SUBNORMAL THEORY<br />

Corollary 3.3.15. An operator S is a cyclic subnormal operator if and <strong>on</strong>ly if S ∼ = S µ<br />

for some compactly supported measure µ.<br />

For any compact K, define<br />

R(K) := the uniform closure in C(K) of Rat (K).<br />

Define ∥f∥ K = sup z∈K |f(z)|. For a subnormal operator S, we may define f(S) for all<br />

functi<strong>on</strong>s f ∈ R (σ(S)) . If f ∈ R (σ(S)), f(S) = f(N)| H . So f(S) is subnormal, so<br />

that<br />

σ(f(S)) = f(σ(S))<br />

∥f∥ σ(S) = ∥f(S)∥ ≤ ∥f(N)∥ = ∥f∥ σ(N) ≤ ∥f∥ σ(S) ,<br />

i.e., the map f ↦→ f(S) is an isometry from R(σ(S)) into B(H).<br />

R(σ(S)),<br />

f(S) := the image of f under this isomorphism.<br />

Then<br />

f(N)H ⊆ H,<br />

f(N)| H = f(S).<br />

Define, for f ∈<br />

Theorem 3.3.16. If S is subnormal and N = mne (S), and for each f ∈ R(σ(S)),<br />

f(S) = f(N)| H then the map f ↦→ f(S) is a multiplicative linear isometry from<br />

R(σ(S)) into B(H) that extends the Riesz functi<strong>on</strong>al calculus for S. Moreover,<br />

Proof. See [C<strong>on</strong>2].<br />

σ(f(S)) = f(σ(S)) for f ∈ R(σ(S)).<br />

Lemma 3.3.17. If S is subnormal and σ(S) ⊆ R then S is hermitian.<br />

Proof. Let N = mne (S), acting <strong>on</strong> K. By Propositi<strong>on</strong> 3.3.11, σ(N) ⊆ σ(S) ⊆ R.<br />

Hence N = N ∗ . Then every invariant subspace for N reduces N. In particular, H<br />

reduces N. By the minimality of N, K = H. So S = N and hence S is hermitian.<br />

Propositi<strong>on</strong> 3.3.18. If S is subnormal and R(σ(S)) = C(σ(S)) then S is normal.<br />

Proof. Let ϕ(z) = Re z and ψ(z) = Im z. By hypothesis, ϕ, ψ ∈ R(σ(S)). By Theorem<br />

3.3.16, ϕ(S) is subnormal and<br />

σ(ϕ(S)) = ϕ(σ(S)) ⊆ R.<br />

Therefore ϕ(S) is hermitian. Similarly, ψ(S) is hermitian. Since ϕ + iψ = z, S =<br />

ϕ(S) + iψ(S). Since ϕ(S)ψ(S) = ψ(S)ϕ(S), it follows S is normal.<br />

Remark. If σ is compact and R(σ) = C(σ) then σ is called thin. It was known [Wer]<br />

that<br />

(i) σ is thin =⇒ intσ = ∅;<br />

(ii) The c<strong>on</strong>verse of (i) fails;<br />

(iii) m(σ) = 0 =⇒ σ thin.<br />

96


CHAPTER 3.<br />

HYPONORMAL AND SUBNORMAL THEORY<br />

3.4 p-Hyp<strong>on</strong>ormal <strong>Operator</strong>s<br />

Recall that the numerical range of T ∈ B(H) is defined by<br />

{<br />

}<br />

W (T ) := ⟨T x, x⟩ : ||x|| = 1<br />

and the numerical radius of T is defined by<br />

{<br />

}<br />

w(T ) := sup |λ| : λ ∈ W (T ) .<br />

It was well-known (cf. [Ha3]) that<br />

(a) W (T ) is c<strong>on</strong>vex (Toeplitz-Haussdorff theorem);<br />

(b) c<strong>on</strong>v σ(T ) ⊂ cl W (T );<br />

(c) r(T ) ≤ w(T ) ≤ ||T ||;<br />

1<br />

(d)<br />

dist ≤ ||(T − (λ,σ(T )) λ)−1 1<br />

|| ≤<br />

dist (λ,cl . W (T ))<br />

Definiti<strong>on</strong> 3.4.1. (a) T is called normaloid if ||T || = r(T );<br />

(b) T is called spectraloid if w(T ) = r(T );<br />

(c) T is called c<strong>on</strong>vexoid if c<strong>on</strong>v σ(T ) = cl W (T );<br />

(d) T is called transaloid if T − λ is normaloid for any λ;<br />

(e) T is siad to satisfy (G 1 )-c<strong>on</strong>diti<strong>on</strong> if<br />

||(T − λ) −1 || ≤<br />

1<br />

dist (λ, σ(T )) , in fact, ||(T − 1<br />

λ)−1 || =<br />

dist (λ, σ(T )) .<br />

(f) T is called paranormal if ||T 2 x|| ≥ ||T x|| 2 for any x with ||x|| = 1.<br />

It was well-known that it T is paranormal then<br />

(i) T n is paranormal for any n;<br />

(ii) T is normaloid;<br />

(iii) T −1 is paranormal if it exists;<br />

and that<br />

hyp<strong>on</strong>ormal ⊂ paranormal ⊂ normaloid ⊂ spectraloid.<br />

Theorem 3.4.2. If T ∈ B(H) then<br />

(a) T is c<strong>on</strong>vexoid ⇐⇒ T − λ is spectraloid for any λ, i.e., w(T − λ) = r(T − λ);<br />

(b) T is c<strong>on</strong>vexoid ⇐⇒ ||(T − λ) −1 || ≤<br />

1<br />

dist (λ, c<strong>on</strong>v σ(T ))<br />

for any λ /∈ c<strong>on</strong>v σ(T ).<br />

97


CHAPTER 3.<br />

HYPONORMAL AND SUBNORMAL THEORY<br />

Proof. (a) Note that<br />

Since cl W (T ) is c<strong>on</strong>vex,<br />

c<strong>on</strong>v X = the intersecti<strong>on</strong> of all disks c<strong>on</strong>taining X<br />

= ∩ {<br />

}<br />

λ : |λ − µ| ≤ sup |x − µ| .<br />

µ<br />

x∈X<br />

cl W (T ) = ∩ µ<br />

c<strong>on</strong>v σ(T ) = ∩ µ<br />

{<br />

}<br />

λ : |λ − µ| ≤ w(T − µ) ;<br />

{<br />

}<br />

λ : |λ − µ| ≤ r(T − µ) .<br />

so the result immediately follows.<br />

(b) (⇒) Clear from the preceding remark.<br />

(⇐) Suppose<br />

or equivalently,<br />

||(T − λ) −1 || ≤<br />

1<br />

dist (λ, c<strong>on</strong>v σ(T ))<br />

||(T − λ) x|| ≥<br />

1<br />

dist (λ, c<strong>on</strong>v σ(T ))<br />

for any λ /∈ c<strong>on</strong>v σ(T ),<br />

for any λ /∈ c<strong>on</strong>v σ(T ) and ||x|| = 1.<br />

Thus<br />

||T x|| 2 − 2 Re ⟨T x, x⟩λ + |λ| 2 ≥<br />

)<br />

inf<br />

(|s| 2 − 2Re sλ + |λ| 2 .<br />

s∈c<strong>on</strong>v σ(T )<br />

Taking λ = |λ|e −i(θ+π) , dividing by |λ| and letting λ → ∞, we have<br />

Re ⟨T x, x⟩e iθ ≥ inf<br />

s∈c<strong>on</strong>v σ(T ) Re ( se iθ) for ||x|| = 1,<br />

which implies cl W (T ) ⊂ c<strong>on</strong>v σ(T ). Therefore cl W (T ) = c<strong>on</strong>v σ(T ).<br />

Corollary 3.4.3. We have:<br />

(a) transaloid ⇒ c<strong>on</strong>vexoid;<br />

(b) (G 1 ) ⇒ c<strong>on</strong>vexoid.<br />

Proof. (a) Clear.<br />

(b) ||(T − λ) −1 || =<br />

1<br />

dist (λ, σ(T )) ≤ 1<br />

dist (λ, c<strong>on</strong>v σ(T ))<br />

Definiti<strong>on</strong> 3.4.4. An operator T ∈ B(H) is said to satisfy the projecti<strong>on</strong> property if<br />

Re σ(T ) = σ(Re T ), where Re T := 1 2 (T + T ∗ ).<br />

98


CHAPTER 3.<br />

HYPONORMAL AND SUBNORMAL THEORY<br />

Theorem 3.4.5. An operator T ∈ B(H) is c<strong>on</strong>vexoid if and <strong>on</strong>ly if<br />

Re c<strong>on</strong>v σ ( e iθ T ) = c<strong>on</strong>v σ ( Re (e iθ T ) )<br />

for any θ ∈ [0, 2π).<br />

Proof. Observe that<br />

Re ( e iθ c<strong>on</strong>v σ(T ) ) = c<strong>on</strong>v σ ( Re (e iθ T ) )<br />

= cl W ( Re (e iθ T ) )<br />

= Re cl W (e iθ T )<br />

= Re ( e iθ cl W (T ) ) .<br />

which implies that c<strong>on</strong>v σ(T ) = cl W (T ) and this argument is reversible.<br />

Example 3.4.6. There exist c<strong>on</strong>vexoid operators which are not normaloid and vice<br />

versa. (see [Ha2, Problem 219]).<br />

Example 3.4.7. (An example of a n<strong>on</strong>-c<strong>on</strong>vexoid and papranormal operator) Let U<br />

be the unilateral shift <strong>on</strong> l 2 , P = diag(1, 0, 0, . . .) and put<br />

T =<br />

[ ] U + I P<br />

.<br />

0 0<br />

Then σ(T ) = σ(U + I) ∪ {0} = {λ : |λ − 1| ≤ 1}. But if x = (− 1 2<br />

, 0, 0, . . .) and<br />

y = ( √ 3<br />

2 , 0, 0, . . .) then ∣ ∣∣∣<br />

∣ ∣∣∣<br />

[ x<br />

y]∣ ∣∣∣<br />

∣ ∣∣∣<br />

= 1 and<br />

W (T ) ∋ ⟨T (x ⊕ y), x ⊕ y⟩ = 1 4 − √<br />

3<br />

4 < 0.<br />

Therefore T is not c<strong>on</strong>vexoid, but T is papranormal (see [T. Furuta, Invitati<strong>on</strong> to<br />

Linear operators]).<br />

Definiti<strong>on</strong> 3.4.8. An operator T ∈ B(H) is called p-hyp<strong>on</strong>ormal if<br />

(T ∗ T ) p ≥ (T T ∗ ) p .<br />

If p = 1, T is hyp<strong>on</strong>ormal ad if p = 1 2<br />

, T is called seminormal. It was known that<br />

q-hyp<strong>on</strong>ormal ⇒ p-hyp<strong>on</strong>ormal for p ≤ q by Löner-Heinz inequality.<br />

Theorem 3.4.9. p-hyp<strong>on</strong>ormal =⇒ paranormal.<br />

Proof. See [An].<br />

99


CHAPTER 3.<br />

HYPONORMAL AND SUBNORMAL THEORY<br />

It was also well-known that if T is p-hyp<strong>on</strong>ormal then<br />

(i) T is normaloid;<br />

(ii) T is reduced by its eigenspaces;<br />

(iii) T −1 is paranormal if it exists.<br />

However p-hyp<strong>on</strong>ormal operators need not be transaloid. In fact, p-hyp<strong>on</strong>ormality is<br />

not translati<strong>on</strong>-invariant. To see this we first recall:<br />

Lemma 3.4.10. If T is p-hyp<strong>on</strong>ormal then T n is p n-hyp<strong>on</strong>ormal for 0 < p ≤ 1.<br />

Proof. See [AW].<br />

Theorem 3.4.11. There exists an operator T satisfying<br />

(i) T is semi-hyp<strong>on</strong>ormal;<br />

(ii) T − λ is not p-hyp<strong>on</strong>ormal for any p > 0 and some λ ∈ C.<br />

Proof. Let<br />

Then we claim that<br />

S ≡ 4U 2 + U ∗2 + 2UU ∗ + 2 (U =the unilateral shift <strong>on</strong> l 2 ).<br />

(a) S is semi-hyp<strong>on</strong>ormal;<br />

(b) S − 4 is not p-hyp<strong>on</strong>ormal for any p > 0, in fact S − 4 is not paranormal.<br />

Indeed, if we put φ(z) = 2z + z −1 the T φ is hyp<strong>on</strong>ormal but T 2 φ is not because Since<br />

T 2 φ = S, so S is semi-hyp<strong>on</strong>ormal. On the other hand, observe that<br />

||(S − 4)e 0 || 2 = 20 and ||(S − 4) 2 e 0 || = √ 384,<br />

so<br />

which is not paranormal.<br />

||(S − 4)e 0 || 2 > ||(S − 4) 2 e 0 ||,<br />

100


CHAPTER 3.<br />

HYPONORMAL AND SUBNORMAL THEORY<br />

3.5 Comments and Problems<br />

The following problem <strong>on</strong> p-hyp<strong>on</strong>ormal operators remains still open:<br />

Problem 3.1.<br />

(a) Is every p-hyp<strong>on</strong>ormal operator c<strong>on</strong>vexoid <br />

(b) Does every p-hyp<strong>on</strong>ormal operator satisfy the (G 1 )-c<strong>on</strong>diti<strong>on</strong> <br />

(c) Does every p-hyp<strong>on</strong>ormal operator satisfy the projecti<strong>on</strong> properry <br />

In fact,<br />

Yes to (b) =⇒ Yes to (a) =⇒ Yes to (c).<br />

It was known that the projecti<strong>on</strong> property holds for every hyp<strong>on</strong>ormal operator. For<br />

a proof, see [Put2].<br />

For a partial answer see [M. Cho, T. Huruya, Y. Kim, J. <str<strong>on</strong>g>Lee</str<strong>on</strong>g>, A note <strong>on</strong> real parts<br />

of some semi-hyp<strong>on</strong>ormal operator.]<br />

It is easily check that every p-hyp<strong>on</strong>ormal weighted shift is hyp<strong>on</strong>ormal. However<br />

we were unable to answer the following:<br />

Problem 3.2. Is every p-hyp<strong>on</strong>ormal Toeplitz operator hyp<strong>on</strong>ormal <br />

We c<strong>on</strong>clude with a problem of hyp<strong>on</strong>ormal operators with finite rank self-commutators.<br />

In general it is quite difficult to determine the subnormality of an operator by definiti<strong>on</strong>.<br />

An alternative descripti<strong>on</strong> of subnormality is given by the Bram-Halmos<br />

criteri<strong>on</strong>, which states that an operator T is subnormal if and <strong>on</strong>ly if<br />

∑<br />

(T i x j , T j x i ) ≥ 0<br />

i,j<br />

for all finite collecti<strong>on</strong>s x 0 , x 1 , · · · , x k ∈ H ([Bra], [C<strong>on</strong>1, II.1.9]). It is easy to see that<br />

this is equivalent to the following positivity test:<br />

⎡<br />

I T ∗ · · · T ∗k ⎤<br />

T T ∗ T · · · T ∗k T<br />

⎢<br />

⎣<br />

.<br />

. . ..<br />

. ⎥ ≥ 0 (all k ≥ 1 ). (3.6)<br />

⎦<br />

T k T ∗ T k · · · T ∗k T k<br />

C<strong>on</strong>diti<strong>on</strong> (3.6) provides a measure of the gap between hyp<strong>on</strong>ormality and subnormality.<br />

An operator T ∈ B(H) is called k-hyp<strong>on</strong>ormal if the (k + 1) × (k + 1) operator<br />

matrix in (3.6) is positive; the Bram-Halmos criteri<strong>on</strong> can be then rephrased as saying<br />

that T is subnormal if and <strong>on</strong>ly if T is k-hyp<strong>on</strong>ormal for every k ≥ 1 ([CMX]). It<br />

now seems to be interesting to c<strong>on</strong>sider the following problem:<br />

Which 2-hyp<strong>on</strong>ormal operators are subnormal (3.7)<br />

The first inquiry involves the self-commutator. The self-commutator of an operator<br />

plays an important role in the study of subnormality. B. Morrel [Mor] showed that<br />

101


CHAPTER 3.<br />

HYPONORMAL AND SUBNORMAL THEORY<br />

a pure subnormal operator with rank-<strong>on</strong>e self-commutator (pure means having no<br />

normal summand) is unitarily equivalent to a linear functi<strong>on</strong> of the unilateral shift.<br />

Morrel’s theorem can be essentially stated (also see [C<strong>on</strong>2, p.162]) that if<br />

⎧<br />

⎪⎨ (i) T is hyp<strong>on</strong>ormal;<br />

(ii) [T<br />

⎪⎩<br />

∗ , T ] is of rank-<strong>on</strong>e; and<br />

(3.8)<br />

(iii) ker [T ∗ , T ] is invariant for T ,<br />

then T −β is quasinormal for some β ∈ C. Now remember that every pure quasinormal<br />

operator is unitarily equivalent to U ⊗ P , where U is the unilateral shift and P is<br />

a positive operator with trivial kernel. Thus if [T ∗ , T ] is of rank-<strong>on</strong>e (and hence so<br />

is [(T − β) ∗ , (T − β)]), we must have P ∼ = α (≠ 0) ∈ C, so that T − β ∼ = α U, or<br />

T ∼ = α U + β. It would be interesting (in the sense of giving a simple sufficiency for<br />

the subnormality) to note that Morrel’s theorem gives that if T satisfies the c<strong>on</strong>diti<strong>on</strong><br />

(3.8) then T is subnormal. On the other hand, it was shown ([CuL2, Lemma 2.2])<br />

that if T is 2-hyp<strong>on</strong>ormal then T (ker [T ∗ , T ]) ⊆ ker [T ∗ , T ]. Therefore by Morrel’s<br />

theorem, we can see that<br />

every 2-hyp<strong>on</strong>ormal operator with rank-<strong>on</strong>e self-commutator is subnormal. (3.9)<br />

On the other hand, M. Putinar [Pu4] gave a matricial model for the hyp<strong>on</strong>ormal<br />

operator T ∈ B(H) with finite rank self-commutator, in the cases where<br />

∞∨<br />

H 0 := T ∗k( ran [T ∗ , T ] ) ∞∨<br />

has finite dimensi<strong>on</strong> d and H = T n H 0 .<br />

k=0<br />

In this case, if we write<br />

H n := G n ⊖ G n−1 (n ≥ 1) and G n :=<br />

n=0<br />

n∨<br />

T k H 0 (n ≥ 0),<br />

then T has the following two-diag<strong>on</strong>al structure relative to the decompositi<strong>on</strong> H =<br />

H 0 ⊕ H 1 ⊕ · · · :<br />

⎡<br />

⎤<br />

B 0 0 0 0 · · ·<br />

A 0 B 1 0 0 · · ·<br />

T =<br />

0 A 1 B 2 0 · · ·<br />

, (3.10)<br />

⎢<br />

⎣<br />

0 0 A 2 B 3 · · · ⎥<br />

⎦<br />

.<br />

. . . . ..<br />

k=0<br />

where<br />

dim (H n ) = dim (H n+1 ) = d (n ≥ 0);<br />

⎧⎪ ⎨<br />

[T ∗ , T ] = ([B0, ∗ B 0 ] + A ∗ 0A 0 ) ⊕ 0 ∞ ;<br />

⎪<br />

[Bn+1, ∗ B n+1 ] + A ∗ n+1A n+1 = A n A ∗ n (n ≥ 0);<br />

⎩<br />

A ∗ nB n+1 = B n A ∗ n (n ≥ 0).<br />

(3.11)<br />

We will refer the operator (3.10) to the Putinar’s matricial model of rank d. This<br />

model was also introduced in [GuP, Pu1, Xi3, Ya1], and etc.<br />

102


CHAPTER 3.<br />

HYPONORMAL AND SUBNORMAL THEORY<br />

We here review a few essential facts c<strong>on</strong>cerning weak subnormality. Note that<br />

the operator [ ] T is subnormal if and <strong>on</strong>ly if there exist operators A and B such that<br />

T A<br />

̂T := is normal, i.e.,<br />

0 B<br />

⎧<br />

⎪⎨ [T ∗ , T ] := T ∗ T − T T ∗ = AA ∗<br />

A<br />

⎪⎩<br />

∗ T = BA ∗<br />

[B ∗ , B] + A ∗ A = 0.<br />

(3.12)<br />

The operator ̂T is called a normal extensi<strong>on</strong> of T . We also say that ̂T in B(K)<br />

is a minimal normal extensi<strong>on</strong> (briefly, m.n.e.) of T if K has no proper subspace<br />

c<strong>on</strong>taining H to which the restricti<strong>on</strong> of ̂T is also a normal extensi<strong>on</strong> of T . It is<br />

known that<br />

̂T = m.n.e.(T ) ⇐⇒ K = ∨ { ̂T ∗n h : h ∈ H, n ≥ 0 } ,<br />

and the m.n.e.(T ) is unique.<br />

An operator T ∈ B(H) is said to be weakly subnormal if there exist operators<br />

A ∈ B(H ′ , H) and B ∈ B(H ′ ) such that the first two c<strong>on</strong>diti<strong>on</strong>s in (3.12) hold:<br />

[T ∗ , T ] = AA ∗ and A ∗ T = BA ∗ , (3.13)<br />

or equivalently, there is an extensi<strong>on</strong> ̂T of T such that ̂T ∗ ̂T f = ̂T ̂T ∗ f for all f ∈ H.<br />

The operator ̂T is called a partially normal extensi<strong>on</strong> (briefly, p.n.e.) of T . We also<br />

say that ̂T in B(K) is a minimal partially normal extensi<strong>on</strong> (briefly, m.p.n.e.) of<br />

T if K has no proper subspace c<strong>on</strong>taining H to which the restricti<strong>on</strong> of ̂T is also a<br />

partially normal extensi<strong>on</strong> of T . It is known ([CuL2, Lemma 2.5 and Corollary 2.7])<br />

that<br />

̂T = m.p.n.e.(T ) ⇐⇒ K = ∨ { ̂T ∗n h : h ∈ H, n = 0, 1 } ,<br />

and the m.p.n.e.(T ) is unique. For c<strong>on</strong>venience, if ̂T = m.p.n.e. (T ) is also weakly<br />

subnormal then we write ̂T (2) := ̂T and more generally, ̂T (n) := ̂̂T (n−1)<br />

, which will<br />

be called the n-th minimal partially normal extensi<strong>on</strong> of T . It was ([CuL2], [CJP])<br />

shown that<br />

2-hyp<strong>on</strong>ormal =⇒ weakly subnormal =⇒ hyp<strong>on</strong>ormal (3.14)<br />

and the c<strong>on</strong>verses of both implicati<strong>on</strong>s in (3.14) are not true in general.<br />

([CuL2]) known that<br />

It was<br />

T is weakly subnormal =⇒ T (ker [T ∗ , T ]) ⊆ ker [T ∗ , T ] (3.15)<br />

and it was ([CJP]) known that if ̂T := m.p.n.e.(T ) then for any k ≥ 1,<br />

T is (k + 1)-hyp<strong>on</strong>ormal ⇐⇒ T is weakly subnormal and ̂T is k-hyp<strong>on</strong>ormal.<br />

(3.16)<br />

103


CHAPTER 3.<br />

HYPONORMAL AND SUBNORMAL THEORY<br />

So, in particular, <strong>on</strong>e can see that if T is subnormal then ̂T is subnormal. It is worth<br />

to noticing that in view of (3.14) and (3.15), Morrel’s theorem gives that every weakly<br />

subnormal operator with rank-<strong>on</strong>e self-commutator is subnormal.<br />

We now have<br />

Theorem 3.5.1. Let T ∈ B(H). If<br />

(i) T is a pure hyp<strong>on</strong>ormal operator;<br />

(ii) [T ∗ , T ] is of rank-two; and<br />

(iii) ker [T ∗ , T ] is invariant for T ,<br />

then the following hold:<br />

1. If T | ker [T ∗ ,T ] has the rank-<strong>on</strong>e self-commutator then T is subnormal;<br />

2. If T | ker [T ∗ ,T ] has the rank-two self-commutator then T is either a subnormal<br />

operator or the Putinar’s matricial model (3.10) of rank two.<br />

Proof. See [LeL3].<br />

Since the operator (3.10) can be c<strong>on</strong>structed from the pair of matrices {A 0 , B 0 },<br />

we know that the pair {A 0 , B 0 } is a complete set of unitary invariants for the operator<br />

(3.10). Many authors used the following Xia’s unitary invariants {Λ, C} to describe<br />

pure subnormal operators with finite rank self-commutators:<br />

C<strong>on</strong>sequently,<br />

Λ := ( T ∗ | ran [T ∗ ,T ]) ∗<br />

and C := [T ∗ , T ]| ran [T ∗ ,T ].<br />

Λ = B 0 and C = [B ∗ 0, B 0 ] + A 2 0.<br />

We know that given Λ and C (or equivalently, A 0 and B 0 ) corresp<strong>on</strong>ding to a pure<br />

subnormal operator we can rec<strong>on</strong>struct T . Now the following questi<strong>on</strong> naturally<br />

arises: “what are the restricti<strong>on</strong>s <strong>on</strong> matrices A 0 and B 0 such that they represent a<br />

subnormal operator ” In the cases where A 0 and B 0 operate <strong>on</strong> a finite dimensi<strong>on</strong>al<br />

Hilbert space, D. Yakubovich [Ya1] showed that such a descripti<strong>on</strong> can be given in<br />

terms of a topological property of a certain algebraic curve, associated with A 0 and<br />

B 0 . However there is a subtle difference between Yakubovich’s criteri<strong>on</strong> and the<br />

Putinar’s model operator (3.10). In fact, in some sense, Yakubovich gave c<strong>on</strong>diti<strong>on</strong>s<br />

<strong>on</strong> A 0 and B 0 such that the operator (3.10) can be c<strong>on</strong>structed so that the c<strong>on</strong>diti<strong>on</strong><br />

(3.11) is satisfied. By comparis<strong>on</strong>, the Putinar’s model operator (3.10) was already<br />

c<strong>on</strong>structed so that it satisfies the c<strong>on</strong>diti<strong>on</strong> (3.11). Thus we would guess that if the<br />

operator (3.10) can be c<strong>on</strong>structed so that the c<strong>on</strong>diti<strong>on</strong> (3.11) is satisfied then two<br />

matrices {A 0 , B 0 } in (2.8) must satisfy the Yakubovich’s criteri<strong>on</strong>. In this viewpoint,<br />

we have the following:<br />

C<strong>on</strong>jecture 3.3. The Putinar’s matricial model (3.10) of rank two is subnormal.<br />

An affirmative answer to the c<strong>on</strong>jecture would show that if T is a hyp<strong>on</strong>ormal<br />

operator with rank-two self-commutator and satisfying that ker [T ∗ , T ] is invariant<br />

for T then T is subnormal. Hence, in particular, <strong>on</strong>e could obtain: Every weakly<br />

subnormal operator with rank-two self-commutator is subnormal.<br />

104


Chapter 4<br />

Weighted Shifts<br />

4.1 Berger’s theorem<br />

Recall that given a bounded sequence of positive numbers α : α 0 , α 1 , α 2 , · · · (called<br />

weights), the (unilateral) weighted shift W α associated with α is the operator l 2 (Z + )<br />

defined by<br />

W α e n = α n e n+1 (n ≥ 0),<br />

where {e n } ∞ n=0 is the can<strong>on</strong>ical orth<strong>on</strong>ormal basis for l 2 . It is straightforward to check<br />

that<br />

W α is compact ⇐⇒ α n → 0.<br />

Indeed, W α = UD, where U is the unilateral shift and D is the diag<strong>on</strong>al operator<br />

whose diag<strong>on</strong>al entries are α n .<br />

We observe:<br />

Propositi<strong>on</strong> 4.1.1. If T ≡ W α is a weighted shift and ω ∈ ∂D then T ∼ = ωT .<br />

Proof. If V e n := ω n e n for all n then V T V ∗ = ωT .<br />

As a c<strong>on</strong>sequence of Propositi<strong>on</strong> 4.1.1, we can see that the spectrum of a weighted<br />

shift must be a circular symmetry:<br />

Indeed we have:<br />

σ(W α ) = σ(ωW α ) = ωσ(W α ).<br />

Theorem 4.1.2. If T ≡ W α is a weighted shift with weight sequence α ≡ {α n } ∞ n=0<br />

such that α n → α + then<br />

(i) σ p (T ) = ∅;<br />

(ii) σ(T ) = {λ : |λ| ≤ α + };<br />

(iii) σ e (T ) = {λ : |λ| = α + };<br />

(iv) |λ| < α + ⇒ ind (T − λ) = −1.<br />

105


CHAPTER 4.<br />

WEIGHTED SHIFTS<br />

Proof. The asserti<strong>on</strong> (i) is straightforward. For the other asserti<strong>on</strong>s, observe that if<br />

α + = 0 then T is compact and quasinilpotent. If instead α + > 0 then T − α + U<br />

(U :=the unilateral shift) is a weighted shift whose weight sequence c<strong>on</strong>verges to 0.<br />

Hence T − α + U is a compact and hence<br />

σ e (T ) = σ e (α + U) = α + σ e (U) = {λ : |λ| = α + }.<br />

If |λ| < α + then T − λ is Fredholm and<br />

index (T − λ) = index (α + U − λ) = −1.<br />

In particular, {λ : |λ| ≤ α + } ⊂ σ(T ). By the asserti<strong>on</strong> (i), we can c<strong>on</strong>clude that<br />

σ(T ) = {λ : |λ| ≤ α + }.<br />

Theorem 4.1.3. If T ≡ W α is a weighted shift with weight sequence α ≡ {α n } ∞ n=0<br />

then<br />

⎡<br />

⎤<br />

α0<br />

2 [T ∗ α1 2 − α 2 0<br />

, T ] = ⎢<br />

⎣<br />

α2 2 − α1<br />

2 ⎥<br />

⎦<br />

. ..<br />

Proof. From a straightforward calculati<strong>on</strong>.<br />

The moments of W α are defined by<br />

β 0 := 1, β n+1 = α 0 · · · α n ,<br />

but we reserve this term for the sequence γ n := βn.<br />

2<br />

Theorem 4.1.4. (Berger’s theorem) Let T ≡ W α be a weighted shift with weight<br />

sequence α ≡ {α n } and define the moment of T by<br />

γ 0 := 1 and γ n := α 2 0α 2 1 · · · α 2 n−1 (n ≥ 1).<br />

Then T is subnormal if and <strong>on</strong>ly if there exists a probability measure ν <strong>on</strong> [ 0, ∥T ∥ 2]<br />

such that<br />

∫<br />

γ n = t n dν(t) (t ≥ 1). (4.1)<br />

[0,∥T ∥ 2 ]<br />

Proof. (⇒) Note that T is cyclic. So if T is subnormal then T ∼ = S µ , i.e., there is an<br />

isomorphism U : L 2 (µ) −→ P 2 (µ) such that<br />

Ue 0 = 1 and UT U −1 = S µ .<br />

Observe T n e 0 = √ γ n e n for all n. Also U(T n e 0 ) = SµUe n 0 = Sµ1 n = z n . So<br />

∫<br />

∫<br />

∫<br />

|z| 2n dµ = |UT n e 0 | 2 dµ = |U( √ ∫<br />

γ n e n )| 2 dµ = γ n |Ue n | 2 dµ = γ n ∥Ue n ∥ = γ n .<br />

106


CHAPTER 4.<br />

WEIGHTED SHIFTS<br />

If ν is defined <strong>on</strong> [0, ||T || 2 ] by<br />

ν(∆) := µ ( {z : |z| 2 ∈ ∆} ) ,<br />

then ν is a probability measure and γ n = ∫ t n dν(t).<br />

(⇐) If ν is the measure satisfying (4.1), define the measure µ by dµ(re iθ ) =<br />

1<br />

2π dθdµ(r). Then we can see that T ∼ = S µ .<br />

Example 4.1.5. (a) The Bergman shift B α is the weighted shift with weight sequence<br />

α ≡ {α n } given by<br />

√<br />

n + 1<br />

α n = (n ≥ 0).<br />

n + 2<br />

Then B α is subnormal: indeed,<br />

γ n := α 2 0α 2 1 · · · α 2 n−1 = 1 2 · 2<br />

3 · · · n<br />

n + 1 = 1<br />

n + 1<br />

and if we define µ(t) = t, i.e., dµ = dt then<br />

∫ 1<br />

0<br />

t n dµ(t) = 1<br />

n + 1 = γ n.<br />

(b) If α n : β, 1, 1, 1, · · · then W α is subnormal: indeed γ n = β 2 and if we define<br />

dµ = β 2 δ 1 + (1 − β 2 )δ 0 then ∫ 1<br />

0 tn dµ = β 2 = γ n .<br />

Remark. Recall that the Bergman space A(D) for D is defined by<br />

{<br />

∫<br />

}<br />

A(D) := f : D → C : f is analytic with |f| 2 dµ < ∞ .<br />

Then the orth<strong>on</strong>ormal basis for A(D) is given by {e n ≡ √ n + 1 z n : n = 0, 1, 2, · · · }<br />

with dµ = 1 π<br />

dA. The Bergman operator T : A(D) → A(D) is defined by<br />

T f = zf.<br />

In this case the matrix (α ij ) of the Bergman operator T with respect to the basis<br />

{e n ≡ √ n + 1 z n : n = 0, 1, 2, · · · } is given by<br />

α ij = ⟨T e j , e i ⟩<br />

= ⟨T √ j + 1 z j , √ i + 1 z i ⟩<br />

= ⟨ √ j + 1 z j+1 , √ i + 1 z i ⟩<br />

= √ ∫<br />

(j + 1)(i + 1) z j+1 z i dµ<br />

D<br />

∫ 2π ∫ 1<br />

= √ (j + 1)(i + 1) 1 π 0<br />

{√<br />

j+1<br />

=<br />

j+2<br />

(i = j + 1)<br />

0 (i ≠ j + 1) :<br />

107<br />

0<br />

D<br />

r j+1+i e i(j+1−i)θ · rdr dθ


CHAPTER 4.<br />

WEIGHTED SHIFTS<br />

therefore<br />

⎡<br />

⎤<br />

√<br />

0<br />

1<br />

2<br />

0<br />

√<br />

2<br />

T =<br />

3<br />

0<br />

√<br />

.<br />

3<br />

⎢<br />

⎣<br />

4<br />

0 ⎥<br />

⎦<br />

. .. . ..<br />

108


CHAPTER 4.<br />

WEIGHTED SHIFTS<br />

4.2 k-Hyp<strong>on</strong>ormality<br />

Given an n-tuple T = (T 1 , . . . , T n ) of operators acting <strong>on</strong> H, we let<br />

⎡<br />

⎤<br />

[T1 ∗ , T 1 ] [T2 ∗ , T 1 ] · · · [Tn, ∗ T 1 ]<br />

[T ∗ [T1 ∗ , T 2 ] [T2 ∗ , T 2 ] · · · [Tn, ∗ T 2 ]<br />

, T] ≡ ⎢<br />

⎣<br />

.<br />

.<br />

.<br />

⎥<br />

. ⎦ .<br />

[T1 ∗ , T k ] [T2 ∗ , T k ] · · · [Tn, ∗ T n ]<br />

By analogy with the case n = 1, we shall say that T is (jointly) hyp<strong>on</strong>ormal if<br />

[T ∗ , T] ≥ 0.<br />

An operator T ∈ B(H) is called k-hyp<strong>on</strong>ormal if (1, T, T 2 , · · · , T k ) is jointly hyp<strong>on</strong>ormal,<br />

i.e.,<br />

( ) k<br />

M k (T ) ≡ [T ∗j , T i ]<br />

i,j=1<br />

⎡<br />

[T ∗ , T ] [T ∗2 , T ] · · · [T ∗k ⎤<br />

, T ]<br />

[T ∗ , T 2 ] [T ∗2 , T 2 ] · · · [T ∗k , T 2 ]<br />

= ⎢<br />

⎥<br />

⎣<br />

.<br />

.<br />

.<br />

. ⎦ ≥ 0<br />

[T ∗ , T k ] [T ∗2 , T k ] · · · [T ∗k , T k ]<br />

An applicati<strong>on</strong> of Choleski algorithm for operator matrices shows that M k (T ) ≥ 0 is<br />

equivalent to the positivity of the following matrix<br />

⎡<br />

1 T ∗ · · · T ∗k ⎤<br />

T T ∗ T · · · T ∗k T<br />

⎢<br />

⎥<br />

⎣<br />

.<br />

.<br />

.<br />

. ⎦ .<br />

T k T ∗ T k · · · T ∗k T k<br />

The Bram-Halmos criteri<strong>on</strong> can be then rephrased as saying that<br />

T is subnormal ⇐⇒ T is k-hyp<strong>on</strong>ormal for every k ≥ 1.<br />

Recall ([Ath],[CMX],[CoS]) that T is called weakly k-hyp<strong>on</strong>ormal if<br />

⎧<br />

⎫<br />

⎨ k∑<br />

⎬<br />

LS(T, T 2 , · · · , T k ) := α<br />

⎩ j T j : α = (α 1 , · · · , α k ) ∈ C k ⎭<br />

j=1<br />

c<strong>on</strong>sists entirely of hyp<strong>on</strong>ormal operators, or equivalently, M k (T ) is weakly positive,<br />

i.e.,<br />

⎡ ⎤ ⎡ ⎤<br />

⟨ λ 1 x λ 1 x ⟩<br />

⎢<br />

M k (T ) ⎣<br />

⎥ ⎢<br />

. ⎦ , ⎣<br />

⎥<br />

. ⎦ ≥ 0 ∀ λ 1 , · · · , λ k ∈ C.<br />

λ k x λ k x<br />

109


CHAPTER 4.<br />

WEIGHTED SHIFTS<br />

Observe that ⟨[<br />

(λ1 T + · · · + λ k T k ) ∗ , (λ 1 T + · · · + λ k T k ) ] x, x ⟩<br />

⎡<br />

⟨<br />

[T ∗ , T ] [T ∗2 , T ] · · · [T ∗k ⎤ ⎡ ⎤ ⎡ ⎤<br />

, T ] λ 1 x λ 1 x<br />

⟩<br />

[T ∗ , T 2 ] [T ∗2 , T 2 ] · · · [T ∗k , T 2 ]<br />

λ 2 x<br />

= ⎢<br />

⎥ ⎢<br />

⎣ . . . . ⎦ ⎣<br />

⎥<br />

. ⎦ , λ 2 x<br />

⎢<br />

⎣<br />

⎥<br />

. ⎦<br />

[T ∗ , T k ] [T ∗2 , T k ] · · · [T ∗k , T k ] λ k x λ k x<br />

(4.2)<br />

If k = 2 then T is said to be quadratically hyp<strong>on</strong>ormal. If k = 3 then T is said<br />

to be cubically hyp<strong>on</strong>ormal. Also T is said to be polynomially hyp<strong>on</strong>ormal if p(T ) is<br />

hyp<strong>on</strong>ormal for every polynomial p ∈ C[z].<br />

Evidently, by (4.2)<br />

k-hyp<strong>on</strong>ormal =⇒ weakly k-hyp<strong>on</strong>ormal.<br />

The classes of (weakly) k-hyp<strong>on</strong>ormal operators have been studied in an attempt to<br />

bridge the gap between subnormality and hyp<strong>on</strong>ormality ([Cu1, Cu2, CuF1, CuF2,<br />

CuF3, CLL, CuL1, CuL2, CuL3, CMX, DPY, McCP]). The study of this gap has been<br />

<strong>on</strong>ly partially successful. For example, such a gap is not yet well described for Toeplitz<br />

operators <strong>on</strong> the Hardy space of the unit circle. For weighted shifts, positive results<br />

appear in [Cu1] and [CuF3], although no c<strong>on</strong>crete example of a weighted shift which<br />

is polynomially hyp<strong>on</strong>ormal but not subnormal has yet been found (the existence of<br />

such weighted shifts was established in [CP1] and [CP2]).<br />

Theorem 4.2.1. Let T ≡ W α be a weighted shift with weight sequence α ≡ {α n } ∞ 0 .<br />

The following are equivalent:<br />

(a) T is k-hyp<strong>on</strong>ormal;<br />

(b) For every n ≥ 0, the Hankel matrix<br />

⎡<br />

⎤<br />

γ n γ n+1 · · · γ n+k+1<br />

(γ n+i+j ) k γ n+1 γ n+2 · · · γ n+k+2<br />

i,j=0 ≡ ⎢<br />

⎥<br />

⎣ . . . . ⎦<br />

γ n+k+1 γ n+k+2 · · · γ n+2k+2<br />

is positive.<br />

Proof. [Cu1, Theorem 4]<br />

Lemma 4.2.2. Let T = (T 1 , T 2 ) be a pair of operators <strong>on</strong> H. Then T is (jointly)<br />

hyp<strong>on</strong>ormal if and <strong>on</strong>ly if<br />

(i) T 1 is hyp<strong>on</strong>ormal<br />

(ii) T 2 is hyp<strong>on</strong>ormal<br />

(iii) |⟨[T ∗ 2 , T 1 ]y, x⟩| 2 ≤ ⟨[T ∗ 1 , T 1 ]x, x⟩⟨[T ∗ 2 , T 2 ]y, y⟩ (for any x, y ∈ H).<br />

110


CHAPTER 4.<br />

WEIGHTED SHIFTS<br />

Proof. [T ∗ , T] ≥ 0 ⇐⇒<br />

Thus<br />

[T ∗ , T ∗ ] ≥ 0 ⇐⇒<br />

⟨ ( x<br />

[T ∗ , T ∗ ]<br />

ty<br />

) ( x<br />

,<br />

ty<br />

⟨[ [T<br />

∗<br />

1 , T 1 ] [T ∗ 2 , T 1 ]<br />

[T ∗ 1 , T 2 ] [T ∗ 2 , T 2 ]] ( x<br />

ty<br />

)⟩<br />

≥ 0 for any x, y ∈ H and t ∈ R.<br />

) ( x<br />

,<br />

ty<br />

)⟩<br />

≥ 0<br />

⇐⇒⟨[T ∗ 1 , T 1 ]x, x⟩ + t 2 ⟨[T ∗ 2 , T 2 ]y, y⟩ + 2tRe ⟨[T ∗ 2 , T 1 ]y, x⟩ ≥ 0 (†)<br />

C<strong>on</strong>versely if (*) holds then<br />

which implies (†) holds.<br />

=⇒If T 1 and T 2 are hyp<strong>on</strong>ormal then<br />

t 2 ⟨[T ∗ 2 , T 2 ]y, y⟩ + 2t ∣ ∣⟨[T ∗ 2 , T 1 ]y, x⟩ ∣ ∣ + ⟨[T ∗ 1 , T 1 ]x, x⟩ ≥ 0<br />

=⇒D/4 ≡ |⟨[T ∗ 2 , T 1 ]y, x⟩| 2 − ⟨[T ∗ 1 , T 1 ]x, x⟩⟨[T ∗ 2 , T 2 ]y, y⟩ ≤ 0<br />

=⇒ ∣ ∣⟨[T ∗ 2 , T 1 ]y, x⟩ ∣ ∣ 2 ≤ ⟨[T ∗ 1 , T 1 ]x, x⟩⟨[T ∗ 2 , T 2 ]y, y⟩ (∗)<br />

Re ⟨[T ∗ 2 , T 1 ]y, x⟩ 2 ≤ ⟨[T ∗ 1 , T 1 ]x, x⟩⟨[T ∗ 2 , T 2 ]y, y⟩,<br />

Corollary 4.2.3. Let T = (T 1 , T 2 ) be a pair of operators <strong>on</strong> H. Then T is hyp<strong>on</strong>ormal<br />

if and <strong>on</strong>ly if T 1 and T 2 are hyp<strong>on</strong>ormal and<br />

[T2 ∗ , T 1 ] = [T1 ∗ , T 1 ] 1 2 D[T<br />

∗<br />

2 , T 2 ] 1 2<br />

for some c<strong>on</strong>tracti<strong>on</strong> D.<br />

Proof. This follows from a theorem of Smul’jan [Smu]:<br />

[ ] A B<br />

B ∗ ≥ 0 ⇐⇒ A ≥ 0, C ≥ 0 and B = √ AD √ C for some c<strong>on</strong>tracti<strong>on</strong> D.<br />

C<br />

Corollary 4.2.4. Let T ≡ W α be a weighted shift with weight sequence α :<br />

α 1 ≤ α 2 ≤ · · · . Then the following are equivalent:<br />

α 0 ≤<br />

(i) T is 2-hyp<strong>on</strong>ormal;<br />

(<br />

(ii) αn+1 2 α<br />

2<br />

n+2 − αn) 2 2 ( (<br />

≤ α<br />

2<br />

n+1 − αn) 2 α<br />

2<br />

n+2 αn+3 2 − αnαn+1) 2 2 for any n ≥ 0;<br />

(<br />

(iii) αn 2 α<br />

2<br />

n+2 − αn+1) 2 2 ( (<br />

≤ α<br />

2<br />

n+2 α<br />

2<br />

n+1 − αn) 2 α<br />

2<br />

n+3 − αn+2) 2 for any n ≥ 0.<br />

Proof. By Corollary 4.2.3,<br />

(T, T 2 ) hyp<strong>on</strong>ormal ⇐⇒ [T ∗2 , T ] = [T ∗ , T ] 1 2 E[T ∗2 , T 2 ] 1 2 for some c<strong>on</strong>tracti<strong>on</strong> E.<br />

111


CHAPTER 4.<br />

WEIGHTED SHIFTS<br />

Observe that [T ∗ , T ] and [T ∗2 , T 2 ] are diag<strong>on</strong>al and that [T ∗2 , T ] is a backward<br />

weighted shift. It follows that E is a backward weighted shift. So it suffices to<br />

check that (n, n + 1)-entries of E. Now,<br />

⟩<br />

⟩<br />

⟨[T ∗2 , T ]e n+1 , e n =<br />

⟨[T ∗ , T ] 1 2 E[T ∗2 , T 2 ] 1 2 en+1 , e n<br />

⟩<br />

=<br />

⟨E[T ∗2 , T 2 ] 1 2 en+1 , [T ∗ , T ] 1 2 en<br />

⟩<br />

⟩ ⟩<br />

=<br />

⟨⟨E[T ∗2 , T 2 ] 1 2 en+1 , e n+1 e n+1 ,<br />

⟨[T ∗ , T ] 1 2 en , e n e n<br />

⟩ ⟩<br />

=<br />

⟨[T ∗2 , T 2 ] 1 2 en+1 , e n+1<br />

⟨[T ∗ , T ] 1 2 en , e n ⟨Ee n+1 , e n+1 ⟩ .<br />

Thus we can see that such a c<strong>on</strong>tracti<strong>on</strong> E exists if and <strong>on</strong>ly if<br />

⟨<br />

⟩∣ ∣ [T ∗2 ∣∣<br />

2 ⟨<br />

⟩<br />

, T ]e n+1 , e n ≤ ⟨[T ∗ , T ]e n , e n ⟩ [T ∗2 , T 2 ]e n+1 , e ∀<br />

n+1 n ≥ 0<br />

which gives<br />

αn+1 2 (<br />

α<br />

2<br />

n+2 − αn) 2 2 (<br />

≤ α<br />

2<br />

n+1 − αn) 2 (<br />

α<br />

2<br />

n+2 αn+3 2 − αnα 2 n+1<br />

2 )<br />

(α 2 1α 0 ) 2 ≤ α 2 0α 2 1α 2 2 (this holds automatically since α 1 ≤ α 2 ),<br />

which gives (i) ⇔ (ii).<br />

Finally, (iii) is just (ii) suitably rewritten.<br />

112


CHAPTER 4.<br />

WEIGHTED SHIFTS<br />

4.3 The Propagati<strong>on</strong><br />

We introduce:<br />

Definiti<strong>on</strong> 4.3.1. If α 0 < α 1 = α 2 = α 3 = · · · then (α n ) is said to be flat.<br />

Propositi<strong>on</strong> 4.3.2. If T ≡ W α is a weighted shift with flat weights then T is subnormal.<br />

Proof. Without loss of generality we may assume that<br />

(α n ) : α, 1, 1, 1, · · · .<br />

Then γ n = α 2 for any n = 0, 1, 2, · · · . Put dµ = α 2 δ 1 + (1 − α 2 )δ 0 , where δ k = the<br />

point mass at k. Then ∫ 1<br />

0 tn dµ = (1 − α 2 ) · 0 + α 2 · 1 = α 2 = γ n . Therefore, T is<br />

subnormal.<br />

Theorem 4.3.3. Let T be a weighted shift with weight sequence {α n } ∞ n=0.<br />

(i) [Sta3] Let T be subnormal. Then<br />

(ii) [Cu2] Let W α be 2-hyp<strong>on</strong>ormal. Then<br />

Proof. (ii) ⇒ (i): Obvious.<br />

(ii) Immediate from Corollary 4.2.4(ii).<br />

α n = α n+1 for some n ≥ 0 =⇒ α is flat<br />

α n = α n+1 for some n ≥ 0 =⇒ α is flat.<br />

Lemma 4.3.4. Let T be a weighted shift whose restricti<strong>on</strong> to ∨ {e 1 , e 2 , · · · } is subnormal<br />

with associated measure µ. Then T is subnormal if and <strong>on</strong>ly if<br />

(i) 1 t ∈ L1 (µ), i.e., ∫ 1<br />

t<br />

dµ < ∞;<br />

(ii) α0 2 ≤ (∥ ∥ 1∥ −1<br />

L 1)<br />

.<br />

t<br />

In particular, T is never subnormal when µ(α) > 0.<br />

Proof. Let S := T | ∨ {e 1,e 2,··· }. Then S has weights α k (S) := α k+1 (k ≥ 0). So the<br />

corresp<strong>on</strong>ding “β numbers” are related by the equati<strong>on</strong><br />

β k (S) = α 1 · · · α k = β k+1<br />

α 0<br />

(k = 1, 2, · · · ).<br />

113


CHAPTER 4.<br />

WEIGHTED SHIFTS<br />

Since β k (S) 2 = ∫ t 2k dµ, we see that<br />

T is subnormal ⇐⇒ ∃ a probability measure ν <strong>on</strong> [0, ∥T ∥] such that<br />

1<br />

α 2 0<br />

∫<br />

t 2(k+1) dν(t) = β2 k+1<br />

α 2 0<br />

∫<br />

= β k (S) 2 =<br />

t 2k dµ (k ≥ 0).<br />

So t 2 dν = α 2 0dµ. Thus<br />

T is subnormal ⇐⇒ dν = λδ 0 + α2 0<br />

dµ for some λ ≥ 0.<br />

t<br />

Thus<br />

{ ∫<br />

α<br />

2 1<br />

T is subnormal ⇐⇒ 0 t dµ ≤ 1 or 1 t ∈ L1 (µ)<br />

∥ 1∥ ≤ 1.<br />

α 2 0<br />

t<br />

Theorem 4.3.5. For x > 0, let T x be the weighted shift whose weight sequence is<br />

given by<br />

(a) T x is subnormal ⇐⇒ 0 < x ≤<br />

x,<br />

√<br />

2<br />

3 , √<br />

3<br />

4 , √<br />

4<br />

5 , √<br />

5<br />

6 , · · ·<br />

√<br />

1<br />

2<br />

(b) T x is k−hyp<strong>on</strong>ormal ⇐⇒ 0 < x ≤ √ k+1<br />

2k(k+2)<br />

In particular, T x is 2-hyp<strong>on</strong>ormal ⇐⇒ 0 < x ≤ 3 4<br />

√<br />

2<br />

(c) T x is quadratically hyp<strong>on</strong>ormal ⇐⇒ 0 < x ≤<br />

3 .<br />

Proof. (a) T x | ∨ {e 1 ,e 2 ,··· } has measure dµ = 2tdt. So, 1 t ∈ L1 (µ) and<br />

1<br />

∥ t ∥ = 2 =⇒ x 2 ≤ 1 √<br />

1<br />

L1 (µ)<br />

2 =⇒ x ≤ 2 .<br />

(b) It is sufficient to show that<br />

⎡ ⎤<br />

γ 0 γ 1 γ 2<br />

⎣γ 1 γ 2 γ 3<br />

⎦ ≥ 0<br />

γ 2 γ 3 γ 4<br />

Since<br />

γ 0 = 1, γ 1 = x 2 , γ 2 = 2 3 x2 , γ 3 = 1 2 x2 , γ 4 = 2 5 x2 ,<br />

114


CHAPTER 4.<br />

WEIGHTED SHIFTS<br />

we have<br />

⎡<br />

⎡<br />

1 x 2 2 3 x2<br />

det ⎣ x 2 2 3 x2 1 2 x2 ⎦ = x 2 det ⎣<br />

⎤<br />

2<br />

3 x2 1 2 x2 2 5 x2<br />

1 2<br />

x<br />

1 2 3<br />

2 1<br />

1<br />

3<br />

2 1 2<br />

3 2 5<br />

= x 2 ( 1<br />

60x 2 − 4<br />

135<br />

⎤<br />

⎦<br />

)<br />

≥ 0 =⇒ x ≤ 3 4 .<br />

(c) See [Cu2]<br />

Let W α be a weighted shift with weights α ≡ {α n } ∞ n=0. For s ∈ C, write<br />

D(s) :=<br />

[ (Wα<br />

+ sWα<br />

2 ) ∗<br />

, Wα + sWα]<br />

2<br />

and let<br />

⎡<br />

⎤<br />

q 0 γ 0 0 · · · 0 0<br />

γ 0 q 1 γ 1 · · · 0 0<br />

0 γ 1 q 2 · · · 0 0<br />

D n (s) := P n D(s)P n =<br />

.<br />

⎢ . . . ..<br />

. ,<br />

.<br />

⎥<br />

⎣ 0 0 0 · · · q n−1 γ n−1<br />

⎦<br />

0 0 0 · · · γ n−1 q n<br />

where P n := the orthog<strong>on</strong>al projecti<strong>on</strong> <strong>on</strong>to the subspace spanned by {e 0 , · · · , e n },<br />

where<br />

⎧<br />

⎨<br />

⎩<br />

{<br />

qn := u n + |s| 2 v n<br />

γ n := s √ w n ,<br />

u n := α 2 n − α 2 n−1<br />

v n := α 2 nα 2 n+1 − α 2 n−1α 2 n−2<br />

w n = α 2 n(α 2 n+1 − α 2 n−1) 2 ,<br />

and, for notati<strong>on</strong>al c<strong>on</strong>venience, α −2 = α −1 = 0.<br />

Clearly,<br />

W α is quadratically hyp<strong>on</strong>ormal ⇐⇒ D n (s) ≥ 0 for any s ∈ C, for any n ≥ 0.<br />

Let d n (·) = detD n (·). Then d n satisfies the following 2-step recursive formula:<br />

d 0 = q 0 , d 1 = q 0 q 1 − |γ 0 | 2 , d n+2 = q n+2 d n+1 − |γ n+1 | 2 d n .<br />

If we let t := |s| 2 , we observe that d n is a polynomial in t of degree n + 1. If we write<br />

n+1<br />

∑<br />

d n ≡ c(n, i)t i ,<br />

i=0<br />

115


CHAPTER 4.<br />

WEIGHTED SHIFTS<br />

then c(n, i) satisfy a double-indexed recursive formula, i.e.,<br />

⎧<br />

c(1, 1) = u 1 v 0 + v 1 u 0 − w 0<br />

⎪⎨<br />

c(n, 0) = u 0 · · · u n<br />

c(n, n + 1) = v 0 · · · v n<br />

⎪⎩<br />

c(n + 2, i) = u n+2 c(n + 1, i) + v n+2 c(n + 1, i − 1) − w n+1 c(n, i − 1).<br />

Theorem 4.3.6. (Outer propagati<strong>on</strong>) Let T be a weighted shift with weight sequence<br />

{α n } ∞ n=0. If T is quadratically hyp<strong>on</strong>ormal then<br />

α n = α n+1 = α n+2 for some n =⇒ α n = α n+1 = α n+2 = α n+3 = · · · .<br />

Proof. We may assume that n = 0 and α 0 = α 1 = α 2 = 1. We want to show that<br />

α 3 = 1. A straightforward calculati<strong>on</strong> shows that<br />

d 0 = 1 + t<br />

d 1 = t 2<br />

d 2 = ( α 2 3 − 1 ) t 3<br />

d 3 = ( α 2 3 − 1 ) ( α 2 3α 2 4 − 1 ) t 4<br />

d 4 = q 4 d 3 − γ 2 3d 2<br />

So<br />

= [( α 2 4 − α 2 3)<br />

+ t<br />

(<br />

α<br />

2<br />

4 α 2 5 − α 2 3)] (<br />

α<br />

2<br />

3 − 1 ) ( α 2 3α 2 4 − 1 ) t 4 − tα 2 3<br />

(<br />

α<br />

2<br />

4 − 1 ) 2 (<br />

α<br />

2<br />

3 − 1 ) t 3 .<br />

which implies that α 3 = 1.<br />

d 4<br />

lim<br />

t→0 + t 4 = ( −α2 4 α<br />

2<br />

3 − 1 ) 3<br />

≥ 0,<br />

Theorem 4.3.7. (Inner Propagati<strong>on</strong>) Let T be a weighted shift with weight sequence<br />

{α n } ∞ n=0. If T is quadratically hyp<strong>on</strong>ormal then<br />

α n = α n+1 = α n+2 for some n =⇒ α 1 = · · · = α n .<br />

Proof. Withou loss of generality we may assume n = 2, i.e., α 2 = α 3 = α 4 = 1. We<br />

want to show that α 1 = 1. We c<strong>on</strong>sider d 3 . Now,<br />

Therefore<br />

d 3 (0) = q 3 (0)d 2 (0) = 0 since q 3 (0) = α 2 3 − α 2 2 = 0<br />

d ′ 3(0) = q ′ 3(0)d 2 (0) − α 2 2(α 2 3 − α 2 1) 2 α 1 (0) = · · · = 0<br />

d ′′<br />

3(0) = 2q ′ 3(0)d ′ 2(0) − 2(1 − α 2 1)α ′ 1(0) = · · · = −2α 4 0(1 − α 2 1) 3 .<br />

Since d 3 ≥ 0 (all t ≥ 0), it follows α 1 = 1.<br />

d 3 (t) = −α 4 0(1 − α 2 1) 3 t 2 + · · · .<br />

116


CHAPTER 4.<br />

WEIGHTED SHIFTS<br />

Theorem 4.3.8. (Propagati<strong>on</strong> of quadratic hyp<strong>on</strong>ormality) Let T be a weighted shift<br />

with weight sequence {α n } ∞ n=0. If T is quadratically hyp<strong>on</strong>ormal then<br />

α n = α n+1 for some n ≥ 1 =⇒ α is flat, i.e., α 1 = α 2 = · · · .<br />

Proof. Without loss of generality we may assume n = 1 and α 1 = α 2 = 1. We want<br />

to show that α 0 = 1 or α 3 = 1. Then we have<br />

so<br />

d 4 (t) = α 2 0α 2 4(α 2 0 − 1)(α 2 3 − 1) 3 t 2 + c(4, 3)t 3 + c(4, 4)t 4 + c(4, 5)t 5 ,<br />

d 4 (t)<br />

lim<br />

t→0+ t 2 = α0α 2 4(α 2 0 2 − 1)(α3 2 − 1) 3 ≥ 0.<br />

Thus α 0 = 1 or α 3 = 1, so that three equal weights are present.<br />

Remark. However the c<strong>on</strong>diti<strong>on</strong> “n ≥ 1” cannot be relaxed to “n ≥ 0”. For example,<br />

in view of Theorem 4.3.5, if<br />

α :<br />

√<br />

2<br />

3 , √<br />

2<br />

3 , √<br />

3<br />

4 , √<br />

4<br />

5 , √<br />

5<br />

6 , · · ·<br />

then W α is quadratically hyp<strong>on</strong>ormal but not subnormal. In fact, W α is not cubically<br />

hyp<strong>on</strong>ormal : if we let<br />

then<br />

c 5 (t) := det (P 5 [(W α + tW 2 α + t 2 W 3 α) ∗ , (W α + tW 2 α + t 2 W 3 α)]P 5 )<br />

c 5 (t)<br />

lim<br />

t→0+ t 8<br />

1<br />

= −<br />

2041200 < 0.<br />

We have a related problem (see Problems 4.1 and 4.2).<br />

Theorem 4.3.9. If W α is a polynomial hyp<strong>on</strong>ormal weighted shift with weight sequence<br />

{α n } ∞ n=0 such that α 0 = α 1 then α is flat.<br />

Proof. Without loss of generality we may assume α 0 = α 1 = 1. We claim that if<br />

α 0 = α 1 = 1 and W α is weakly k-hyp<strong>on</strong>ormal then<br />

(2 − α 2 k−1)α 2 k ≥ 1 for all k ≥ 3. (4.3)<br />

For (4.3) suppose W α is weakly k-hyp<strong>on</strong>ormal. Then T k := W α + sWα k is hyp<strong>on</strong>ormal<br />

for every s ∈ R. For k ≥ 3,<br />

⎡<br />

⎤<br />

q k,0 0 0 · · · γ k,0 0<br />

0 q k,1 0 · · · 0 γ k,1<br />

D k = P k [Tk ∗ 0 0 q k,2 · · · 0 0<br />

, T k ]P k =<br />

.<br />

⎢ . . . ..<br />

. ,<br />

.<br />

⎥<br />

⎣γ k,0 0 0 · · · q k,k−1 0 ⎦<br />

0 γ k,1 0 · · · 0 q k,k<br />

117


where<br />

⎧⎪⎨<br />

⎪ ⎩<br />

q n = (α 2 n − α 2 n−1) + (α 2 nα 2 n+1|t| 2 + (α 2 nα 2 n+1α 2 n+2 − α 2 n−3α 2 n−2α 2 n−1)|t| 2<br />

CHAPTER 4.<br />

WEIGHTED SHIFTS<br />

where<br />

q k,j :=<br />

{<br />

(αj 2 − α2 j−1 ) + s2 (αk+j−1 2 α2 k+j−2 · · · α2 j ) (0 ≤ j ≤ k − 1)<br />

(αk 2 − α2 k−1 ) + s2 (α2k−1 2 α2 2k−2 · · · α2 k − α2 k−1 α2 k−2 · · · α2 0) (j = k)<br />

γ k,0 = sα 0 α 1 · · · α k−2 α 2 k−1<br />

γ k,1 = sα 1 α 2 · · · α k−1 (α 2 k − α 2 0).<br />

Thus<br />

det D k =<br />

{<br />

(q k,k q k,1 − γk,1 2 )(q k,k−1q k,0 − γk,0 2 )q k,k−2q k,k−3 · · · q k,2 (k ≥ 4)<br />

(q 3,3 q 3,1 − γ3,1)(q 2 3,2 q 3,0 − γ3,0) 2 (k = 3)<br />

If α 0 = α 1 = 0 and if we let t := s 2 then<br />

det D k<br />

lim<br />

t→0+<br />

Since det D k ≥ 0 it follows that<br />

t k = (2α 2 k − α 2 k−1α 2 k − 1)<br />

k−1<br />

∏<br />

j=2<br />

(2 − α 2 k−1)α 2 k − 1 ≥ 0,<br />

α 2 j(α 2 j − α 2 j−1).<br />

which proves (4.3). If lim t→0+ α 2 n = α then (2 − α 2 )α − 1 ≥ 0, i.e.,<br />

(α − 1) 2 ≤ 0, i.e., α = 1.<br />

C<strong>on</strong>sider the case of cubic hyp<strong>on</strong>ormality. Let W α be a hyp<strong>on</strong>ormal weighted shift<br />

with {α n } ∞ n=0. For s, t ∈ C, let<br />

C n (s, t) := P n [(W α + sW 2 α + tW 3 α) ∗ W α + sW 2 α + tW 3 α]P n .<br />

Then C n (s, t) is a pentadiag<strong>on</strong>al matrix :<br />

⎡<br />

q 0 γ 0 υ 0 0 0 · · · 0<br />

⎤<br />

· · · · · · · · · · · · · · · υ n−2 γ n−1 q n γ 0 q 1 γ 1 υ 1 0 0 · · · 0<br />

υ 0 γ 1 q 2 γ 2 υ 2 0 · · · 0<br />

0 υ 1 γ 2 q 3 γ 3 υ 3 · · · 0<br />

C n (s, t) =<br />

. .. . .. . .. . .. . .. . .. · · · 0<br />

,<br />

. .. . .. . .. . .. . .. . .. · · · υ n−2<br />

⎢<br />

⎣<br />

. .. . .. . .. . .. . .. . .. . ⎥<br />

.. γn−1<br />

⎦<br />

γ n = α n (α 2 n+1 − α 2 n−1)s + α n (α 2 n+1α 2 n+2 − α 2 n−1α 2 n−2)st<br />

υ n = α n α n+1 (α 2 n+2 − α 2 n−1)t<br />

118


CHAPTER 4.<br />

WEIGHTED SHIFTS<br />

and α −1 = α −2 = α −3 = 0. Then<br />

W α is cubically hyp<strong>on</strong>ormal ⇐⇒ det C n (s, t) ≥ 0 (s, t ∈ C, n ≥ 0).<br />

In particular, if d k := det C k (s, t) then<br />

(<br />

d k = q k−1 − γ k−3γ k−2 υ<br />

) (<br />

k−3<br />

|γ k−3 | 2 d k−1 − |γ k−2 | 2 − q k−2γ k−3 γ k−2 υ<br />

)<br />

k−3<br />

|γ k−3 | 2 d k−2<br />

)<br />

+<br />

(|υ k−3 | 2 q k−2 − γ k−3 γ k−2 υ k−3 d k−3 + |υ k−4 | 2( |υ k−3 | 2 − q k−3γ k−3 γ k−2 υ<br />

)<br />

k−3<br />

|γ k−3 | 2 d k−4<br />

+ |υ k−4| 2 |υ k−2 | 2 γ k−3 γ k−2 υ k−3<br />

|γ k−3 | 2 d k−5 .<br />

119


CHAPTER 4.<br />

WEIGHTED SHIFTS<br />

4.4 The Perturbati<strong>on</strong>s<br />

Recall the Bram-Halmos criteri<strong>on</strong> for subnormality, which states that an operator T<br />

is subnormal if and <strong>on</strong>ly if<br />

∑<br />

(T i x j , T j x i ) ≥ 0<br />

i,j<br />

for all finite collecti<strong>on</strong>s x 0 , x 1 , · · · , x k ∈ H, or equivalently,<br />

⎡<br />

I T ∗ . . . T ∗k ⎤<br />

T T ∗ T . . . T ∗k T<br />

⎢<br />

⎣<br />

.<br />

.<br />

. .. .<br />

⎥ ≥ 0 (all k ≥ 1). (4.4)<br />

⎦<br />

T k T ∗ T k . . . T ∗k T k<br />

C<strong>on</strong>diti<strong>on</strong> (4.4) provides a measure of the gap between hyp<strong>on</strong>ormality and subnormality.<br />

In fact, the positivity c<strong>on</strong>diti<strong>on</strong> (4.4) for k = 1 is equivalent to the hyp<strong>on</strong>ormality<br />

of T , while subnormality requires the validity of (4.4) for all k. Let<br />

[A, B] := AB − BA denote the commutator of two operators A and B, and define T<br />

to be k-hyp<strong>on</strong>ormal whenever the k × k operator matrix<br />

M k (T ) := ([T ∗j , T i ]) k i,j=1 (4.5)<br />

is positive. An applicati<strong>on</strong> of the Choleski algorithm for operator matrices shows that<br />

the positivity of (4.5) is equivalent to the positivity of the (k + 1) × (k + 1) operator<br />

matrix in (4.4); the Bram-Halmos criteri<strong>on</strong> can be then rephrased as saying that T is<br />

subnormal if and <strong>on</strong>ly if T is k-hyp<strong>on</strong>ormal for every k ≥ 1.<br />

Recall also that T ∈ L(H) is said to be weakly k-hyp<strong>on</strong>ormal if<br />

⎧<br />

⎫<br />

⎨ k∑<br />

⎬<br />

LS(T, T 2 , · · · , T k ) := α<br />

⎩ j T j : α = (α 1 , · · · , α k ) ∈ C k ⎭<br />

j=1<br />

c<strong>on</strong>sists entirely of hyp<strong>on</strong>ormal operators, or equivalently, M k (T ) is weakly positive,<br />

i.e.,<br />

⎡ ⎤ ⎡ ⎤<br />

λ 0 x λ 0 x<br />

⎢<br />

(M k (T )<br />

⎥ ⎢<br />

⎣ . ⎦ ,<br />

⎥<br />

⎣ . ⎦) ≥ 0 for x ∈ H and λ 0 , · · · , λ k ∈ C. (4.6)<br />

λ k x λ k x<br />

If k = 2 then T is said to be quadratically hyp<strong>on</strong>ormal, and if k = 3 then T is said to<br />

be cubically hyp<strong>on</strong>ormal. Similarly, T ∈ B(H) is said to be polynomially hyp<strong>on</strong>ormal<br />

if p(T ) is hyp<strong>on</strong>ormal for every polynomial p ∈ C[z]. It is known that k-hyp<strong>on</strong>ormal<br />

⇒ weakly k-hyp<strong>on</strong>ormal, but the c<strong>on</strong>verse is not true in general.<br />

In the present secti<strong>on</strong> we renew our efforts to help describe the gap between subnormality<br />

and hyp<strong>on</strong>ormality, with particular emphasis <strong>on</strong> polynomial hyp<strong>on</strong>ormality.<br />

We focus <strong>on</strong> the class of unilateral weighted shifts, and initiate a study of how the<br />

120


CHAPTER 4.<br />

WEIGHTED SHIFTS<br />

above menti<strong>on</strong>ed noti<strong>on</strong>s behave under finite perturbati<strong>on</strong>s of the weight sequence.<br />

We first obtain three c<strong>on</strong>crete results:<br />

(i) the subnormality of W α is never stable under n<strong>on</strong>zero finite rank perturbati<strong>on</strong>s<br />

unless the perturbati<strong>on</strong> is c<strong>on</strong>fined to the zeroth weight;<br />

(ii) 2-hyp<strong>on</strong>ormality implies positive quadratic hyp<strong>on</strong>ormality, in the sense that<br />

the Maclaurin coefficients of D n (s) := det P n [(W α + sWα) 2 ∗ , W α + sWα] 2 P n are n<strong>on</strong>negative,<br />

for every n ≥ 0, where P n denotes the orthog<strong>on</strong>al projecti<strong>on</strong> <strong>on</strong>to the basis<br />

vectors {e 0 , · · · , e n }; and<br />

(iii) if α is strictly increasing and W α is 2-hyp<strong>on</strong>ormal then for α ′ a small perturbati<strong>on</strong><br />

of α, the shift W α ′ remains positively quadratically hyp<strong>on</strong>ormal.<br />

Al<strong>on</strong>g the way we establish two related results, each of independent interest:<br />

(iv) an integrality criteri<strong>on</strong> for a subnormal weighted shift to have an n-step subnormal<br />

extensi<strong>on</strong>; and<br />

(v) a proof that the sets of k-hyp<strong>on</strong>ormal and weakly k-hyp<strong>on</strong>ormal operators are<br />

closed in the str<strong>on</strong>g operator topology.<br />

C. Berger’s characterizati<strong>on</strong> of subnormality for unilateral weighted shifts states<br />

that W α is subnormal if and <strong>on</strong>ly if there exists a Borel probability measure µ supported<br />

in [0, ||W α || 2 ], with ||W α || 2 ∈ supp µ, such that<br />

∫<br />

γ n = t n dµ(t) for all n ≥ 0.<br />

Given an initial segment of weights α : α 0 , · · · α m , the sequence ̂α ∈ l ∞ (Z + ) such<br />

that ̂α : α i (i = 0, · · · , m) is said to be recursively generated by α if there exists r ≥ 1<br />

and φ 0 , · · · , φ r−1 ∈ R such that<br />

γ n+r = φ 0 γ n + · · · + φ r−1 γ n+r−1 (all n ≥ 0),<br />

where γ 0 = 1, γ n = α 2 0 · · · α 2 n−1 (n ≥ 1). In this case, Ŵα with weights ̂α is said to<br />

be recursively generated. If we let<br />

g(t) := t r − (φ r−1 t r−1 + · · · + φ 0 )<br />

then g has r distinct real roots 0 ≤ s 0 < · · · < s r−1 . Then Ŵα is a subnormal shift<br />

whose Berger measure µ is given by<br />

µ = ρ 0 δ s0 + · · · + ρ r−1 δ sr−1,<br />

where (ρ 0 , · · · , ρ r−1 ) is the unique soluti<strong>on</strong> of the Vanderm<strong>on</strong>de equati<strong>on</strong><br />

⎡<br />

⎤ ⎡ ⎤ ⎡ ⎤<br />

1 1 · · · 1 ρ 0 γ 0<br />

s 0 s 1 · · · s r−1<br />

ρ 1<br />

⎢<br />

⎥ ⎢<br />

⎣ . . . ⎦ ⎣<br />

⎥<br />

. ⎦ = γ 1<br />

⎢<br />

⎣<br />

⎥<br />

. ⎦ .<br />

s r−1<br />

0 s r−1<br />

1 · · · s r−1<br />

r−1<br />

ρ r−1 γ r−1<br />

121


CHAPTER 4.<br />

WEIGHTED SHIFTS<br />

For example, given α 0 < α 1 < α 2 , W (α0,α ̂ is the recursive weighted shift whose<br />

1,α 2)<br />

weights are calculated according to the recursive relati<strong>on</strong><br />

αn+1 2 1<br />

= φ 1 + φ 0<br />

αn<br />

2 ,<br />

where φ 0 = − α2 0 α2 1 (α2 2 −α2 1 ) and φ<br />

α 2 1 = − α2 1 (α2 2 −α2 0 ) . In this case, W<br />

1 −α2 0<br />

α 2 1 −α2 ̂<br />

0<br />

(α0<br />

is subnormal<br />

with 2-atomic Berger measure. Write W x(α0 ̂ for the weighted shift whose<br />

,α 1 ,α 2 )<br />

,α 1 ,α 2 )<br />

weight sequence c<strong>on</strong>sists of the initial weight x followed by the weight sequence of<br />

W<br />

.<br />

(α0,α ̂ 1,α 2)<br />

By the Density Theorem ([CuF2, Theorem 4.2 and Corollary 4.3]), we know that<br />

if W α is a subnormal weighted shift with weights α = {α n } and ϵ > 0, then there<br />

exists a n<strong>on</strong>zero compact operator K with ||K|| < ϵ such that W α +K is a recursively<br />

generated subnormal weighted shift; in fact W α + K = W for some m ≥ 1, where<br />

̂α(m)<br />

α (m) : α 0 , · · · , α m . The following result shows that K cannot generally be taken to<br />

be finite rank.<br />

Theorem 4.4.1. (Finite Rank Perturbati<strong>on</strong>s of Subnormal Shifts) If W α is a subnormal<br />

weighted shift then there exists no n<strong>on</strong>zero finite rank operator F (≠ cP {e0 }) such<br />

that W α + F is a subnormal weighted shift. C<strong>on</strong>cretely, suppose W α is a subnormal<br />

weighted shift with weight sequence α = {α n } ∞ n=0 and assume α ′ = {α n} ′ is a n<strong>on</strong>zero<br />

perturbati<strong>on</strong> of α in a finite number of weights except the initial weight; then W α ′ is<br />

not subnormal.<br />

We next c<strong>on</strong>sider the selfcommutator [(W α + s W 2 α) ∗ , W α + s W 2 α]. Let W α be a<br />

hyp<strong>on</strong>ormal weighted shift. For s ∈ C, we write<br />

D(s) := [(W α + s W 2 α) ∗ , W α + s W 2 α]<br />

and we let<br />

⎡<br />

⎤<br />

q 0 ¯r 0 0 . . . 0 0<br />

r 0 q 1 ¯r 1 . . . 0 0<br />

D n (s) := P n [(W α +s Wα) 2 ∗ , W α +s Wα]P 2 0 r 1 q 2 . . . 0 0<br />

n =<br />

⎢ .<br />

.<br />

.<br />

. .. . . , (4.7)<br />

⎥<br />

⎣ 0 0 0 . . . q n−1 ¯r n−1<br />

⎦<br />

0 0 0 . . . r n−1 q n<br />

where P n is the orthog<strong>on</strong>al projecti<strong>on</strong> <strong>on</strong>to the subspace generated by {e 0 , · · · , e n },<br />

⎧<br />

q n := u n + |s| 2 v n<br />

⎪⎨ r n := s √ w n<br />

u n := αn 2 − αn−1<br />

2 (4.8)<br />

v n := αnα 2 n+1 2 − αn−1α 2 n−2<br />

2 ⎪⎩<br />

w n := αn(α 2 n+1 2 − αn−1) 2 2 ,<br />

122


CHAPTER 4.<br />

WEIGHTED SHIFTS<br />

and, for notati<strong>on</strong>al c<strong>on</strong>venience, α −2 = α −1 = 0. Clearly, W α is quadratically hyp<strong>on</strong>ormal<br />

if and <strong>on</strong>ly if D n (s) ≥ 0 for all s ∈ C and all n ≥ 0. Let d n (·) := det (D n (·)).<br />

Then d n satisfies the following 2–step recursive formula:<br />

d 0 = q 0 , d 1 = q 0 q 1 − |r 0 | 2 , d n+2 = q n+2 d n+1 − |r n+1 | 2 d n .<br />

If we let t := |s| 2 , we observe that d n is a polynomial in t of degree n + 1, and if<br />

we write d n ≡ ∑ n+1<br />

i=0 c(n, i)ti , then the coefficients c(n, i) satisfy a double-indexed<br />

recursive formula, namely<br />

c(n + 2, i) = u n+2 c(n + 1, i) + v n+2 c(n + 1, i − 1) − w n+1 c(n, i − 1),<br />

c(n, 0) = u 0 · · · u n , c(n, n + 1) = v 0 · · · v n , c(1, 1) = u 1 v 0 + v 1 u 0 − w 0<br />

(4.9)<br />

(n ≥ 0, i ≥ 1). We say that W α is positively quadratically hyp<strong>on</strong>ormal if c(n, i) ≥ 0<br />

for every n ≥ 0, 0 ≤ i ≤ n + 1. Evidently, positively quadratically hyp<strong>on</strong>ormal =⇒<br />

quadratically hyp<strong>on</strong>ormal. The c<strong>on</strong>verse, however, is not true in general.<br />

The following theorem establishes a useful relati<strong>on</strong> between 2-hyp<strong>on</strong>ormality and<br />

positive quadratic hyp<strong>on</strong>ormality.<br />

Theorem 4.4.2. Let α ≡ {α n } ∞ n=0 be a weight sequence and assume that W α is 2-<br />

hyp<strong>on</strong>ormal. Then W α is positively quadratically hyp<strong>on</strong>ormal. More precisely, if W α<br />

is 2-hyp<strong>on</strong>ormal then<br />

c(n, i) ≥ v 0 · · · v i−1 u i · · · u n (n ≥ 0, 0 ≤ i ≤ n + 1). (4.10)<br />

In particular, if α is strictly increasing and W α is 2-hyp<strong>on</strong>ormal then the Maclaurin<br />

coefficients of d n (t) are positive for all n ≥ 0.<br />

If W α is a weighted shift with weight sequence α = {α n } ∞ n=0, then the moments<br />

of W α are usually defined by β 0 := 1, β n+1 := α n β n (n ≥ 0); however, we prefer to<br />

reserve this term for the sequence γ n := βn 2 (n ≥ 0). A criteri<strong>on</strong> for k-hyp<strong>on</strong>ormality<br />

can be given in terms of these moments ([Cu2, Theorem 4]): if we build a (k + 1) ×<br />

(k + 1) Hankel matrix A(n; k) by<br />

⎡<br />

⎤<br />

γ n γ n+1 . . . γ n+k<br />

γ n+1 γ n+2 . . . γ n+k+1<br />

A(n; k) := ⎢<br />

⎣<br />

.<br />

.<br />

⎥ (n ≥ 0),<br />

. ⎦<br />

γ n+k γ n+k+1 . . . γ n+2k<br />

then<br />

W α is k-hyp<strong>on</strong>ormal ⇐⇒ A(n; k) ≥ 0 (n ≥ 0). (4.11)<br />

In particular, for α strictly increasing, W α is 2-hyp<strong>on</strong>ormal if and <strong>on</strong>ly if<br />

⎡<br />

⎤<br />

γ n γ n+1 γ n+2<br />

det ⎣γ n+1 γ n+2 γ n+3<br />

⎦ ≥ 0 (n ≥ 0). (4.12)<br />

γ n+2 γ n+3 γ n+4<br />

123


CHAPTER 4.<br />

WEIGHTED SHIFTS<br />

One might c<strong>on</strong>jecture that if W α is a k-hyp<strong>on</strong>ormal weighted shift whose weight<br />

sequence is strictly increasing then W α remains weakly k-hyp<strong>on</strong>ormal under a small<br />

perturbati<strong>on</strong> of the weight sequence. We will show below that this is true for k = 2<br />

([]).<br />

In [CuF3, Theorem 4.3], it was shown that the gap between 2-hyp<strong>on</strong>ormality and<br />

quadratic hyp<strong>on</strong>ormality can be detected by unilateral shifts with a weight sequence<br />

α : √ x, ( √ a, √ b, √ c) ∧ . In particular, there exists a maximum value H 2 ≡ H 2 (a, b, c)<br />

of x that makes W √ x,( √ a, √ b, √ c)<br />

2-hyp<strong>on</strong>ormal; H ∧ 2 is called the modulus of 2-<br />

hyp<strong>on</strong>ormality (cf. citeCuF3). Any value of x > H 2 yields a n<strong>on</strong>-2-hyp<strong>on</strong>ormal<br />

weighted shift. However, if x − H 2 is small enough, W √ x,( √ a, √ b, √ c)<br />

is still quadratically<br />

hyp<strong>on</strong>ormal. The following theorem shows that, more generally, for finite rank<br />

∧<br />

perturbati<strong>on</strong>s of weighted shifts with strictly increasing weight sequences, there always<br />

exists a gap between 2-hyp<strong>on</strong>ormality and quadratic hyp<strong>on</strong>ormality.<br />

Theorem 4.4.3. (Finite Rank Perturbati<strong>on</strong>s of 2-hyp<strong>on</strong>ormal Shifts) Let α = {α n } ∞ n=0<br />

be a strictly increasing weight sequence. If W α is 2-hyp<strong>on</strong>ormal then W α remains positively<br />

quadratically hyp<strong>on</strong>ormal under a small n<strong>on</strong>zero finite rank perturbati<strong>on</strong> of α.<br />

We are ready for:<br />

Proof of Theorem 4.4.1. It suffices to show that if T is a weighted shift whose restricti<strong>on</strong><br />

to ∨ {e n , e n+1 , · · · } (n ≥ 2) is subnormal then there is at most <strong>on</strong>e α n−1 for<br />

which T is subnormal.<br />

Let W := T | ∨ {e n−1 ,e n ,e n+1 ,··· } and S := T | ∨ {e n ,e n+1 ,··· }, where n ≥ 2. Then<br />

W and S have weights α k (W ) := α k+n−1 and α k (S) := α k+n (k ≥ 0). Thus the<br />

corresp<strong>on</strong>ding moments are related by the equati<strong>on</strong><br />

γ k (S) = αn 2 · · · αn+k−1 2 = γ k+1(W )<br />

αn−1<br />

2 .<br />

We now adapt the proof of [Cu2, Propositi<strong>on</strong> 8]. Suppose S is subnormal with associated<br />

Berger measure µ. Then γ k (S) = ∫ ||T || 2<br />

t k dµ. Thus W is subnormal if and<br />

0<br />

<strong>on</strong>ly if there exists a probability measure ν <strong>on</strong> [0, ||T || 2 ] such that<br />

1<br />

α 2 n−1<br />

∫ ||T ||<br />

2<br />

0<br />

t k+1 dν(t) =<br />

∫ ||T ||<br />

2<br />

0<br />

t k dµ(t) for all k ≥ 0,<br />

which readily implies that t dν = α 2 n−1 dµ. Thus W is subnormal if and <strong>on</strong>ly if the<br />

formula<br />

dν := λ · δ 0 + α2 n−1<br />

dµ<br />

t<br />

defines a probability measure for some λ ≥ 0, where δ 0 is the point mass at the<br />

origin. In particular 1 t<br />

∈ L1 (µ) and µ({0}) = 0 whenever W is subnormal. If we<br />

repeat the above argument for W and V := T | ∨ {e n−2,e n−1,··· }, then we should have<br />

124


CHAPTER 4.<br />

WEIGHTED SHIFTS<br />

that ν({0}) = 0 whenever V is subnormal. Therefore we can c<strong>on</strong>clude that if V is<br />

subnormal then λ = 0, and hence<br />

dν = α2 n−1<br />

dµ.<br />

t<br />

Thus we have<br />

so that<br />

1 =<br />

∫ ||T ||<br />

2<br />

0<br />

α 2 n−1 =<br />

dν(t) = α 2 n−1<br />

( ∫ ||T ||<br />

2<br />

0<br />

∫ ||T ||<br />

2<br />

0<br />

1<br />

t dµ(t) ) −1<br />

,<br />

1<br />

t dµ(t),<br />

which implies that α n−1 is determined uniquely by {α n , α n+1 , · · · } whenever T is<br />

subnormal. This completes the proof.<br />

□<br />

Theorem 4.4.1 says that a n<strong>on</strong>zero finite rank perturbati<strong>on</strong> of a subnormal shift is<br />

never subnormal unless the perturbati<strong>on</strong> occurs at the initial weight. However, this is<br />

not the case for k-hyp<strong>on</strong>ormality. To see this √we use a close relative of the Bergman<br />

n+1<br />

shift B + (whose weights are given by α = {<br />

n+2 }∞ n=0); it is well known that B + is<br />

subnormal.<br />

Example 4.4.4. For x > 0, let T x be the weighted shift whose weights are given by<br />

Then we have:<br />

α 0 :=<br />

√<br />

1<br />

2 , α 1 := √ x, and α n :=<br />

(i)<br />

T x is subnormal ⇐⇒ x = 2 3 ;<br />

(ii)<br />

T x is 2-hyp<strong>on</strong>ormal ⇐⇒ 63−√ 129<br />

80<br />

≤ x ≤ 24<br />

35 .<br />

√<br />

n + 1<br />

n + 2<br />

(n ≥ 2).<br />

Proof. Asserti<strong>on</strong> (i) follows from Theorem 4.4.1. For asserti<strong>on</strong> (ii) we use (4.12): T x<br />

is 2-hyp<strong>on</strong>ormal if and <strong>on</strong>ly if<br />

⎡<br />

det ⎣<br />

1<br />

1<br />

2<br />

1<br />

2<br />

1<br />

2 x 3<br />

1<br />

2 x 3 8 x 3<br />

or equivalently, 63−√ 129<br />

80<br />

≤ x ≤ 24<br />

35 .<br />

1<br />

2 x ⎤<br />

⎡<br />

8 x ⎦ ≥ 0 and det ⎣<br />

10 x<br />

1 1<br />

2 2 x 3<br />

8 x<br />

1<br />

2 x 3<br />

8 x 3<br />

10 x<br />

3<br />

8 x 3<br />

10 x 1<br />

⎤<br />

⎦ ≥ 0,<br />

4 x<br />

For perturbati<strong>on</strong>s of recursive subnormal shifts of the form W (<br />

√ a,<br />

√<br />

b,<br />

√ c) ∧, subnormality<br />

and 2-hyp<strong>on</strong>ormality coincide.<br />

125


CHAPTER 4.<br />

WEIGHTED SHIFTS<br />

Theorem 4.4.5. Let α = {α n } ∞ n=0 be recursively generated by √ a, √ b, √ c. If T x is<br />

the weighted shift whose weights are given by α x : α 0 , · · · , α j−1 , √ x, α j+1 , · · · , then<br />

we have<br />

{<br />

x = αj 2 if j ≥ 1;<br />

T x is subnormal ⇐⇒ T x is 2-hyp<strong>on</strong>ormal ⇐⇒<br />

x ≤ a if j = 0.<br />

Proof. Since α is recursively generated by √ a, √ b, √ c, we have that α 2 0 = a, α 2 1 =<br />

b, α 2 2 = c,<br />

α3 2 = b(c2 − 2ac + ab)<br />

, and α4 2 = bc3 − 4abc 2 + 2ab 2 c + a 2 bc − a 2 b 2 + a 2 c 2<br />

c(b − a)<br />

(b − a)(c 2 .<br />

− 2ac + ab)<br />

(4.13)<br />

Case 1 (j = 0): It is evident that T x is subnormal if and <strong>on</strong>ly if x ≤ a. For<br />

2-hyp<strong>on</strong>ormality observe by (4.11) that T x is 2-hyp<strong>on</strong>ormal if and <strong>on</strong>ly if<br />

det<br />

⎡<br />

⎣ 1 x x bx bx<br />

bcx<br />

⎤<br />

⎦ ≥ 0,<br />

bx bcx α3bcx<br />

2<br />

or equivalently, x ≤ a.<br />

Case 2 (j ≥ 1): Without loss of generality we may assume that j = 1 and a = 1.<br />

Thus α 1 = √ x. Then by Theorem 4.4.1, T x is subnormal if and <strong>on</strong>ly if x = b. On the<br />

other hand, by (4.12), T x is 2-hyp<strong>on</strong>ormal if and <strong>on</strong>ly if<br />

det<br />

⎡<br />

⎣ 1 1 x ⎤<br />

1 x cx<br />

x cx α3cx<br />

2<br />

⎦ ≥ 0 and det<br />

⎡<br />

⎣ 1 x cx ⎤<br />

x cx α3cx<br />

2 ⎦ ≥ 0.<br />

cx α3cx 2 α3α 2 4cx<br />

2<br />

Thus a direct calculati<strong>on</strong> with the specific forms ( of α 3 , α 4 ) given in (4.13) shows that<br />

T x is 2-hyp<strong>on</strong>ormal if and <strong>on</strong>ly if (x − b) x − b(c2 −2c+b)<br />

b−1<br />

≤ 0 and x ≤ b. Since<br />

b ≤ b(c2 −2c+b)<br />

b−1<br />

, it follows that T x is 2-hyp<strong>on</strong>ormal if and <strong>on</strong>ly if x = b. This completes<br />

the proof.<br />

With the notati<strong>on</strong> in (4.8), we let<br />

We then have:<br />

p n := u n v n+1 − w n (n ≥ 0).<br />

Lemma 4.4.6. If α ≡ {α n } ∞ n=0 is a strictly increasing weight sequence then the<br />

following statements are equivalent:<br />

(i)<br />

W α is 2-hyp<strong>on</strong>ormal;<br />

(ii)<br />

αn+1(u 2 n+1 + u n+2 ) 2 ≤ u n+1 v n+2 (n ≥ 0);<br />

126


CHAPTER 4.<br />

WEIGHTED SHIFTS<br />

(iii)<br />

α 2 n<br />

α 2 n+2<br />

u n+2<br />

u n+3<br />

≤ un+1<br />

u n+2<br />

(n ≥ 0);<br />

(iv)<br />

p n ≥ 0 (n ≥ 0).<br />

Proof. This follows from a straightforward calculati<strong>on</strong>.<br />

We are ready for:<br />

Proof of Theorem 4.4.2. If α is not strictly increasing then α is flat, by the argument<br />

of [Cu2, Corollary 6], i.e., α 0 = α 1 = α 2 = · · · . Then<br />

[ ]<br />

α<br />

2<br />

D n (s) = 0 + |s| 2 α0 4 ¯sα0<br />

3<br />

sα0 3 |s| 2 α0<br />

4 ⊕ 0 ∞<br />

(cf. (4.7)), so that (4.10) is evident. Thus we may assume that α is strictly increasing,<br />

so that u n > 0, v n > 0 and w n > 0 for all n ≥ 0. Recall that if we write d n (t) :=<br />

∑ n+1<br />

i=0 c(n, i)ti then the c(n, i)’s satisfy the following recursive formulas (cf. (4.9)):<br />

c(n+2, i) = u n+2 c(n+1, i)+v n+2 c(n+1, i−1)−w n+1 c(n, i−1) (n ≥ 0, 1 ≤ i ≤ n).<br />

(4.14)<br />

Also, c(n, n + 1) = v 0 · · · v n (again by (4.9) and p n := u n v n+1 − w n ≥ 0 (n ≥ 0), by<br />

Lemma 4.4.6. A straightforward calculati<strong>on</strong> shows that<br />

d 0 (t) = u 0 + v 0 t;<br />

d 1 (t) = u 0 u 1 + (v 0 u 1 + p 0 ) t + v 0 v 1 t 2 ;<br />

d 2 (t) = u 0 u 1 u 2 + (v 0 u 1 u 2 + u 0 p 1 + u 2 p 0 ) t + (v 0 v 1 u 2 + v 0 p 1 + v 2 p 0 ) t 2 + v 0 v 1 v 2 t 3 .<br />

(4.15)<br />

Evidently,<br />

c(n, i) ≥ 0 (0 ≤ n ≤ 2, 0 ≤ i ≤ n + 1). (4.16)<br />

Define<br />

β(n, i) := c(n, i) − v 0 · · · v i−1 u i · · · u n<br />

(n ≥ 1, 1 ≤ i ≤ n).<br />

For every n ≥ 1, we now have<br />

⎧<br />

⎪⎨ u 0 · · · u n ≥ 0 (i = 0)<br />

c(n, i) = v 0 · · · v i−1 u i · · · u n + β(n, i) (1 ≤ i ≤ n)<br />

⎪⎩<br />

v 0 · · · v n ≥ 0 (i = n + 1).<br />

For notati<strong>on</strong>al c<strong>on</strong>venience we let β(n, 0) := 0 for every n ≥ 0.<br />

Claim 1. For n ≥ 1,<br />

c(n, n) ≥ u n c(n − 1, n) ≥ 0.<br />

(4.17)<br />

Proof of Claim 1. We use mathematical inducti<strong>on</strong>. For n = 1,<br />

c(1, 1) = v 0 u 1 + p 0 ≥ u 1 c(0, 1) ≥ 0,<br />

127


CHAPTER 4.<br />

WEIGHTED SHIFTS<br />

and<br />

c(n + 1, n + 1) = u n+1 c(n, n + 1) + v n+1 c(n, n) − w n c(n − 1, n)<br />

≥ u n+1 c(n, n + 1) + v n+1 u n c(n − 1, n) − w n c(n − 1, n)<br />

= u n+1 c(n, n + 1) + p n c(n − 1, n)<br />

≥ u n+1 c(n, n + 1),<br />

which proves Claim 1.<br />

Claim 2. For n ≥ 2,<br />

(by inductive hypothesis)<br />

β(n, i) ≥ u n β(n − 1, i) ≥ 0 (0 ≤ i ≤ n − 1). (4.18)<br />

Proof of Claim 2. We use mathematical inducti<strong>on</strong>. If n = 2 and i = 0, this is trivial.<br />

Also,<br />

β(2, 1) = u 0 p 1 + u 2 p 0 = u 0 p 1 + u 2 β(1, 1) ≥ u 2 β(1, 1) ≥ 0.<br />

Assume that (4.18) holds. We shall prove that<br />

β(n + 1, i) ≥ u n+1 β(n, i) ≥ 0<br />

(0 ≤ i ≤ n).<br />

For,<br />

β(n + 1, i) + v 0 · · · v i−1 u i · · · u n+1 = c(n + 1, i) (by (4.14))<br />

= u n+1 c(n, i) + v n+1 c(n, i − 1) − w n c(n − 1, i − 1)<br />

(<br />

)<br />

= u n+1 β(n, i) + v 0 · · · v i−1 u i · · · u n<br />

+ v n+1<br />

(<br />

β(n, i − 1) + v 0 · · · v i−2 u i−1 · · · u n<br />

)<br />

− w n<br />

(<br />

β(n − 1, i − 1) + v 0 · · · v i−2 u i−1 · · · u n−1<br />

),<br />

so that<br />

β(n + 1, i) = u n+1 β(n, i) + v n+1 β(n, i − 1) − w n β(n − 1, i − 1)<br />

+ v 0 · · · v i−2 u i−1 · · · u n−1 (u n v n+1 − w n )<br />

= u n+1 β(n, i) + v n+1 β(n, i − 1) − w n β(n − 1, i − 1) + (v 0 · · · v i−2 u i−1 · · · u n−1 ) p n<br />

≥ u n+1 β(n, i) + v n+1 u n β(n − 1, i − 1) − w n β(n − 1, i − 1)<br />

(by the inductive hypothesis and Lemma 4.4.6;<br />

observe that i − 1 ≤ n − 1, so (4.18) applies)<br />

= u n+1 β(n, i) + p n β(n − 1, i − 1)<br />

≥ u n+1 β(n, i),<br />

which proves Claim 2.<br />

By Claim 2 and (4.17), we can see that c(n, i) ≥ 0 for all n ≥ 0 and 1 ≤ i ≤ n − 1.<br />

Therefore (4.16), (4.17), Claim 1 and Claim 2 imply<br />

c(n, i) ≥ v 0 · · · v i−1 u i · · · u n (n ≥ 0, 0 ≤ i ≤ n + 1).<br />

128


CHAPTER 4.<br />

WEIGHTED SHIFTS<br />

This completes the proof.<br />

□<br />

To prove Theorem 4.4.3 we need:<br />

Lemma 4.4.7. ([CuF3, Lemma 2.3]) Let α ≡ {α n } ∞ n=0 be a strictly increasing weight<br />

sequence. If W α is 2-hyp<strong>on</strong>ormal then the sequence of quotients<br />

Θ n := u n+1<br />

u n+2<br />

(n ≥ 0)<br />

is bounded away from 0 and from ∞. More precisely,<br />

1 ≤ Θ n ≤ u 1<br />

u 2<br />

( ||Wα || 2<br />

α 0 α 1<br />

) 2<br />

for sufficiently large n.<br />

In particular, {u n } ∞ n=0 is eventually decreasing.<br />

We are ready for:<br />

Proof of Theorem 4.4.3. By Theorem 4.4.2, W α is strictly positively quadratically<br />

hyp<strong>on</strong>ormal, in the sense that all coefficients of d n (t) are positive for all n ≥ 0. Note<br />

that finite rank perturbati<strong>on</strong>s of α affect a finite number of values of u n , v n and w n .<br />

More c<strong>on</strong>cretely, if α ′ is a perturbati<strong>on</strong> of α in the weights {α 0 , · · · , α N }, then u n , v n ,<br />

w n and p n are invariant under α ′ for n ≥ N + 3. In particular, p n ≥ 0 for n ≥ N + 3.<br />

Claim 1. For n ≥ 3, 0 ≤ i ≤ n + 1,<br />

c(n, i) =u n c(n − 1, i) + p n−1 c(n − 2, i − 1) +<br />

where<br />

+ v n · · · v 3 ρ i−n+1 ,<br />

(cf. [CuF3, Proof of Theorem 4.3]).<br />

n∑<br />

k=4<br />

p k−2<br />

⎛<br />

⎝<br />

n∏<br />

j=k<br />

⎧<br />

0 (i < n − 1)<br />

⎪⎨<br />

u 0 p 1 (i = n − 1)<br />

ρ i−n+1 =<br />

v 0 p 1 + v 2 p 0 (i = n)<br />

⎪⎩<br />

v 0 v 1 v 2 (i = n + 1)<br />

Proof of Claim 1. We use inducti<strong>on</strong>. For n = 3, 0 ≤ i ≤ 4,<br />

v j<br />

⎞<br />

⎠ c(k − 3, i − n + k − 2)<br />

(4.19)<br />

c(3, i) = u 3 c(2, i) + v 3 c(2, i − 1) − w 2 c(1, i − 1)<br />

)<br />

= u 3 c(2, i) + v 3<br />

(u 2 c(1, i − 1) + v 2 c(1, i − 2) − w 1 c(0, i − 2) − w 2 c(1, i − 1)<br />

)<br />

= u 3 c(2, i) + p 2 c(1, i − 1) + v 3<br />

(v 2 c(1, i − 2) − w 1 c(0, i − 2)<br />

= u 3 c(2, i) + p 2 c(1, i − 1) + v 3 ρ i−2 ,<br />

129


CHAPTER 4.<br />

WEIGHTED SHIFTS<br />

where by (4.15),<br />

Now,<br />

⎧<br />

0 (i < 2)<br />

⎪⎨<br />

u 0 p 1 (i = 2)<br />

ρ i−2 =<br />

v 0 p 1 + v 2 p 0 (i = 3)<br />

⎪⎩<br />

v 0 v 1 v 2 (i = 4).<br />

c(n + 1, i) = u n+1 c(n, i) + v n+1 c(n, i − 1) − w n c(n − 1, i − 1)<br />

= u n+1 c(n, i) + v n+1<br />

(u n c(n − 1, i − 1) + p n−1 c(n − 2, i − 2)<br />

⎛ ⎞<br />

n∑<br />

n∏<br />

)<br />

+ p k−2<br />

⎝ v j<br />

⎠ c(k − 3, i − n + k − 3) + v n · · · v 3 ρ i−n − w n c(n − 1, i − 1)<br />

k=4<br />

j=k<br />

= u n+1 c(n, i) + p n c(n − 1, i − 1) + v n+1 p n−1 c(n − 2, i − 2)<br />

⎛ ⎞<br />

∑<br />

n n∏<br />

+ v n+1 p k−2<br />

⎝ v j<br />

⎠ c(k − 3, i − n + k − 3) + v n+1 · · · v 3 ρ i−n<br />

k=4<br />

j=k<br />

(by inductive hypothesis)<br />

n+1<br />

∑<br />

= u n+1 c(n, i) + p n c(n − 1, i − 1) +<br />

+ v n+1 · · · v 3 ρ i−n ,<br />

which proves Claim 1.<br />

k=4<br />

⎛<br />

∏<br />

p k−2<br />

n+1<br />

⎝<br />

j=k<br />

v j<br />

⎞<br />

⎠ c(k − 3, i − n + k − 3)<br />

Write u ′ n, v n, ′ w n, ′ p ′ n, ρ ′ n, and c ′ (·, ·) for the entities corresp<strong>on</strong>ding to α ′ . If p n > 0<br />

for every n = 0, · · · , N +2, then in view of Claim 1, we can choose a small perturbati<strong>on</strong><br />

such that p ′ n > 0 (0 ≤ n ≤ N + 2) and therefore c ′ (n, i) > 0 for all n ≥ 0 and<br />

0 ≤ i ≤ n + 1, which implies that W α ′ is also positively quadratically hyp<strong>on</strong>ormal. If<br />

instead p n = 0 for some n = 0, · · · , N + 2, careful inspecti<strong>on</strong> of (4.19) reveals that<br />

without loss of generality we may assume p 0 = · · · = p N+2 = 0. By Theorem 4.4.2,<br />

we have that for a sufficiently small perturbati<strong>on</strong> α ′ of α,<br />

c ′ (n, i) > 0 (0 ≤ n ≤ N + 2, 0 ≤ i ≤ n + 1) and c ′ (n, n + 1) > 0 (n ≥ 0). (4.20)<br />

Write<br />

Claim 2. {k n } ∞ n=2 is bounded.<br />

Proof of Claim 2. Observe that<br />

k n := v n<br />

u n<br />

(n = 2, 3, · · · ).<br />

k n = v n<br />

u n<br />

= α2 nα 2 n+1 − α 2 n−1α 2 n−2<br />

α 2 n − α 2 n−1<br />

= αn 2 + αn−1 2 + αn<br />

2 αn+1 2 − αn<br />

2<br />

αn 2 − αn−1<br />

2 + αn−1<br />

2 αn−1 2 − αn−2<br />

2<br />

αn 2 − αn−1<br />

2 .<br />

130


CHAPTER 4.<br />

WEIGHTED SHIFTS<br />

Therefore if W α is 2-hyp<strong>on</strong>ormal then by Lemma 4.4.7, the sequences<br />

{ α<br />

2<br />

n+1 − α 2 n<br />

α 2 n − α 2 n−1<br />

} ∞<br />

n=2<br />

and<br />

{ α<br />

2<br />

n−1 − α 2 n−2<br />

α 2 n − α 2 n−1<br />

} ∞<br />

n=2<br />

are both bounded, so that {k n } ∞ n=2 is bounded. This proves Claim 2.<br />

Write k := sup n k n . Without loss of generality we assume k < 1 (this is possible<br />

from the observati<strong>on</strong> that cα induces {c 2 k n }). Choose a sufficiently small perturbati<strong>on</strong><br />

α ′ of α such that if we let<br />

h :=<br />

sup<br />

0≤l≤N+2;0≤m≤1<br />

N+4<br />

∑<br />

∣<br />

k=4<br />

p ′ k−2<br />

⎛<br />

∏<br />

⎝<br />

N+3<br />

v j<br />

′<br />

j=k<br />

⎞<br />

⎠ c ′ (k − 3, l) + v N+3 ′ · · · v 3 ′ ρ ′ m<br />

∣<br />

(4.21)<br />

then<br />

c ′ (N + 3, i) − 1 h > 0 (0 ≤ i ≤ N + 3) (4.22)<br />

1 − k<br />

(this is always possible because by Theorem 4.4.2, we can choose a sufficiently small<br />

|p ′ i | such that<br />

c ′ (N + 3, i) > v 0 · · · v i−1 u i · · · u N+3 − ϵ and |h| < (1 − k) ( v 0 · · · v i−1 u i · · · u N+3 − ϵ )<br />

for any small ϵ > 0).<br />

Claim 3. For j ≥ 4 and 0 ≤ i ≤ N + j,<br />

)<br />

∑j−3<br />

c ′ (N + j, i) ≥ u N+j · · · u N+4<br />

(c ′ (N + 3, i) − k n h . (4.23)<br />

Proof of Claim 3. We use inducti<strong>on</strong>. If j = 4 then by Claim 1 and (4.21),<br />

c ′ (N + 4, i) = u ′ N+4c ′ (N + 3, i) + p ′ N+3c ′ (N + 2, i − 1)<br />

⎛ ⎞<br />

N+4<br />

∑<br />

N+3<br />

∏<br />

+ v N+4<br />

′ p ′ ⎝<br />

k−2 v j<br />

′ ⎠ c ′ (k − 3, i − N + k − 6) + v N+4 ′ · · · v 3ρ ′ ′ i−(N+3)<br />

k=4<br />

j=k<br />

n=1<br />

≥ u ′ N+4c ′ (N + 3, i) + p ′ N+3c ′ (N + 2, i − 1) − v ′ N+4h<br />

≥ u N+4<br />

(<br />

c ′ (N + 3, i) − k N+4 h )<br />

≥ u N+4<br />

(<br />

c ′ (N + 3, i) − k h )<br />

because u ′ N+4 = u N+4, v ′ N+4 = v N+4 and p ′ N+3 = p N+3 ≥ 0. Now suppose (4.23)<br />

131


CHAPTER 4.<br />

WEIGHTED SHIFTS<br />

holds for some j ≥ 4. By Claim 1, we have that for j ≥ 4,<br />

c ′ (N + j + 1, i) = u ′ N+j+1c ′ (N + j, i) + p ′ N+jc(N + j − 1, i − 1)<br />

⎛ ⎞<br />

N+j+1<br />

∑<br />

N+j+1<br />

∏<br />

+ p ′ ⎝<br />

k−2 v j<br />

′ ⎠ c ′ (k − 3, i − N + k − j − 3) + v N+j+1 ′ · · · v 3ρ ′ ′ i−(N+j)<br />

k=4<br />

j=k<br />

= u ′ N+j+1c ′ (N + j, i) + p ′ N+jc(N + j − 1, i − 1)<br />

⎛ ⎞<br />

N+j+1<br />

∑<br />

N+j+1<br />

∏<br />

+<br />

⎝ ⎠ c ′ (k − 3, i − N + k − j − 3)<br />

k=N+5<br />

∑<br />

p ′ k−2<br />

N+4<br />

+ p ′ k−2<br />

k=4<br />

⎛<br />

⎝<br />

∏<br />

j=k<br />

N+j+1<br />

v j<br />

′<br />

j=k<br />

v ′ j<br />

⎞<br />

⎠ c ′ (k − 3, i − N + k − j − 3) + v ′ N+j+1 · · · v ′ 3ρ ′ i−(N+j) .<br />

Since p ′ n = p n > 0 for n ≥ N + 3 and c ′ (n, l) > 0 for 0 ≤ n ≤ N + j by the inductive<br />

hypothesis, it follows that<br />

⎛ ⎞<br />

N+j+1<br />

∑<br />

N+j+1<br />

∏<br />

p ′ N+jc(N +j−1, i−1)+<br />

⎝ ⎠ c ′ (k−3, i−N +k−j−3) ≥ 0. (4.24)<br />

k=N+5<br />

p ′ k−2<br />

By inductive hypothesis and (4.24),<br />

c ′ (N + j + 1, i)<br />

N+4<br />

∑<br />

≥ u ′ N+j+1c ′ (N + j, i) +<br />

k=4<br />

p ′ k−2<br />

⎛<br />

⎝<br />

∏<br />

j=k<br />

N+j+1<br />

v j<br />

′<br />

j=k<br />

)<br />

∑j−3<br />

≥ u N+j+1 u N+j · · · u N+4<br />

(c ′ (N + 3, i) − k n h<br />

⎛<br />

∑<br />

+ v N+j+1 v N+j · · · v N+4<br />

N+4<br />

⎝<br />

k=4<br />

p ′ k−2<br />

n=1<br />

⎛<br />

∏<br />

⎝<br />

v ′ j<br />

N+3<br />

v j<br />

′<br />

j=k<br />

⎞<br />

⎠ c ′ (k − 3, i − N + k − j − 3) + v ′ N+j+1 · · · v ′ 3ρ ′ i−(N+j)<br />

⎞<br />

⎠ c ′ (k − 3, i − N + k − j − 3) + v N+3 ′ · · · v 3ρ ′ ′ ⎠<br />

i−(N+j)<br />

)<br />

∑j−3<br />

≥ u N+j+1 u N+j · · · u N+4<br />

(c ′ (N + 3, i) − k n h − v N+j+1 v N+j · · · v N+4 h<br />

)<br />

∑j−3<br />

= u N+j+1 u N+j · · · u N+4<br />

(c ′ (N + 3, i) − k n h − k N+j+1 k N+j · · · k N+4 h<br />

n=1<br />

n=1<br />

)<br />

∑j−2<br />

≥ u N+j+1 u N+j · · · u N+4<br />

(c ′ (N + 3, i) − k n h ,<br />

which proves Claim 3.<br />

Since ∑ j<br />

n=1 kn < 1<br />

1−k<br />

n=1<br />

for every j > 1, it follows from Claim 3 and (4.22) that<br />

c ′ (N + j, i) > 0 for j ≥ 4 and 0 ≤ i ≤ N + j. (4.25)<br />

⎞<br />

132


CHAPTER 4.<br />

WEIGHTED SHIFTS<br />

It thus follows from (4.20) and (4.25) that c ′ (n, i) > 0 for every n ≥ 0 and 0 ≤ i ≤ n+1.<br />

Therefore W α ′ is also positively quadratically hyp<strong>on</strong>ormal. This completes the proof.<br />

□<br />

Corollary 4.4.8. Let W α be a weighted shift such that α j−1 < α j for some j ≥ 1,<br />

and let T x be the weighted shift with weight sequence<br />

α x : α 0 , · · · , α j−1 , x, α j+1 , · · · .<br />

Then {x : T x is 2-hyp<strong>on</strong>ormal} is a proper closed subset of {x : T x is quadratically hyp<strong>on</strong>ormal}<br />

whenever the latter set is n<strong>on</strong>-empty.<br />

Proof. Write<br />

H 2 := {x : T x is 2-hyp<strong>on</strong>ormal}.<br />

Without loss of generality, we can assume that H 2 is n<strong>on</strong>-empty, and that j = 1.<br />

Recall that a 2-hyp<strong>on</strong>ormal weighted shift with two equal weights is of the form<br />

α 0 = α 1 = α 2 = · · · or α 0 < α 1 = α 2 = α 3 = · · · . Let x m := inf H 2 . By Propositi<strong>on</strong><br />

4.4.14 below, T xm is hyp<strong>on</strong>ormal. Then x m > α 0 . By assumpti<strong>on</strong>, x m < α 2 . Thus<br />

α 0 , x m , α 2 , α 3 , · · · is strictly increasing. Now we apply Theorem 4.4.3 to obtain x ′<br />

such that α 0 < x ′ < x m and T x ′ is quadratically hyp<strong>on</strong>ormal. However T x ′ is not<br />

2-hyp<strong>on</strong>ormal by the definiti<strong>on</strong> of x m . The proof is complete.<br />

The following questi<strong>on</strong> arises naturally:<br />

Questi<strong>on</strong>. Let α be a strictly increasing weight sequence and let k ≥ 3. If W α is a<br />

k-hyp<strong>on</strong>ormal weighted shift, does it follow that W α is weakly k-hyp<strong>on</strong>ormal under a<br />

small perturbati<strong>on</strong> of the weight sequence <br />

Let α : α 0 , α 1 , · · · be a weight sequence, let x i > 0 for 1 ≤ i ≤ n, and let<br />

(x n , · · · x 1 )α : x n , · · · , x 1 , α 0 , α 1 , · · · be the augmented weight sequence. We say that<br />

W (xn ,··· ,x 1 )α is an extensi<strong>on</strong> (or n-step extensi<strong>on</strong>) of W α . Observe that<br />

W (xn,··· ,x 1)α| ∨ {e n,e n+1,··· } ∼ = W α .<br />

The hypothesis F ≠ c P {e0 } in Theorem 4.4.1 is essential. Indeed, there exist infinitely<br />

many <strong>on</strong>e-step subnormal extensi<strong>on</strong> of a subnormal weighted shift whenever<br />

<strong>on</strong>e such extensi<strong>on</strong> exists. Recall ([Cu2, Propositi<strong>on</strong> 8]) that if W α is a weighted shift<br />

whose restricti<strong>on</strong> to ∨ {e 1 , e 2 , · · · } is subnormal with associated measure µ, then W α<br />

is subnormal if and <strong>on</strong>ly if<br />

(i) 1 t ∈ L1 (µ);<br />

(ii) α 2 0 ≤ ( || 1 t || L 1 (µ)) −1.<br />

Also note that there may not exist any <strong>on</strong>e-step subnormal extensi<strong>on</strong> of the subnormal<br />

weighted shift: for example, if W α is the Bergman shift then the corresp<strong>on</strong>ding Berger<br />

measure is µ(t) = t, and hence 1 t<br />

is not integrable with respect to µ; therefore W α<br />

does not admit any subnormal extensi<strong>on</strong>. A similar situati<strong>on</strong> arises when µ has an<br />

atom at {0}.<br />

More generally we have:<br />

133


CHAPTER 4.<br />

WEIGHTED SHIFTS<br />

Theorem 4.4.9. (Subnormal Extensi<strong>on</strong>s) Let W α be a subnormal weighted shift<br />

with weights α : α 0 , α 1 , · · · and let µ be the corresp<strong>on</strong>ding Berger measure. Then<br />

W (xn,··· ,x 1)α is subnormal if and <strong>on</strong>ly if<br />

(i) 1<br />

t<br />

∈ L 1 (µ);<br />

n<br />

(ii) x j =<br />

(iii) x n ≤<br />

(<br />

|| 1<br />

t j−1 || L 1 (µ)<br />

|| 1<br />

t j || L 1 (µ)<br />

(<br />

|| 1<br />

t n−1 || L 1 (µ)<br />

|| 1<br />

t n || L 1 (µ)<br />

In particular, if we put<br />

) 1<br />

2<br />

) 1<br />

2<br />

.<br />

for 1 ≤ j ≤ n − 1;<br />

S := {(x 1 , · · · , x n ) ∈ R n : W (xn,··· ,x 1)α<br />

is subnormal}<br />

then either S = ∅ or S is a line segment in R n .<br />

Proof. Write W j := W (xn ,··· ,x 1 )α| ∨ {e n−j ,e n−j+1 ,··· } (1 ≤ j ≤ n) and hence W n =<br />

W (xn ,··· ,x 1 )α. By the argument used to establish (3.2) we have that W 1 is subnormal<br />

with associated measure ν 1 if and <strong>on</strong>ly if<br />

(i) 1 t ∈ L1 (µ);<br />

(ii) dν 1 = x2 1<br />

t<br />

dµ, or equivalently, x 2 1 =<br />

( ∫ ||Wα || 2<br />

0<br />

) −1.<br />

1<br />

t dµ(t)<br />

Inductively W n−1 is subnormal with associated measure ν n−1 if and <strong>on</strong>ly if<br />

(i) W n−2 is subnormal;<br />

(ii)<br />

1<br />

t n−1 ∈ L 1 (µ);<br />

(iii) dν n−1 = x2 n−1<br />

t<br />

dν n−2 = · · · = x2 n−1···x2 1<br />

t n−1 dµ, or equivalently, x 2 n−1 =<br />

Therefore W n is subnormal if and <strong>on</strong>ly if<br />

(i) W n−1 is subnormal;<br />

(ii) 1<br />

t<br />

∈ L 1 (µ);<br />

n<br />

( ∫<br />

(iii) x 2 ||Wα||<br />

n ≤<br />

2<br />

0<br />

) −1 (<br />

1<br />

∫<br />

t dν ||Wα||<br />

n−1 = 2<br />

0<br />

x 2 n−1···x2 1<br />

t<br />

dµ(t)<br />

n<br />

) −1<br />

=<br />

∫ ||W α|| 2<br />

∫ ||W α|| 2 1<br />

0<br />

tn−2 dµ(t)<br />

∫ ||Wα|| 2<br />

1<br />

0<br />

tn−1 dµ(t).<br />

1<br />

0<br />

tn−1 dµ(t)<br />

∫ .<br />

||W α|| 2 1<br />

0<br />

t n dµ(t)<br />

Corollary 4.4.10. If W α is a subnormal weighted shift with associated measure µ,<br />

there exists an n-step subnormal extensi<strong>on</strong> of W α if and <strong>on</strong>ly if<br />

1<br />

t n ∈ L 1 (µ).<br />

Corollary 4.4.11. A recursively generated subnormal shift with φ 0 ≠ 0 admits an<br />

n-step subnormal extensi<strong>on</strong> for every n ≥ 1.<br />

Proof. The assumpti<strong>on</strong> about φ 0 implies that the zeros of g(t) are positive, so that<br />

1<br />

s 0 > 0. Thus for every n ≥ 1,<br />

t<br />

is integrable with respect to the corresp<strong>on</strong>ding<br />

n<br />

Berger measure µ = ρ 0 δ s0 + · · · + ρ r−1 δ sr−1 . By Corollary 4.4.10, there exists an<br />

n-step subnormal extensi<strong>on</strong>.<br />

134


CHAPTER 4.<br />

WEIGHTED SHIFTS<br />

We need not expect that for arbitrary recursively generated shifts, √ 2-hyp<strong>on</strong>ormality<br />

1<br />

and subnormality coincide as in Theorem 4.4.5. For example, if α :<br />

2 , √ x, ( √ √ √<br />

10<br />

3,<br />

3 , 17<br />

5 )∧<br />

then by (4.12) and Theorem 4.4.9,<br />

(i) T x is 2-hyp<strong>on</strong>ormal ⇐⇒ 4 − √ 6 ≤ x ≤ 2;<br />

(ii) T x is subnormal ⇐⇒ x = 2.<br />

A straightforward calculati<strong>on</strong> shows, however, that T x is 3-hyp<strong>on</strong>ormal if and <strong>on</strong>ly if<br />

x = 2; for,<br />

⎡<br />

1 1<br />

1<br />

2 2 x 3<br />

2 x ⎤<br />

1 1<br />

A(0; 3) := ⎢ 2 2<br />

⎣<br />

x 3<br />

2 x 5x<br />

1<br />

2 x ⎥<br />

3<br />

2x 5x 17x⎦ ≥ 0 ⇐⇒ x = 2.<br />

3<br />

2 x 5x 17x 58x<br />

This behavior is typical of general recursively generated weighted shifts: we show in<br />

[CJL] that subnormality is equivalent to k-hyp<strong>on</strong>ormality for some k ≥ 2.<br />

Next, we will show that can<strong>on</strong>ical rank-<strong>on</strong>e perturbati<strong>on</strong>s of k-hyp<strong>on</strong>ormal weighted<br />

shifts which preserve k-hyp<strong>on</strong>ormality form a c<strong>on</strong>vex set. To see this we need an auxiliary<br />

result.<br />

Lemma 4.4.12. Let I = {1, · · · , n} × {1, · · · , n} and let J be a symmetric subset of<br />

I. Let A = (a ij ) ∈ M n (C) and let C = (c ij ) ∈ M n (C) be given by<br />

{<br />

c a ij if (i, j) ∈ J<br />

c ij =<br />

(c > 0).<br />

if (i, j) ∈ I \ J<br />

a ij<br />

If A and C are positive semidefinite then B = (b ij ) ∈ M n (C) defined by<br />

{<br />

b a ij if (i, j) ∈ J<br />

b ij =<br />

(b ∈ [1, c] or [c, 1])<br />

if (i, j) ∈ I \ J<br />

a ij<br />

is also positive semidefinite.<br />

Proof. Without loss of generality we may assume c > 1. If b = 1 or b = c the asserti<strong>on</strong><br />

is trivial. Thus we assume 1 < b < c. The result is now a c<strong>on</strong>sequence of the following<br />

observati<strong>on</strong>. If [D] (i,j) denotes the (i, j)-entry of the matrix D then<br />

[ ( c − b<br />

A + b − 1<br />

c − 1 c − b<br />

)](i,j)<br />

C =<br />

=<br />

⎧<br />

⎨<br />

c−b<br />

c−1<br />

⎩ c−b<br />

c−1<br />

{<br />

b a ij<br />

a ij<br />

= [B] (i,j) ,<br />

( )<br />

1 + b−1<br />

c−b c a ij if (i, j) ∈ J<br />

( )<br />

1 + b−1<br />

c−b<br />

a ij if (i, j) ∈ I \ J<br />

if (i, j) ∈ J<br />

if (i, j) ∈ I \ J<br />

which is positive semidefinite because positive semidefinite matrices in M n (C) form<br />

a c<strong>on</strong>e.<br />

135


CHAPTER 4.<br />

WEIGHTED SHIFTS<br />

An immediate c<strong>on</strong>sequence of Lemma 4.4.12 is that positivity of a matrix forms<br />

a c<strong>on</strong>vex set with respect to a fixed diag<strong>on</strong>al locati<strong>on</strong>; i.e., if<br />

⎡<br />

A x = ⎣ ∗ ∗ ∗ ⎤<br />

∗ x ∗⎦<br />

∗ ∗ ∗<br />

then {x : A x is positive semidefinite} is c<strong>on</strong>vex.<br />

We now have:<br />

Theorem 4.4.13. Let α = {α n } ∞ n=0 be a weight sequence, let k ≥ 1, and let j ≥ 0.<br />

Define α (j) (x) : α 0 , · · · , α j−1 , x, α j+1 , · · · . Assume W α is k-hyp<strong>on</strong>ormal and define<br />

Then Ω k,j<br />

α<br />

Ω k,j<br />

α<br />

is a closed interval.<br />

:= {x : W α (j) (x) is k-hyp<strong>on</strong>ormal}.<br />

Proof. Suppose x 1 , x 2 ∈ Ω k,j<br />

α with x 1 < x 2 . Then by ([]), the (k +1)×(k +1) Hankel<br />

matrix<br />

⎡<br />

⎤<br />

γ n γ n+1 . . . γ n+k<br />

γ n+1 γ n+2 . . . γ n+k+1<br />

A xi (n; k) := ⎢<br />

⎣<br />

.<br />

.<br />

⎥ (n ≥ 0; i = 1, 2)<br />

. ⎦<br />

γ n+k γ n+k+1 . . . γ n+2k<br />

is positive, where A xi corresp<strong>on</strong>ds to α (j) (x i ). We must show that tx 1 +(1−t)x 2 ∈ Ω k,j<br />

α<br />

(0 < t < 1), i.e.,<br />

A tx1 +(1−t)x 2<br />

(n; k) ≥ 0 (n ≥ 0, 0 < t < 1).<br />

Observe that it suffices to establish the positivity of the 2k Hankel matrices corresp<strong>on</strong>ding<br />

to α (j) (tx 1 + (1 − t)x 2 ) such that tx 1 + (1 − t)x 2 appears as a factor in at<br />

least <strong>on</strong>e entry but not in every entry. A moment’s thought reveals that without loss<br />

of generality we may assume j = 2k. Observe that<br />

A z1 (n; k) − A z2 (n; k) = (z 2 1 − z 2 2) H(n; k)<br />

for some Hankel matrix H(n; k). For notati<strong>on</strong>al c<strong>on</strong>venience, we abbreviate A z (n; k)<br />

as A z . Then<br />

⎧<br />

⎨t 2 A x1 + (1 − t) 2 A x2 + 2t(1 − t)A √ x 1 x 2<br />

for 0 ≤ n ≤ 2k<br />

A tx1 +(1−t)x 2<br />

= (<br />

) 2<br />

⎩ t + (1 − t) x2<br />

x Ax1<br />

1<br />

for n ≥ 2k + 1.<br />

Since A x1 ≥ 0, A x2 ≥ 0 and A √ x 1x 2<br />

have the form described by Lemma 4.4.12<br />

and since x 1 < √ x 1 x 2 < x 2 it follows from Lemma 4.4.12 that A √ x 1 x 2<br />

≥ 0. Thus<br />

evidently, A tx1+(1−t)x 2<br />

≥ 0, and therefore tx 1 + (1 − t)x 2 ∈ Ω k,j<br />

α . This shows that<br />

Ω k,j<br />

α is an interval. The closedness of the interval follows from Propositi<strong>on</strong> 4.4.14<br />

below.<br />

136


CHAPTER 4.<br />

WEIGHTED SHIFTS<br />

In [CP1] and [CP2], it was shown that there exists a n<strong>on</strong>-subnormal polynomially<br />

hyp<strong>on</strong>ormal operator. Also in [McCP], it was shown that there exists a n<strong>on</strong>-subnormal<br />

polynomially hyp<strong>on</strong>ormal operator if and <strong>on</strong>ly if there exists <strong>on</strong>e which is also a<br />

weighted shift. However, no c<strong>on</strong>crete weighted shift has yet been found. As a strategy<br />

for finding such a shift, we would like to suggest the following:<br />

Questi<strong>on</strong> Does it follow that the polynomial hyp<strong>on</strong>ormality of the weighted shift is<br />

stable under small perturbati<strong>on</strong>s of the weight sequence <br />

If the answer to the above questi<strong>on</strong> were affirmative then we would easily find<br />

a polynomially hyp<strong>on</strong>ormal n<strong>on</strong>-subnormal (even n<strong>on</strong>-2-hyp<strong>on</strong>ormal) weighted shift;<br />

for example, if<br />

α : 1, √ x, ( √ √ √<br />

10 17<br />

3,<br />

3 , 5 )∧<br />

and T x is the weighted shift associated with α, then by Theorem 4.4.5, T x is subnormal<br />

⇔ x = 2, whereas T x is polynomially hyp<strong>on</strong>ormal ⇔ 2 − δ 1 < x < 2 + δ 2 for some<br />

δ 1 , δ 2 > 0 provided the answer to the above questi<strong>on</strong> is yes; therefore for sufficiently<br />

small ϵ > 0,<br />

α ϵ : 1, √ 2 + ϵ, ( √ √ √<br />

10 17<br />

3,<br />

3 , 5 )∧<br />

would induce a n<strong>on</strong>-2-hyp<strong>on</strong>ormal polynomially hyp<strong>on</strong>ormal weighted shift.<br />

The answer to the above questi<strong>on</strong> for weak k-hyp<strong>on</strong>ormality is negative. In fact<br />

we have:<br />

Propositi<strong>on</strong> 4.4.14. (i) The set of k-hyp<strong>on</strong>ormal operators is sot-closed.<br />

(ii) The set of weakly k-hyp<strong>on</strong>ormal operators is sot-closed.<br />

Proof. Suppose T η ∈ L(H) and T η → T in sot. Then, by the Uniform Boundedness<br />

Principle, {||T η ||} η is bounded. Thus Tη ∗i Tη<br />

j → T ∗i T j in sot for every i, j, so that<br />

M k (T η ) → M k (T ) in sot (where M k (T ) is as in (4.5). (i) In this case M k (T η ) ≥ 0 for<br />

all η, so M k (T ) ≥ 0, i.e., T is k-hyp<strong>on</strong>ormal.<br />

(ii) Here, M k (T η ) is weakly positive for all η. By (4.6), M k (T ) is also weakly<br />

positive, i.e., T is weakly k-hyp<strong>on</strong>ormal.<br />

137


CHAPTER 4.<br />

WEIGHTED SHIFTS<br />

4.5 The Extensi<strong>on</strong>s<br />

In [Sta3], J. Stampfli showed that given α : √ a, √ b, √ c with 0 < a < b < c, there<br />

always exists a subnormal completi<strong>on</strong> of α, but that for α : √ a, √ b, √ c, √ d (a < b <<br />

c < d) such a subnormal completi<strong>on</strong> may not exist.<br />

There are instances where k-hyp<strong>on</strong>ormality implies subnormality for weighted<br />

shifts. For example, in [CuF3], it was shown that if α(x) : √ x, ( √ a, √ b, √ c) ∧ (a <<br />

b < c) then W α(x) is 2-hyp<strong>on</strong>ormal if and <strong>on</strong>ly if it is subnormal: more c<strong>on</strong>cretely,<br />

W α(x) is 2-hyp<strong>on</strong>ormal if and <strong>on</strong>ly if<br />

√ x ≤ H2 ( √ a, √ √<br />

b, √ ab(c − b)<br />

c) :=<br />

(b − a) 2 + b(c − b) ,<br />

in which case W α(x) is subnormal. In this secti<strong>on</strong> we extend the above result to weight<br />

sequences of the form α : x n , · · · , x 1 , (α 0 , · · · , α k ) ∧ with 0 < α 0 < · · · < α k . We here<br />

show:<br />

Extensi<strong>on</strong>s of Recursively Generated Weighted Shifts.<br />

If α : x n , · · · , x 1 , (α 0 , · · · , α k ) ∧ then<br />

{<br />

W α is ([ k+1<br />

W α is subnormal ⇐⇒<br />

2<br />

] + 1)-hyp<strong>on</strong>ormal (n = 1)<br />

W α is ([ k+1<br />

2<br />

] + 2)-hyp<strong>on</strong>ormal (n > 1).<br />

In particular, the above theorem shows that the subnormality of an extensi<strong>on</strong> of<br />

the recursive shift is independent of its length if the length is bigger than 1.<br />

Given an initial segment of weights<br />

α : α 0 , · · · , α 2k (k ≥ 0),<br />

suppose ˆα ≡ (α 0 , · · · , α 2k ) ∧ , i.e., ˆα is recursively generated by α. Write<br />

⎡ ⎤<br />

γ ṇ<br />

⎢<br />

v n := ⎣ .<br />

⎥<br />

⎦ (0 ≤ n ≤ k + 1).<br />

γ n+k<br />

Then {v 0 , · · · , v k+1 } is linearly dependent in R k+1 . Now the rank of α is defined<br />

by the smallest integer i (1 ≤ i ≤ k + 1) such that v i is a linear combinati<strong>on</strong> of<br />

v 0 , · · · , v i−1 . Since {v 0 , · · · , v i−1 } is linearly independent, there exists a unique i-<br />

tuple φ ≡ (φ 0 , · · · , φ i−1 ) ∈ R i such that v i = φ 0 v 0 + · · · + φ i−1 v i−1 , or equivalently,<br />

γ j = φ i−1 γ j−1 + · · · + φ 0 γ j−i<br />

(i ≤ j ≤ k + i),<br />

which says that (α 0 , · · · , α k+i ) is recursively generated by (α 0 , · · · , α i ). In this case,<br />

W α is said to be i-recursive (cf. [CuF3, Definiti<strong>on</strong> 5.14]).<br />

We begin with:<br />

138


CHAPTER 4.<br />

WEIGHTED SHIFTS<br />

Lemma 4.5.1. ([CuF2, Propositi<strong>on</strong>s 2.3, 2.6, and 2.7]) Let A, B ∈ M n (C), Ã, ˜B ∈<br />

M n+1 (C) (n ≥ 1) be such that<br />

[ [ ]<br />

A ∗ ∗ ∗<br />

à = and ˜B = .<br />

∗]<br />

∗ B<br />

Then we have:<br />

(i) If A ≥ 0 and if à is a flat extensi<strong>on</strong> of A (i.e., rank(Ã) = rank(A)) then à ≥ 0;<br />

(ii) If A ≥ 0 and à ≥ 0 then det(A) = 0 implies det(Ã) = 0;<br />

(iii) If B ≥ 0 and ˜B ≥ 0 then det(B) = 0 implies det( ˜B) = 0.<br />

Lemma 4.5.2. If α ≡ (α 0 , · · · , α k ) ∧ then<br />

W α is subnormal ⇐⇒ W α is ([ k ] + 1)-hyp<strong>on</strong>ormal. (4.26)<br />

2<br />

In the cases where W α is subnormal and i := rank(α), then we have that α =<br />

(α 0 , · · · , α 2i−2 ) ∧ .<br />

Proof. We <strong>on</strong>ly need to establish the sufficiency c<strong>on</strong>diti<strong>on</strong> in (4.26). Let i := rank(α).<br />

Since W α is i-recursive, [CuF3, Propositi<strong>on</strong> 5.15] implies the subnormality of W α<br />

follows after we verify that A(0, i − 1) ≥ 0 and A(1, i − 1) ≥ 0. Now observe that<br />

i − 1 ≤ [ k 2 ] + 1 and A(j, [ k [ A(j, i − 1)<br />

2 ] + 1) = ∗<br />

] ∗<br />

∗<br />

(j = 0, 1),<br />

so the positivity of A(0, i − 1) and A(1, i − 1) is a c<strong>on</strong>sequence of the positivity<br />

of the ([ k 2 ] + 1)-hyp<strong>on</strong>ormality of W α. For the sec<strong>on</strong>d asserti<strong>on</strong>, observe that if<br />

i := rank(α) then det A(n, i) = 0 for all n ≥ 0. By assumpti<strong>on</strong> A(n, i + 1) ≥ 0,<br />

so by Lemma 4.5.1 (ii) we have det A(n, i + 1) = 0, which says that (α 0 , · · · , α 2i−1 ) ⊂<br />

(α 0 , · · · , α 2i−2 ) ∧ . By iterati<strong>on</strong> we obtain (α 0 , · · · , α k ) ⊂ (α 0 , · · · , α 2i−2 ) ∧ , and therefore<br />

(α 0 , · · · , α k ) ∧ = (α 0 , · · · , α 2i−2 ) ∧ . This proves the lemma.<br />

In what follows, and for notati<strong>on</strong>al c<strong>on</strong>venience, we shall set x −j := α j (0 ≤ j ≤ k).<br />

Theorem 4.5.3. (Subnormality Criteri<strong>on</strong>) If α : x n , · · · , x 1 , (α 0 , · · · , α k ) ∧ then<br />

{<br />

W α is ([ k+1<br />

W α is subnormal ⇐⇒<br />

2<br />

] + 1)-hyp<strong>on</strong>ormal (n = 1)<br />

W α is ([ k+1<br />

2 ] + 2)-hyp<strong>on</strong>ormal (n > 1). (4.27)<br />

Furthermore, in the cases where the above equivalence holds, if rank(α 0 , · · · , α k ) = i<br />

then<br />

{<br />

W α is i-hyp<strong>on</strong>ormal (n = 1)<br />

W α is subnormal ⇐⇒<br />

(4.28)<br />

W α is (i + 1)-hyp<strong>on</strong>ormal (n > 1).<br />

139


CHAPTER 4.<br />

WEIGHTED SHIFTS<br />

In fact,<br />

⎧<br />

x 1 = H i (α 0 , · · · , α 2i−2 )<br />

⎪⎨<br />

· · · · · · · · ·<br />

x n−1 = H i (x n−2 , · · · , · · · , α 2i−n )<br />

⎪⎩<br />

x n ≤ H i (x n−1 , · · · , α 2i−n−1 ),<br />

where H i is the modulus of i-hyp<strong>on</strong>ormality (cf. [CuF3, Propositi<strong>on</strong> 3.4 and (3.4)]),<br />

i.e.,<br />

H i (α) := sup{x > 0 : W xα is i-hyp<strong>on</strong>ormal}.<br />

Therefore, W α = W xn (x n−1 ,··· ,α 2i−n−1 ) ∧.<br />

Proof. C<strong>on</strong>sider the (k + 1) × (l + 1) “Hankel” matrix A(n; k, l) by<br />

⎡<br />

⎤<br />

γ n γ n+1 . . . γ n+l<br />

γ n+1 γ n+2 . . . γ n+1+l<br />

A(n; k, l) := ⎢<br />

⎥<br />

⎣ . .<br />

. ⎦<br />

γ n+k γ n+k+1 . . . γ n+k+l<br />

(n ≥ 0).<br />

Case 1 (α : x 1 , (α 0 , · · · , α k ) ∧ ): Let Â(n; k, l) and A(n; k, l) denote the Hankel<br />

matrices corresp<strong>on</strong>ding to the weight sequences (α 0 , · · · , α k ) ∧ and α, respectively.<br />

Suppose W α is ([ k+1<br />

2 ] + 1)-hyp<strong>on</strong>ormal. Then by Lemma 4.5.2, W (α 0,··· ,α k ) ∧ is subnormal.<br />

Observe that<br />

A(n + 1; m, m) = x 2 1 Â(n; m, m) for all n ≥ 0 and all m ≥ 0.<br />

Thus it suffices to show that A(0; m, m) ≥ 0 for all m ≥ [ k+1<br />

2<br />

] + 2. Also note that<br />

if ˜B denotes the (k − 1) × k matrix obtained by eliminating the first row of a k × k<br />

matrix B then<br />

Ã(0; m, m) = x 2 1<br />

+ 1<br />

Â(0; m − 1, m) for all m ≥ [k ] + 2.<br />

2<br />

Therefore for every m ≥ [ k+1<br />

k+1<br />

2<br />

] + 2, A(0; m, m) is a flat extensi<strong>on</strong> of A(0; [<br />

2 ] +<br />

1, [ k+1<br />

k+1<br />

2<br />

] + 1). This implies A(0; m, m) ≥ 0 for all m ≥ [<br />

2 ] + 2 and therefore W α is<br />

subnormal.<br />

Case 2 (α : x n , · · · , x 1 , (α 0 , · · · , α k ) ∧ )): As in Case 1, let Â(n; k, l) and A(n; k, l)<br />

denote the Hankel matrices corresp<strong>on</strong>ding to the weight sequences (α 0 , · · · , α k ) ∧ and<br />

k+1 k+1<br />

α, respectively. Observe that det Â(n; [<br />

2<br />

] + 1, [<br />

2<br />

] + 1) = 0 for all n ≥ 0. Suppose<br />

W α is ([ k+1<br />

2<br />

] + 2)-hyp<strong>on</strong>ormal. Observe that<br />

so that<br />

A(n + 1; [ k + 1<br />

2<br />

] + 1, [ k + 1 ] + 1) = x 2 1 · · · x<br />

2<br />

det A(n + 1; [ k + 1<br />

2<br />

2 + 1<br />

nÂ(1; [k<br />

2<br />

] + 1, [ k + 1 ] + 1),<br />

2<br />

] + 1, [ k + 1 ] + 1) = 0. (4.29)<br />

2<br />

140


CHAPTER 4.<br />

WEIGHTED SHIFTS<br />

Also observe<br />

A(n − 1; [ k + 1<br />

2<br />

Since W α is ([ k+1<br />

2<br />

det A(n − 1; [ k+1<br />

2<br />

A(n − 1; [ k + 1<br />

2<br />

] + 2, [ k + 1 [ x<br />

2<br />

] + 2) = 2 · · · x 2 n ∗<br />

2<br />

]<br />

∗ A(n + 1; [ k+1 k+1<br />

2<br />

] + 1, [<br />

2 ] + 1) .<br />

] + 1)-hyp<strong>on</strong>ormal it follows from Lemma 4.5.1 (iii) and (4.29) that<br />

k+1<br />

] + 1, [<br />

2<br />

] + 1) = 0. Note that<br />

] + 1, [ k + 1<br />

2<br />

⎡<br />

] + 1) = x 2 1 · · · x 2 n ⎢<br />

⎣<br />

1<br />

x 2 1<br />

ˆγ 0 . . . ˆγ [<br />

k+1<br />

ˆγ 0 ˆγ 1 . . . ˆγ [<br />

k+1<br />

.<br />

.<br />

ˆγ [<br />

k+1<br />

2 ]+1 ˆγ [<br />

k+1<br />

2 ]+2 . . . ˆγ 2[<br />

k+1<br />

⎤<br />

2 ]+1<br />

2 ]+2<br />

⎥<br />

.<br />

2 ]+2<br />

where ˆγ j denotes the moments corresp<strong>on</strong>ding to the weight sequence (α 0 , · · · , α k ) ∧ .<br />

Therefore x 1 is determined uniquely by {α 0 , · · · , α k } such that (x 1 , α 0 , · · · , α k−1 ) ∧ =<br />

x 1 , (α 0 , · · · , α k ) ∧ : more precisely, if i := rank (α) and φ 0 , · · · , φ i−1 denote the coefficients<br />

of recursi<strong>on</strong> in (α 0 , · · · , α k ) ∧ then<br />

[<br />

] 1<br />

x 1 = H i [(α 0 , · · · , α k ) ∧ 2<br />

φ 0<br />

] =<br />

ˆγ i−1 − φ i−1ˆγ i−2 − · · · − φ 1ˆγ 0<br />

(cf. [CuF3, (3.4)]). C<strong>on</strong>tinuing this process we can see that x 1 , · · · , x n−1 are determined<br />

uniquely by a telescoping method such that<br />

(x n−1 , · · · , x n−1−k ) ∧ = x n−1 , · · · , x 1 , (α 0 , · · · , α k ) ∧<br />

and W (xn−1 ,··· ,x n−1−k ) ∧ is subnormal. Therefore, after (n − 1) steps, Case 2 reduces<br />

to Case 1. This proves the first asserti<strong>on</strong>. For the sec<strong>on</strong>d asserti<strong>on</strong>, note that if<br />

rank(α 0 , · · · , α k ) = i then<br />

det Â(n; i, i) = 0.<br />

Now applying the above argument with i in place of [ k+1<br />

2 ] + 1 gives that x 1, · · · , x n−1<br />

are determined uniquely by α 0 , · · · , α 2i−2 such that W (xn−1 ,··· ,x n−2i−1 ) ∧ is subnormal.<br />

Thus the sec<strong>on</strong>d asserti<strong>on</strong> immediately follows. Finally, observe that the preceding<br />

argument also establish the remaining asserti<strong>on</strong>s.<br />

⎦ ,<br />

Remark 4.5.4. (a) From Theorem 4.5.3 we note that the subnormality of an extensi<strong>on</strong><br />

of a recursive shift is independent of its length if the length is bigger than<br />

1.<br />

(b) In Theorem 4.5.3, “[ k+1<br />

2 ]” can not be relaxed to “[ k 2<br />

]”. For example c<strong>on</strong>sider<br />

the following weight sequences:<br />

√ √<br />

1<br />

(i) α :<br />

2 , ( 3<br />

2 , √ √ √<br />

10<br />

3,<br />

3<br />

√ √<br />

,<br />

(ii) α ′ 1<br />

:<br />

2 , 3<br />

2 , (√ 3,<br />

√ √<br />

10<br />

3 , 17<br />

5 )∧ .<br />

17<br />

5 )∧ with φ 0 = 0;<br />

141


CHAPTER 4.<br />

WEIGHTED SHIFTS<br />

Observe that α equals α ′ . Then a straightforward calculati<strong>on</strong> shows that W α (and<br />

hence W α ′) is 2-hyp<strong>on</strong>ormal but not 3-hyp<strong>on</strong>ormal (and hence, not subnormal). Note<br />

that k = 3 and n = 1 in (i) and k = 2 and n = 2 in (ii).<br />

(c) The sec<strong>on</strong>d asserti<strong>on</strong> of Theorem 4.5.3 does not imply that if rank(α 0 , · · · , α k ) =<br />

i then (4.28) holds in general. Theorem 4.5.3 says <strong>on</strong>ly that when W α is ([ k+1<br />

2 ] + 1)-<br />

hyp<strong>on</strong>ormal (n = 1), i-hyp<strong>on</strong>ormality and subnormality coincide, and that when W α<br />

is ([ k+1 ] + 2)-hyp<strong>on</strong>ormal (n > 1), (i + 1)-hyp<strong>on</strong>ormality and subnormality coincide.<br />

2<br />

For example c<strong>on</strong>sider the weight sequence<br />

ˆα ≡ ( √ 2, √ √ √<br />

10 17<br />

3,<br />

3 ,<br />

Since ( √ 2, √ √ √<br />

10<br />

3,<br />

3 , 17<br />

5 ) ⊂ (√ 2, √ 3,<br />

5 , 2)∧ with φ 0 = 0 (here φ 1 = 0 also).<br />

√<br />

10<br />

β ≡ 1, ( √ 2, √ 3,<br />

3 )∧ , we can see that rank(α) = 2. Put<br />

√ √<br />

10 17<br />

3 , 5 , 2)∧ .<br />

If (4.28) held true without assuming (4.27), then 2-hyp<strong>on</strong>ormality would imply subnormality<br />

for W β . However, a straightforward calculati<strong>on</strong> shows that W β is 2-<br />

hyp<strong>on</strong>ormal but not 3-hyp<strong>on</strong>ormal (and hence not subnormal): in fact, det A(n, 2) = 0<br />

for all n ≥ 0 except for n = 2 and det A(2, 2) = 160 > 0, while since<br />

φ 3 = − α2 3α4(α 2 5 2 − α4)<br />

2<br />

α4 2 − α2 3<br />

√<br />

(so that α 6 = √ φ 4 − φ 3<br />

=<br />

α 2 5<br />

= −102 and φ 4 = α2 4(α 2 5 − α 2 3)<br />

α 2 4 − α2 3<br />

17<br />

2<br />

), we have that<br />

⎡<br />

1 2 6 20<br />

det A(1, 3) = det ⎢ 2 6 20 68<br />

⎣ 6 20 68 272<br />

20 68 272 2312<br />

⎤<br />

⎥<br />

⎦ = −3200 < 0.<br />

= 34<br />

(d) On the other hand, Theorem 4.5.3 does show that if α ≡ (α 0 , · · · , α k ) is such<br />

that rank(α) = i and Wˆα is subnormal with associated Berger measure µ, then Wˆα has<br />

1<br />

an n-step (i + 1)-hyp<strong>on</strong>ormal extensi<strong>on</strong> W xn ,··· ,x 1 ,ˆα (n ≥ 2) if and <strong>on</strong>ly if<br />

t<br />

∈ L 1 (µ),<br />

n<br />

and<br />

x j+1 =<br />

[<br />

x n ≤<br />

φ 0<br />

γ (j)<br />

i−1 − φ i−1γ (j)<br />

i−2 − · · · − φ 1γ (j)<br />

0<br />

[<br />

γ (n−1)<br />

i−1<br />

φ 0<br />

− φ i−1 γ (n−1)<br />

i−2<br />

] 1<br />

2<br />

− · · · − φ 1 γ (n−1)<br />

0<br />

(0 ≤ j ≤ n − 2),<br />

] 1<br />

2<br />

,<br />

where φ 0 , · · · , φ i−1 denote the coefficients of recursi<strong>on</strong> in (α 0 , · · · , α 2i−2 ) ∧ and γ (j)<br />

m<br />

(0 ≤ m ≤ i − 1) are the moments corresp<strong>on</strong>ding to the weight sequence<br />

(x j , · · · , x 1 , α 0 , · · · , α k−j ) ∧ with γ (0)<br />

m = γ m .<br />

142


CHAPTER 4.<br />

WEIGHTED SHIFTS<br />

We now observe that the determinati<strong>on</strong> of k-hyp<strong>on</strong>ormality and subnormality<br />

for can<strong>on</strong>ical perturbati<strong>on</strong>s of recursive shifts falls within the scope of the theory of<br />

extensi<strong>on</strong>s.<br />

Corollary 4.5.5. Let α ≡ {α n } ∞ n=0 = (α 0 , · · · , α k ) ∧ . If W α ′ is a perturbati<strong>on</strong> of W α<br />

at the j-th weight then<br />

W α ′ is subnormal ⇐⇒<br />

{<br />

W α ′ is ([ k+1<br />

2<br />

] + 1)-hyp<strong>on</strong>ormal (j = 0)<br />

W α ′ is ([ k+1<br />

.<br />

2<br />

] + 2)-hyp<strong>on</strong>ormal (j ≥ 1)<br />

Proof. Observe that if j = 0 then α ′ = x, (α 1 , · · · , α k+1 ) ∧ and if instead j ≥ 1 then<br />

α ′ = α 0 , · · · , α j−1 , x, (α j+1 , · · · , α j+k+1 ) ∧ . Thus the result immediately follows from<br />

Theorem 4.5.3.<br />

In Corollary 4.5.5, we showed that if α(x) is a can<strong>on</strong>ical rank-<strong>on</strong>e perturbati<strong>on</strong><br />

of a recursive weight sequence then subnormality and k-hyp<strong>on</strong>ormality for the corresp<strong>on</strong>ding<br />

shift coincide. We now c<strong>on</strong>sider a c<strong>on</strong>verse - an “extremality” problem:<br />

Let α(x) be a can<strong>on</strong>ical rank-<strong>on</strong>e perturbati<strong>on</strong> of a weight sequence α. If there exists<br />

k ≥ 1 such that (k+1)-hyp<strong>on</strong>ormality and k-hyp<strong>on</strong>ormality for the corresp<strong>on</strong>ding shift<br />

W α(x) coincide, does it follow that α(x) is recursively generated <br />

In [CuF3], the following extremality criteri<strong>on</strong> was established.<br />

Lemma 4.5.6. (Extremality Criteri<strong>on</strong>) [CuF3, Theorem 5.12, Propositi<strong>on</strong> 5.13] Let<br />

α be a weight sequence and let k ≥ 1.<br />

(i) If W α is k-extremal (i.e., det A(j, k) = 0 for all j ≥ 0) then W α is recursive<br />

subnormal.<br />

(ii) If W α is k-hyp<strong>on</strong>ormal and if det A(i 0 , j 0 ) = 0 for some i 0 ≥ 0 and some<br />

j 0 < k then W α is recursive subnormal.<br />

In particular, Lemma 4.5.6 (ii) shows that if W α is subnormal and if detA(i 0 , j 0 ) =<br />

0 for some i ≥ 0 and some j ≥ 0 then W α is recursive subnormal.<br />

We now answer the above questi<strong>on</strong> affirmatively.<br />

Theorem 4.5.7. Let α ≡ {α n } ∞ n=0 be a weight sequence and let α j (x) be a can<strong>on</strong>ical<br />

perturbati<strong>on</strong> of α in the j-th weight. Write<br />

H k := {x ∈ R + : W αj(x) is k-hyp<strong>on</strong>ormal}.<br />

If H k = H k+1 for some k ≥ 1 and x ∈ H k , then α j (x) is recursively generated, i.e.,<br />

W αj(x) is recursive subnormal.<br />

Proof. Suppose H k = H k+1 and let H k := sup x H k . To avoid triviality we assume<br />

α j−1 < x < α j+1 .<br />

Case 1 (j = 0): In this case, clearly Hk 2 is the n<strong>on</strong>zero root of the equati<strong>on</strong><br />

detA(0, k) = 0 and for x ∈ (0, H k ], W α0(x) is k-hyp<strong>on</strong>ormal. By assumpti<strong>on</strong> H k =<br />

143


CHAPTER 4.<br />

WEIGHTED SHIFTS<br />

H k+1 , so W α0(H k+1 ) is (k + 1)-hyp<strong>on</strong>ormal. The result now follows from Lemma 4.5.6<br />

(ii).<br />

Case 2 (j ≥ 1): Let A x (n, k) denote the Hankel matrix corresp<strong>on</strong>ding to α j (x).<br />

Since W αj (x) is (k + 1)-hyp<strong>on</strong>ormal for x ∈ H k , we have that A x (n, k + 1) ≥ 0 for all<br />

n ≥ 0 and all x ∈ H k . Observe that if n ≥ j + 1 then<br />

⎡<br />

⎤<br />

˜γ n−j−1 . . . ˜γ n−j−1+k<br />

A x (n, k) = α0 2 · · · αj−1 2 x 2 ⎢<br />

⎣<br />

.<br />

⎥<br />

. ⎦ ,<br />

˜γ n−j−1+k . . . ˜γ n−j−1+2k<br />

where ˜γ ∗ denotes the moments corresp<strong>on</strong>ding to the subsequence α j+1 , α j+2 , · · · .<br />

Therefore for n ≥ j + 1, the positivity of A x (n, k) is independent of the values of<br />

x > 0. This gives<br />

W αj (x) is k-hyp<strong>on</strong>ormal ⇐⇒ A x (n, k) ≥ 0 for all n ≤ j.<br />

Write<br />

H (i)<br />

k := {<br />

x : detA x (i, k) ≥ 0 and α j−1 < x < α j+1<br />

}<br />

(0 ≤ i ≤ j)<br />

and<br />

H (i)<br />

k<br />

= sup H (i)<br />

k<br />

x<br />

(0 ≤ i ≤ j).<br />

Since det A x (i, k) is a polynomial in x we have detA (i) H<br />

(i, k) = 0. Observe that<br />

k<br />

∩ j i=0 H(i) k = H k and sup H (i)<br />

k<br />

= H k .<br />

0≤i≤j<br />

Since by [CuL2, Theorem 2.11], H k is a closed interval, it follows that H k ∈ H k . Say,<br />

H k = H (p)<br />

k<br />

for some 0 ≤ p ≤ j. Then detA (p) H<br />

(p, k) = 0 and W (p)<br />

k<br />

α(H<br />

is (k + 1)-<br />

k )<br />

hyp<strong>on</strong>ormal. Therefore it follows from Lemma 4.1 (ii) that W α is recursive subnormal.<br />

This completes the proof.<br />

We c<strong>on</strong>clude this secti<strong>on</strong> with two corollaries of independent interest.<br />

Corollary 4.5.8. With the notati<strong>on</strong>s in Theorem 4.5.7, if j ≥ 1 and H k = H k+1 for<br />

some k, then H k is a singlet<strong>on</strong> set.<br />

Proof. By [CuL2, Theorem 2.2],<br />

H ∞ := {x ∈ R + : W αj(x) is subnormal}<br />

is a singlet<strong>on</strong> set. By Theorem 4.5.7, we have that H k = H ∞ .<br />

144


CHAPTER 4.<br />

WEIGHTED SHIFTS<br />

Corollary 4.5.9. If W α is a n<strong>on</strong>recursive shift with weight sequence α = {α n } ∞ n=0<br />

and if α(x) is a can<strong>on</strong>ical rank-<strong>on</strong>e perturbati<strong>on</strong> of α, then for every k ≥ 1 there<br />

always exists a gap between k-hyp<strong>on</strong>ormality and (k + 1)-hyp<strong>on</strong>ormality for W α(x) .<br />

More c<strong>on</strong>cretely, if we let<br />

H k := {x : W α(x) is k-hyp<strong>on</strong>ormal}<br />

then {H k } ∞ k=1<br />

is a strictly decreasing nested sequence of closed intervals in (0, ∞)<br />

except when the perturbati<strong>on</strong> occurs in the first weight. In that case, the intervals are<br />

of the form (0, H k ].<br />

Proof. Straightforward from Theorem 4.5.7.<br />

We now illustrate our result with some examples. C<strong>on</strong>sider<br />

α(y, x) : √ y, √ x, ( √ a, √ b, √ c) ∧<br />

(a < b < c).<br />

Without loss of generality, we assume a = 1. Observe that<br />

H 2 (1, √ b, √ √<br />

bc − b<br />

2 (<br />

c) =<br />

and H 2 ( √ x, 1, √ ) 2 x(b − 1)<br />

b) =<br />

1 + bc − 2b<br />

(x − 1) 2 := f(x).<br />

+ (b − 1)<br />

√<br />

According to Theorem , W α(y,x) is 2-hyp<strong>on</strong>ormal if and <strong>on</strong>ly if 0 < x ≤<br />

and 0 < y ≤ f(x). To completely describe the regi<strong>on</strong><br />

we study the graph of f. Observe that<br />

R := {(x, y) : W α(y,x) is 2-hyp<strong>on</strong>ormal},<br />

f ′ (x) = (b − 1)(b − x2 )<br />

(b − 2x + x 2 ) 2 > 0 and f ′′ (x) = 2(b − 1)(2b − 3bx + x3 )<br />

(b − 2x + x 2 ) 3 .<br />

bc−b 2<br />

1+bc−2b<br />

Note that b − 2x + x 2 = (b − 1) + (1 − x) 2 > 0 and f ′ ( √ b) = 0. To c<strong>on</strong>sider the<br />

sign of f ′′ , we let g(x) := 2b − 3bx + x 3 . Then g ′ ( √ b) = 0, g(1) = −b + 1 < 0, and<br />

g ′′ (x) > 0 (x > 0). Hence there exists x 0 ∈ (0, 1) such that f ′′ (x 0 ) = 0, f ′′ (x) > 0 <strong>on</strong><br />

0 < x < x 0 , and f ′′ (x) < 0 <strong>on</strong> x 0 < x ≤ 1. We investigate which of the two values x 0<br />

or ˜H := H 2 (1, √ b, √ c) 2 is bigger. By a simple calculati<strong>on</strong>, we have<br />

where<br />

g( ˜H) = (−1 + b)b · g 1(b, c)<br />

(1 − 2b + bc) 3 ,<br />

g 1 (b, c) = −(2 − 10b + 17b 2 − 11b 3 + b 4 + 3bc − 9b 2 c + 9b 3 c − 3b 3 c 2 + b 2 c 3 ).<br />

For notati<strong>on</strong>al c<strong>on</strong>venience we let b := 1 + h, c := 1 + h + k. Then<br />

g 1 (b, c) = 2h 5 + (3h 3 + 3h 4 )k + (−1 − 2h − h 2 )k 3 .<br />

145


CHAPTER 4.<br />

WEIGHTED SHIFTS<br />

If h is sufficiently small (i.e., b is sufficiently close to 1), then g 1 < 0, i.e., ˜H > x0 .<br />

If k is sufficiently small (i.e., b is sufficiently close to c), then g 1 > 0, i.e., ˜H < x0 .<br />

Thus, if ˜H ≤ x 0 , then f is c<strong>on</strong>vex downward <strong>on</strong> x ≤ ˜H. If ˜H > x 0 , then (x 0 , f(x 0 ))<br />

is an inflecti<strong>on</strong> point. Thus, f is c<strong>on</strong>vex downward <strong>on</strong> 0 < x < x 0 and c<strong>on</strong>vex<br />

upward <strong>on</strong> x 0 < x ≤ ˜H. Moreover, W α(y,x) is 2-hyp<strong>on</strong>ormal if and <strong>on</strong>ly if (x, y) ∈<br />

{(x, y)|0 ≤ y ≤ f(x), 0 < x ≤ ˜H}, and W α(y,x) is k-hyp<strong>on</strong>ormal (k ≥ 3) if and <strong>on</strong>ly<br />

if (x, y) ∈ {( ˜H, y)|0 ≤ y ≤ f(x)}.<br />

Example 4.5.10. (b = 2, c = 3)<br />

f(x) =<br />

Notice that f is c<strong>on</strong>vex in this case.<br />

x<br />

1 + (1 − x) 2 .<br />

Example 4.5.11. (b = 11<br />

10 , c = 10)<br />

f(x) =<br />

x<br />

11 − 20x + 10x 2 .<br />

In this case, f has an inflecti<strong>on</strong> point at x ≈ 0.633892.<br />

146


CHAPTER 4.<br />

WEIGHTED SHIFTS<br />

4.6 The Completi<strong>on</strong> Problem<br />

We begin with:<br />

Definiti<strong>on</strong> 4.6.1. Let α : α 0 , · · · α m , (m ≥ 0) be an initial segment of positive<br />

weights and let ϖ = {ϖ n } ∞ n=0 be a bounded sequence. We say that W ϖ is a completi<strong>on</strong><br />

of α if<br />

ϖ n = α n (0 ≤ n ≤ m)<br />

and we write α ⊆ ϖ.<br />

The completi<strong>on</strong> problem for a property (P ) entails finding necessary and sufficient<br />

c<strong>on</strong>diti<strong>on</strong>s <strong>on</strong> α to ensure the existence of a weight sequence ϖ ⊃ α such that<br />

W α satisfies (P ).<br />

In 1966, Stampfli [Sta3] showed that for arbitrary α 0 < α 1 < α 2 , there exists a<br />

subnormal shift W α whose first three weights are α 0 , α 1 , α 2 ; he also proved that<br />

given four or more weights it may not be possible to find a subnormal completi<strong>on</strong>.<br />

Theorem 4.6.2. [CuF3]<br />

(a)(Minimality of Norm)<br />

{<br />

}<br />

||Ŵα || = inf ∥W ω ∥ : α ⊆ ω and W ω is subnormal .<br />

(b) (Minimality of Moments) If α ⊆ ω and W ω is subnormal then<br />

∫<br />

∫<br />

t n dµ̂α (t) ≤ t n dµ ω (t) (n ≥ 0).<br />

Proof. See [CuF3].<br />

Theorem 4.6.3. (Subnormal Completi<strong>on</strong> Problem) [CuF3] If α : α 0 , α 1 , · · · , α m (m ≥<br />

0) is an initial segment then the followings are equivalent:<br />

(i) α has a subnormal completi<strong>on</strong>.<br />

(ii) α has a recursively generated subnormal completi<strong>on</strong>.<br />

(iii) The Hankel matrices<br />

⎡<br />

⎤<br />

γ 0 γ 1 · · · γ k<br />

γ 1 γ k+1<br />

A(k) := ⎢<br />

⎣<br />

.<br />

⎥<br />

. ⎦<br />

γ k γ k+1 · · · γ 2k<br />

⎡<br />

⎤<br />

γ 1 γ 2 · · · γ 2<br />

and B(l − 1) := γ 1 γ l+1<br />

⎢<br />

⎣<br />

.<br />

⎥<br />

. ⎦<br />

γ l γ l+1 · · · γ 2l<br />

147


CHAPTER 4.<br />

WEIGHTED SHIFTS<br />

are both positive<br />

( [<br />

k =<br />

] [<br />

m+1<br />

2<br />

, l =<br />

⎡ ⎤<br />

γ k+1<br />

⎢<br />

⎣<br />

⎥<br />

. ⎦<br />

γ 2k+1<br />

] )<br />

m<br />

2<br />

+ 1 and the vector<br />

⎡ ⎤<br />

(<br />

γ k+1<br />

)<br />

⎢<br />

resp. ⎣<br />

⎥<br />

. ⎦<br />

γ 2k<br />

is in the range of A(k) (resp. B(l − 1)) when m is even (resp. odd).<br />

Theorem 4.6.4. (k-Hyp<strong>on</strong>ormal Completi<strong>on</strong> Problem) [CuF3] If α : α 0 , α 1 · · · , α 2m (m ≥<br />

1) is an initial segment then for 1 ≤ k ≤ m, the followings are equivalent:<br />

(i) α has k-hyp<strong>on</strong>ormal completi<strong>on</strong>.<br />

(ii) The Hankel matrix<br />

⎡<br />

⎤<br />

γ j · · · γ j+k<br />

⎢<br />

A(j, k) :=<br />

⎥<br />

⎣ .<br />

. ⎦<br />

γ j+k · · · γ j+2k<br />

is positive for all j, 0 ≤ j ≤ 2m − 2k + 1 and the vector<br />

⎡ ⎤<br />

⎢<br />

⎣<br />

γ 2m−k+2<br />

.<br />

γ 2m+1<br />

is in the range of A(2m − 2k + 2, k − 1).<br />

Theorem 4.6.5. (Quadraically Hyp<strong>on</strong>ormal Completi<strong>on</strong> Problem) Let m ≥ 2 and<br />

let α : α 0 < α 1 , · · · < α m be an initial segment. Then the followings are equivalent:<br />

(i) α has a quadratically hyp<strong>on</strong>ormal completi<strong>on</strong>.<br />

(ii) D m−1 (t) > 0 for all t ≥ 0.<br />

Moreover, a quadratically hyp<strong>on</strong>ormal completi<strong>on</strong> ω of L can be obtained by<br />

where α m+1 is chosen sufficiently large.<br />

⎥<br />

⎦<br />

ω : α 0 , α 1 , · · · , α m−2 (α m−1 , α m , α m+1 ) ∧ ,<br />

Proof. First of all, note that D m−1 (t) > 0 for all t ≥ 0 if and <strong>on</strong>ly if d n (t) > 0<br />

for all t ≥ 0 and for n = 0, · · · , m − 1; this follows from the Nested Determinants<br />

Test (see [12, Remark 2.4]) or Choleski’s Algorithm (see [CuF2, Propositi<strong>on</strong> 2.3]). A<br />

straightforward calculati<strong>on</strong> gives<br />

d 0 (t) = α 2 0 + α 2 0α 2 1 t<br />

d 1 (t) = α 2 0(α 2 1 − α 2 0) + α 2 0α 2 1(α 2 2 − α 2 0) t + α 2 0α 4 1α 2 2 t 2<br />

d 2 (t) = α0(α 2 1 2 − α0)(α 2 2 2 − α1) 2 + α0α 2 2(α 2 1 2 − α0)(α 2 3 2 − α1) 2 t<br />

{<br />

}<br />

+ α0α 2 1α 2 2<br />

2 α3(α 2 2 2 − α0) 2 − α1(α 2 1 2 − α0)<br />

2 t 2 + α0α 2 1α 4 2(α 2 2α 2 3 2 − α1α 2 0) 2 t 3 ,<br />

148


CHAPTER 4.<br />

WEIGHTED SHIFTS<br />

which shows that all coefficients of d i (i = 0, 1, 2) are positive, so that d i (t) > 0 for<br />

all t ≥ 0 and i = 0, 1, 2.<br />

Now suppose α has a quadratically hyp<strong>on</strong>ormal completi<strong>on</strong>. Then evidently,<br />

d n (t) ≥ 0 for all t ≥ 0 and all n ≥ 0. In view of the propagati<strong>on</strong> property, {α n } ∞ n=m<br />

is strictly increasing. Thus d n (0) = u 0 · · · u n = ∏ n<br />

i=0 (α2 i − α2 i−1 ) > 0 for all n ≥ 0.<br />

If d n0 (t 0 ) = 0 for some t 0 > 0 and the first such n 0 > 0 (3 ≤ n 0 ≤ m − 1), then<br />

(1.6) implies that 0 ≤ d n0 +1(t 0 ) = −|r n0 (t 0 )| 2 d n0 −1(t 0 ) ≤ 0, which forces r n0 (t 0 ) = 0,<br />

so that α n0+1 = α n0−1, a c<strong>on</strong>tradicti<strong>on</strong>. Therefore d n (t) > 0 for all t ≥ 0 and for<br />

n = 0, · · · , m − 1. This proves the implicati<strong>on</strong> (i) ⇒ (ii).<br />

For the reverse implicati<strong>on</strong>, we must find a bounded sequence {α n } ∞ n=m+1 such<br />

that d n (t) ≥ 0 for all t ≥ 0 and all n ≥ 0. Suppose d n (t) > 0 for all t ≥ 0 and for<br />

n = 0, · · · , m − 1. We now claim that there exists a c<strong>on</strong>stant M k > 0 for which<br />

d k−1 (t)<br />

d k (t)<br />

≤ M k for all t ≥ 0 and for k = 1, · · · , m − 1.<br />

Indeed, since d k−1(t)<br />

d k (t)<br />

is a c<strong>on</strong>tinuous functi<strong>on</strong> of t <strong>on</strong> [0, ∞), and deg (d k−1 ) < deg (d k ),<br />

it follows that<br />

d k−1 (t)<br />

max<br />

t∈[0, ∞) d k (t)<br />

{<br />

≤ max 1, max<br />

t∈[0, ξ]<br />

}<br />

d k−1 (t)<br />

=: M k ,<br />

d k (t)<br />

where ξ is the largest root of the equati<strong>on</strong> d k−1 (t) = d k (t). Now a straightforward<br />

calculati<strong>on</strong> shows that<br />

d m (t) = q m (t)d m−1 (t) − |r m−1 (t)| 2 d m−2 (t)<br />

[ (<br />

) ]<br />

d m−2 (t)<br />

= u m + v m − w m−1 t d m−1 (t) .<br />

d m−1 (t)<br />

d<br />

So if we write e m (t) := v m − w m−2 (t)<br />

m−1 d m−1 (t) , then by (3.1), e m(t) ≥ v m − w m−1 M m−1 .<br />

Now choose α m+1 so that v m − w m−1 M m−1 > 0, i.e.,<br />

α 2 m+1 > max<br />

{<br />

α 2 m,<br />

αm−1<br />

2 [<br />

M (α<br />

2<br />

αm<br />

2 m − αm−2) 2 2 + αm−2<br />

2 ] } ,<br />

where M := max t∈[0,∞)<br />

d m−2 (t)<br />

d m−1(t) . Then e m(t) ≥ 0 for all t ≥ 0, so that<br />

d m (t) = (u m + e m (t) t)d m−1 (t) ≥ u m d m−1 (t) > 0.<br />

Therefore, d m−1 (t) ≤ d m(t)<br />

u m<br />

. With α m+2 to be chosen later, we now c<strong>on</strong>sider d m+1 .<br />

We have<br />

d m+1 (t) = q m+1 (t)d m (t) − |r m (t)| 2 d m−1 (t)<br />

≥ 1 [<br />

]<br />

u m q m+1 (t) − |r m (t)| 2 d m (t)<br />

u m<br />

= 1 [<br />

]<br />

u m u m+1 + (u m v m+1 − w m )t d m (t)<br />

u m<br />

= u m+1 d m (t) + t<br />

u m<br />

(u m v m+1 − w m ) d m (t).<br />

149


CHAPTER 4.<br />

WEIGHTED SHIFTS<br />

Write f m+1 := u m v m+1 −w m . If we choose α m+2 such that f m+1 ≥ 0, then d m+1 (t) ≥<br />

0 for all t > 0. In particular we can choose α m+2 so that f m+1 = 0. i.e., u m v m+1 =<br />

w m , or<br />

α 2 m+2 := α2 m(α 2 m+1 − α 2 m−1) 2 + α 2 m−1α 2 m(α 2 m − α 2 m−1)<br />

α 2 m+1 (α2 m − α 2 m−1 ) ,<br />

or equivalently,<br />

α 2 m+2 := α 2 m+1 + α2 m−1(α 2 m+1 − α 2 m) 2<br />

α 2 m+1 (α2 m − α 2 m−1 ) .<br />

In this case, d m+1 (t) ≥ u m+1 d m (t) ≥ 0. Repeating the argument (with α m+3 to be<br />

chosen later), we obtain<br />

d m+2 (t) = q m+2 (t)d m+1 (t) − |r m+1 (t)| 2 d m (t)<br />

≥ 1 [<br />

u m+1 q m+2 (t) − |r m+1 (t)|<br />

]d 2 m+1 (t)<br />

u m+1<br />

= 1 [<br />

]<br />

u m+1 u m+2 + (u m+1 v m+2 − w m+1 ) t d m+1 (t)<br />

u m+1<br />

= u m+2 d m+1 (t) + t<br />

u m+1<br />

(u m+1 v m+2 − w m+1 ) d m+1 (t).<br />

Write f m+2 := u m+1 v m+2 − w m+1 . If we choose α m+3 such that f m+2 = 0, i.e.,<br />

α 2 m+3 := α 2 m+2 + α2 m(α 2 m+2 − α 2 m+1) 2<br />

α 2 m+2 (α2 m+1 − α2 m) ,<br />

then d m+2 (t) ≥ u m+2 d m+1 (t) ≥ 0. C<strong>on</strong>tinuing this process with the sequence {α n } ∞ n=m+2<br />

defined recursively by<br />

and<br />

φ 1 := α2 m(αm+1 2 − αm−1)<br />

2<br />

αm 2 − αm−1<br />

2 , φ 0 := − α2 m−1αm(α 2 m+1 2 − αm)<br />

2<br />

αm 2 − αm−1<br />

2<br />

α 2 n+1 := φ 1 + φ 0<br />

α 2 n<br />

(n ≥ m + 1),<br />

we obtain that d n (t) ≥ 0 for all t > 0 and all n ≥ m + 2. On the other hand, by<br />

an argument of [Sta3, Theorem 5], the sequence {α n } ∞ n=m+2 is bounded. Therefore,<br />

a quadratically hyp<strong>on</strong>ormal completi<strong>on</strong> {α n } ∞ n=0 is obtained. The above recursive<br />

relati<strong>on</strong> shows that the sequence {α n } ∞ n=m+2 is obtained recursively from α m−1 , α m<br />

and α m+1 , that is, {α n } ∞ n=m−1 = (α m−1 , α m , α m+1 ) ∧ . This completes the proof.<br />

Given four weights α : α 0 < α 1 < α 2 < α 3 , it may not be possible to find<br />

a 2-hyp<strong>on</strong>ormal completi<strong>on</strong>. In fact, by the preceding criteri<strong>on</strong> for subnormal and<br />

k-hyp<strong>on</strong>ormal completi<strong>on</strong>s, the following statements are equivalent:<br />

(i) α has a subnormal completi<strong>on</strong>;<br />

(ii) α has a 2-hyp<strong>on</strong>ormal completi<strong>on</strong>;<br />

150


CHAPTER 4.<br />

WEIGHTED SHIFTS<br />

⎡<br />

(iii) det ⎣ γ ⎤<br />

0 γ 1 γ 2<br />

γ 1 γ 2 γ 3<br />

⎦ ≥ 0.<br />

γ 2 γ 3 γ 4<br />

By c<strong>on</strong>trast, a quadratically hyp<strong>on</strong>ormal completi<strong>on</strong> always exists for four weights.<br />

Corollary 4.6.6. For arbitrary α : α 0 < α 1 < α 2 < α 3 , there always exists a<br />

quadratically hyp<strong>on</strong>ormal completi<strong>on</strong> ω of α.<br />

Proof. In the proof of Theorem 4.6.5, we showed that d n (t) > 0 for all t ≥ 0 and for<br />

n = 0, 1, 2. Thus the result immediately follows from Theorem 4.6.5.<br />

Remark. To discuss the hypothesis α 0 < α 1 < · · · < α m in Theorem 4.6.5, we c<strong>on</strong>sider<br />

the case where α : α 0 , α 1 , · · · , α m admits equal weights:<br />

(i) If α 0 < α 1 = · · · = α m then there exists a trivial quadratically hyp<strong>on</strong>ormal<br />

completi<strong>on</strong> (in fact, a subnormal completi<strong>on</strong>) ω : α 0 < α 1 = · · · = α n = α n+1 = · · · .<br />

(ii) If {α n } m n=0 is such that α j = α j+1 for some j = 1, 2, · · · , m − 1, and α j ≠ α k<br />

for some 1 ≤ j, k ≤ m, then by the propagati<strong>on</strong> property there does not exist any<br />

quadratically hyp<strong>on</strong>ormal completi<strong>on</strong> of α.<br />

(iii) If α 0 = α 1 , the c<strong>on</strong>clusi<strong>on</strong> of 4.6.5 may fail: for example, if α : 1, 1, 2, 3<br />

then d n (t) > 0 for all t ≥ 0 and for n = 0, 1, 2, whereas α admits no quadratically<br />

hyp<strong>on</strong>ormal completi<strong>on</strong> because we must have α 2 2 < 2.<br />

Problem. Given α : α 0 = α 1 < α 2 < · · · < α m , find necessary and sufficient c<strong>on</strong>diti<strong>on</strong>s<br />

for the existence of a quadratically hyp<strong>on</strong>ormal completi<strong>on</strong> ω of α.<br />

In [CuJ], related to the above problem, weighted shifts of the form 1, (1, √ b, √ c) ∧<br />

have been studied and their quadratic hyp<strong>on</strong>ormality completely characterized in<br />

terms of b and c.<br />

Remark. In Theorem 4.6.5, the recursively quadratically hyp<strong>on</strong>ormal completi<strong>on</strong><br />

requires a sufficiently large α m+1 . One might c<strong>on</strong>jecture that if the quadratically<br />

hyp<strong>on</strong>ormal completi<strong>on</strong> of α : α 0 < α 1 < α 2 < · · · < α m exists, then<br />

ω : α 0 , · · · , α m−3 , (α m−2 , α m−1 , α m ) ∧<br />

√<br />

9<br />

is such a completi<strong>on</strong>. However, if α :<br />

10 , √ 1, √ 2, √ √<br />

9<br />

3 then ω :<br />

10 , (√ 1, √ 2, √ 3) ∧<br />

is not quadratically hyp<strong>on</strong>ormal (by [CuF3, Theorem 4.3]), even though by Corollary<br />

4.6.6 a quadratically hyp<strong>on</strong>ormal completi<strong>on</strong> does exist.<br />

We c<strong>on</strong>clude this secti<strong>on</strong> by establishing that for five or more weights, the gap<br />

between 2-hyp<strong>on</strong>ormal and quadratically hyp<strong>on</strong>ormal completi<strong>on</strong>s can be extremal.<br />

Propositi<strong>on</strong> 4.6.7. For a < b < c, let η : ( √ a, √ b, √ c) ∧ be a recursively generated<br />

weight sequence, and c<strong>on</strong>sider α(x) : √ a, √ b, √ c, √ x, η 4 (five weights). Then<br />

(i) α has a subnormal completi<strong>on</strong> ⇐⇒ x = η 3 ;<br />

(ii) α has a 2-hyp<strong>on</strong>ormal completi<strong>on</strong> ⇐⇒ x = η 3 ;<br />

(iii) α has a quadratically hyp<strong>on</strong>ormal completi<strong>on</strong> ⇐⇒ c < x < η 2 4.<br />

151


CHAPTER 4.<br />

WEIGHTED SHIFTS<br />

Proof. Asserti<strong>on</strong>s (i) and (ii) follow from the argument used in the proof of . For<br />

asserti<strong>on</strong> (iii), observe that by Theorem 3.1, α has a quadratically hyp<strong>on</strong>ormal completi<strong>on</strong><br />

if and <strong>on</strong>ly if d 3 (t) > 0 for all t ≥ 0. Without loss of generality, we write<br />

a = 1, b = 1 + r, c = 1 + r + s, and x = 1 + r + s + u (r > 0, s > 0, u > 0). A<br />

straightforward calculati<strong>on</strong> using Mathematica shows that the Maclaurin coefficients<br />

c(3, i) of d 3 (t) are given by<br />

c(3, 0) = rsu;<br />

c(3, 1) = s 3 (r + s)(1 + r + s + u)(r + r 2 + 2rs + s 2 ) −1 ;<br />

c(3, 2) = (1 + r + s)(s 4 + rsu + 4r 2 su + 5r 3 su + 2r 4 su + 2rs 2 u + 7r 2 s 2 u + 5r 3 s 2 u<br />

+ 2s 3 u + 4rs 3 u + 4r 2 s 3 u + s 4 u + rs 4 u + r 2 u 2 + 2r 3 u 2 + r 4 u 2 + 3r 2 su 2<br />

+ 3r 3 su 2 + 2rs 2 u 2 + 3r 2 s 2 u 2 + s 3 u 2 + rs 3 u 2 )(r + r 2 + 2rs + s 2 ) −1 ;<br />

c(3, 3) = (1 + r)(r + s)(1 + r + s)(1 + r + s + u)(r 2 s 2 + r 3 s 2 + s 3 + 2rs 3<br />

+ 2r 2 s 3 + s 4 + rs 4 + r 2 u + 2r 3 u + r 4 u + 3r 2 su + 3r 3 su + 2rs 2 u + 3r 2 s 2 u<br />

+ s 3 u + rs 3 u)(r 2 + r 3 + 2r 2 s + rs 2 ) −1 ; and<br />

c(3, 4) = (1 + r) 2 (1 + r + s)(r + r 2 + 2s + 2rs + s 2 + u + ru + su)(r 2 s + 2r 3 s + r 4 s<br />

+ rs 2 + 4r 2 s 2 + 3r 3 s 2 + s 3 + 3rs 3 + 3r 2 s 3 + s 4 + rs 4 + r 2 u + 2r 3 u + r 4 u<br />

+ 3r 2 su + 3r 3 su + 2rs 2 u + 3r 2 s 2 u + s 3 u + rs 3 u)(r 2 + r 3 + 2r 2 s + rs 2 ) −1 .<br />

This readily shows that for c < x < α 2 4, all Maclaurin coefficients of d 3 (t) are positive,<br />

so that d 3 (t) > 0 for all t ≥ 0. Moreover if x = c or α 2 4 then Theorem 1.2 shows that<br />

no quadratically hyp<strong>on</strong>ormal completi<strong>on</strong> exists. This proves asserti<strong>on</strong> (iii).<br />

152


CHAPTER 4.<br />

WEIGHTED SHIFTS<br />

4.7 Comments and Problems<br />

Problem 4.1. Let T x be a weighted shift with weights α ≡ {α n } given by<br />

α : x,<br />

√<br />

2<br />

3 , √<br />

3<br />

4 , √<br />

4<br />

5 , · · · .<br />

Describe the set {x : T x is cubically hyp<strong>on</strong>ormal}. More generally, describe {x :<br />

T x is weakly k-hyp<strong>on</strong>ormal}.<br />

Problem 4.2. Let T be the weighted shift with weights α ≡ {α n } given by<br />

If T is cubically hyp<strong>on</strong>ormal, is α flat<br />

α 0 = α 1 ≤ α 2 ≤ α 3 ≤ · · · .<br />

Problem 4.3. (Minimality of Weights Problem) If α : α 0 , α 1 , · · · , α 2k admits a<br />

subnormal completi<strong>on</strong> and if α ⊆ ω with W ω subnormal, does it follow that<br />

α n ≤ ω n for all n ≥ 0 <br />

A combinati<strong>on</strong> of Theorem 4.6.2 (a) and (b) show that α n ≤ ω n for 0 ≤ n ≤ 2k + 1<br />

and also for large n.<br />

Problem 4.4. Given α : α 0 = α 1 < α 2 < · · · < α m , find necessary and sufficient<br />

c<strong>on</strong>diti<strong>on</strong>s for the existence of a quadratically hyp<strong>on</strong>ormal completi<strong>on</strong> ω of α.<br />

In [CuF2] it was shown that<br />

∃ 1 < b < c such that W 1(1,<br />

√<br />

b,<br />

√ c) ∧<br />

is quadratically hyp<strong>on</strong>ormal. In fact, it was shown that if we write<br />

H 2 := {(b, c) : W 1(1,<br />

√<br />

b,<br />

√ c) ∧<br />

is quadratically hyp<strong>on</strong>ormal}<br />

then<br />

where<br />

H 2 := {(b, c) : b(bc − 1) + b(b − 1)(c − 1)K − (b − 1) 2 K 2 ≥ 0},<br />

K =<br />

b(c − 1)<br />

(b(c 2 − 1) + √ )<br />

b 2 (c − 1) 2 − 4b(b − 1)(c − b)<br />

2(b − 1) 2 .<br />

(c − b)<br />

Problem 4.5. Does there exists 1 < b < c such that W 1,(1,<br />

√<br />

b,<br />

√ c) ∧ is cubically<br />

hyp<strong>on</strong>ormal More generally, describe the set<br />

{(b, c) : W 1,(1,<br />

√<br />

b,<br />

√ c) ∧<br />

is cubically hyp<strong>on</strong>ormal}.<br />

153


CHAPTER 4.<br />

WEIGHTED SHIFTS<br />

We remember the following questi<strong>on</strong> (Due to P. Halmos):<br />

Whether every polynomially hyp<strong>on</strong>ormal operator is subnormal <br />

In 1993, R. Curto and M. Putinar [CP2] have answered it negatively:<br />

There exits a polynomially hyp<strong>on</strong>ormal operator which is not 2-hyp<strong>on</strong>ormal.<br />

In 1989, S. M. McCullough and V. Paulsen [McCP] proved the following: Every<br />

polynomially hyp<strong>on</strong>ormal operator is subnormal if and <strong>on</strong>ly if every polynomially hyp<strong>on</strong>ormal<br />

weighted shift is subnormal.<br />

However we did not find a c<strong>on</strong>crete example of such a weighted shift:<br />

Problem 4.6. Find a weighted shift which is polynomially hyp<strong>on</strong>ormal but not subnormal.<br />

Problem 4.7. Does there exists a polynomially hyp<strong>on</strong>ormal weighted shift which is<br />

not 2-hyp<strong>on</strong>ormal <br />

Let B 1 be the weighted shift whose weight are given by<br />

√ x,<br />

√<br />

2<br />

3 , √<br />

5<br />

4 , √<br />

4<br />

5 , · · ·<br />

Let B 2 be the weighted shift whose weight are given by<br />

√<br />

1<br />

2 , √ x,<br />

√<br />

3<br />

4 , √<br />

4<br />

5 , · · ·<br />

A straightforward calculati<strong>on</strong> shows that<br />

We c<strong>on</strong>jecture that<br />

B 1 subnormal ⇐⇒ 0 < x ≤ 1 2 ;<br />

B 1 2-hyp<strong>on</strong>ormal ⇐⇒ 0 < x ≤ 9 16 ;<br />

B 1 quadratically hyp<strong>on</strong>ormal ⇐⇒ 0 < x ≤ 2 3 ;<br />

B 2 subnormal ⇐⇒ x = 2 3 ;<br />

[<br />

B 2 2-hyp<strong>on</strong>ormal ⇐⇒ x ∈<br />

63 − √ 129<br />

,<br />

80<br />

]<br />

24<br />

.<br />

35<br />

9<br />

16 < sup{x : B 1 is polynomially hyp<strong>on</strong>ormal}<br />

24<br />

35 < sup{x : B 2 is polynomially hyp<strong>on</strong>ormal}<br />

154


CHAPTER 4.<br />

WEIGHTED SHIFTS<br />

Problem 4.8. Is the above c<strong>on</strong>verse true <br />

We here suggest related problems:<br />

Problem 4.9.<br />

(a) Does there exists a Toeplitz operator which is polynomially hyp<strong>on</strong>ormal but not<br />

subnormal <br />

(b) Classify the polynomially hyp<strong>on</strong>ormal operators with finite rank self commutators.<br />

(c) Is there an analogue of Berger’s theorem for polynomially hyp<strong>on</strong>ormal weighted<br />

shift <br />

An operator T ∈ B(H) is called M-hyp<strong>on</strong>ormal if<br />

∃ M > 0 such that ||(T − λ) ∗ x|| ≤ M ||(T −λ)x|| for any λ ∈ C and for any x ∈ H.<br />

If M ≤ 1 then M-hyp<strong>on</strong>ormality ⇒ hyp<strong>on</strong>ormality. It was shown [HLL] that it<br />

T ≡ W α is a weighted shift with weight sequence α then<br />

α is eventually increasing =⇒ T is hyp<strong>on</strong>ormal.<br />

We w<strong>on</strong>der if the c<strong>on</strong>verse is also true.<br />

Problem 4.10. (M-hyp<strong>on</strong>ormality of weighted shifts) Does it follow that<br />

W α is M-hyp<strong>on</strong>ormal =⇒ α is eventually increasing <br />

Problem 4.11 (Perturbati<strong>on</strong>s of weighted shifts) Let α be a strictly increasing weighted<br />

sequence.<br />

(a) If W α is k-hyp<strong>on</strong>ormal, dose it follow that W α is weakly k-hyp<strong>on</strong>ormal under<br />

small perturbati<strong>on</strong>s of the weighted shifts <br />

(b) Does it follow that the polynomiality of the weighted shifts is stable under small<br />

perturbati<strong>on</strong>s of the weighted sequence <br />

It was shown [CuL5] that the answer to Problem 4.10 (a) is affirmative if k = 2.<br />

155


CHAPTER 4.<br />

WEIGHTED SHIFTS<br />

156


Chapter 5<br />

Toeplitz <strong>Theory</strong><br />

5.1 Preliminaries<br />

5.1.1 Fourier Transform and Beurling’s Theorem<br />

A trig<strong>on</strong>ometric polynomial is a functi<strong>on</strong> p ∈ C(T) of the form ∑ n<br />

k=−n a kz k . It was<br />

well-known that the set of trig<strong>on</strong>ometric polynomials is uniformly dense in C(T) and<br />

hence is dense in L 2 (T). In fact, if e n := z n , (n ∈ Z) then {e n : n ∈ Z} forms<br />

an orth<strong>on</strong>ormal basis for L 2 (T). The Hardy space H 2 (T) is spanned by {e n : n =<br />

0, 1, 2, · · · }. Write H ∞ (T) := L ∞ (T) ∩ H 2 (T). Then H ∞ is a subalgebra of L ∞ .<br />

Let m :=the normalized Lebesgue measure <strong>on</strong> T and write L 2 := L 2 (T). If f ∈ L 2<br />

then the Fourier transform of f, ̂f : Z → C, is defined by<br />

∫<br />

̂f(n) ≡ ⟨f, e n ⟩ =<br />

T<br />

fz n dm = 1<br />

2π<br />

∫ 2π<br />

0<br />

f(t)e −int dt,<br />

which is called the n-th Fourier coefficient of f. By Parseval’s identity,<br />

∞∑<br />

f = ̂f(n)z n ,<br />

n=−∞<br />

which c<strong>on</strong>verges in the norm of L 2 . This series is called the Fourier series of f.<br />

Propositi<strong>on</strong> 5.1.1. We have:<br />

(i) f ∈ L 2 ⇒ ̂f ∈ l 2 (Z);<br />

(ii) If V : L 2 → l 2 (Z) is defined by V f = ̂f then V is an isomorphism.<br />

(iii) If W = N m <strong>on</strong> L 2 then V W V −1 is the bilateral shift <strong>on</strong> l 2 (Z).<br />

157


CHAPTER 5.<br />

TOEPLITZ THEORY<br />

Proof. (i) Since by Parseval’s identity, ∑ | ̂f(n)| 2 = ||f|| 2 < ∞, it follows ̂f ∈ l 2 (Z).<br />

(ii) We claim that ||V f|| = ||f||: indeed, ||V f|| 2 = || ̂f|| 2 = ∑ | ̂f(n)| 2 = ||f|| 2 . If<br />

f = z n then<br />

{<br />

0 if k ≠ n<br />

̂f(k) =<br />

1 if k = n,<br />

so that ̂f is the n-th basis vector in l 2 (Z). Thus ran V is dense and hence V is an<br />

isomorphism.<br />

(iii) If {e n } is an orth<strong>on</strong>ormal basis for l 2 (Z) then by (ii), V z n = e n . Thus<br />

V W z n = V (z n+1 ) = e n+1 = UV z n .<br />

If T ∈ B(H), write Lat T for the set of all invariant subspaces for T , i.e.,<br />

Lat T := {M ⊂ H : T M ⊂ M}.<br />

Theorem 5.1.2. If µ is a compactly supported measure <strong>on</strong> T and M ∈ LatN µ then<br />

M = ϕH 2 ⊕ L 2 (µ|∆),<br />

where ϕ ∈ L ∞ (µ) and ∆ is a Borel set of T such that ϕ|∆ = 0 a.e. and |ϕ| 2 µ =<br />

m(:=the normalized Lebesgue measure).<br />

Proof. See [C<strong>on</strong>3, p.121].<br />

Now c<strong>on</strong>sider the case where µ = m (in this case, N µ is the bilateral shift). Observe<br />

ϕ ∈ L 2 , |ϕ| 2 m = m =⇒ |ϕ| = 1 a.e.,<br />

so that there is no Borel set ∆ such that ϕ|∆ = 0 and m(∆) ≠ 0. Therefore every<br />

invariant subspace for the bilateral shift must have <strong>on</strong>e form or the other. We thus<br />

have:<br />

Corollary 5.1.3. If W is the bilateral shift <strong>on</strong> L 2 and M ∈ Lat W then<br />

either M = L 2 (m|∆) or M = ϕH 2<br />

for a Borel set ∆ and a functi<strong>on</strong> ϕ ∈ L ∞ such that |ϕ| = 1 a.e.<br />

Definiti<strong>on</strong> 5.1.4. A functi<strong>on</strong> ϕ ∈ L ∞<br />

functi<strong>on</strong> if |ϕ| = 1 a.e.<br />

[ϕ ∈ H ∞ ] is called a unimodular [inner]<br />

The following theorem has had an enormous influence <strong>on</strong> the development in<br />

operator theory and functi<strong>on</strong> theory.<br />

Theorem 5.1.5 (Beurling’s Theorem). If U is the unilateral shift <strong>on</strong> H 2 then<br />

Lat U = {ϕH 2 : ϕ is an inner functi<strong>on</strong>}.<br />

158


CHAPTER 5.<br />

TOEPLITZ THEORY<br />

Proof. Let W be the bilateral shift <strong>on</strong> L 2 . If M ∈ Lat U then M ∈ Lat W . By<br />

Corollary 5.1.3, M = L 2 (m|∆) or M = ϕH 2 , where ϕ is a unimodular functi<strong>on</strong>.<br />

Since U is a shift, ∩<br />

U n M ⊂ ∩ U n H 2 = {0},<br />

so the first alternative is impossible. Hence ϕH 2 = M ⊂ H 2 . Since ϕ = ϕ · 1 ∈ M, it<br />

follows ϕ ∈ L ∞ ∩ H 2 = H ∞ .<br />

159


CHAPTER 5.<br />

TOEPLITZ THEORY<br />

5.1.2 Hardy Spaces<br />

If f ∈ H 2 and f(z) = ∑ ∞<br />

n=0 a nz n is its Fourier series expansi<strong>on</strong>, this series c<strong>on</strong>verges<br />

uniformly <strong>on</strong> compact subsets of D. Indeed, if |z| ≤ r < 1, then<br />

(<br />

∞∑<br />

∑ ∞<br />

|a n z n | ≤<br />

n=m<br />

n=m<br />

|a n | 2 ) 1<br />

2 ( ∞ ∑<br />

n=m<br />

|z| 2n ) 1<br />

2<br />

≤ ||f|| 2<br />

( ∞<br />

∑<br />

n=m<br />

r 2n ) 1<br />

2<br />

.<br />

Therefore it is possible to identify H 2 with the space of analytic functi<strong>on</strong>s <strong>on</strong> the unit<br />

disk whose Taylor coefficients are square summable.<br />

Propositi<strong>on</strong> 5.1.6. If f is a real-valued functi<strong>on</strong> in H 1 then f is c<strong>on</strong>stant.<br />

Proof. Let α = ∫ fdm. By hypothesis, we have α ∈ R. Since f ∈ H 1 , we have<br />

∫<br />

fz n dm = 0 for n ≥ 1. So ∫ (f − α)z n dm = 0 for n ≥ 0. Also,<br />

∫<br />

0 =<br />

∫<br />

(f − α)z n dm =<br />

(f − α)z −n dm (n ≥ 0),<br />

so that ∫ (f − α)z n dm = 0 for all integers n. Thus f − α annihilates all the trig<strong>on</strong>ometric<br />

polynomials. Therefore, f − α = 0 in L 1 .<br />

Corollary 5.1.7. If ϕ is inner such that ϕ = 1 ϕ ∈ H2 then ϕ is c<strong>on</strong>stant.<br />

Proof. By hypothesis, ϕ+ϕ and ϕ−ϕ<br />

i<br />

5.1.6, they are c<strong>on</strong>stant, so is ϕ.<br />

are real-valued functi<strong>on</strong>s in H 2 . By Propositi<strong>on</strong><br />

The proof of the following important theorem uses Beurling’s theorem.<br />

Theorem ( 5.1.8 (The F. and)<br />

M. Riesz Theorem). If f is a n<strong>on</strong>zero functi<strong>on</strong> in H 2 ,<br />

then m {z ∈ ∂D : f(z) = 0} = 0. Hence, in particular, if f, g ∈ H 2 and if fg = 0<br />

a.e. then f = 0 a.e. or g = 0 a.e.<br />

Proof. Let △ be a Borel set of ∂D and put<br />

M := {h ∈ H 2 : h(z) = 0 a.e. <strong>on</strong> △}.<br />

Then M is an invariant subspace for the unilateral shift. By Beurling’s theorem,<br />

if M ̸= {0}, then there exists an inner functi<strong>on</strong> ϕ such that M = ϕH 2 . Since<br />

ϕ = ϕ · 1 ∈ M, it follows ϕ = 0 <strong>on</strong> △. But |ϕ| = 1 a.e., and hence M = {0}.<br />

A functi<strong>on</strong> f in H 2 is called an outer functi<strong>on</strong> if<br />

H 2 = ∨ {z n f : n ≥ 0}.<br />

So f is outer if and <strong>on</strong>ly if it is a cyclic vector for the unilateral shift.<br />

160


CHAPTER 5.<br />

TOEPLITZ THEORY<br />

Theorem 5.1.9 (Inner-Outer Factorizati<strong>on</strong>). If f is a n<strong>on</strong>zero functi<strong>on</strong> in H 2 , then<br />

∃ an inner functi<strong>on</strong> ϕ and an outer functi<strong>on</strong> g in H 2<br />

such that f = ϕg.<br />

In particular, if f ∈ H ∞ , then g ∈ H ∞ .<br />

Proof. Observe M ≡ ∨ {z n f : n ≥ 0} ∈ Lat U. By Beurling’s theorem,<br />

∃ an inner functi<strong>on</strong> ϕ s.t. M = ϕH 2 .<br />

Let g ∈ H 2 be such that f = ϕg. We want to show that g is outer. Put N ≡ ∨ {z n g :<br />

n ≥ 0}. Again there exists an inner functi<strong>on</strong> ψ such that N = ψH 2 . Note that<br />

ϕH 2 := ∨ {z n f : n ≥ 0} = ∨ {z n ϕg : n ≥ 0} = ϕψH 2 .<br />

Therefore there exists a functi<strong>on</strong> h ∈ H 2 such that ϕ = ϕψh so that ψ = h ∈ H 2 .<br />

Hence ψ is a c<strong>on</strong>stant by Corollary 5.1.7. So N = H 2 and g is outer. Assume f ∈ H ∞<br />

with f = ϕg. Thus |g| = |f| a.e. <strong>on</strong> ∂D, so that g must be bounded, i.e., g ∈ H ∞ .<br />

161


CHAPTER 5.<br />

TOEPLITZ THEORY<br />

5.1.3 Toeplitz <strong>Operator</strong>s<br />

Let P be the orthog<strong>on</strong>al projecti<strong>on</strong> of L 2 (T) <strong>on</strong>to H 2 (T). For φ ∈ L ∞ (T), the Toeplitz<br />

operator T φ with symbol φ is defined by<br />

T φ f = P (φf) for f ∈ H 2 .<br />

Remember that {z n : n = 0, 1, 2, · · · } is an orth<strong>on</strong>ormal basis for H 2 . Thus if<br />

φ ∈ L ∞ has the Fourier coefficients<br />

̂φ(n) = 1<br />

2π<br />

∫ 2π<br />

0<br />

φz n dt,<br />

then the matrix (a ij ) for T φ with respect to the basis {z n : n = 0, 1, 2, · · · } is given<br />

by:<br />

a ij = (T φ z j , z i ) = 1 ∫ 2π<br />

φz i−j dt = ̂φ(i − j).<br />

2π<br />

Thus the matrix for T φ is c<strong>on</strong>stant <strong>on</strong> diag<strong>on</strong>als:<br />

⎡<br />

⎤<br />

c 0 c −1 c −2 c −3 · · ·<br />

c 1 c 0 c −1 c −2 · · ·<br />

(a ij ) =<br />

c 2 c 1 c 0 c −1 · · ·<br />

, where c<br />

⎢<br />

⎣<br />

c 3 c 2 c 1 c 0 · · · ⎥<br />

j = ̂φ(j) :<br />

⎦<br />

.<br />

. .. . .. . .. . ..<br />

Such a matrix is called a Toeplitz matrix.<br />

Lemma 5.1.10. Let A ∈ B(H 2 ). The matrix A relative to the orth<strong>on</strong>ormal basis<br />

{z n : n = 0, 1, 2, · · · } is a Toeplitz matrix if and <strong>on</strong>ly if<br />

U ∗ AU = A, where U is the unilateral shift.<br />

Proof. The hypothesis <strong>on</strong> the matrix entries a ij = ⟨Az j , z i ⟩ of A if and <strong>on</strong>ly if<br />

Noting Uz n = z n+1 for n ≥ 0, we get<br />

0<br />

a i+1,j+1 = a ij (i, j = 0, 1, 2, · · · ). (5.1)<br />

(5.1) ⇐⇒ ⟨U ∗ AUz j , z i ⟩ = ⟨AUz j , Uz i ⟩ = ⟨Az j+1 , z i+1 ⟩ = ⟨Az j , z i ⟩, ∀ i, j<br />

⇐⇒ U ∗ AU = A.<br />

Remark. AU = UA ⇔ A is an analytic Toeplitz operator (i.e., A = T φ with φ ∈ H ∞ ).<br />

C<strong>on</strong>sider the mapping ξ : L ∞<br />

→ B(H 2 ) defined by ξ(φ) = T φ . We have:<br />

162


CHAPTER 5.<br />

TOEPLITZ THEORY<br />

Propositi<strong>on</strong> 5.1.11. ξ is a c<strong>on</strong>tractive ∗-linear mapping from L ∞ to B(H 2 ).<br />

Proof. It is obvious that ξ is c<strong>on</strong>tractive and linear. To show that ξ(φ) ∗ = ξ(φ), let<br />

f, g ∈ H 2 . Then<br />

⟨T φ f, g⟩ = ⟨P (φf), g⟩ = ⟨φf, g⟩ = ⟨f, φg⟩ = ⟨f, P (φg)⟩ = ⟨f, T φ g⟩ = ⟨T ∗ φf, g⟩,<br />

so that ξ(φ) ∗ = T ∗ φ = T φ = ξ(φ).<br />

Remark. ξ is not multiplicative. For example, T z T z ≠ I = T 1 = T |z| 2<br />

is not a homomorphism.<br />

In special cases, ξ is multiplicative.<br />

= T zz . Thus ξ<br />

Propositi<strong>on</strong> 5.1.12. T φ T ψ = T φψ<br />

⇐⇒ either ψ or φ is analytic.<br />

Proof. (⇐) Recall that if f ∈ H 2 and ψ ∈ H ∞ then ψf ∈ H 2 . Thus, T ψ f = P (ψf) =<br />

ψf. So<br />

T φ T ψ f = T φ (ψf) = P (φψf) = T φψ f, i.e., T φ T ψ = T φψ .<br />

Taking adjoints reduces the sec<strong>on</strong>d part to the first part.<br />

(⇒) From a straightforward calculati<strong>on</strong>.<br />

Write M φ for the multiplicati<strong>on</strong> operator <strong>on</strong> L 2 with symbol<br />

(<br />

φ ∈ L ∞ . The<br />

essential range of φ ∈ L ∞ ≡ R(φ) :=the set of all λ for which µ {x : |f(x) − λ| <<br />

)<br />

ϵ} > 0 for any ϵ > 0.<br />

Lemma 5.1.13. If φ ∈ L ∞ (µ) then σ(M φ ) = R(φ).<br />

Proof. If λ /∈ R(φ) then<br />

(<br />

)<br />

∃ ε > 0 such that µ {x : |φ(x) − λ| < ε} = 0, i.e., |φ(x) − λ| ≥ ϵ a.e. [µ].<br />

So<br />

g(x) :=<br />

1<br />

φ(x) − λ ∈ L∞ (X, µ).<br />

Hence M g is the inverse of M φ − λ, i.e., λ /∈ σ(M φ ). For the c<strong>on</strong>verse, suppose<br />

λ ∈ R(φ). We will show that<br />

∃ a sequence {g n } of unit vectors ∈ L 2 with the property ||M φ g n − λg n || → 0,<br />

showing that M φ − λ is not bounded below, and hence λ ∈ σ(M φ ). By assumpti<strong>on</strong>,<br />

{x ∈ T : |φ(x) − λ| ≤ 1 n<br />

} has a positive measure. So we can find a subset<br />

{<br />

E n ⊆ x ∈ T : |φ(x) − λ| ≤ 1 }<br />

n<br />

163


CHAPTER 5.<br />

TOEPLITZ THEORY<br />

satisfying 0 < µ(E n ) < ∞. Letting g n :=<br />

and hence ||(φ − λ)g n || L 2 ≤ 1 n<br />

−→ 0.<br />

√ χ En<br />

, we have that<br />

µ(En)<br />

∣<br />

∣(φ(x) − λ)g n (x) ∣ ∣ ≤ 1 n |g n(x)|,<br />

Propositi<strong>on</strong> 5.1.14. If φ ∈ L ∞ is such that T φ is invertible, then φ is invertible in<br />

L ∞ .<br />

Proof. In view of Lemma 5.1.13, it suffices to show that<br />

If T φ is invertible then<br />

So for n ∈ Z and f ∈ H 2 ,<br />

T φ is invertible =⇒ M φ is invertible.<br />

∃ ε > 0 such that ||T φ f|| ≥ ε||f||, ∀f ∈ H 2 .<br />

||M φ (z n f)|| = ||φz n f|| = ||φf|| ≥ ||P (φf)|| = ||T φ f|| ≥ ε||f|| = ε||z n f||.<br />

Since {z n f : f ∈ H 2 , n ∈ Z} is dense in L 2 , it follows ||M φ g|| ≥ ε||g|| for g ∈<br />

L 2 . Similarly, ||M φ f|| ≥ ε||f|| since T ∗ φ = T φ is also invertible. Therefore M φ is<br />

invertible.<br />

Theorem 5.1.15 (Hartman-Wintner). If φ ∈ L ∞ then<br />

(i) R(φ) = σ(M φ ) ⊂ σ(T φ )<br />

(ii) ||T φ || = ||φ|| ∞ (i.e., ξ is an isometry).<br />

Proof. (i) From Lemma 5.1.13 and Propositi<strong>on</strong> 5.1.14.<br />

(ii) ||φ|| ∞ = sup λ∈R(φ) |λ| ≤ sup λ∈σ(Tφ)|λ| = r(T φ ) ≤ ||T φ || ≤ ||φ|| ∞ .<br />

From Theorem 5.1.15 we can see that<br />

(i) If T φ is quasinilpotent then T φ = 0 because R(φ) ⊆ σ(T φ ) = {0} ⇒ φ = 0.<br />

(ii) If T φ is self-adjoint then φ is real-valued because R(φ) ⊆ σ(T φ ) ⊆ R.<br />

If S ⊆ L ∞ , write T (S) := the smallest closed subalgebra of L(H 2 ) c<strong>on</strong>taining<br />

{T φ : φ ∈ S}.<br />

If A is a C ∗ -algebra then its commutator ideal C is the closed ideal generated by<br />

the commutators [a, b] := ab − ba (a, b ∈ A). In particular, C is the smallest closed<br />

ideal in A such that A/C is abelian.<br />

164


CHAPTER 5.<br />

TOEPLITZ THEORY<br />

Theorem 5.1.16. If C is the commutator ideal in T (L ∞ ), then the mapping ξ c<br />

induced from L ∞ to T (L ∞ )/C by ξ is a ∗-isometrical isomorphism. Thus there is a<br />

short exact sequence<br />

Proof. See [Do1].<br />

0 −→ C −→ T (L ∞ ) −→ L ∞ −→ 0.<br />

The commutator ideal C c<strong>on</strong>tains compact operators.<br />

Propositi<strong>on</strong> 5.1.17. The commutator ideal in T (C(T)) = K(H 2 ). Hence the commutator<br />

ideal of T (L ∞ ) c<strong>on</strong>tains K(H 2 ).<br />

Proof. Since T z is the unilateral shift, we can see that the commutator ideal of<br />

T (C(T)) c<strong>on</strong>tains the rank <strong>on</strong>e operator Tz ∗ T z − T z Tz ∗ . Moreover, T (C(T)) is irreducible<br />

since T z has no proper reducing subspaces by Beurling’s theorem. Therefore<br />

T (C(T)) c<strong>on</strong>tains K(H 2 ). Since T z is normal modulo a compact operator and generates<br />

the algebra T (C(T)), it follows that T (C(T))/K(H 2 ) is commutative. Hence<br />

K(H 2 ) c<strong>on</strong>tains the commutator ideal of T (C(T)). But since K(H 2 ) is simple (i.e., it<br />

has no n<strong>on</strong>trivial closed ideal), we can c<strong>on</strong>clude that K(H 2 ) is the commutator ideal<br />

of T (C(T)).<br />

Corollary 5.1.18. There exists a ∗-homomorphism ζ : T (L ∞ )/K(H 2 ) −→ L ∞<br />

such that the following diagram commutes:<br />

T (L ∞ )<br />

❏ ❏❏❏❏❏❏❏❏<br />

ρ<br />

π<br />

ζ<br />

♣♣♣♣♣♣♣♣♣♣♣<br />

L ∞ (T)<br />

T (L ∞ )/K(H 2 )<br />

Corollary 5.1.19. Let φ ∈ L ∞ . If T φ is Fredholm then φ is invertible in L ∞ .<br />

Proof. If T φ is Fredholm then π(T φ ) is invertible in T (L ∞ )/K(H 2 ), so φ = ρ(T φ ) =<br />

(ζ ◦ π)(T φ ) is invertible in L ∞ .<br />

From Corollary 5.1.18, we have:<br />

(i) ||T φ || ≤ ||T φ + K|| for every compact operator K because ||T φ || = ||φ|| ∞ =<br />

||ζ(T φ + K)|| ≤ ||T φ + K||.<br />

(ii) The <strong>on</strong>ly compact Toeplitz operator is 0 because ||K|| ≤ ||K + K|| ⇒ K = 0.<br />

Propositi<strong>on</strong> 5.1.20. If φ is invertible in L ∞ such that R(φ) ⊆ the open right halfplane,<br />

then T φ is invertible.<br />

165


CHAPTER 5.<br />

TOEPLITZ THEORY<br />

Proof. If ∆ ≡ {z ∈ C : |z − 1| < 1} then there exists ϵ > 0 such that ϵ R(φ) ⊆<br />

∆. Hence ||ϵφ − 1|| < 1, which implies ||I − T ϵφ || < 1. Therefore T ϵφ = ϵT φ is<br />

invertible.<br />

Corollary 5.1.21 (Brown-Halmos). If φ ∈ L ∞ , then σ(T φ ) ⊆ c<strong>on</strong>v R(φ).<br />

Proof. It is sufficient to show that every open half-plane c<strong>on</strong>taining R(φ) c<strong>on</strong>tains<br />

σ(T φ ). This follow at <strong>on</strong>ce from Propositi<strong>on</strong> 5.1.20 after a translati<strong>on</strong> and rotati<strong>on</strong> of<br />

the open half-plane to coincide with the open right half-plane.<br />

Propositi<strong>on</strong> 5.1.22. If φ ∈ C(T) and ψ ∈ L ∞ then<br />

Proof. If ψ ∈ L ∞ , f ∈ H 2 then<br />

T φ T ψ − T φψ and T ψ T φ − T ψφ are compact.<br />

T ψ T z f = T ψ P (zf) = T ψ (zf − ̂f(0)z)<br />

= P M ψ<br />

(zf − ̂f(0)z<br />

)<br />

= P (ψzf) − ̂f(0)P (ψz)<br />

= T ψz f − ̂f(0)P (ψz),<br />

which implies that T ψ T z − T ψz is at most a rank <strong>on</strong>e operator. Suppose T ψ T z n − T ψz n<br />

is compact for every ψ ∈ L ∞ and n = 1, · · · , N. Then<br />

T ψ T z N+1 − T ψz N+1 = (T ψ T z N − T ψz N ) T z + (T ψz N T z − T (ψz N )z),<br />

which is compact. Also, since T ψ T z n = T ψz n (n ≥ 0), it follows that T ψ T p − T ψp<br />

is compact for every trig<strong>on</strong>ometric polynomial p. But since the set of trig<strong>on</strong>ometric<br />

polynomials is dense in C(T) and ξ is isometric, we can c<strong>on</strong>clude that T ψ T φ − T ψφ is<br />

compact for ψ ∈ L ∞ and φ ∈ C(T).<br />

Theorem 5.1.23. T (C(T)) c<strong>on</strong>tains K(H 2 ) as its commutator and the sequence<br />

0 −→ K(H 2 ) −→ T (C(T)) −→ C(T) −→ 0<br />

is a short exact sequence, i.e., T (C(T))/K(H 2 ) is ∗-isometrically isomorphic to C(T).<br />

Proof. By Propositi<strong>on</strong> 5.1.22 and Corollary 5.1.18.<br />

Propositi<strong>on</strong> 5.1.24. [Co] If φ ≠ 0 a.e. in L ∞ , then<br />

either ker T φ = {0} or ker T ∗ φ = {0}.<br />

166


CHAPTER 5.<br />

TOEPLITZ THEORY<br />

Proof. If f ∈ ker T φ and g ∈ ker T ∗ φ, i.e., P (φf) = 0 and P (φg)=0, then<br />

φf ∈ zH 2 and φg ∈ zH 2 .<br />

Thus φfg, φgf ∈ zH 1 and therefore φfg = 0. If neither f nor g is 0, then by F. and<br />

M. Riesz theorem, φ = 0 a.e. <strong>on</strong> T, a c<strong>on</strong>tradicti<strong>on</strong>.<br />

Corollary 5.1.25. If φ ∈ C(T) then T φ is Fredholm if and <strong>on</strong>ly if φ vanishes nowhere.<br />

Proof. By Theorem 5.1.23,<br />

T φ is Fredholm ⇐⇒ π(T φ ) is invertible in T (C(T))/K(H 2 )<br />

⇐⇒ φ is invertible in C(T).<br />

Corollary 5.1.26. If φ ∈ C(T), then σ e (T φ ) = φ(T).<br />

Proof. σ e (T φ ) = σ(T φ + K(H 2 )) = σ(φ) = φ(T).<br />

Theorem 5.1.27. If φ ∈ C(T) is such that T φ is Fredholm, then<br />

index (T φ ) = −wind (φ).<br />

Proof. We claim that if φ and ψ determine homotopic curves in C \ {0}, then<br />

index (T φ ) = index (T ψ ).<br />

To see this, let Φ be a c<strong>on</strong>stant map from [0, 1] × T to C \ {0} such that<br />

Φ(0, e it ) = φ(e it ) and Φ(1, e it ) = ψ(e it ).<br />

If we set Φ λ (e it ) = Φ(λ, e it ), then the mapping λ ↦→ T Φλ is norm c<strong>on</strong>tinuous and<br />

each T Φλ is a Fredholm operator. Since the map index is c<strong>on</strong>tinuous, index(T φ ) =<br />

index(T ψ ). Now if n = wind(φ) then φ is homotopic in C \ {0} to z n . Since<br />

index (T z n) = −n, we have that index (T φ ) = −n.<br />

Theorem 5.1.28. If U is the unilateral shift <strong>on</strong> H 2 then comm(U) = {T φ : φ ∈<br />

H ∞ }.<br />

Proof. It is straightforward that UT φ = T φ U for φ ∈ H ∞ , i.e., {T φ : φ ∈ H ∞ } ⊂<br />

comm(U). For the reverse we suppose T ∈ comm(U), i.e., T U = UT . Put φ := T (1).<br />

So φ ∈ H 2 and T (p) = φp for every polynomial p. If f ∈ H 2 , let {p n } be a sequence<br />

of polynomials such that p n → f in H 2 . By passing to a subsequence, we can assume<br />

p n (z) → f(z) a.e. [m]. Thus φp n = T (p n ) → T (f) in H 2 and φp n → φf a.e. [m].<br />

167


CHAPTER 5.<br />

TOEPLITZ THEORY<br />

Therefore T f = φf for all f ∈ H 2 . We want to show that φ ∈ L ∞ and hence φ ∈ H ∞ .<br />

We may assume, without loss of generality, that ||T || = 1. Observe<br />

T k f = φ k f for f ∈ H 2 , k ≥ 1.<br />

Hence ||φ k f|| 2 ≤ ||f|| 2 for all k ≥ 1. Taking f = 1 shows that ∫ |φ| 2k dm ≤ 1 for all<br />

k ≥ 1. If ∆ := {z ∈ ∂D : |φ(z)| > 1} then ∫ ∆ |φ|2k dm ≤ 1 for all k ≥ 1. If m(∆) ≠ 0<br />

then ∫ ∆ |φ|2k dm → ∞ as k → ∞, a c<strong>on</strong>tradicti<strong>on</strong>. Therefore m(∆) = 0 and hence φ<br />

is bounded. Therefore T = T φ for φ ∈ H ∞ .<br />

D. Saras<strong>on</strong> [Sa] gave a generalizati<strong>on</strong> of Theorem 5.1.28.<br />

Theorem 5.1.29 (Saras<strong>on</strong>’s Interpolati<strong>on</strong> Theorem). Let<br />

(i) U =the unilateral shift <strong>on</strong> H 2 ;<br />

(ii) K := H 2 ⊖ ψH 2 (ψ is an inner functi<strong>on</strong>);<br />

(iii) S := P U| K , where P is the projecti<strong>on</strong> of H 2 <strong>on</strong>to K.<br />

If T ∈ comm(S) then there exists a functi<strong>on</strong> φ ∈ H ∞ such that T = T φ | K with<br />

||φ|| ∞ = ||T ||.<br />

Proof. See [Sa].<br />

168


CHAPTER 5.<br />

TOEPLITZ THEORY<br />

5.2 Hyp<strong>on</strong>ormality of Toeplitz operators<br />

An elegant and useful theorem of C. Cowen [Cow3] characterizes the hyp<strong>on</strong>ormality<br />

of a Toeplitz operator T φ <strong>on</strong> the Hardy space H 2 (T) of the unit circle T ⊂ C by<br />

properties of the symbol φ ∈ L ∞ (T). This result makes it possible to answer an<br />

algebraic questi<strong>on</strong> coming from operator theory – namely, is T φ hyp<strong>on</strong>ormal - by<br />

studying the functi<strong>on</strong> φ itself. Normal Toeplitz operators were characterized by a<br />

property of their symbol in the early 1960’s by A. Brown and P.R. Halmos [BH], and<br />

so it is somewhat of a surprise that 25 years passed before the exact nature of the<br />

relati<strong>on</strong>ship between the symbol φ ∈ L ∞ and the positivity of the selfcommutator<br />

[Tφ, ∗ T φ ] was understood (via Cowen’s theorem). As Cowen notes in his survey paper<br />

[Cow2], the intensive study of subnormal Toeplitz operators in the 1970’s and early<br />

80’s is <strong>on</strong>e explanati<strong>on</strong> for the relatively late appearance of the sequel to the Brown-<br />

Halmos work. The characterizati<strong>on</strong> of hyp<strong>on</strong>ormality via Cowen’s theorem requires<br />

<strong>on</strong>e to solve a certain functi<strong>on</strong>al equati<strong>on</strong> in the unit ball of H ∞ . However the<br />

case of arbitrary trig<strong>on</strong>ometric polynomials φ, though solved in principle by Cowen’s<br />

theorem, is in practice very complicated. Indeed it may not even be possible to find<br />

tractable necessary and sufficient c<strong>on</strong>diti<strong>on</strong>s for the hyp<strong>on</strong>ormality of T φ in terms of<br />

the Fourier coefficients of φ unless certain assumpti<strong>on</strong>s are made about φ. In this<br />

chapter we present some recent development in this research.<br />

5.2.1 Cowen’s Theorem<br />

In this secti<strong>on</strong> we present Cowen’s theorem. Cowen’s method is to recast the operatortheoretic<br />

problem of hyp<strong>on</strong>ormality of Toeplitz operators into the problem of finding<br />

a soluti<strong>on</strong> of a certain functi<strong>on</strong>al equati<strong>on</strong> involving its symbol. This approach has<br />

been put to use in the works [CLL, CuL1, CuL2, CuL3, FL1, FL2, Gu1, HKL1, HKL2,<br />

HwL3, KL, NaT, Zh] to study Toeplitz operators.<br />

We begin with:<br />

Lemma 5.2.1. A necessary and sufficient c<strong>on</strong>diti<strong>on</strong> that two Toeplitz operators commute<br />

is that either both be analytic or both be co-analytic or <strong>on</strong>e be a linear functi<strong>on</strong><br />

of the other.<br />

Proof. Let φ = ∑ i α iz i and ψ = ∑ j β jz j . Then a straightforward calculati<strong>on</strong> shows<br />

that<br />

T φ T ψ = T ψ T φ ⇐⇒ α i+1 β −j−1 = β i+1 α −j−1 (i, j ≥ 0).<br />

Thus either α −j−1 = β −j−1 = 0 for j ≥ 0, i.e., φ and ψ are both analytic, or<br />

α i+1 = β i+1 = 0 for i ≥ 0, i.e., φ and ψ are both co-analytic, or there exist i 0 , j 0<br />

such that α i0 +1 ≠ 0 and α −j0 −1 ≠ 0. So for the last case, if the comm<strong>on</strong> value of<br />

β −j0−1/α −j0−1 and β i0+1/α i0+1 is denoted by λ, then<br />

β i+1 = λ α i+1 (i ≥ 0) and β −j−1 = λ α −j−1 (j ≥ 0).<br />

Therefore, β k = λ α k (k ≠ 0).<br />

169


CHAPTER 5.<br />

TOEPLITZ THEORY<br />

Theorem 5.2.2 (Brown-Halmos). Normal Toeplitz operators are translati<strong>on</strong>s and<br />

rotati<strong>on</strong>s of hermitian Toeplitz operators i.e.,<br />

T φ normal ⇐⇒ ∃ α, β ∈ C, a real valued ψ ∈ L ∞ such that T φ = αT ψ + β1.<br />

Proof. If φ = ∑ i α iz i , then<br />

φ = ∑ i<br />

α i z i = ∑ i<br />

α −i z i .<br />

So if φ is real, then α i = α −i . Thus no real φ can be analytic or co-analytic unless<br />

φ is a c<strong>on</strong>stant. Write T φ = T φ1 +iφ 2<br />

, where φ 1 , φ 2 are real-valued. Then by Lemma<br />

5.2.1, T φ T φ = T φ T φ iff T φ1 T φ2 = T φ2 T φ1 iff either φ 1 and φ 2 are both analytic or φ 1<br />

and φ 2 are both co-analytic or φ 1 = αφ 2 + β (α, β ∈ C). So if φ ≠ a c<strong>on</strong>stant, then<br />

φ = αφ 2 + β + iφ 2 = (α + i)φ 2 + β.<br />

For ψ ∈ L ∞ , the Hankel operator H ψ is the operator <strong>on</strong> H 2 defined by<br />

H ψ f = J(I − P )(ψf) (f ∈ H 2 ),<br />

where J is the unitary operator from (H 2 ) ⊥ <strong>on</strong>to H 2 :<br />

J(z −n ) = z n−1 (n ≥ 1).<br />

Denoting v ∗ (z) := v(z), another way to put this is that H ψ is the operator <strong>on</strong> H 2<br />

defined by<br />

< zuv, ψ >=< H ψ u, v ∗ > for all v ∈ H ∞ .<br />

If ψ has the Fourier series expansi<strong>on</strong> ψ := ∑ ∞<br />

n=−∞ a nz n , then the matrix of H ψ is<br />

given by<br />

⎡<br />

⎤<br />

a −1 a −2 a −3 · · ·<br />

a −2 a −3<br />

H ψ ≡<br />

⎢<br />

.<br />

⎣<br />

a ..<br />

−3<br />

⎥<br />

⎦ .<br />

.<br />

.<br />

..<br />

The following are basic properties of Hankel operators.<br />

1. H ∗ ψ = H ψ ∗;<br />

2. H ψ U = U ∗ H ψ (U is the unilateral shift);<br />

3. KerH ψ = {0} or θH 2 for some inner functi<strong>on</strong> θ (by Beurling’s theorem);<br />

4. T φψ − T φ T ψ = H ∗ φ H ψ;<br />

5. H φ T h = H φh = T ∗ h ∗H φ (h ∈ H ∞ ).<br />

We are ready for:<br />

Theorem 5.2.3 (Cowen’s Theorem). If φ ∈ L ∞ is such that φ = g + f (f, g ∈ H 2 ),<br />

then<br />

T φ is hyp<strong>on</strong>ormal ⇐⇒ g = c + T h<br />

f<br />

for some c<strong>on</strong>stant c and some h ∈ H ∞ (D) with ||h|| ∞ ≤ 1.<br />

170


CHAPTER 5.<br />

TOEPLITZ THEORY<br />

Proof. Let φ = f + g (f, g ∈ H 2 ). For every polynomial p ∈ H 2 ,<br />

⟨(T ∗ φT φ − T φ T ∗ φ)p, p⟩ = ⟨T φ p, T φ p⟩ − ⟨T ∗ φp, T ∗ φp⟩<br />

Since polynomials are dense in H 2 ,<br />

= ⟨fp + P gp, fp + P gp⟩ − ⟨P fp + gp, P fp + gp⟩<br />

= ⟨fp, fp⟩ − ⟨P fp, P fp⟩ − ⟨gp, gp⟩ + ⟨P gp, P gp⟩<br />

= ⟨fp, (I − P )fp⟩ − ⟨gp, (I − p)gp⟩<br />

= ⟨(I − P )fp, (I − P )fp⟩ − ⟨(I − P )gp, (I − P )gp⟩<br />

= ||H f<br />

p|| 2 − ||H g p|| 2 .<br />

T φ hyp<strong>on</strong>ormal ⇐⇒ ||H g u|| ≤ ||H f<br />

u||, ∀u ∈ H 2 (5.2)<br />

Write K := cl ran(H f<br />

) and let S be the compressi<strong>on</strong> of the unilateral shift U to K.<br />

Since K is invariant for U ∗ (why: H f<br />

U = U ∗ H f<br />

), we have S ∗ = U ∗ | K . Suppose T φ is<br />

hyp<strong>on</strong>ormal. Define A <strong>on</strong> ran(H f<br />

) by<br />

Then A is well defined because by (5.3)<br />

A(H f<br />

u) = H g u. (5.3)<br />

H f<br />

u 1 = H f<br />

u 2 =⇒ H f<br />

(u 1 − u 2 ) = 0 =⇒ H g (u 1 − u 2 ) = 0.<br />

By (5.2), ||A|| ≤ 1, so A has an extensi<strong>on</strong> to K, which will also be denoted A. Observe<br />

that<br />

H g U = AH f<br />

U = AU ∗ H f<br />

= AS ∗ H f<br />

and H g U = U ∗ H g = U ∗ AH f<br />

= S ∗ AH f<br />

.<br />

Thus AS ∗ = S ∗ A <strong>on</strong> K since ranH f<br />

is dense in K, and hence SA ∗ = A ∗ S. By<br />

Saras<strong>on</strong>’s interpolati<strong>on</strong> theorem,<br />

∃ k ∈ H ∞ (D) with ||k|| ∞ = ||A ∗ || = ||A|| s.t. A ∗ = the compressi<strong>on</strong> of T k to K.<br />

Since Tk ∗H f = H f T k ∗, we have that K is invariant for T k ∗ = T k<br />

, which means that A<br />

is the compressi<strong>on</strong> of T k<br />

to K and<br />

H g = T k<br />

H f<br />

(by (5.3)). (5.4)<br />

C<strong>on</strong>versely, if (5.4) holds for some k ∈ H ∞ (D) with ||k|| ∞ ≤ 1, then (5.2) holds for<br />

all u, and hence T φ is hyp<strong>on</strong>ormal. C<strong>on</strong>sequently,<br />

T φ hyp<strong>on</strong>ormal ⇐⇒ H g = T k<br />

H f<br />

.<br />

But H g = T k<br />

H f<br />

if and <strong>on</strong>ly if ∀ u, v ∈ H ∞ ,<br />

⟨zuv, g⟩ = ⟨H g u, v ∗ ⟩ = ⟨T k<br />

H f<br />

u, v ∗ ⟩ = ⟨H f<br />

u, kv ∗ ⟩<br />

= ⟨zuk ∗ v, f⟩ = ⟨zuv, k ∗ f⟩ = ⟨zuv, T k ∗f⟩.<br />

Since ∨ {zuv : u, v ∈ H ∞ } = zH 2 , it follows that<br />

H g = T k<br />

H f<br />

⇐⇒ g = c + T h<br />

f for h = k ∗ .<br />

171


CHAPTER 5.<br />

TOEPLITZ THEORY<br />

Theorem 5.2.4 (Nakazi-Takahashi Variati<strong>on</strong> of Cowen’s Theorem). For φ ∈ L ∞ ,<br />

put<br />

E(φ) := {k ∈ H ∞ : ||k|| ∞ ≤ 1 and φ − kφ ∈ H ∞ }.<br />

Then T φ is hyp<strong>on</strong>ormal if and <strong>on</strong>ly if E(φ) ≠ ∅.<br />

Proof. Let φ = f + g ∈ L ∞ (f, g ∈ H 2 ). By Cowen’s theorem,<br />

T φ is hyp<strong>on</strong>ormal ⇐⇒ g = c + T k<br />

f<br />

for some c<strong>on</strong>stant c and some k ∈ H ∞ with ||k|| ∞ ≤ 1. If φ = kφ + h (h ∈ H ∞ ) then<br />

φ − kφ = g − kf + f − kg ∈ H ∞ . Thus g − kf ∈ H 2 , so that P (g − kf) = c (c = a<br />

c<strong>on</strong>stant), and hence g = c + T k<br />

f for some c<strong>on</strong>stant c. Thus T φ is hyp<strong>on</strong>ormal. The<br />

argument is reversible.<br />

172


CHAPTER 5.<br />

TOEPLITZ THEORY<br />

5.2.2 The Case of Trig<strong>on</strong>ometric Polynomial Symbols<br />

In this secti<strong>on</strong> we c<strong>on</strong>sider the hyp<strong>on</strong>ormality of Toeplitz operators with trig<strong>on</strong>ometric<br />

polynomial<br />

[<br />

symbols.<br />

]<br />

To do this we first review the dilati<strong>on</strong> theory.<br />

A ∗<br />

If B = , then B is called a dilati<strong>on</strong> of A and A is called a compressi<strong>on</strong> of B.<br />

∗ ∗<br />

It was well-known that every c<strong>on</strong>tracti<strong>on</strong> has a unitary dilati<strong>on</strong>: indeed if ||A|| ≤ 1,<br />

then<br />

[<br />

]<br />

A (I − AA<br />

B ≡<br />

∗ ) 1 2<br />

(I − A ∗ A) 1 2 −A ∗<br />

is unitary.<br />

On the other hand, an operator B is called a power (or str<strong>on</strong>g) dilati<strong>on</strong> of A if B n<br />

is a dilati<strong>on</strong> of A n for all n [ = 1, 2, ] 3, · · · . So if B is a (power) dilati<strong>on</strong> of A then B<br />

A 0<br />

should be of the form B = . Sometimes, B is called a lifting of A and A is<br />

∗ ∗<br />

said to be lifted to B. It was also well-known that every c<strong>on</strong>tracti<strong>on</strong> has a isometric<br />

(power) dilati<strong>on</strong>. In fact, the minimal isometric dilati<strong>on</strong> of a c<strong>on</strong>tracti<strong>on</strong> A is given<br />

by<br />

⎡<br />

A 0 0 0 · · ·<br />

(I − A ∗ A) 1 2 0 0 0 · · ·<br />

B ≡<br />

0 I 0 0 · · ·<br />

.<br />

⎢ 0 0 I 0 · · · ⎥<br />

We then have:<br />

⎣<br />

.<br />

.<br />

.<br />

⎤<br />

⎦<br />

. ..<br />

Theorem 5.2.5 (Commutant Lifting Theorem). Let A be a c<strong>on</strong>tracti<strong>on</strong> and T be a<br />

minimal isometric dilati<strong>on</strong> of A. If BA = AB then there exists a dilati<strong>on</strong> S of B<br />

such that<br />

[ ] B 0<br />

S = , ST = T S, and ||S|| = ||B||.<br />

∗ ∗<br />

Proof. See [GGK, p.658].<br />

We next c<strong>on</strong>sider the following interpolati<strong>on</strong> problem, called the Carathéodory-<br />

Schur Interpolati<strong>on</strong> Problem (CSIP).<br />

Given c 0 , · · · , c N−1 in C, find an analytic functi<strong>on</strong> φ <strong>on</strong> D such that<br />

(i) ̂φ(j) = c j (j = 0, · · · , N − 1);<br />

(ii) ||φ|| ∞ ≤ 1.<br />

The following is a soluti<strong>on</strong> of CSIP.<br />

173


CHAPTER 5.<br />

TOEPLITZ THEORY<br />

Theorem 5.2.6.<br />

⎡<br />

⎤<br />

c 0<br />

c 1 c 0 O<br />

CSIP is solvable ⇐⇒ C ≡<br />

c 2 c 1 c 0<br />

⎢<br />

⎣<br />

.<br />

.<br />

. .. . ⎥ .. ⎦<br />

c N−1 c N−2 · · · c 1 c 0<br />

is a c<strong>on</strong>tracti<strong>on</strong>.<br />

Moreover, φ is a soluti<strong>on</strong> if and <strong>on</strong>ly if T φ is a c<strong>on</strong>tractive lifting of C which commutes<br />

with the unilateral shift.<br />

Proof. (⇒) Assume that we have a soluti<strong>on</strong> φ. Then the c<strong>on</strong>diti<strong>on</strong> (ii) implies<br />

⎡<br />

⎤<br />

φ 0<br />

T φ =<br />

φ 1 φ 0 O<br />

⎢<br />

⎣φ 2 φ 1 φ 0<br />

⎥<br />

⎦<br />

.<br />

. . .. . ..<br />

(φ j := ̂φ(j))<br />

is a c<strong>on</strong>tracti<strong>on</strong> because ||T φ || = ||φ|| ∞ ≤ 1. So the compressi<strong>on</strong> of T φ is also c<strong>on</strong>tractive.<br />

In particular,<br />

⎡<br />

⎤<br />

φ 0<br />

φ 1 φ 0 O<br />

⎢<br />

⎣<br />

.<br />

. ⎥<br />

. .. ⎦<br />

φ n−1 φ n−2 · · · φ 0<br />

must have norm less than or equal to 1 for all n. Therefore if CSIP is solvable, then<br />

||C|| ≤ 1.<br />

(⇐) Let<br />

⎡<br />

⎤<br />

c 0<br />

c 1 c 0 O<br />

C ≡<br />

c 2 c 1 c 0<br />

with ||C|| ≤ 1<br />

⎢<br />

⎣<br />

.<br />

.<br />

. .. . ⎥ .. ⎦<br />

c N−1 c N−2 · · · c 1 c 0<br />

and let<br />

⎡<br />

⎤<br />

0<br />

1 0<br />

A :=<br />

1 0<br />

: C N → C N .<br />

⎢<br />

⎣<br />

. .. . ..<br />

⎥<br />

⎦<br />

1 0<br />

Then A and C are c<strong>on</strong>tracti<strong>on</strong>s and AC = CA. Observe that the unilateral shift U<br />

is the minimal isometric dilati<strong>on</strong> of A (please check it!). By the Commutant Lifting<br />

Theorem, C can be lifted to a c<strong>on</strong>tracti<strong>on</strong> S such that SU = US. But then S is an<br />

174


CHAPTER 5.<br />

TOEPLITZ THEORY<br />

analytic Toeplitz operator, i.e., S = T φ with φ ∈ H ∞ . Since S is a lifting of C we<br />

must have<br />

̂φ(j) = c j (j = 0, 1, · · · , N − 1).<br />

Since S is a c<strong>on</strong>tracti<strong>on</strong>, it follows that ||φ|| ∞ = ||T φ || ≤ 1.<br />

Now suppose φ is a trig<strong>on</strong>ometric polynomial of the form<br />

φ(z) =<br />

N∑<br />

n=−N<br />

a n z n (a N ≠ 0).<br />

If a functi<strong>on</strong> k ∈ H ∞ (T) satisfies φ − kφ ∈ H ∞ then k necessarily satisfies<br />

N∑<br />

N∑<br />

k a n z −n − a −n z −n ∈ H ∞ . (5.5)<br />

n=1<br />

n=1<br />

From (5.5) <strong>on</strong>e compute the Fourier coefficients ̂k(0), · · · , ̂k(N − 1) to be ̂k(n) =<br />

c n (n = 0, 1, · · · , N − 1), where c 0 , c 1 , · · · , c N−1 are determined uniquely from the<br />

coefficients of φ by the following relati<strong>on</strong><br />

⎡<br />

c<br />

⎡<br />

⎤−1 0 a 1 a 2 a 3 · · · a ⎡ N<br />

a −1<br />

⎢<br />

c 1..<br />

⎥ ⎢ a 2 a 3 . . . · ⎥ ⎢ ⎢⎢⎢⎢⎢⎣<br />

a −2<br />

⎥<br />

⎤<br />

=<br />

⎢<br />

⎥ ⎢<br />

⎣ . ⎦ ⎣<br />

c N−1<br />

a 3 . . . · · ·<br />

. · · · O<br />

a N<br />

Thus if k(z) = ∑ ∞<br />

j=0 c jz j is a functi<strong>on</strong> in H ∞ then<br />

φ − kφ ∈ H ∞ ⇐⇒ c 0 , c 1 , · · · , c N−1 are given by (5.6).<br />

⎥<br />

⎦<br />

⎤<br />

.<br />

. (5.6)<br />

⎥<br />

. ⎦<br />

a −N<br />

Thus by Cowen’s theorem, if c 0 , c 1 , · · · , c N−1 are given by (5.6) then the hyp<strong>on</strong>ormality<br />

of T φ is equivalent to the existence of a functi<strong>on</strong> k ∈ H ∞ such that<br />

{̂k(j) = cj (j = 0, · · · , N − 1)<br />

||k|| ∞ ≤ 1,<br />

which is precisely the formulati<strong>on</strong> of CSIP. Therefore we have:<br />

Theorem 5.2.7. If φ(z) = ∑ N<br />

n=−N a nz n , where a N ≠ 0 and if c 0 , c 1 , · · · , c N−1 are<br />

given by (5.6) then<br />

⎡<br />

⎤<br />

c 0<br />

c 1 c 0 O<br />

T φ is hyp<strong>on</strong>ormal ⇐⇒ C ≡<br />

c 2 c 1 c 0<br />

is a c<strong>on</strong>tracti<strong>on</strong>.<br />

⎢<br />

⎣<br />

.<br />

. . .. . ⎥ .. ⎦<br />

c N−1 c N−2 · · · c 1 c 0<br />

175


CHAPTER 5.<br />

TOEPLITZ THEORY<br />

5.2.3 The Case of Rati<strong>on</strong>al Symbols<br />

A functi<strong>on</strong> φ ∈ L ∞ is said to be of bounded type (or in the Nevanlinna class) if there<br />

are functi<strong>on</strong>s ψ 1 , ψ 2 in H ∞ (D) such that<br />

φ(z) = ψ 1(z)<br />

ψ 2 (z)<br />

for almost all z in T. Evidently, rati<strong>on</strong>al functi<strong>on</strong>s in L ∞ are of bounded type.<br />

If θ is an inner functi<strong>on</strong>, the degree of θ, denoted by deg(θ), is defined by the<br />

number of zeros of θ lying in the open unit disk D if θ is a finite Blaschke product of<br />

the form<br />

∏ n<br />

θ(z) = e iξ z − β j<br />

(|β j | < 1 for j = 1, · · · , n),<br />

1 − β j z<br />

j=1<br />

otherwise the degree of θ is infinite. For an inner functi<strong>on</strong> θ, write<br />

Note that for f ∈ H 2 ,<br />

H(θ) := H 2 ⊖ θH 2 .<br />

⟨[T ∗ φ, T φ ]f, f⟩ = ||T φ f|| 2 − ||T φ f|| 2 = ||φf|| 2 − ||H φ f|| 2 − (||φf|| 2 − ||H φ f|| 2 )<br />

Thus we have<br />

= ||H φ f|| 2 − ||H φ f|| 2 .<br />

T φ hyp<strong>on</strong>ormal ⇐⇒ ||H φ f|| ≥ ||H φ f|| (f ∈ H 2 ).<br />

Now let φ = g + f ∈ L ∞ , where f and g are in H 2 . Since H φ U = U ∗ H φ (U =the<br />

unilateral shift), it follows from the Beurling’s theorem that<br />

ker H f<br />

= θ 0 H 2 and ker H g = θ 1 H 2 for some inner functi<strong>on</strong>s θ 0 , θ 1 .<br />

Thus if T φ is hyp<strong>on</strong>ormal then since ||H f<br />

h|| ≥ ||H g h|| (h ∈ H 2 ), we have<br />

θ 0 H 2 = ker H f<br />

⊂ ker H g = θ 1 H 2 , (5.7)<br />

which implies that θ 1 divides θ 0 , so that θ 0 = θ 1 θ 2 for some inner functi<strong>on</strong> θ 2 .<br />

On the other hand, note that if f ∈ H 2 and f is of bounded type, i.e., f = ψ 2 /ψ 1<br />

(ψ i ∈ H ∞ ), then dividing the outer part of ψ 1 into ψ 2 <strong>on</strong>e obtain f = ψ/θ with θ<br />

inner and ψ ∈ H ∞ , and hence f = θψ. But since f ∈ H 2 we must have ψ ∈ H(θ).<br />

Thus if f ∈ H 2 and f is of bounded type then we can write<br />

f = θψ (θ inner, ψ ∈ H(θ)). (5.8)<br />

Therefore if φ = g + f is of bounded type and T φ is hyp<strong>on</strong>ormal then by (5.7) and<br />

(5.8), we can write<br />

f = θ 1 θ 2 a and g = θ 1 b,<br />

where a ∈ H(θ 1 θ 2 ) and b ∈ H(θ 1 ).<br />

We now have:<br />

176


CHAPTER 5.<br />

TOEPLITZ THEORY<br />

Lemma 5.2.8. Let φ = g + f ∈ L ∞ , where f and g are in H 2 . Assume that<br />

f = θ 1 θ 2 a and g = θ 1 b (5.9)<br />

for a ∈ H(θ 1 θ 2 ) and b ∈ H(θ 1 ). Let ψ := θ 1 P H(θ1 )(a) + g. Then T φ is hyp<strong>on</strong>ormal if<br />

and <strong>on</strong>ly if T ψ is.<br />

Proof. This asserti<strong>on</strong> follows at <strong>on</strong>ce from [Gu2, Corollary 3.5].<br />

In view of Lemma 5.2.8, when we study the hyp<strong>on</strong>ormality of Toeplitz operators<br />

with bounded type symbols φ, we may assume that the symbol φ = g + f ∈ L ∞ is of<br />

the form<br />

f = θa and g = θb, (5.10)<br />

where θ is an inner functi<strong>on</strong> and a, b ∈ H(θ) such that the inner parts of a, b and θ<br />

are coprime.<br />

On the other hand, let f ∈ H ∞ be a rati<strong>on</strong>al functi<strong>on</strong>. Then we may write<br />

f = p m (z) +<br />

n∑<br />

l∑<br />

i−1<br />

i=1 j=0<br />

a ij<br />

(1 − α i z) li−j (0 < |α i | < 1),<br />

where p m (z) denotes a polynomial of degree m. Let θ be a finite Blaschke product of<br />

the form<br />

∏<br />

n ( ) li<br />

z −<br />

θ = z m αi<br />

.<br />

1 − α i z<br />

Observe that<br />

i=1<br />

a ij<br />

1 − α i z = α ia ij<br />

1 − |α i | 2 ( z − αi<br />

1 − α i z + 1 α i<br />

)<br />

.<br />

Thus f ∈ H(zθ). Letting a := θf, we can see that a ∈ H(zθ) and f = θa. Thus if<br />

φ = g + f ∈ L ∞ , where f and g are rati<strong>on</strong>al functi<strong>on</strong>s and if T φ is hyp<strong>on</strong>ormal, then<br />

we can write<br />

f = θa and g = θb<br />

for a finite Blaschke product θ with θ(0) = 0 and a, b ∈ H(θ).<br />

Now let θ be a finite Blaschke product of degree d. We can write<br />

θ = e iξ<br />

n ∏<br />

i=1<br />

B ni<br />

i , (5.11)<br />

where B i (z) :=<br />

z−αi<br />

1−α , (|α iz i| < 1), n i ≥ 1 and ∑ n<br />

i=1 n i = d. Let θ = e iξ ∏ d<br />

j=1 B j<br />

and each zero of θ be repeated according to its multiplicity. Note that this Blaschke<br />

product is precisely the same Blaschke product in (5.11). Let<br />

ϕ j :=<br />

d j<br />

1 − α j z B j−1B j−2 · · · B 1 (1 ≤ j ≤ d),<br />

177


CHAPTER 5.<br />

TOEPLITZ THEORY<br />

where ϕ 1 := d 1 (1 − α 1 z) −1 and d j := (1 − |α j | 2 ) 1 2 . It is well known that {ϕ j } d 1 is an<br />

orth<strong>on</strong>ormal basis for H(θ) (cf. [FF, Theorem X.1.5]). Let φ = g + f ∈ L ∞ , where<br />

g = θb and f = θa for a, b ∈ H(θ) and write<br />

C(φ) := {k ∈ H ∞ : φ − kφ ∈ H ∞ }.<br />

Then k is in C(φ) if and <strong>on</strong>ly if θb − kθa ∈ H 2 , or equivalently,<br />

b − ka ∈ θH 2 . (5.12)<br />

Note that θ (n) (α i ) = 0 for all 0 ≤ n < n i . Thus the c<strong>on</strong>diti<strong>on</strong> (5.12) is equivalent to<br />

the following equati<strong>on</strong>: for all 1 ≤ i ≤ n,<br />

where<br />

⎡ ⎤<br />

k i,0<br />

k i,1<br />

k i,2<br />

⎢ .<br />

⎥<br />

⎣k i,ni<br />

⎦<br />

−2<br />

k i,ni−1<br />

⎡<br />

⎤<br />

a i,0 0 0 0 · · · 0<br />

a i,1 a i,0 0 0 · · · 0<br />

a i,2 a i,1 a i,0 0 · · · 0<br />

=<br />

. . .. . .. . .. . .. . ⎢<br />

⎣<br />

.<br />

a i,ni−2 a .. . ⎥<br />

.. i,ni−3<br />

ai,0 0 ⎦<br />

a i,ni −1 a i,ni −2 . . . a i,2 a i,1 a i,0<br />

k i,j := k(j) (α i )<br />

, a i,j := a(j) (α i )<br />

j!<br />

j!<br />

−1 ⎡<br />

⎢<br />

⎣<br />

b i,0<br />

b i,1<br />

b i,2<br />

. .<br />

b i,ni −2<br />

b i,ni−1<br />

and b i,j := b(j) (α i )<br />

.<br />

j!<br />

⎤<br />

, (5.13)<br />

⎥<br />

⎦<br />

C<strong>on</strong>versely, if k ∈ H ∞ satisfies the equality (5.13) then k must be in C(φ). Thus k<br />

bel<strong>on</strong>gs to C(φ) if and <strong>on</strong>ly if k is a functi<strong>on</strong> in H ∞ for which<br />

k (j) (α i )<br />

j!<br />

= k i,j (1 ≤ i ≤ n, 0 ≤ j < n i ), (5.14)<br />

where the k i,j are determined by the equati<strong>on</strong> (5.13). If in additi<strong>on</strong> ||k|| ∞ ≤ 1 is<br />

required then this is exactly the classical Hermite-Fejér Interpolati<strong>on</strong> Problem (HFIP).<br />

Therefore we have:<br />

Theorem 5.2.9. Let φ = g + f ∈ L ∞ , where f and g are rati<strong>on</strong>al functi<strong>on</strong>s. Then<br />

T φ is hyp<strong>on</strong>ormal if and <strong>on</strong>ly if the corresp<strong>on</strong>ding HFIP (5.14) is solvable.<br />

Now we can summarize that tractable criteria for the hyp<strong>on</strong>ormality of Toeplitz<br />

operators T φ are accomplished for the cases where the symbol φ is a trig<strong>on</strong>ometric<br />

polynomial or a rati<strong>on</strong>al functi<strong>on</strong> via soluti<strong>on</strong>s of some interpolati<strong>on</strong> problems.<br />

We c<strong>on</strong>clude this secti<strong>on</strong> with:<br />

Problem 5.1. Let φ ∈ L ∞ be arbitrary. Find necessary and sufficient c<strong>on</strong>diti<strong>on</strong>s,<br />

in terms of the coefficients of φ, for T φ to be hyp<strong>on</strong>ormal. In particular, for the cases<br />

where φ is of bounded type.<br />

178


CHAPTER 5.<br />

TOEPLITZ THEORY<br />

5.3 Subnormality of Toeplitz operators<br />

The present chapter c<strong>on</strong>cerns the questi<strong>on</strong>: Which Toeplitz operators are subnormal <br />

Recall that a Toeplitz operator T φ is called analytic if φ is in H ∞ , that is, φ is<br />

a bounded analytic functi<strong>on</strong> <strong>on</strong> D. These are easily seen to be subnormal: T φ h =<br />

P (φh) = φh = M φ h for h ∈ H 2 , where M φ is the normal operator of multiplicati<strong>on</strong> by<br />

φ <strong>on</strong> L 2 . P.R. Halmos raised the following problem, so-called the Halmos’s Problem<br />

5 in his 1970 lectures “Ten Problems in Hilbert Space” [Ha1], [Ha2]:<br />

Is every subnormal Toeplitz operator either normal or analytic <br />

The questi<strong>on</strong> is natural because the two classes, the normal and analytic Toeplitz<br />

operators, are fairly well understood and are obviously subnormal.<br />

5.3.1 Halmos’s Problem 5<br />

We begin with a brief survey of research related to P.R. Halmos’s Problem 5.<br />

In 1976, M. Abrahamse [Ab] gave a general sufficient c<strong>on</strong>diti<strong>on</strong> for the answer to<br />

the Halmos’s Problem 5 to be affirmative.<br />

Theorem 5.3.1 (Abrahamse’s Theorem). If<br />

(i) T φ is hyp<strong>on</strong>ormal;<br />

(ii) φ or φ is of bounded type;<br />

(iii) ker[T ∗ φ, T φ ] is invariant for T φ ,<br />

then T φ is normal or analytic.<br />

Proof. See [Ab].<br />

On the other hand, observe that if S is a subnormal operator <strong>on</strong> H and if N := mne (S)<br />

then<br />

ker[S ∗ , S] = {f : < f, [S ∗ , S]f >= 0} = {f : ||S ∗ f|| = ||Sf||} = {f : N ∗ f ∈ H}.<br />

Therefore, S(ker[S ∗ , S]) ⊆ ker[S ∗ , S].<br />

By Theorem 5.3.1 and the preceding remark we get:<br />

Corollary 5.3.2. If T φ is subnormal and if φ or φ is of bounded type, then T φ is<br />

normal or analytic.<br />

Lemma 5.3.3. A functi<strong>on</strong> φ is of bounded type if and <strong>on</strong>ly if kerH φ ≠ {0}.<br />

179


CHAPTER 5.<br />

TOEPLITZ THEORY<br />

Proof. If kerH φ ≠ {0} then since H φ f = 0 ⇒ (1 − P )φf = 0 ⇒ φf = P φf := g, we<br />

have<br />

∃ f, g ∈ H 2 s.t. φf = g.<br />

Hence φ = g f . Remembering that if 1 φ ∈ L∞ then φ is outer if and <strong>on</strong>ly if 1 φ ∈ H∞<br />

and dividing the outer part of f into g gives<br />

φ = ψ θ<br />

(ψ ∈ H ∞ , θ inner).<br />

C<strong>on</strong>versely, if φ = ψ θ<br />

(ψ ∈ H∞ , θ inner), then θ ∈ kerH φ because φθ = ψ ∈ H ∞ ⇒<br />

(1 − P )φθ = 0 ⇒ H φ θ = 0.<br />

From Theorem 5.3.1 we can see that<br />

φ = ψ θ (θ, ψ inner), T φ subnormal ⇒ T φ normal or analytic (5.15)<br />

The following propositi<strong>on</strong> strengthen the c<strong>on</strong>clusi<strong>on</strong> of (5.15), whereas weakens<br />

the hypothesis of (5.15).<br />

Propositi<strong>on</strong> 5.3.4. If φ = ψ θ (θ, ψ inner) and if T φ is hyp<strong>on</strong>ormal, then T φ is<br />

analytic.<br />

Proof. Observe that<br />

1 = ||θ|| = ||P (θ)|| = ||P (φθφ)|| = ||P (φψ)||<br />

= ||T φ (ψ)|| ≤ ||T φ (ψ)|| = ||P ( ψ2<br />

θ<br />

)|| ≤ ||ψ2 || = 1,<br />

θ<br />

which implies that ψ2<br />

θ<br />

∈ H 2 , so θ divides ψ 2 . Thus if <strong>on</strong>e choose ψ and θ to be<br />

relatively prime (i.e., if φ = ψ θ<br />

is in lowest terms), then θ is c<strong>on</strong>stant. Therefore T φ<br />

is analytic.<br />

Propositi<strong>on</strong> 5.3.5. If A is a weighted shift with weights a 0 , a 1 , a 2 , · · · such that<br />

0 ≤ a 0 ≤ a 1 ≤ · · · < a N = a N+1 = · · · = 1,<br />

then A is not unitarily equivalent to any Toeplitz operator.<br />

Proof. Note that A is hyp<strong>on</strong>ormal, ||A|| = 1 and A attains its norm. If A is unitarily<br />

equivalent to T φ then by a result of Brown and Douglas [BD], T φ is hyp<strong>on</strong>ormal and<br />

φ = ψ θ (θ, ψ inner). By Propositi<strong>on</strong> 5.3.4, T φ ≡ T ψ is an isometry, so a 0 = 1, a<br />

c<strong>on</strong>tradicti<strong>on</strong>.<br />

√<br />

Recall that the Bergman shift (whose weights are given by<br />

The following questi<strong>on</strong> arises naturally:<br />

n+1<br />

n+2<br />

) is subnormal.<br />

Is the Bergman shift unitarily equivalent to a Toeplitz operator (5.16)<br />

180


CHAPTER 5.<br />

TOEPLITZ THEORY<br />

An affirmative answer to the questi<strong>on</strong> (5.16) gives a negative answer to Halmos’s<br />

Problem 5. To see this, assume that the Bergman shift S is unitarily equivalent to<br />

T φ , then<br />

R(φ) ⊆ σ e (T φ ) = σ e (S) = the unit circle T.<br />

Thus φ is unimodular. Since S is not an isometry it follows that φ is not inner.<br />

Therefore T φ is not an analytic Toeplitz operator.<br />

To answer the questi<strong>on</strong> (5.16) we need an auxiliary lemma:<br />

Lemma 5.3.6. If a Toeplitz operator T φ is a weighted shift with weights {a n } ∞ n=0<br />

with respect to the orth<strong>on</strong>ormal basis {e n } ∞ n=0, i.e.,<br />

then e 0 (z) is an outer functi<strong>on</strong>.<br />

T φ e n = a n e n+1 (n ≥ 0) (5.17)<br />

Proof. By Coburn’s theorem, ker T φ = {0} or ker T ∗ φ = {0}. The expressi<strong>on</strong> (5.17)<br />

gives e 0 ∈ ker T ∗ φ, and hence ker T φ = {0}. Thus a n > 0 (n ≥ 0). Write<br />

Because T ∗ φe 0 = 0, we get<br />

e 0 := gF, where g is inner and F is outer.<br />

T ∗ φF = T φ (ge 0 ) = T g T φ e 0 = T g T ∗ φe 0 = 0.<br />

Note that dim ker T ∗ φ = 1. So we have F = ce 0 (c =a c<strong>on</strong>stant), so that g is a c<strong>on</strong>stant,<br />

and hence e 0 is an outer functi<strong>on</strong>.<br />

Theorem 5.3.7 (Sun’s Theorem). Let T be a weighted shift with a strictly increasing<br />

weight sequence {a n } ∞ n=0. If T ∼ = T φ then<br />

a n = √ 1 − α 2n+2 ||T φ || (0 < α < 1).<br />

Proof. Assume T ∼ = T φ . We assume, without loss of generality, that ||T || = 1 (so<br />

a n < 1). Since T is a weighted shift, σ e (T ) = {z : |z| = 1}. Since R(φ) ⊂ σ e (T φ ), it<br />

follows that |φ| = 1, i.e., φ is unimodular. By Lemma 5.3.6,<br />

∃ an orth<strong>on</strong>ormal basis {e n } ∞ n=0 such that (5.17) holds.<br />

Expressi<strong>on</strong> (5.17) can be written as follows:<br />

{<br />

φe n = a n e n+1 + √ 1 − a 2 n η n<br />

φe n+1 = a n e n + √ 1 − a 2 n ξ n<br />

(5.18)<br />

where η n , ξ n ∈ (H 2 ) ⊥ and ||η n || = ||ξ n || = 1. Since {φe n } ∞ n=0 is an orth<strong>on</strong>omal system<br />

and a n < 1, we have<br />

{<br />

0, l ≠ k<br />

< η l , η k >=< ξ l , ξ k >=<br />

(5.19)<br />

1, l = k<br />

181


CHAPTER 5.<br />

TOEPLITZ THEORY<br />

From (5.18) we have<br />

e n = φ<br />

(a n e n+1 + √ )<br />

√<br />

1 − a 2 n η n = a 2 ne n + a n 1 − a<br />

2 n ξ n + √ 1 − a 2 n φ η n . (5.20)<br />

Then (5.20) is equivalent to<br />

φη n = −a n ξ n + √ 1 − a 2 n e n . (5.21)<br />

Set d n := η n<br />

t<br />

and ρ n := ξ n<br />

t<br />

(|t| = 1). Then () is equivalent to<br />

φd n = −a n ρ n + √ 1 − a 2 n<br />

e n<br />

t . (5.22)<br />

Since en t<br />

∈ (H 2 ) ⊥ and {d n } ∞ n=0 is an orth<strong>on</strong>ormal basis for H 2 , we can see that<br />

{<br />

||T φ d 0 || = a 0 = inf ||x||=1 ||T φ x|| = ||T φ e 0 ||<br />

(5.23)<br />

||T φ d l || = a l = ||T φ e l || .<br />

Then (5.17) and (5.23) imply<br />

d n = r n e n (|r n | = 1). (5.24)<br />

Substituting (5.24) into (5.23) and comparing it with (5.18) gives<br />

a n e n+1 + √ 1 − a 2 n η n = φe n = − a n<br />

r n<br />

ρ n +<br />

√<br />

1 − a<br />

2 n<br />

r n<br />

e n<br />

t ,<br />

which implies {<br />

−r n ρ n = e n+1<br />

r n<br />

e nt<br />

= η n .<br />

Therefore (5.18) is reduced to:<br />

{<br />

φe n = a n e n+1 + √ 1 − a 2 n r n<br />

e nt<br />

φe n+1 = a n e n − √ 1 − a 2 n r n<br />

e n+1<br />

t<br />

(5.25)<br />

(5.26)<br />

Put e −(n+1) := e n<br />

t<br />

∈ (H 2 ) ⊥ (n ≥ 0). We now claim that<br />

φe 0 = re −1 (|r| = 1) : (5.27)<br />

( )<br />

indeed, T φe0<br />

φ t<br />

= P ( e0 t ) = 0, so e 0 = r φe0<br />

t<br />

for |r| = 1, and hence φe 0 = re −1 . From<br />

(5.26) we have<br />

√<br />

φe 0 = a 0 e 1 + r 0<br />

√1 − a 2 0 e −1 = a 0 e 1 + r 0 r 1 − a 2 0 φe 0, (5.28)<br />

or, equivalently, (<br />

)<br />

φ − r 0 r<br />

√1 − a 2 0 φ e 0 = a 0 e 1 . (5.29)<br />

182


CHAPTER 5.<br />

TOEPLITZ THEORY<br />

Write<br />

√<br />

ψ ≡ φ − r 0 r 1 − a 2 0 φ. (5.30)<br />

Evidently,<br />

V := {x ∈ H 2 : ψx ∈ H 2 }<br />

is not empty. Moreover, since V is invariant for U, it follows from Beurling’s theorem<br />

that<br />

V = χH 2 for an inner functi<strong>on</strong> χ.<br />

Since e 0 ∈ V and e 0 is an outer functi<strong>on</strong>, we must have χ = 1. This means that<br />

ψ = ψ · 1 ∈ H 2 . Therefore ψe 1 = T ψ e 1 ∈ H 2 . On the other hand, by (5.26),<br />

(<br />

)<br />

ψe 1 = φ − r 0 r<br />

√1 − a 2 0 φ e 1<br />

√<br />

= a 1 e 2 + r 1<br />

√1 − a 2 1 e −2 − r 0 r<br />

= a 1 e 2 − r 0 ra 0<br />

√1 − a 2 1 e 0 +<br />

(<br />

)<br />

a 0 e 0 −<br />

√1 − a 2 0 r 0 e −2<br />

)<br />

1 − a 2 0<br />

√ (r 1 1 − a 2 1 + r r 0 2 (1 − a 2 1)<br />

e −2 .<br />

Thus we have<br />

r 1<br />

√1 − a 2 1 + r r 0 2 (1 − a 2 0) = 0<br />

So, √ 1 − a 2 1 = 1 − a2 0, i.e., a 1 = √ 1 − (1 − a 2 0 )2 . If we put α 2 ≡ 1 − a 2 0, i.e.,<br />

a 0 = (1 − α 2 ) 1 2 then a 1 = (1 − α 4 ) 1 2 . Inductively, we get a n = (1 − α 2n+2 ) 1 2 .<br />

Corollary 5.3.8. The Bergman shift is not unitarily equivalent to any Toeplitz operator.<br />

Proof.<br />

n+1<br />

n+2 ≠ 1 − α2n+2 for any α > 0.<br />

Lemma 5.3.9. The weighted shift T ≡ W α with weights α n ≡ (1 − α 2n+2 ) 1 2 (0 <<br />

α < 1) is subnormal.<br />

Proof. Write r n := α0α 2 1 2 · · · αn−1 2 for the moment of W . Define a discrete measure µ<br />

<strong>on</strong> [0, 1] by<br />

{<br />

Π<br />

∞<br />

j=1 (1 − α 2j ) (z = 0)<br />

µ(z) =<br />

α 2k<br />

Π ∞ j=1 (1 − α2j )<br />

(1−α 2 )···(1−α 2k ) (z = αk ; k = 1, 2, · · · ).<br />

Then r n = ∫ 1<br />

0 tn dµ. By Berger’s theorem, T is subnormal.<br />

Corollary 5.3.10. If T φ<br />

∼ = a weighted shift, then Tφ is subnormal.<br />

183


CHAPTER 5.<br />

TOEPLITZ THEORY<br />

Remark 5.3.11. If T φ<br />

∼ = a weighted shift, what is the form of φ A careful analysis<br />

of the proof of Theorem 5.3.7 shows that<br />

ψ = φ − αφ ∈ H ∞ .<br />

But<br />

⎡<br />

⎤<br />

0 −αa 0<br />

a 0 0 −αa 1<br />

T ψ = T φ − αTφ ∗ =<br />

a 1 0 −αa 2<br />

⎢<br />

.<br />

⎣<br />

a 2 0 ..<br />

⎥<br />

⎦<br />

. .. . ..<br />

⎡<br />

⎤<br />

0 −α<br />

1 0 −α<br />

=<br />

1 0 −α<br />

⎢<br />

.<br />

⎣ 1 0 ..<br />

+ K (K compact)<br />

⎥<br />

⎦<br />

. .. . ..<br />

∼= T z−αz + K.<br />

Thus ran (ψ) = σ e (T ψ ) = σ e (T z−αz ) = ran(z −αz). Thus ψ is a c<strong>on</strong>formal mapping of<br />

D <strong>on</strong>to the interior of the ellipse with vertices ±i(1+α) and passing through ±(1−α).<br />

On the other hand, ψ = φ − αφ. So αψ = αφ − α 2 φ, which implies<br />

We now have:<br />

φ = 1 (ψ + αψ).<br />

1 − α2 Theorem 5.3.12 (Cowen and L<strong>on</strong>g’s Theorem). For 0 < α < 1, let ψ be a c<strong>on</strong>formal<br />

map of D <strong>on</strong>to the interior of the ellipse with vertices ±i(1−α) −1 and passing through<br />

±(1 + α) −1 . Then T ψ+αψ<br />

is a subnormal weighted shift that is neither analytic nor<br />

normal.<br />

Proof. Let φ = ψ + αψ. Then φ is a c<strong>on</strong>tinuous map of D <strong>on</strong>to D with wind(φ) = 1.<br />

Let<br />

K := 1 − T φ T φ = T φφ − T φ T φ = H ∗ φH φ ,<br />

which is compact since φ is c<strong>on</strong>tinuous. Now φ − αφ = (1 − α 2 )ψ ∈ H ∞ , so H ψ = 0<br />

and hence, H φ = αH φ . Thus<br />

so that<br />

K = H ∗ φH φ = α 2 H ∗ φH φ = α 2 (1 − T φ T φ ),<br />

KT φ = α 2 (1 − T φ T φ )T φ = α 2 T φ (1 − T φ T φ ) = α 2 T φ K.<br />

By Coburn’s theorem, ker T φ = {0} or ker T φ = {0}. But since<br />

ind(T φ ) = −wind(φ) = −1,<br />

184


CHAPTER 5.<br />

TOEPLITZ THEORY<br />

it follows<br />

Let e 0 ∈ ker T φ and ||e 0 || = 1. Write<br />

ker T φ = {0} and dim ker T φ = 1.<br />

e n+1 :=<br />

T φe n<br />

||T φ e n || .<br />

We claim that Ke n = α 2n+2 e n : indeed, Ke 0 = α 2 (1 − T φ T φ )e 0 = α 2 e 0 and if we<br />

assume Ke j = α 2j+2 e j then<br />

Ke j+1 = ||T φ e j || −1 (KT φ e j ) = ||T φ e j || −1 (α 2 T φ Ke j ) = ||T φ e j || −1 (α 2j+4 T φ e j ) = α 2j+4 e j+1 .<br />

Thus we can see that<br />

{<br />

α 2 , α 4 , α 6 , · · · are eigenvalues of K ;<br />

{e n } ∞ n=0 is an orth<strong>on</strong>ormal set since K is self-adjoint.<br />

We will then prove that {e n } forms an orth<strong>on</strong>ormal basis for H 2 . Observe<br />

Thus<br />

tr(H ∗ φH φ ) = the sum of its eigenvalues.<br />

∞∑<br />

α 2n+2 ≤ tr(HφH ∗ φ ) = ||H φ || 2 2<br />

n=0<br />

Since ψ ∈ H ∞ , we have<br />

(|| · || 2 denotes the Hilbert-Schmidt norm).<br />

(5.31)<br />

||H φ || 2 2 = ||H ψ + αH ψ<br />

|| 2 2 = α 2 ||H ψ<br />

|| 2 2 = α 2 tr(H ∗ ψ H ψ ) = α2 tr [T ψ<br />

, T ψ ]<br />

which together with (5.31) implies that<br />

≤ α2<br />

π µ(σ(T ψ)) = α2<br />

π µ(ψ(D)) =<br />

α2<br />

1 − α 2 ,<br />

∑<br />

α 2n+2 ≤ ||H φ || 2 2 ≤<br />

α2<br />

∞ 1 − α 2 = ∑<br />

α 2n+2 ,<br />

so tr(H ∗ φH φ ) = ∑ ∞<br />

n=0 α2n+2 , which say that {α 2n+2 } ∞ n=0 is a complete set of n<strong>on</strong>zero<br />

eigenvalues for K ≡ H ∗ φH φ and each has multiplicity <strong>on</strong>e. Now, by Beurling’s<br />

theorem,<br />

n=0<br />

kerK = kerH ∗ φH φ = kerH φ = bH 2 , where b is inner or b = 0.<br />

Since KT φ = α 2 T φ K, we see that<br />

f ∈ kerK ⇒ T φ f ∈ kerK<br />

185


CHAPTER 5.<br />

TOEPLITZ THEORY<br />

So, since b ∈ kerK, it follows<br />

T φ b = bφ − H φ b = bφ ∈ kerK,<br />

which means that bφ = bh for some h ∈ H 2 . Since φ /∈ H 2 it follows that b = 0 and<br />

kerK = 0. Thus 0 is not an eigenvalue. Therefore {e n } ∞ n=0 is an <strong>on</strong>th<strong>on</strong>ormal basis<br />

for H 2 . Remember that T φ e n = ||T φ e n ||e n+1 . So we can see that T φ is a weighted<br />

shift with weights {||T φ e n ||}. Since<br />

we have<br />

so that<br />

α 2n+2 e n = Ke n = (1 − T φ T φ )e n ,<br />

(1 − α 2n+2 )e n = T φ T φ e n ,<br />

1 − α 2n+2 = ⟨(1 − α 2n+2 )e n , e n ⟩ = ⟨T φ T φ e n , e n ⟩ = ||T φ e n || 2 .<br />

Thus the weights are (1 − α 2n+2 ) 1 2 . By Lemma 5.3.9, T φ is subnormal. Evidently,<br />

φ /∈ H ∞ and T φ is not normal since ran(φ) is not c<strong>on</strong>tained in a line segment.<br />

Corollary 5.3.13. If φ = ψ + αψ is as in Theorem 5.3.12, then neither φ nor φ is<br />

bounded type.<br />

Proof. From Abrahamse’s theorem and Theorem 5.3.12.<br />

We will present a couple of open problems which are related to the subnormality<br />

of Toeplitz operators. They are of particular interest in operator theory.<br />

Problem 5.2.<br />

<br />

For which f ∈ H ∞ , is there λ (0 < λ < 1) with T f+λf<br />

subnormal<br />

Problem 5.3. Suppose ψ is as in Theorem 5.3.12 (i.e., the ellipse map). Are there<br />

g ∈ H ∞ , g ≠ λψ + c, such that T ψ+g is subnormal <br />

Problem 5.4.<br />

More generally, if ψ ∈ H ∞ , define<br />

S(ψ) := {g ∈ H ∞ : T ψ+g is subnormal }.<br />

Describe S(ψ). For example, for which ψ ∈ H ∞ , is it balanced, or is it c<strong>on</strong>vex, or<br />

is it weakly closed What is ext S(ψ) For which ψ ∈ H ∞ , is it strictly c<strong>on</strong>vex ,<br />

i.e., ∂S(ψ) ⊂ ext S(ψ) <br />

In general, S(ψ) is not c<strong>on</strong>vex. In the below (Theorem 5.3.14), we will show that<br />

if ψ is as in Theorem 5.3.12 then {λ : T ψ+λψ<br />

is subnormal} is a n<strong>on</strong>-c<strong>on</strong>vex set.<br />

C. Cowen gave an interesting remark with no dem<strong>on</strong>strati<strong>on</strong> in [Cow3]: If T φ is<br />

subnormal then E(φ) = {λ} with |λ| < 1. However we were unable to decide whether<br />

or not it is true. By comparis<strong>on</strong>, if T φ is normal then E(φ) = {e iθ }.<br />

Problem 5.5. Is the above Cowen’s remark true That is, if T φ is subnormal, does<br />

it follow that E(φ) = {λ} with |λ| < 1 <br />

186


CHAPTER 5.<br />

TOEPLITZ THEORY<br />

If the answer to Problem 5.5 is affirmative, i.e., the Cowen’s remark is true then<br />

for φ = g + f,<br />

T φ is subnormal =⇒ g − λf ∈ H 2<br />

with |λ| < 1 =⇒ g = λf + c (c a c<strong>on</strong>stant),<br />

which says that the answer to Problem 5.3 is negative.<br />

When ψ is as in Theorem 5.3.12, we examine the questi<strong>on</strong>: For which λ, is T ψ+λψ<br />

subnormal <br />

We then have:<br />

Theorem 5.3.14. Let λ ∈ C and 0 < α < 1. Let ψ be the c<strong>on</strong>formal map of the disk<br />

<strong>on</strong>to the interior of the ellipse with vertices ±(1 + α)i passing through ±(1 − α). For<br />

φ = ψ + λψ, T φ is subnormal if and <strong>on</strong>ly if λ = α or λ = αk e iθ +α<br />

1+α k+1 e iθ (−π < θ ≤ π).<br />

To prove Theorem 5.3.14, we need an auxiliary lemma:<br />

Propositi<strong>on</strong> 5.3.15. Let T be the weighted shift with weights<br />

w 2 n =<br />

n∑<br />

α 2j .<br />

Then T + µT ∗ is subnormal if and <strong>on</strong>ly if µ = 0 or |µ| = α k (k = 0, 1, 2, · · · ).<br />

Proof. See [CoL].<br />

j=0<br />

Proof of Theorem 5.3.14. By Theorem 5.3.12, T ψ+αψ<br />

∼ = (1 − α 2 ) 3 2 T , where T is a<br />

weighted shift of Propositi<strong>on</strong> 5.3.15. Thus T ψ<br />

∼ = (1 − α 2 ) 1 2 (T − αT ∗ ), so<br />

(<br />

T φ = T ψ + λTψ ∗ ∼ = (1 − α 2 ) 1 2 (1 − λα) T + λ − α )<br />

1 − λα T ∗ .<br />

Applying Propositi<strong>on</strong> 5.3.15 with λ−α<br />

1−λα<br />

in place of µ gives that for k = 0, 1, 2, · · · ,<br />

λ − α<br />

∣1 − λα∣ = αk ⇐⇒ λ − α<br />

1 − λα = αk e iθ<br />

⇐⇒ λ − α = α k e iθ − λα k+1 e iθ<br />

⇐⇒ λ(1 + α k+1 e iθ ) = α + α k e iθ<br />

⇐⇒ λ =<br />

α + αk e iθ<br />

1 + α k+1 e iθ (−π < θ ≤ π) □<br />

However we find that, surprisingly, some analytic Toeplitz operators are unitarily<br />

equivalent to some n<strong>on</strong>-analytic Toeplitz operators. So C. Cowen noted that subnormality<br />

of Toeplitz operators may not be the wr<strong>on</strong>g questi<strong>on</strong> to be studying.<br />

187


CHAPTER 5.<br />

TOEPLITZ THEORY<br />

Example 5.3.16. Let ψ be the ellipse map as in the example of Cowen and L<strong>on</strong>g.<br />

Then<br />

T ψ<br />

∼ = Tφ with φ = i e− iθ 2 (1 + α 2 e iθ (<br />

)<br />

1 − α 2 ψ +<br />

Proof. Note that<br />

Thus we have<br />

)<br />

αeiθ + α<br />

1 + α 2 e iθ ψ<br />

T ∼ = e iθ 2 T and T + λT<br />

∗ ∼ = e<br />

iθ<br />

2 T + λe<br />

− iθ 2 T ∗ .<br />

T ψ<br />

∼ = (1 − α 2 ) 1 2 (T − αT ∗ )<br />

∼= (1 − α 2 ) 1 2 i(T + αT ∗ )<br />

∼= (1 − α 2 ) 1 2 ie<br />

− iθ 2 (T + αe iθ T ∗ )<br />

(<br />

)<br />

∼= (1 − α 2 ) −1 ie − iθ 2 T ψ + αT ψ<br />

+ αe iθ (T ψ<br />

+ αT ψ )<br />

∼= (1 − α 2 ) −1 i e − iθ 2 T(1+α 2 e iθ )ψ+α(1+e iθ )ψ<br />

(−π < θ < π)<br />

∼= i e− iθ 2 (1 + α 2 e iθ )<br />

1 − α 2 T ψ+<br />

αe iθ +α<br />

1+α 2 e iθ ψ (−π < θ ≤ π).<br />

(−π < θ ≤ π)<br />

188


CHAPTER 5.<br />

TOEPLITZ THEORY<br />

5.3.2 Weak Subnormality<br />

Now it seems to be interesting to understand the gap between k-hyp<strong>on</strong>ormality and<br />

subnormality for Toeplitz operators. As a candidate for the first questi<strong>on</strong> in this line<br />

we posed the following ([CuL1]):<br />

Problem 5.7. Is every 2-hyp<strong>on</strong>ormal Toeplitz operator subnormal <br />

In [CuL1], the following was shown:<br />

Theorem 5.3.17. [CuL1] Every trig<strong>on</strong>ometric Toeplitz operator whose square is<br />

hyp<strong>on</strong>ormal must be normal or analytic. Hence, in particular, every 2-hyp<strong>on</strong>ormal<br />

trig<strong>on</strong>ometric Toeplitz operator is subnormal.<br />

It is well known ([Cu1]) that there is a gap between hyp<strong>on</strong>ormality and 2–hyp<strong>on</strong>ormality<br />

for weighted shifts. Theorem 5.3.17 also shows that there is a big gap between hyp<strong>on</strong>ormality<br />

and 2-hyp<strong>on</strong>ormality for Toeplitz operators. For example, if<br />

φ(z) =<br />

N∑<br />

n=−m<br />

a n z n (m < N)<br />

is such that T φ is hyp<strong>on</strong>ormal then by Theorem 5.3.17, T φ is never 2-hyp<strong>on</strong>ormal<br />

because T φ is neither analytic nor normal (recall that if φ(z) = ∑ N<br />

n=−m a nz n is such<br />

that T φ is normal then m = N (cf. [FL1])).<br />

We can extend Theorem 5.3.17 First of all we observe:<br />

Propositi<strong>on</strong> 5.3.18. [CuL2] If T ∈ L(H) is 2-hyp<strong>on</strong>ormal then<br />

T ( ker [T ∗ , T ] ) ⊆ ker [T ∗ , T ]. (5.32)<br />

Proof. Suppose that [T ∗ , T ]f = 0. Since T is 2-hyp<strong>on</strong>ormal, it follows that (cf. [CMX,<br />

Lemma 1.4])<br />

|⟨[T ∗2 , T ]g, f⟩| 2 ≤ ⟨[T ∗ , T ]f, f⟩⟨[T ∗2 , T 2 ]g, g⟩ for all g ∈ H.<br />

By assumpti<strong>on</strong>, we have that for all g ∈ H, 0 = ⟨[T ∗2 , T ]g, f⟩ = ⟨g, [T ∗2 , T ] ∗ f⟩, so<br />

that [T ∗2 , T ] ∗ f = 0, i.e., T ∗ T 2 f = T 2 T ∗ f. Therefore,<br />

[T ∗ , T ]T f = (T ∗ T 2 − T T ∗ T )f = (T 2 T ∗ − T T ∗ T )f = T [T ∗ , T ]f = 0,<br />

which proves (5.32).<br />

Corollary 5.3.19. If T φ is 2-hyp<strong>on</strong>ormal and if φ or ¯φ is of bounded type then T φ<br />

is normal or analytic, so that T φ is subnormal.<br />

Proof. This follows at <strong>on</strong>ce from Abrahamse’s theorem and Propositi<strong>on</strong> 5.3.18.<br />

189


CHAPTER 5.<br />

TOEPLITZ THEORY<br />

Corollary 5.3.20. If T φ is a 2-hyp<strong>on</strong>ormal operator such that E(φ) c<strong>on</strong>tains at least<br />

two elements then T φ is normal or analytic, so that T φ is subnormal.<br />

Proof. This follows from Corollary 5.3.19 and the fact ([NaT, Propositi<strong>on</strong> 8]) that if<br />

E(φ) c<strong>on</strong>tains at least two elements then φ is of bounded type.<br />

From Corollaries 5.3.19 and 5.3.20, we can see that if T φ is 2-hyp<strong>on</strong>ormal but not<br />

subnormal then φ is not of bounded type and E(φ) c<strong>on</strong>sists of exactly <strong>on</strong>e element.<br />

For a strategy to answer Problem 5.7 we will introduce the noti<strong>on</strong> of “weak subnormality,”<br />

which was introduced by R. Curto and W.Y. <str<strong>on</strong>g>Lee</str<strong>on</strong>g> [CuL2]. Recall that<br />

the operator [ ] T is subnormal if and <strong>on</strong>ly if there exist operators A and B such that<br />

T A<br />

̂T := is normal, i.e.,<br />

0 B<br />

⎧<br />

⎪⎨ [T ∗ , T ] := T ∗ T − T T ∗ = AA ∗<br />

A<br />

⎪⎩<br />

∗ T = BA ∗<br />

[B ∗ , B] + A ∗ A = 0.<br />

(5.33)<br />

We now introduce:<br />

Definiti<strong>on</strong> 5.3.21. [CuL2] An operator T ∈ B(H) is said to be weakly subnormal if<br />

there exist operators A ∈ L(H ′ , H) and B ∈ L(H ′ ) such that the first two c<strong>on</strong>diti<strong>on</strong>s<br />

in (5.33) hold: [T ∗ , T ] = AA ∗ and A ∗ T = BA ∗ . The operator ̂T is said to be a<br />

partially normal extensi<strong>on</strong> of T .<br />

Clearly,<br />

subnormal =⇒ weakly subnormal =⇒ hyp<strong>on</strong>ormal. (5.34)<br />

The c<strong>on</strong>verses of both implicati<strong>on</strong>s in (5.34) are not true in general. Moreover, we<br />

can easily see that the following statements are equivalent for T ∈ B(H):<br />

(a) T is weakly subnormal;<br />

(b) There is an extensi<strong>on</strong> ̂T of T such that ̂T ∗ ̂T f = ̂T ̂T ∗ f for all f ∈ H;<br />

(c) There is an extensi<strong>on</strong> ̂T of T such that H ⊆ ker [ ̂T ∗ , ̂T ].<br />

Weakly subnormal operators possess the following invariance properties:<br />

(i) (Unitary equivalence) if T is weakly subnormal with a partially normal extensi<strong>on</strong><br />

( T 0 B A ) then for every unitary U, ( ) ( )<br />

U ∗ T U U ∗ A<br />

0 B (= U ∗ 0<br />

0 I (<br />

T A<br />

0 B ) ( U 0 I 0 )) is a<br />

partially normal extensi<strong>on</strong> of U ∗ T U, i.e., U ∗ T U is also weakly subnormal.<br />

(ii) (Translati<strong>on</strong>) if T ∈ L(H) is weakly subnormal then T − λ is also weakly subnormal<br />

for every λ ∈ C: indeed if T has a partially normal extensi<strong>on</strong> ̂T then<br />

̂T − λ := ̂T − λ satisfies the properties in Definiti<strong>on</strong> 5.3.21.<br />

190


CHAPTER 5.<br />

TOEPLITZ THEORY<br />

(iii) (Restricti<strong>on</strong>) if T ∈ L(H) is weakly subnormal and if M ∈ Lat T then T | M is also<br />

weakly subnormal because for a partially normal extensi<strong>on</strong> ̂T of T , ̂T |M := ̂T<br />

still satisfies the required properties.<br />

How does <strong>on</strong>e find partially normal extensi<strong>on</strong>s of weakly subnormal operators <br />

Since weakly subnormal operators are hyp<strong>on</strong>ormal, <strong>on</strong>e possible soluti<strong>on</strong> of the equati<strong>on</strong><br />

AA ∗ = [T ∗ , T ] is A := [T ∗ , T ] 1 2 . Indeed this is the case.<br />

Theorem 5.3.22. [CuL2] If T ∈ B(H) is weakly subnormal then T has a partially<br />

normal extensi<strong>on</strong> ̂T <strong>on</strong> K of the form<br />

[ ]<br />

T [T<br />

(3.2.6.1) ̂T ∗ , T ] 1 2<br />

= <strong>on</strong> K := H ⊕ H.<br />

0 B<br />

The proof of Theorem 5.3.22 will make use of the following elementary fact.<br />

Lemma 5.3.23. If T is weakly subnormal then<br />

T (ker [T ∗ , T ]) ⊆ ker [T ∗ , T ].<br />

Proof. By definiti<strong>on</strong>, there exist operators A and B such that [T ∗ , T ] = AA ∗ and<br />

A ∗ T = BA ∗ . If [T ∗ , T ]f = 0 then AA ∗ f = 0 and hence A ∗ f = 0. Therefore<br />

as desired.<br />

[T ∗ , T ]T f = AA ∗ T f = ABA ∗ f = 0,<br />

Definiti<strong>on</strong> 5.3.24. Let T be a weakly subnormal operator <strong>on</strong> H and let ̂T be a partially<br />

normal extensi<strong>on</strong> of T <strong>on</strong> K. We shall say that ̂T is a minimal partially normal<br />

extensi<strong>on</strong> of T if K has no proper subspace c<strong>on</strong>taining H to which the restricti<strong>on</strong> of<br />

̂T is also a partially normal extensi<strong>on</strong> of T . We write ̂T := m.p.n.e.(T ).<br />

Lemma 5.3.25. Let T be a weakly subnormal operator <strong>on</strong> H and let ̂T be a partially<br />

normal extensi<strong>on</strong> of T <strong>on</strong> K. Then ̂T = m.p.n.e.(T ) if and <strong>on</strong>ly if<br />

Proof. See [CuL2].<br />

K = ∨ { ̂T ∗n h : h ∈ H, n = 0, 1 } . (5.35)<br />

It is well known (cf. [C<strong>on</strong>2, Propositi<strong>on</strong> II.2.4]) that if T is a subnormal operator<br />

<strong>on</strong> H and N is a normal extensi<strong>on</strong> of T then N is a minimal normal extensi<strong>on</strong> of T<br />

if and <strong>on</strong>ly if<br />

K = ∨ { ̂T ∗n h : h ∈ H, n ≥ 0}.<br />

Thus if T is a subnormal operator then T may have a partially normal extensi<strong>on</strong><br />

different from a normal extensi<strong>on</strong>. For, c<strong>on</strong>sider the unilateral (unweighted) shift U +<br />

acting <strong>on</strong> l 2 (Z + ). Then m.n.e. (U + ) = U, the bilateral shift acting <strong>on</strong> l 2 (Z), with<br />

orth<strong>on</strong>ormal basis {e n } ∞ n=−∞. It is easy to verify that m.p.n.e. (U + ) = U| L , where<br />

L :=< e −1 > ⊕ l 2 (Z + ).<br />

191


CHAPTER 5.<br />

TOEPLITZ THEORY<br />

Theorem 5.3.26. Let T ∈ B(H).<br />

(i) If T is 2-hyp<strong>on</strong>ormal then [T ∗ , T ] 1 2 T [T ∗ , T ] − 1 2 | Ran[T ∗ ,T ] is bounded;<br />

(ii) T is (k + 1)-hyp<strong>on</strong>ormal if and <strong>on</strong>ly if T is weakly subnormal and ̂T :=<br />

m.p.n.e.(T ) is k-hyp<strong>on</strong>ormal.<br />

Proof. See [CJP, Theorems 2.7 and 3.2].<br />

In 1966, Stampfli [Sta3] explicitly exhibited for a subnormal weighted shift A 0 its<br />

minimal normal extensi<strong>on</strong><br />

⎡<br />

⎤<br />

A 0 B 1 0<br />

A 1 B 2<br />

N :=<br />

⎢<br />

.<br />

⎣ A ..<br />

2<br />

⎥<br />

⎦ , (5.36)<br />

.<br />

0<br />

..<br />

where A n is a weighted shift with weights {a (n)<br />

0 , a(n) 1 , · · · }, B n := diag{b (n)<br />

0 , b(n) 1 , · · · },<br />

and these entries satisfy:<br />

(I) (a (n)<br />

j ) 2 − (a (n)<br />

j−1 )2 + (b (n)<br />

j ) 2 ≥ 0 (b (0)<br />

j = 0 for all j);<br />

(II) b (n)<br />

j<br />

= 0 =⇒ b (n)<br />

j+1 = 0;<br />

(III) there exists a c<strong>on</strong>stant M such that |a (n)<br />

j | ≤ M and |b (n)<br />

j | ≤ M for n = 0, 1, · · ·<br />

and j = 0, 1, · · · , where<br />

(if b (n)<br />

j 0<br />

b (n+1)<br />

j<br />

:= [(a (n)<br />

j ) 2 − (a (n)<br />

j−1 )2 + (b (n)<br />

j ) 2 ] 1 2 and a (n+1)<br />

j<br />

= 0, then a (n)<br />

j 0<br />

is taken to be 0).<br />

:= a (n) b (n+1)<br />

j+1<br />

j<br />

b (n+1)<br />

j<br />

We will now discuss analogues of the preceding results for k-hyp<strong>on</strong>ormal operators.<br />

Our criteri<strong>on</strong> <strong>on</strong> k-hyp<strong>on</strong>ormality follows:<br />

Theorem 5.3.27. An operator A 0 ∈ B(H 0 ) is k-hyp<strong>on</strong>ormal if and <strong>on</strong>ly if the following<br />

three c<strong>on</strong>diti<strong>on</strong>s hold for all n such that 0 ≤ n ≤ k − 1:<br />

(I n ) D n ≥ 0;<br />

(II n ) A n−1 (ker D n−1 ) ⊆ ker D n−1 (n ≥ 1);<br />

(III n ) D 1 2<br />

n−1 A n−1 D − 1 2<br />

n−1 | Ran (D n−1) (n ≥ 1) is bounded,<br />

where<br />

D 0 := [A ∗ 0, A 0 ], D n+1 := D n | Hn+1 + [A ∗ n+1, A n+1 ], H n+1 := ran (D n )<br />

and A n+1 denotes the bounded extensi<strong>on</strong> of D 1 2 n A n D − 1 2<br />

n<br />

Ran (D n ).<br />

to ran (D n )(= H n+1 ) from<br />

192


CHAPTER 5.<br />

TOEPLITZ THEORY<br />

Proof. Suppose A 0 is k-hyp<strong>on</strong>ormal. We now use inducti<strong>on</strong> <strong>on</strong> k. If k = 2 then A 0 is<br />

2-hyp<strong>on</strong>ormal, and so D 0 := [A ∗ 0, A 0 ] ≥ 0. By Theorem 5.3.26 (i), D 1 2<br />

0 A 0 D − 1 2<br />

0 | ran (D0)<br />

is bounded. Let A 1 be the bounded extensi<strong>on</strong> of D 1 2<br />

0 A 0 D − 1 2<br />

0[<br />

from Ran<br />

]<br />

(D 0 ) to H 1 :=<br />

Ran (D 0 ) and D 1 := D 0 | H1 + [A ∗ 1, A 1 ]. Writing Â0 :=<br />

A 0 D 1 2<br />

0 , we have<br />

0 A Â0 =<br />

1<br />

m.p.n.e. (A 0 ), which is hyp<strong>on</strong>ormal by Theorem 5.3.26(ii). Thus<br />

[Â0∗<br />

, Â 0 ] =<br />

[ ]<br />

0 0<br />

0 D 0 | H1 + [A ∗ ≥ 0.<br />

1, A 1 ]<br />

and hence D 1 ≥ 0. Also by [CuL2, Lemma 2.2], A 0 (ker D 0 ) ⊆ ker D 0 whenever A 0 is<br />

2-hyp<strong>on</strong>ormal. Thus (I n ), (II n ), and (III n ) hold for n = 0, 1. Assume now that if A 0<br />

is k-hyp<strong>on</strong>ormal then (I n ),(II n ) and (III n ) hold for all 0 ≤ n ≤ k − 1. Suppose A 0 is<br />

(k + 1)-hyp<strong>on</strong>ormal. We must show that (I n ),(II n ) and (III n ) hold for n = k. Define<br />

⎡<br />

⎤<br />

A 0 D 1 2 0 0<br />

A 1 D 1 2<br />

1<br />

S :=<br />

. .. . ..<br />

⊕k−1<br />

⊕k−1<br />

: H i −→ H i .<br />

⎢<br />

⎣<br />

. ..<br />

1 ⎥ i=0<br />

i=0<br />

D<br />

2 ⎦<br />

k−2<br />

0 A k−1<br />

By our inductive assumpti<strong>on</strong>, D k−1 ≥ 0. Writing ̂T (n) := m.p.n.e.( ̂T (n−1) ) when it<br />

exists, we can see by our assumpti<strong>on</strong> that S = Â0(k−1)<br />

: indeed, if<br />

⎡<br />

⎤<br />

A 0 D 1 2 0 0<br />

A 1 D 1 2<br />

1<br />

S l :=<br />

. .. . ..<br />

⎢<br />

⎣<br />

. ⎥ ..<br />

D 1 2 ⎦<br />

l−2<br />

0 A l−1<br />

then since by assumpti<strong>on</strong> [Sl ∗, S l] = 0 ⊕ D l and A l = D 1 2<br />

l−1<br />

A l−1 D − 1 2<br />

l−1 | Ran (D l−1 ), it<br />

follows that S l is the minimal partially normal extensi<strong>on</strong> of S l−1 (1 ≤ l ≤ k − 1). But<br />

since by our assumpti<strong>on</strong> A 0 is (k + 1)-hyp<strong>on</strong>ormal, it follows from Lemma 5.3.26(ii)<br />

that S is 2-hyp<strong>on</strong>ormal. Thus by Theorem 5.3.26(i), [S ∗ , S] 1 2 S[S ∗ , S] − 1 2 | Ran ([S∗ ,S]) is<br />

bounded, which says that D 1 2<br />

k−1<br />

A k−1 D − 1 2<br />

k−1 | Ran (D k−1 ) is bounded, proving (III n ) for<br />

]<br />

n = k. Observe that A k , H k and D k are well-defined. Writing<br />

[S Ŝ := D 1 2<br />

k−1 ,<br />

0 A k<br />

we can see that Ŝ = m.p.n.e.(S), [ ] which is hyp<strong>on</strong>ormal, again by Theorem 5.3.26(ii).<br />

0 0<br />

Thus, since [Ŝ∗ , Ŝ] = ≥ 0, we have D<br />

0 D k ≥ 0, proving (I n ) for n = k. On the<br />

k<br />

193


CHAPTER 5.<br />

TOEPLITZ THEORY<br />

other hand, [ since S]<br />

is 2-hyp<strong>on</strong>ormal, it follows that S(ker[S ∗ , S]) ⊆ ker[S ∗ , S]. Since<br />

0 0<br />

[S ∗ , S] =<br />

, we have ker [S<br />

0 D ∗ , S] = ⊕ k−2<br />

i=0 H ⊕<br />

i ker (Dk−1 ). Thus, since<br />

k−1<br />

⎡<br />

⎤<br />

A 0 D 1 2 0 0<br />

⎡<br />

A 1 D 1 2<br />

1<br />

. .. . ..<br />

⎢<br />

⎢<br />

⎣<br />

. ⎥ ..<br />

⎣<br />

D 1 2 ⎦<br />

k−2<br />

0 A k−1<br />

H 0<br />

H 1<br />

. .<br />

H k−2<br />

ker (D k−1 )<br />

⎤<br />

⎡<br />

⊆<br />

⎥ ⎢<br />

⎦ ⎣<br />

H 0<br />

H 1<br />

. .<br />

H k−2<br />

ker (D k−1 )<br />

we must have that A k−1 (ker (D k−1 )) ⊆ ker (D k−1 ), proving (II n ) for n = k. This<br />

proves the necessity c<strong>on</strong>diti<strong>on</strong>.<br />

Toward sufficiency, suppose that c<strong>on</strong>diti<strong>on</strong>s (I n ), (II n ) and (III n ) hold for all n<br />

such that 0 ≤ n ≤ k − 1. Define<br />

⎡<br />

⎤<br />

A 0 D 1 2 0 0<br />

A 1 D 1 2<br />

1<br />

S n :=<br />

. .. . ..<br />

(1 ≤ n ≤ k − 1).<br />

⎢<br />

⎣<br />

. ⎥ ..<br />

D 1 2 ⎦<br />

n−2<br />

0 A n−1<br />

Then S k−2 is weakly subnormal and S k−1 [ = m.p.n.e. ] (S k−2 ). Since, by assumpti<strong>on</strong>,<br />

D k−1 ≥ 0, we have [Sk−1 ∗ , S k−1] =<br />

≥ 0. It thus follows from<br />

0 0<br />

0 D k−1<br />

Theorem 5.3.26(ii) that S k−2 is 2-hyp<strong>on</strong>ormal. Note that S n = m.p.n.e. (S n−1 ) for<br />

n = 1, · · · , k−1 (S 0 := A 0 ). Thus, again by Theorem 5.3.26(ii), S k−3 is 3-hyp<strong>on</strong>ormal.<br />

Now repeating this argument, we can c<strong>on</strong>clude that S 0 ≡ A 0 is k-hyp<strong>on</strong>ormal. This<br />

completes the proof.<br />

⎤<br />

,<br />

⎥<br />

⎦<br />

Corollary 5.3.28. An operator A 0 ∈ B(H 0 ) is subnormal if and <strong>on</strong>ly if the c<strong>on</strong>diti<strong>on</strong>s<br />

(I n ), (II n ), and (III n ) hold for all n ≥ 0. In this case, the minimal normal extensi<strong>on</strong><br />

N of A 0 is given by<br />

⎡<br />

⎤<br />

A 0 D 1 2 0 0<br />

A 1 D 1 2<br />

1<br />

∞⊕<br />

∞⊕<br />

N =<br />

⎢<br />

.<br />

⎣ A ..<br />

: H<br />

2<br />

⎥ i → H i .<br />

⎦ i=0<br />

i=0<br />

.<br />

0<br />

..<br />

194


CHAPTER 5.<br />

TOEPLITZ THEORY<br />

5.3.3 Gaps between k-Hyp<strong>on</strong>ormality and Subnormality<br />

We find gaps between subnormality and k-hyp<strong>on</strong>ormality for Toeplitz operators.<br />

Theorem 5.3.29. [Gu2],[CLL] Let 0 < α < 1 and let ψ be the c<strong>on</strong>formal map of the<br />

unit disk <strong>on</strong>to the interior of the ellipse with vertices ±(1 + α)i and passing through<br />

±(1 − α). Let φ = ψ + λ ¯ψ and let T φ be the corresp<strong>on</strong>ding Toeplitz operator ∣ <strong>on</strong> H 2 .<br />

∣<br />

Then T φ is k-hyp<strong>on</strong>ormal if and <strong>on</strong>ly if λ is in the circle ∣z − α(1−α2j ) ∣∣<br />

1−α =<br />

α j (1−α 2 )<br />

2j+2 1−α ∣ 2j+2<br />

∣<br />

for j = 0, 1, · · · , k − 2 or in the closed disk ∣z − α(1−α2(k−1) ) ∣∣<br />

1−α ≤<br />

α k−1 (1−α 2 )<br />

.<br />

2k 1−α 2k<br />

For 0 < α < 1, let T ≡ W β be the weighted shift with weight sequence β =<br />

{β n } ∞ n=0, where (cf. [Cow2, Propositi<strong>on</strong> 9])<br />

β n := (<br />

n∑<br />

α 2j ) 1 2 for n = 0, 1, · · · . (5.37)<br />

j=0<br />

Let D be the diag<strong>on</strong>al operator, D = diag (α n ), and let S λ ≡ T + λ T ∗ (λ ∈ C). Then<br />

we have that<br />

[T ∗ , T ] = D 2 = diag (α 2n ) and [S ∗ λ, S λ ] = (1 − |λ| 2 )[T ∗ , T ] = (1 − |λ| 2 )D 2 .<br />

Define<br />

A l := α l T + λ α l T ∗ (l = 0, ±1, ±2, · · · ).<br />

It follows that A 0 = S λ and<br />

DA l = A l+1 D and A ∗ l D = DA ∗ l+1 (l = 0, ±1, ±2, · · · ). (5.38)<br />

Theorem 5.3.30. Let 0 < α < 1 and T ≡ W β<br />

sequence β = {β n } ∞ n=0, where<br />

be the weighted shift with weight<br />

β n = (<br />

n∑<br />

α 2j ) 1 2 for n = 0, 1, · · · .<br />

j=0<br />

Then A 0 := T + λT ∗ is k-hyp<strong>on</strong>ormal if and <strong>on</strong>ly if |λ| ≤ α k−1 or |λ| = α j for some<br />

j = 0, 1, · · · , k − 2.<br />

Proof. Observe that<br />

[A ∗ l , A l] = [α l T ∗ + λ α l T, α l T + λ α l T ∗ ]<br />

= α 2l [T ∗ , T ] − |λ|2<br />

α 2l [T ∗ , T ] =<br />

(α 2l − |λ|2<br />

α 2l )<br />

D 2 .<br />

(5.39)<br />

195


CHAPTER 5.<br />

TOEPLITZ THEORY<br />

Since ker D = {0} and DA n = A n+1 D, it follows that H n = H for all n; if we use A l<br />

for the operator A n in Theorem 5.3.27 then we have, by (5.39) and the definiti<strong>on</strong> of<br />

D j , that<br />

D j = D j−1 + [A ∗ j , A j ] = D j−2 + [A ∗ j−1, A j−1 ] + [A ∗ j , A j ] = · · ·<br />

= [A ∗ 0, A 0 ] + [A ∗ 1, A 1 ] + · · · + [A ∗ j , A j ] = (1 − |λ| 2 )D 2 + · · · +<br />

=<br />

( ) ( )<br />

1 − α<br />

2(j+1)<br />

1 − α 2 1 − |λ|2<br />

α 2j D 2 .<br />

(<br />

)<br />

α 2j − |λ|2<br />

α 2j D 2<br />

By Theorem 5.3.27, A 0 is k-hyp<strong>on</strong>ormal if and <strong>on</strong>ly if D k−1 ≥ 0 or D j = 0 for<br />

some j such that 0 ≤ j ≤ k − 2 (in this case A 0 is subnormal). Note that D j = 0 if<br />

and <strong>on</strong>ly if |λ| = α j . On the other hand, if D j > 0 for j = 0, 1, · · · , k − 2, then<br />

D k−1 =<br />

( 1 − α<br />

2k<br />

1 − α 2 ) (<br />

1 − |λ|2<br />

α 2(k−1) )<br />

D 2 ≥ 0<br />

if and <strong>on</strong>ly if |λ| ≤ α k−1 . Therefore A 0 is k-hyp<strong>on</strong>ormal if and <strong>on</strong>ly if |λ| ≤ α k−1 or<br />

|λ| = α j for some j, j = 0, 1, · · · , k − 2.<br />

We are ready for:<br />

Proof. of Theorem 5.3.29 It was shown in [CoL] that T ψ+α ¯ψ is unitarily equivalent<br />

to (1 − α 2 ) 3 2 T , where T is the weighted shift in Theorem 5.3.30. Thus T ψ is unitarily<br />

equivalent to (1 − α 2 ) 1 2 (T − αT ∗ ), so T φ is unitarily equivalent to<br />

(1 − α 2 ) 1 2 (1 − λ α)(T +<br />

λ − α<br />

1 − λ α T ∗ ) (cf. [Cow1, Theorem 2.4]).<br />

Applying Theorem 5.3.30 with λ−α<br />

1−λ α<br />

λ − α<br />

∣1 − λ α∣ ≤ αk ⇐⇒ |λ − α| 2 ≤ α 2k |1 − λ α| 2<br />

This completes the proof.<br />

in place of λ, we have that for k = 0, 1, 2, · · · ,<br />

⇐⇒ |λ| 2 − α(1 − α2k )<br />

1 − α 2k+2 (λ + ¯λ) + α2 − α 2k<br />

1 − α 2k+2 ≤ 0<br />

⇐⇒<br />

∣ λ − α(1 − ∣ α2k ) ∣∣∣<br />

1 − α 2k+2 ≤ αk (1 − α 2 )<br />

1 − α 2k+2 .<br />

196


CHAPTER 5.<br />

TOEPLITZ THEORY<br />

5.4 Comments and Problems<br />

From Corollary 5.3.19 we can see that if T φ is a 2-hyp<strong>on</strong>ormal operator such that φ<br />

or ¯φ is of bounded type then T φ has a n<strong>on</strong>trivial invariant subspace. The following<br />

questi<strong>on</strong> is naturally raised:<br />

Problem 5.8. Does every 2-hyp<strong>on</strong>ormal Toeplitz operator have a n<strong>on</strong>trivial invariant<br />

subspace More generally, does every 2-hyp<strong>on</strong>ormal operator have a n<strong>on</strong>trivial<br />

invariant subspace <br />

It is well known ([Bro]) that if T is a hyp<strong>on</strong>ormal operator such that R(σ(T )) ≠<br />

C(σ(T )) then T has a n<strong>on</strong>trivial invariant subspace. But it remains still open whether<br />

every hyp<strong>on</strong>ormal operator with R(σ(T )) = C(σ(T )) (i.e., a thin spectrum) has a<br />

n<strong>on</strong>trivial invariant subspace. Recall that T ∈ B(H) is called a v<strong>on</strong>-Neumann operator<br />

if σ(T ) is a spectral set for T , or equivalently, f(T ) is normaloid (i.e., norm equals<br />

spectral radius) for every rati<strong>on</strong>al functi<strong>on</strong> f with poles off σ(T ). Recently, B. Prunaru<br />

[Pru] has proved that polynomially hyp<strong>on</strong>ormal operators have n<strong>on</strong>trivial invariant<br />

subspaces. It was also known ([Ag1]) that v<strong>on</strong>-Neumann operators enjoy the same<br />

property. The following is a sub-questi<strong>on</strong> of Problem G.<br />

Problem 5.9.<br />

operator <br />

Is every 2-hyp<strong>on</strong>ormal operator with thin spectrum a v<strong>on</strong>-Neumann<br />

Although the existence of a n<strong>on</strong>-subnormal polynomially hyp<strong>on</strong>ormal weighted<br />

shift was established in [CP1] and [CP2], it is still an open questi<strong>on</strong> whether the implicati<strong>on</strong><br />

“polynomially hyp<strong>on</strong>ormal ⇒ subnormal” can be disproved with a Toeplitz<br />

operator.<br />

Problem 5.10. Does there exist a Toeplitz operator which is polynomially hyp<strong>on</strong>ormal<br />

but not subnormal <br />

In [CuL2] it was shown that every pure 2-hyp<strong>on</strong>ormal operator with rank-<strong>on</strong>e selfcommutator<br />

is a linear functi<strong>on</strong> of the unilateral shift. McCarthy and Yang [McCYa]<br />

classified all rati<strong>on</strong>ally cyclic subnormal operators with finite rank self-commutators.<br />

However it remains still open what are the pure subnormal operators with finite rank<br />

self-commutators.<br />

Now the following questi<strong>on</strong> comes up at <strong>on</strong>ce:<br />

Problem 5.11. If T φ is a 2-hyp<strong>on</strong>ormal Toeplitz operator with n<strong>on</strong>zero finite rank<br />

self-commutator, does it follow that T φ is analytic <br />

For affirmativeness to Problem J we shall give a partial answer. To do this we recall<br />

Theorem 15 in [NaT] which states that if T φ is subnormal and φ = q ¯φ, where q is a<br />

finite Blaschke product then T φ is normal or analytic. But from a careful examinati<strong>on</strong><br />

of the proof of the theorem we can see that its proof uses subnormality assumpti<strong>on</strong><br />

<strong>on</strong>ly for the fact that ker [T ∗ φ, T φ ] is invariant under T φ . Thus in view of Propositi<strong>on</strong><br />

3.2.2, the theorem is still valid for “2–hyp<strong>on</strong>ormal” in place of “subnormal”. We thus<br />

have:<br />

197


CHAPTER 5.<br />

TOEPLITZ THEORY<br />

Theorem 5.4.1. If T φ is 2–hyp<strong>on</strong>ormal and φ = q ¯φ, where q is a finite Blaschke<br />

product then T φ is normal or analytic.<br />

We now give a partial answer to Problem 5.11.<br />

Theorem 5.4.2. Suppose log |φ| is not integrable. If T φ is a 2–hyp<strong>on</strong>ormal operator<br />

with n<strong>on</strong>zero finite rank self-commutator then T φ is analytic.<br />

Proof. If T φ is hyp<strong>on</strong>ormal such that log |φ| is not integrable then by an argument<br />

of [NaT, Theorem 4], φ = q ¯φ for some inner functi<strong>on</strong> q. Also if T φ has a finite rank<br />

self-commutator then by [NaT, Theorem 10], there exists a finite Blaschke product<br />

b ∈ E(φ). If q ≠ b, so that E(φ) c<strong>on</strong>tains at least two elements, then by Corollary<br />

5.3.20, T φ is normal or analytic. If instead q = b then by Theorem 5.4.1, T φ is also<br />

normal or analytic.<br />

Theorem 5.4.2 reduces Problem 5.11 to the class of Toeplitz operators such that<br />

log |φ| is integrable. If log |φ| is integrable then there exists an outer functi<strong>on</strong> e such<br />

that |φ| = |e|. Thus we may write φ = ue, where u is a unimodular functi<strong>on</strong>. Since<br />

by the Douglas-Rudin theorem (cf. [Ga, p.192]), every unimodular functi<strong>on</strong> can be<br />

approximated by quotients of inner functi<strong>on</strong>s, it follows that if log |φ| is integrable<br />

then φ can be approximated by functi<strong>on</strong>s of bounded type. Therefore if we could<br />

obtain such a sequence ψ n c<strong>on</strong>verging to φ such that T ψn is 2–hyp<strong>on</strong>ormal with finite<br />

rank self-commutator for each n, then we would answer Problem J affirmatively. On<br />

the other hand, if T φ attains its norm then by a result of Brown and Douglas [BD],<br />

φ is of the form φ = λ ψ θ<br />

with λ > 0, ψ and θ inner. Thus φ is of bounded type.<br />

Therefore by Corollary 5.3.20, if T φ is 2–hyp<strong>on</strong>ormal and attains its norm then T φ is<br />

normal or analytic. However we were not able to decide that if T φ is a 2–hyp<strong>on</strong>ormal<br />

operator with finite rank self-commutator then T φ attains its norm.<br />

198


Chapter 6<br />

A Brief Survey <strong>on</strong> the<br />

Invariant Subspace Problem<br />

6.1 A Brief History<br />

Let H be a separable complex Hilbert space. If T ∈ L(H) then T is said to have a<br />

n<strong>on</strong>trivial invariant subspace if there is a subspace M of H such that {0} ≠ M ≠ H<br />

and T M ⊂ M. In this case we can represent T as<br />

[ ] ∗ ∗<br />

T = <strong>on</strong> M ⊕ M ⊥ .<br />

0 ∗<br />

Example 6.1.1. If T has aigenvalue λ, put<br />

M λ := {x : T x = λx} ≡ the eigenspace corresp<strong>on</strong>ding to λ.<br />

Then evidently T M λ ⊆ M λ . If T ≠ λ then M λ is n<strong>on</strong>trivial.<br />

Invariant Subspace Problem (1932, J. v<strong>on</strong> Neumann) Let X ≡ a Banach space<br />

of dim ≥ 2 and T ∈ B(X ). Does T have a n<strong>on</strong>trivial invariant subspace <br />

Let K(H) be the set of compact operators <strong>on</strong> H. If K ∈ K(H) has a polar<br />

decompositi<strong>on</strong> K = U |T |, where |T | := (T ∗ T ) 1 2 and U is a partial isometry, then |T | ∈<br />

K(H) and so has a diag<strong>on</strong>al matrix diag (λ 1 , λ 2 , · · · ) relative to some orth<strong>on</strong>ormal<br />

basis for H. For p ≥ 1 we define<br />

{<br />

}<br />

∞∑<br />

C p (H) := K ∈ K(H) : λ p n < ∞ ,<br />

199<br />

n=1


CHAPTER 6.<br />

A BRIEF SURVEY ON THE INVARIANT SUBSPACE PROBLEM<br />

which is called the Schatten p-ideal. The ideal C 1 (H) is known as the trace class and<br />

the ideal C 2 (H) as the Hilbert-Schmidt class.<br />

1984 C.J. Read answered ISP in the negative (for l 1 ).<br />

Comment. However ISP is still open for a separable Hilbert space.<br />

1934 J. v<strong>on</strong> Neumann (unpublished): T ∈ K(H) =⇒ T has n.i.s.<br />

1954 N. Ar<strong>on</strong>szajn and K. Smith (Ann. of Math.): T ∈ K(H) =⇒ T has n.i.s.<br />

1966 A. Bernstein and A. Robins<strong>on</strong> (Pacific J. of Math.)<br />

T is polynomially compact (i.e., p(T ) is compact for a polynomial p) =⇒ T has n.i.s.<br />

1966 P. Halmos (Pacific J. of Math.) reproved Bernstein-Robins<strong>on</strong> theorem via<br />

analysis technique.<br />

1973 K. Lom<strong>on</strong>osov (Funk. Anal. Pril.)<br />

T (≠ λ) commutes with a n<strong>on</strong>zero compact operator =⇒ T has n.i.s.<br />

1978 S. Brown (Int. Eq. Op. Th.): T is subnormal =⇒ T has n.i.s.<br />

1986 S. Brown, Chevreau, C. Pearcy (J. Funct. Anal.)<br />

||T || ≤ 1, σ(T ) ⊇ T (= the unit circle) =⇒ T has n.i.s.<br />

1987 S. Brown (Ann. of Math.)<br />

T is hyp<strong>on</strong>ormal with int σ(T ) ≠ ∅ =⇒ T has n.i.s.<br />

Problem. Prove or disprove ISP for hyp<strong>on</strong>ormal operators.<br />

200


CHAPTER 6.<br />

A BRIEF SURVEY ON THE INVARIANT SUBSPACE PROBLEM<br />

6.2 Basic Facts<br />

The spectral picture of T ∈ B(H), SP(T ), is the structure c<strong>on</strong>sisting of σ e (T ), the<br />

collecti<strong>on</strong> of holes and pseudoholes in σ e (T ), and the indices associated with these<br />

holes and pseudoholes.<br />

If H i (i = 1, 2) is a separable Hilbert space and T i ∈ B(H i ), then T 1 and T 2 are said<br />

to be compalent (notati<strong>on</strong>:T 1 ∼ T 2 ) if there exists a unitary operator W ∈ B(H 1 , H 2 )<br />

and a compact operator K ∈ K(H 2 ) such that W T 1 W ∗ + K = T 2 .<br />

Propositi<strong>on</strong> 6.2.1. The relati<strong>on</strong> of compalence <strong>on</strong> B(H) is an equivalence relati<strong>on</strong><br />

and partiti<strong>on</strong>s B(H) into equivalence classes.<br />

Definiti<strong>on</strong> 6.2.2. An operator T ∈ B(H) is called essentially normal if [T ∗ , T ] ∈<br />

K(H), or equivalently, if π(T ) is normal in B(H)/K(H). We write (EN)(H) for the<br />

set of all essentially normal operators in B(H).<br />

Theorem 6.2.3. (BDF Theorem) [BDF] If T ∈ (EN)(H 1 ) and T 2 ∈ (EN)(H 2 ) then<br />

T 1 ∼ T 2 ⇐⇒ SP(T 1 ) = SP(T 2 ).<br />

Suppose there exists a unitary operator W and a compact operator K such that<br />

W T 1 W ∗ +K = T 2 . If ||K|| < ϵ then T 1 and T 2 are said to be ϵ-compalent. (Notati<strong>on</strong>:<br />

T 1 ∼ T 2 (ϵ)).<br />

Theorem 6.2.4. [Ber] If N ∈ B(H) is normal then for any ϵ > 0, there exists a<br />

diag<strong>on</strong>al operator D ϵ such that N ∼ D ϵ (ϵ).<br />

201


CHAPTER 6.<br />

A BRIEF SURVEY ON THE INVARIANT SUBSPACE PROBLEM<br />

6.3 Quasitriangular operators<br />

An operator T ∈ B(H) is called quasitriangular if there exists a sequence {P n } of<br />

projecti<strong>on</strong>s of finite rank that<br />

P n → 1 weakly and ||P n T P n − T P n || → 0.<br />

We write QT (H) for the set of all quasitriangular operators in B(H).<br />

Compact operators are quasitriangular. Indeed, if P n is a projecti<strong>on</strong> such that<br />

P n → I weakly and K is compact then ||P n KP n − K|| → 0. So<br />

||P n KP n − KP n || = ||P n (KP n )P n − (KP n )|| = ||P n K ′ P n − K ′ || → 0.<br />

A trivial example of a quasitriangular operator is an upper triangular operator: indeed<br />

if P n denotes the orthog<strong>on</strong>al projecti<strong>on</strong> <strong>on</strong>to ∨ {e 1 , · · · , e n }, then T P n H ⊂ P n H, so<br />

P n T P n = T P n .<br />

Definiti<strong>on</strong> 6.3.1. An operator T ∈ B(H) is called triangular if there exists an<br />

orth<strong>on</strong>ormal basis {e n } for H such that T is upper triangular.<br />

Evidently, triangular ⇒ quasitriangular.<br />

Theorem 6.3.2. (P.Halmos, Quasitriangular operators, Acta Sci. Math. (Szeged)<br />

29 (1968), 283–293)<br />

QT (H) is norm closed.<br />

Theorem 6.3.3. normal ⇒ quasitriangular.<br />

Proof. By Theorem 6.2.4, if T is normal then for any ϵ > 0,<br />

T ∼ D ϵ (ϵ) with a diag<strong>on</strong>al D ϵ ,<br />

i.e., W ϵ T Wϵ ∗ = D ϵ + K ϵ with ||K ϵ || < ϵ. So, ||T − WnD ∗ n W n || = ϵ → 0.<br />

Theorem 6.3.4. (P.Halmos, Quasitriangular operators, Acta Sci. Math. (Szeged)<br />

29 (1968), 283–293)<br />

QT (H) = T riangular + Compact.<br />

Theorem 6.3.5. (R. Douglas and C. Pearcy, A note <strong>on</strong> quasitriangular operators,<br />

Duke math. J. 37(1970), 177-188)<br />

Similarity preserves quasitriangularity<br />

202


CHAPTER 6.<br />

A BRIEF SURVEY ON THE INVARIANT SUBSPACE PROBLEM<br />

Corollary 6.3.6. Compalence preserves quasitriangularity.<br />

Theorem 6.3.7. (AFV Theorem) (Apostol, Foias, and Voiculescu, Some results <strong>on</strong><br />

n<strong>on</strong>-quasitriangular operators II, Rev. Roumaine Math. Pure Appl. 18(1973), 159–<br />

181) If T ∈ B(H) then T is quasitriangular if and <strong>on</strong>ly if SP(T ) c<strong>on</strong>tains no hole or<br />

pseudohole associated with a negative number.<br />

Definiti<strong>on</strong> 6.3.8. An operator T ∈ B(H) is said to have a n<strong>on</strong>trivial hyperinvariant<br />

subspace if there exists a n<strong>on</strong>trivial closed subspace M such that<br />

T ′ M ⊂ M for every T ′ with T T ′ = T ′ T.<br />

Definiti<strong>on</strong> 6.3.9. An operator T ∈ B(H) is called biquasitriangular if T, T ∗ ∈ B(H).<br />

We write (BQT )(H) for the set of all biquasitriangular operators <strong>on</strong> H.<br />

Theorem 6.3.10. If T /∈ (BQT )(H) then either T or T ∗ has an eigenvalue and so<br />

T has a n<strong>on</strong>trivial hyperinvariant subspace.<br />

Proof. We c<strong>on</strong>sider T /∈ (QT )(H). By the AFV theorem, there exists λ 0 such that<br />

T − λ 0 is semi-Fredholm with −∞ ≤ index (T − λ 0 ) < 0. Thus dim ker (T ∗ − λ 0 ) > 0,<br />

and hence λ 0 is an eugenvalue for T ∗ . In fact, M = {x ∈ H : T ∗ x = λx} is<br />

hyperinvariant for T ∗ . Since λ 0 cannot be n<strong>on</strong>quasitriangular, we have M ≠ H.<br />

Thus M ⊥ is a n<strong>on</strong>trivial hyperinvariant subspace for T .<br />

1973 Berger-Shaw If T is hyp<strong>on</strong>ormal and cyclic (i.e., there exists a vector e 0 such<br />

that H = cl {p(T )e 0 : p = a polynomial} then [T ∗ , T ] ∈ C 1 .<br />

If T is not cyclic, and so<br />

then T M ⊆ M.<br />

M ≡ cl {p(T )e : e = a vector} ̸= H<br />

1979 Voiculescu: Normal ∼ = Diag<strong>on</strong>al normal + C 2 .<br />

Sub-C<strong>on</strong>clusi<strong>on</strong>. The <strong>on</strong>ly hyp<strong>on</strong>ormal operators without known n.i.s. bel<strong>on</strong>g to<br />

T ∼ = Diag<strong>on</strong>al normal + Compact with [T ∗ , T ] ∈ C 1 .<br />

203


CHAPTER 6.<br />

A BRIEF SURVEY ON THE INVARIANT SUBSPACE PROBLEM<br />

6.4 <strong>Operator</strong>s whose spectra are Carathéodory regi<strong>on</strong>s<br />

In this secti<strong>on</strong> it is shown that if an operator T satisfies ||p(T )|| ≤ ||p|| σ(T ) for every<br />

polynomial p and the polynomially c<strong>on</strong>vex hull of σ(T ) is a Carathéodory regi<strong>on</strong><br />

whose accessible boundary points lie in rectifiable Jordan arcs <strong>on</strong> its boundary, then<br />

T has a n<strong>on</strong>trivial invariant subspace. As a corollary, it is also shown that if T is<br />

a hyp<strong>on</strong>ormal operator and the outer boundary of σ(T ) has at most finitely many<br />

prime ends corresp<strong>on</strong>ding to singular points <strong>on</strong> ∂D and has a tangent at almost every<br />

point <strong>on</strong> each Jordan arc, then T has a n<strong>on</strong>trivial invariant subspace.<br />

To prove the main theorem we first review some definiti<strong>on</strong>s and auxiliary lemmas.<br />

Let K be a compact subset of C. Write ηK for the polynomially c<strong>on</strong>vex hull of K.<br />

The outer boundary of K means ∂(ηK), i.e., the boundary of ηK. If Γ is a Jordan<br />

curve then int Γ means the bounded comp<strong>on</strong>ent of C\Γ. If K is a compact subset of<br />

C then C(K) denotes the set of all complex-valued c<strong>on</strong>tinuous functi<strong>on</strong>s <strong>on</strong> K; P (K)<br />

for the uniform closure of all polynomials in C(K); R(K) for the uniform closure of<br />

all rati<strong>on</strong>al functi<strong>on</strong>s with poles off K in C(K); and A(K) for the set of all functi<strong>on</strong>s<br />

<strong>on</strong> K which are analytic <strong>on</strong> int K and c<strong>on</strong>tinuous <strong>on</strong> K. A compact set K is called a<br />

spectral set for an operator T if σ(T ) ⊂ K and ∥f(T )∥ ≤ ∥f∥ K for any f ∈ R(K) and<br />

is called a k-spectral set for an operator T if σ(T ) ⊂ K and there exists a c<strong>on</strong>stant<br />

k > 0 such that<br />

∥f(T )∥ ≤ k∥f∥ K for any f ∈ R(K).<br />

A functi<strong>on</strong> algebra <strong>on</strong> a compact space K is a closed subalgebra A of C(K) that<br />

c<strong>on</strong>tains the c<strong>on</strong>stant functi<strong>on</strong>s and separates the points of K. A functi<strong>on</strong> algebra A<br />

<strong>on</strong> a set K is called a Dirichlet algebra <strong>on</strong> K if Re A ≡ {Re f : f ∈ A} is dense in<br />

C R (K) which is the set of all real-valued c<strong>on</strong>tinuous functi<strong>on</strong>s <strong>on</strong> K.<br />

The following lemma will be used for proving our main theorem.<br />

Lemma 6.4.1. [Ag1, Propositi<strong>on</strong> 1] Let T ∈ B(H). Suppose that K is a spectral<br />

set for T and R(K) is a Dirichlet algebra. If T has no n<strong>on</strong>trivial reducing subspaces<br />

then there exists a norm c<strong>on</strong>tractive algebra homomorphism φ : H ∞ (int K) → B(H)<br />

such that φ(z) = T . Furthermore, φ is c<strong>on</strong>tinuous when domain and range have their<br />

weak ∗ topologies.<br />

We recall [Co] that a Carathéodory domain is an open c<strong>on</strong>nected subset of C whose<br />

boundary coincides with its outer boundary. We can easily show that a Carathéodory<br />

domain G is a comp<strong>on</strong>ent of int ηG and hence is simply c<strong>on</strong>nected. The noti<strong>on</strong> of a<br />

Carathéodory domain was much focused in giving an exact descripti<strong>on</strong> of the functi<strong>on</strong>s<br />

in P 2 (G) ≡ the closure of the polynomials in L 2 (G): for example, P 2 (G) is exactly<br />

204


CHAPTER 6.<br />

A BRIEF SURVEY ON THE INVARIANT SUBSPACE PROBLEM<br />

the Bergman space L 2 a(G) if G is a bounded Carathéodory domain (cf. [Co, Theorem<br />

8.15]). Throughout this secti<strong>on</strong>, a Carathéodory regi<strong>on</strong> means a closed set in C whose<br />

interior is a Carathéodory domain.<br />

We note that the boundary of a bounded Carathéodory domain need not be a<br />

Jordan arc. A simple example is a Cornucopia, which is an open ribb<strong>on</strong> G that winds<br />

about the unit circle so that each point of ∂D bel<strong>on</strong>gs to ∂G. In this case, ∂G is<br />

not a Jordan curve because every point c of ∂D is not an accessible boundary point,<br />

in the sense that it cannot be joined with an arbitrary point of the domain G by a<br />

c<strong>on</strong>tinuous curve that entirely lies in G except for the end point c. Of course, ∂G\∂D<br />

is a Jordan arc. In particular, ∂D is called a prime end of a Cornucopia G (for the<br />

definiti<strong>on</strong> of prime ends, see [Go, p.39]). We note that if φ is a c<strong>on</strong>formal map from<br />

D <strong>on</strong>to G then φ can be extended to a homeomorphism from cl D \ {<strong>on</strong>e point <strong>on</strong> ∂D}<br />

<strong>on</strong>to G ∪ (∂G \ ∂D) (cf. [Go, pp.40-44]).<br />

If f is a c<strong>on</strong>formal mapping of D <strong>on</strong>to the inside of a Jordan curve Γ, then f has<br />

a c<strong>on</strong>tinuous <strong>on</strong>e-to-<strong>on</strong>e extensi<strong>on</strong> up to ∂ D and when thus extended takes ∂ D <strong>on</strong>to<br />

Γ. If Γ has a tangent at a point, we have:<br />

Lemma 6.4.2. (Lindelöf theorem)[Ko, p.40] Let G be a simply c<strong>on</strong>nected domain<br />

bounded by a Jordan curve Γ and 0 ∈ Γ. Suppose that f maps D c<strong>on</strong>formally <strong>on</strong>to G<br />

and f(1) = 0. If Γ has a tangent at 0, then for a c<strong>on</strong>stant c,<br />

arg f(z) − arg (1 − z) → c for |z| < 1, z → 1.<br />

Note that Lemma 6.4.2 says that the c<strong>on</strong>formal images of sectors in D with their<br />

vertices at 1 are asymptotically like sectors in G of the same opening with their<br />

vertices at 0.<br />

We can extend Lemma 6.4.2 slightly.<br />

Lemma 6.4.3. (An extensi<strong>on</strong> of Lindelöf theorem) Let G be a simply c<strong>on</strong>nected<br />

domain and suppose a c<strong>on</strong>formal map φ : D → G can be extended to a homeomorphism<br />

˜φ : cl D \ { z i ∈ ∂D : i ∈ N } → G ∪ { J i : i ∈ N } ,<br />

where the J i are Jordan arcs <strong>on</strong> ∂ G. If 0 ∈ J 1 , ˜φ −1 (0) = 1 /∈ cl {z i : i ∈ N}, and J 1<br />

has a tangent at 0, then for a c<strong>on</strong>stant c,<br />

arg φ(z) − arg (1 − z) → c for |z| < 1, z → 1.<br />

Proof. C<strong>on</strong>sider open disks D i = D i (z i , r i ) (i = 1, 2, · · · ), where r i is chosen so that<br />

1 ∉ cl D i . Let D = D\ ∪ ∞ i=1 cl D i. Then D is simply c<strong>on</strong>nected. So by Riemann’s<br />

mapping theorem there exists a c<strong>on</strong>formal map ψ from D <strong>on</strong>to D such that ψ(0) = 0<br />

and ψ(1) = 1. Then φ ◦ ψ is a c<strong>on</strong>formal map from D <strong>on</strong>to a simply c<strong>on</strong>nected<br />

domain bounded by a Jordan curve. Clearly, the Jordan curve has a tangent at<br />

φ ◦ ψ(1) = φ(1) = 0. Note that 1 − ψ is a c<strong>on</strong>formal map from D <strong>on</strong>to 1 − D. Also<br />

∂(1 − D) is a Jordan curve and ∂(1 − D) has a tangent at 1 − ψ(1) = 0. Now applying<br />

Lemma 6.4.2 with φ ◦ ψ and 1 − ψ gives the result.<br />

205


CHAPTER 6.<br />

A BRIEF SURVEY ON THE INVARIANT SUBSPACE PROBLEM<br />

Applying Lemma 6.4.2, we can show the following geometric property of a bounded<br />

Carathéodory domain whose accessible boundary points lie in rectifiable Jordan arcs<br />

<strong>on</strong> its boundary. The following property was proved for the open unit disk in [Ber].<br />

But our case is little subtle. The following lemma plays a key role in proving our<br />

main theorem.<br />

Lemma 6.4.4. Let G be a bounded Carathéodory domain whose accessible boundary<br />

points lie in rectifiable Jordan arcs <strong>on</strong> its boundary. If a subset Λ ⊂ G is not dominating<br />

for G, i.e., there exists h ∈ H ∞ (G) such that ∥h∥ G > sup |h(λ)|, then we can<br />

λ∈Λ<br />

c<strong>on</strong>struct two rectifiable simple closed curves Γ and Γ ′ satisfying<br />

(i) Γ and Γ ′ are exterior to each other;<br />

(ii) Γ (resp. Γ ′ ) meets a Jordan arc J (resp. J ′ ) at two points, where J ⊂ ∂G (resp.<br />

J ′ ⊂ ∂G);<br />

(iii) Γ and Γ ′ cross Jordan arcs al<strong>on</strong>g line segments which are orthog<strong>on</strong>al to the<br />

tangent lines of the Jordan arcs;<br />

(iv) Γ ∩ Λ = ϕ and Γ ′ ∩ Λ = ϕ.<br />

Proof. Let φ be a c<strong>on</strong>formal map from D <strong>on</strong>to the domain G. Then it is well known<br />

(cf. [Go, pp. 41-42]) that there exists a <strong>on</strong>e-<strong>on</strong>e corresp<strong>on</strong>dence between points <strong>on</strong><br />

∂D and the prime ends of the domain G and that every prime end of G c<strong>on</strong>tains no<br />

more than <strong>on</strong>e accessible boundary point of G. Since G is a simply c<strong>on</strong>nected domain,<br />

the map φ −1 can be extended to a homeomorphism which maps a Jordan arc γ <strong>on</strong><br />

∂G, no interior point of which is a cluster point for ∂G \ γ, <strong>on</strong>to an arc <strong>on</strong> ∂D (cf.<br />

[Go, p.44, Theorem 4’]). But since by our assumpti<strong>on</strong>, every accessible boundary<br />

point of ∂G lies in a Jordan arc of ∂G and the set of all points <strong>on</strong> ∂D corresp<strong>on</strong>ding<br />

to accessible boundary points of ∂G is dense in ∂D (cf. [Go, p.37, Theorem 1]), it<br />

follows that every prime end which c<strong>on</strong>tains no accessible boundary point of ∂G must<br />

be corresp<strong>on</strong>ded to an end point of an arc <strong>on</strong> ∂D corresp<strong>on</strong>ding to a Jordan arc <strong>on</strong><br />

∂G or a limit point of a sequence of disjoint Jordan arcs <strong>on</strong> ∂D. Thus the points <strong>on</strong><br />

∂D corresp<strong>on</strong>ding to the prime ends which c<strong>on</strong>tain no accessible boundary points of<br />

∂G form a countable set. Now let V be the set of ‘singular’ points, that is, points <strong>on</strong><br />

∂D corresp<strong>on</strong>ding to the prime ends which c<strong>on</strong>tain no accessible boundary points of<br />

∂G. Then V is countable and the map φ can be extended to a homeomorphism from<br />

cl D \ V <strong>on</strong>to G ∪ { J i : i = 1, 2, · · · },<br />

where the J i are rectifiable Jordan arcs <strong>on</strong> ∂G.<br />

We denote this homeomorphism by still φ. Then we claim that<br />

Λ ′ = φ −1 (Λ) is not dominating for D. (6.1)<br />

Indeed, by our assumpti<strong>on</strong>, ∥h∥ G > sup λ∈Λ |h(λ)| for some h ∈ H ∞ (G). Since ∥h∥ G =<br />

∥h ◦ φ∥ D and φ is c<strong>on</strong>formal <strong>on</strong> D, we have that h ◦ φ ∈ H ∞ (D). Also, since<br />

sup |h(λ)| = sup |h(φ(λ))| = sup |(h ◦ φ)(λ)|,<br />

λ∈Λ<br />

λ∈Λ ′ λ∈Λ ′<br />

206


CHAPTER 6.<br />

A BRIEF SURVEY ON THE INVARIANT SUBSPACE PROBLEM<br />

it follows that<br />

giving ([]). Write<br />

ω :=<br />

∥h ◦ φ∥ D > sup<br />

λ∈Λ ′ |(h ◦ φ)(λ)|,<br />

{λ ∈ ∂D : λ is not approached n<strong>on</strong>tangentially by points in Λ ′ }<br />

.<br />

Remember that S ≡ {α n } ⊂ D is dominating for D if and <strong>on</strong>ly if almost every point<br />

<strong>on</strong> ∂D is approached n<strong>on</strong>tangentially by points of S (cf. [BSZ, Theorem 3]). It thus<br />

follows that ω has a positive measure. We put<br />

W := { x ∈ J i : J i does not have a tangent at x for i = 1, 2, · · · }.<br />

Then W has measure zero since the J i are rectifiable and every rectifiable Jordan arc<br />

has a tangent almost everywhere. Now let W ′ = φ −1 (W ). Also W ′ has measure zero.<br />

Let θ be a fixed angle with 3 4 π < θ < π and let A λ be the sector whose vertex is λ<br />

and whose radius is r λ , of opening θ. Then for each λ ∈ ω we can find a rati<strong>on</strong>al<br />

number r λ ∈ (0, 1) such that the sector A λ c<strong>on</strong>tains no point in Λ ′ . Write<br />

˜ω ≡ ω \ (V ∪ W ′ ).<br />

Since ˜ω has a positive measure and hence it is uncountable, there exist a rati<strong>on</strong>al<br />

number r ∈ (0, 1) and an uncountable set ω ′ ⊂ ˜ω such that r = r λ for all λ ∈ ω ′ .<br />

Clearly, we can find distinct points λ 1 , λ 2 , λ 3 , λ 4 in ω ′ such that<br />

A λ1 ∩ A λ2 ≠ ϕ and A λ3 ∩ A λ4 ≠ ϕ.<br />

We can thus c<strong>on</strong>struct two rectifiable arcs Γ ◦ 1 and Γ ◦ 2 in cl D such that<br />

and<br />

Γ ◦ 1 ∩ D ⊂ A λ1 ∪ A λ2 , Γ ◦ 1 ∩ T = {λ 1 , λ 2 }<br />

Γ ◦ 2 ∩ D ⊂ A λ3 ∪ A λ4 , Γ ◦ 2 ∩ T = {λ 3 , λ 4 }.<br />

Let η i := φ(λ i ) for i = 1, . . . , 4. Then, since φ is a homeomorphism, η i ’s are distinct.<br />

Also, each η i is c<strong>on</strong>tained in a Jordan arc of ∂G. Let B i := φ(A λi ). Then, since<br />

3<br />

4 π < θ < π, we can, by Lemma 6.4.3, find a line segment l i ⊂ B i which is orthog<strong>on</strong>al<br />

to the tangent line at η i . Let L i := φ −1 (l i ). Then, by cutting off the end parts of<br />

Γ ◦ 1 and Γ ◦ 2 and joining L i ’s, we can c<strong>on</strong>struct two new rectifiable arcs ˜Γ ◦ 1 and ˜Γ ◦ 2. Let<br />

˜Γ := φ(˜Γ ◦ 1 ) and ˜Γ ′ := φ(˜Γ ◦ 2 ). Since G is a Carathéodory domain and the end parts<br />

of ˜Γ and ˜Γ ′ are line segments, by extending straightly the end parts of ˜Γ and ˜Γ ′ in<br />

the unbounded comp<strong>on</strong>ent of C\cl G, we can c<strong>on</strong>struct two Jordan curves ̂Γ and ̂Γ ′<br />

whose end parts cross the boundary of G through line segments. Therefore, by joining<br />

end points of ̂Γ (resp., the end points of ̂Γ ′ ) by a rectifiable arc in the unbounded<br />

comp<strong>on</strong>ent, we can find a simple closed rectifiable curve Γ (resp., Γ ′ ) satisfying the<br />

given c<strong>on</strong>diti<strong>on</strong>s.<br />

207


CHAPTER 6.<br />

A BRIEF SURVEY ON THE INVARIANT SUBSPACE PROBLEM<br />

We are ready for proving:<br />

Theorem 6.4.5. Let T ∈ B(H) be such that ∥p(T )∥ ≤ ∥p∥ σ(T ) for every polynomial<br />

p. If ησ(T ) is a Carathéodory regi<strong>on</strong> whose accessible boundary points lie in rectifiable<br />

Jordan arcs <strong>on</strong> its boundary, then T has a n<strong>on</strong>trivial invariant subspace.<br />

Proof. To investigate the invariant subspaces, we may assume that T has no n<strong>on</strong>trivial<br />

reducing subspace and σ(T ) = σ ap (T ), where σ ap (T ) denotes the approximate<br />

point spectrum of T . Since the complement of ησ(T ) is c<strong>on</strong>nected, we have that by<br />

Mergelyan’s theorem, R(ησ(T )) = P (ησ(T )). We thus have<br />

∥f(T )∥ ≤ ∥f∥ ησ(T ) for any f ∈ R(ησ(T )),<br />

which says that ησ(T ) is a spectral set for T . On the other hand, we note that<br />

R(ησ(T )) (= P (ησ(T ))) is a Dirichlet algebra. Thus if σ(T ) ⊄ cl (int ησ(T )), then<br />

it follows from a theorem of J. Stampfli [St, Propositi<strong>on</strong> 1] that T has a n<strong>on</strong>trivial<br />

invariant subspace. So we may, without loss of generality, assume that σ(T ) ⊂<br />

cl (int ησ(T )). In this case we have that ησ(T ) = cl (int ησ(T )). Hence int ησ(T ) is a<br />

Carathéodory domain.<br />

Now since ησ(T ) is a spectral set, R(ησ(T )) is a Dirichlet algebra, and T has<br />

no n<strong>on</strong>trivial reducing subspaces, it follows from Lemma 6.4.1 that there exists an<br />

extensi<strong>on</strong> of the functi<strong>on</strong>al calculus of T to a norm c<strong>on</strong>tractive algebra homomorphism<br />

ϕ : H ∞ (int ησ(T )) → B(H). (6.2)<br />

Moreover, ϕ is weak ∗ -weak ∗ c<strong>on</strong>tinuous. Let 0 < ε < 1 2<br />

. C<strong>on</strong>sider the following set:<br />

{<br />

Λ(ε) = λ ∈ int ησ(T ) : ∃ a unit vector x such that ∥(T − λ)x∥ < ε dist ( λ, ∂(ησ(T )) )} .<br />

There are two cases to c<strong>on</strong>sider.<br />

Case 1: Λ(ε) is not dominating for int ησ(T ). Since int ησ(T ) is a Carathéodory<br />

domain, we can find two rectifiable simple closed curves Γ and Γ ′ satisfying the c<strong>on</strong>diti<strong>on</strong>s<br />

given in Lemma 6.4.4; in particular, Γ ∩ Λ(ϵ) = ∅ and Γ ′ ∩ Λ(ϵ) = ∅. Let<br />

Γ ∩ ∂(ησ(T )) = {λ 1 , λ 2 } and Γ ′ ∩ ∂(ησ(T )) = {λ 3 , λ 4 }.<br />

Since σ(T ) = σ ap (T ), it is clear that Λ(ε) ⊃ int ησ(T ) ∩ σ(T ). So T − λ is invertible<br />

for any λ in Γ\{λ 1 , λ 2 } and Γ ′ \{λ 3 , λ 4 }. If λ ∈ Γ\ησ(T ), then since the functi<strong>on</strong>al<br />

calculus in (6.2) is c<strong>on</strong>tractive, we have<br />

{ }<br />

1<br />

∥(λ − T ) −1 ∥ ≤ sup<br />

|λ − µ| : µ ∈ int ησ(T ) 1<br />

=<br />

dist(λ, ∂ (ησ(T )) .<br />

Let λ ∈ Γ ∩ int ησ(T ). Since Γ ∩ Λ(ε) = ∅, we have that for any unit vector x,<br />

∥(T − λ)x∥ ≥ ε dist (λ, ∂(ησ(T ))),<br />

208


CHAPTER 6.<br />

A BRIEF SURVEY ON THE INVARIANT SUBSPACE PROBLEM<br />

which implies that<br />

∥(λ − T ) −1 ∥ ≤<br />

1<br />

ε dist (λ, ∂(ησ(T ))) .<br />

On the other hand, Since ∂(ησ(T )) has a tangent at λ i , it follows that in a sufficiently<br />

small neighborhood N i of λ i , ∂(ησ(T )) lies in a double-sector A i of opening 2θ i<br />

(0 < θ i < π 2<br />

) for each i = 1, 2. But since Γ is a line segment in a sufficiently small<br />

neighborhood of each λ i (i = 1, 2), it follows that if λ ∈ N i ∩ Γ, then<br />

We thus have<br />

|λ − λ i |<br />

dist (λ, ∂(ησ(T ))) ≤ |λ − λ i|<br />

dist (λ, A i ) = 1 =: c.<br />

cos θ i<br />

∥(λ − λ 1 )(λ − λ 2 )(λ − T ) −1 ∥ ≤ c ε |λ − λ 2| ≤ M <strong>on</strong> N 1 ∩ Γ,<br />

which says that S λ ≡ (λ − λ 1 )(λ − λ 2 )(λ − T ) −1 is bounded <strong>on</strong> N 1 ∩ Γ. Also S λ has<br />

at most two disc<strong>on</strong>tinuities <strong>on</strong> Γ. So the following operator A is well-defined ([Ap1]):<br />

A := 1 ∫<br />

(λ − λ 1 )(λ − λ 2 )(λ − T ) −1 dλ.<br />

2πi<br />

Γ<br />

Now, using the argument of [Ber, Lemma 3.1]), we can c<strong>on</strong>clude that ker(A) is a<br />

n<strong>on</strong>trivial invariant subspace for T .<br />

Case 2: Λ(ε) is dominating for int ησ(T ).<br />

isometric, i.e.,<br />

In this case, we can show that ϕ is<br />

∥h(T )∥ = ∥h∥ int ησ(T )<br />

for all h ∈ H ∞ (int ησ(T )),<br />

by using the same argument as the well-known method due to Apostol (cf. [Ap1]),<br />

in which it was shown that the Sz.-Nagy-Foias calculus is isometric. Now c<strong>on</strong>sider a<br />

c<strong>on</strong>formal map φ : D → int ησ(T ) and then define the functi<strong>on</strong> ψ by<br />

ψ = φ −1 : int ησ(T ) → D.<br />

Then ψ ∈ H ∞ (int ησ(T )). Define A := ψ(T ). Then A is an absolutely c<strong>on</strong>tinuous<br />

c<strong>on</strong>tracti<strong>on</strong> with norm 1. Thus we can easily show that<br />

∥h(A)∥ = ∥h∥ D<br />

for any h ∈ H ∞ (D).<br />

Thus if λ 0 ∈ T, then<br />

lim ||(A −<br />

λ→λ 0 ,|λ|>1 λ)−1 || = lim ||(z −<br />

λ→λ 0 ,|λ|>1 λ)−1 || D = ∞,<br />

which implies that A − λ 0 is not invertible, so that we get T ⊂ σ(A). Since every<br />

c<strong>on</strong>tracti<strong>on</strong> whose spectrum c<strong>on</strong>tains the unit circle has a n<strong>on</strong>trivial invariant subspace<br />

([BCP2]), A has a n<strong>on</strong>trivial invariant subspace. On the other hand, since T ∈<br />

weak ∗ -cl {p(A) : p is a polynomial}, we can c<strong>on</strong>clude that T has a n<strong>on</strong>trivial invariant<br />

subspace.<br />

209


CHAPTER 6.<br />

A BRIEF SURVEY ON THE INVARIANT SUBSPACE PROBLEM<br />

A simple example for the set satisfying ||p(T )|| ≤ ||p|| σ(T ) for every polynomial p<br />

is the set of ‘polynomially normaloid’ operators, in the sense that p(T ) is normaloid<br />

(i.e., norm equals spectral radius) for every polynomial p. Indeed if p(T ) is normaloid<br />

then ||p(T )|| = sup λ∈σ(p(T )) |λ| = ||p|| σ(T ) by the spectral mapping theorem.<br />

Remark. We were unable to decide whether in Theorem 6.4.5, the c<strong>on</strong>diti<strong>on</strong> “||p(T )|| ≤<br />

||p|| σ(T ) ” can be relaxed to the c<strong>on</strong>diti<strong>on</strong> “||p(T )|| ≤ k ||p|| σ(T ) for some k > 0”. However<br />

we can prove that if T ∈ B(H) is such that ||p(T )|| ≤ k ||p|| σ(T ) for every<br />

polynomial p and some k > 0 and if the outer boundary of σ(T ) is a Jordan curve<br />

then T has a n<strong>on</strong>trivial invariant subspace. This is a corollary of the theorem of C.<br />

Ambrozie and V. Müller [AM, Theorem A]. The proof goes as follows. Since ∂ (ησ(T ))<br />

is a Jordan curve, then by Carathéodory’s theorem <strong>on</strong> extensi<strong>on</strong>s of the c<strong>on</strong>formal<br />

representati<strong>on</strong>s, a c<strong>on</strong>formal map φ : int ησ(T ) → D can be extended to a homeomorphism<br />

ψ : ησ(T ) → cl D. Since C \ ησ(T ) is c<strong>on</strong>nected, we can find polynomials p n<br />

such that p n → ψ uniformly <strong>on</strong> ησ(T ). Since the spectrum functi<strong>on</strong> σ : B(H) → C is<br />

upper semi-c<strong>on</strong>tinuous, it follows that<br />

ψ(∂(ησ(T ))) ⊂ ψ(σ(T )) = lim sup p n (σ(T )) = lim sup σ(p n (T )) ⊂ σ(ψ(T )).<br />

But since ψ is a homeomorphism we have that ∂D ⊂ σ(ψ(T )). By our assumpti<strong>on</strong> we<br />

can also see that<br />

||(p ◦ ψ)(T )|| ≤ k ||p ◦ ψ|| int ησ(T )<br />

= k ||p|| D for every polynomail p,<br />

which says that ψ(T ) is a polynomially bounded operator. Therefore by the theorem<br />

of C. Ambrozie and V. Müller[AM], ψ(T ) has a n<strong>on</strong>trivial invariant subspace. Hence<br />

we can c<strong>on</strong>clude that T has a n<strong>on</strong>trivial invariant subspace.<br />

We c<strong>on</strong>clude with a result <strong>on</strong> the invariant subspaces for hyp<strong>on</strong>ormal operators<br />

(this applies, in particular, to the case when ησ(T ) is the closure of a Cornucopia).<br />

Corollary 6.4.6. Let T ∈ B(H) be a hyp<strong>on</strong>ormal operator. If the outer boundary<br />

of σ(T ) has at most finitely many prime ends corresp<strong>on</strong>ding to singular points <strong>on</strong><br />

∂D and has a tangent at almost every point <strong>on</strong> each Jordan arc, with respect to a<br />

c<strong>on</strong>formal map from D <strong>on</strong>to int ησ(T ), then T has a n<strong>on</strong>trivial invariant subspace.<br />

Proof. Suppose that<br />

∥h∥ int ησ(T )<br />

= sup { |h(λ)| : λ ∈ σ(T ) ∩ int ησ(T ) }<br />

for all h ∈ H ∞ (int ησ(T )). Then σ(T ) ∩ int ησ(T ) is dominating for int ησ(T ). Thus<br />

by the well-known theorem due to S. Brown [Br2, Theorem 2], T has a n<strong>on</strong>trivial<br />

invariant subspace. Suppose instead that<br />

∥h∥ int ησ(T )<br />

> sup { |h(λ)| : λ ∈ σ(T ) ∩ int ησ(T ) }<br />

for some h ∈ H ∞ (int ησ(T )). By an analysis of the proof of Lemma 6.4.4, we can<br />

c<strong>on</strong>struct two rectifiable curves Γ and Γ ′ satisfying the c<strong>on</strong>diti<strong>on</strong>s (i) - (iv). Let<br />

210


CHAPTER 6.<br />

A BRIEF SURVEY ON THE INVARIANT SUBSPACE PROBLEM<br />

Γ∩∂ησ(T ) = {λ 1 , λ 2 } and Γ ′ ∩∂ησ(T ) = {λ 3 , λ 4 }. Since T is a hyp<strong>on</strong>ormal operator,<br />

we have<br />

∥(λ − T ) −1 ∥ =<br />

1<br />

dist(λ, σ(T ))<br />

<strong>on</strong> λ ∈ ( Γ\{λ 1 , λ 2 } ) ∪ ( Γ ′ \{λ 3 , λ 4 } ) .<br />

Now the same argument as in Case 1 of the proof of Theorem 6.4.5 shows that T has<br />

a n<strong>on</strong>trivial invariant subspace.<br />

211


CHAPTER 6.<br />

A BRIEF SURVEY ON THE INVARIANT SUBSPACE PROBLEM<br />

212


Bibliography<br />

[Ab]<br />

M. B. Abrahamse, Subnormal Toeplitz operators and functi<strong>on</strong>s of bounded<br />

type, Duke Math. J. 43(1976), 597–604.<br />

[Ag1] J. Agler, An invariant subspace problem, J. Funct. Anal. 38(1980), 315–323.<br />

[Ag2] J. Agler, Hyperc<strong>on</strong>tracti<strong>on</strong>s and subnormality, J. <strong>Operator</strong> <strong>Theory</strong>,<br />

13(1985), 203–217.<br />

[Ai]<br />

[ACG]<br />

P. Aiena, Fredholm and local spectral theory, with applicati<strong>on</strong>s to multipliers,<br />

Kluwer Academic Publishers, L<strong>on</strong>d<strong>on</strong>, 2004<br />

P. Aiena, M. L.Colasante and M. G<strong>on</strong>zález, <strong>Operator</strong>s which have a closed<br />

quasinilpotent part, Proc. Amer. Math. Soc. 130(2002), 2701–2710.<br />

[Ale] A. Aleman, Subnormal operators with compact selfcommutator,<br />

Manuscripta Math., 91(1996), 353–367.<br />

[Al]<br />

[AM]<br />

[AW]<br />

[AIW]<br />

[An]<br />

[Ap1]<br />

A. Aluthge, On p-hyp<strong>on</strong>ormal operators for 0 < p < 1, Integral Equati<strong>on</strong>s<br />

<strong>Operator</strong> <strong>Theory</strong>, 13(1990), 307–315<br />

C. Ambrozie and V. Müller, Invariant subspaces for polynomially bounded<br />

opoerators, J. Funct. Anal. 213(2004), 321–345.<br />

A. Aluthge and D. Wang, w-hyp<strong>on</strong>ormal operators, Integral Equati<strong>on</strong>s <strong>Operator</strong><br />

<strong>Theory</strong> 36(2000), 1–10<br />

I. Amemiya, T. Ito, and T.K. W<strong>on</strong>g, On quasinormal Toeplitz operators,<br />

Proc. Amer. Math. Soc. 50(1975), 254–258.<br />

T. Ando, <strong>Operator</strong>s with a norm c<strong>on</strong>diti<strong>on</strong>, Acta Sci. Math. (Szeged)<br />

33(1972), 169–178.<br />

C. Apostol, Ultraweakly closed operator algebras, J. <strong>Operator</strong> <strong>Theory</strong><br />

2(1979), 49–61.<br />

[Ap2] C. Apostol, The reduced minimum modulus, Michigan Math. J. 32(1985),<br />

279–294<br />

[AFV]<br />

C. Apostol, C. Foias and D. Voiculescu, Some results <strong>on</strong> n<strong>on</strong>-quasitriangular<br />

operators, IV Rev. Roum. Math. Pures Appl. 18(1973), 487–514<br />

213


BIBLIOGRAPHY<br />

[AT]<br />

S.C. Arora and J.K. Thukral, On a class of operators, Glasnik Math.<br />

21(1986), 381–386<br />

[Ar] W. Arves<strong>on</strong>, A short course <strong>on</strong> spectral theory, Springer, New York, 2002.<br />

[Ath]<br />

[Be1]<br />

[Be2]<br />

[Be3]<br />

[Ber]<br />

A. Athavale, On joint hyp<strong>on</strong>ormality of operators, Proc. Amer. Math. Soc.<br />

103(1988), 417–423.<br />

S.K. Berberian, An extensi<strong>on</strong> of Weyl’s theorem to a class of not necessarily<br />

normal operators, Michigan Math. Jour. 16 (1969), 273-279.<br />

S.K. Berberian, The Weyl spectrum of an operator, Indiana Univ. Math.<br />

Jour. 20 (1970), 529–544<br />

S.K. Berberian, <str<strong>on</strong>g>Lecture</str<strong>on</strong>g>s in Functi<strong>on</strong>al Analysis and <strong>Operator</strong> <strong>Theory</strong>,<br />

Springer, New York, 1974.<br />

J.D. Berg, An extensi<strong>on</strong> of the Weyl-v<strong>on</strong> Neumann theorem, Trans. Amer.<br />

Math. Soc. 160 (1971), 365–371.<br />

[Bo] R.H. Bouldin, The essential minimum modulus, Indiana Univ. Math. J.<br />

30(1981), 513–517<br />

[BGS]<br />

A. Bottcher, S. Grudsky and I. Spitkovsky, The spectrum is disc<strong>on</strong>tinuous<br />

<strong>on</strong> the manifold of Toeplitz operators, Arch. Math. (Basel) 75(2000), 46–52<br />

[Bra] J. Bram, Subnormal operators, Duke Math. J. 22(1955), 75–94.<br />

[Br]<br />

[BD]<br />

A. Brown, Unitary equivalence of binormal operators, Amer. J. Math.<br />

76(1954), 413–434<br />

A. Brown and R.G. Douglas, Partially isometric Toeplitz operators, Proc.<br />

Amer. Math. Soc. 16(1965), 681–682.<br />

[BH] A. Brown and P.R. Halmos, Algebraic properties of Toeplitz operators, J.<br />

Regine. Angew. Math. 213(1963/1964), 89–102<br />

[BDF]<br />

[Bro]<br />

L. Brown, R. Douglas, and P. Fillmore, Unitary equivalence modulo the compact<br />

operators and extensi<strong>on</strong>s of C ∗ -algebras, Proc. C<strong>on</strong>f. Op. Th., <str<strong>on</strong>g>Lecture</str<strong>on</strong>g><br />

<str<strong>on</strong>g>Notes</str<strong>on</strong>g> in Mathematics, vol. 346, Springer (1973), 58–128.<br />

S. Brown, Hyp<strong>on</strong>ormal operators with thick spectra have invariant subspaces,<br />

Ann. of Math. 125(1987), 93–103.<br />

[BDW] J. Bu<strong>on</strong>i, A. Dash and B. Wadhwa, Joint Browder spectrum, Pacific J.<br />

Math. 94 (1981), 259–263<br />

[CaG]<br />

S.L. Campbell and R. Gellar, Linear operators for which T ∗ T and T + T ∗<br />

commute. II, Trans. Amer. Math. Soc., 236(1977), 305–319.<br />

[Ch1] M. Cho, On the joint Weyl spectrum II, Acta Sci. Math. (Szeged) 53(1989),<br />

381–384<br />

214


BIBLIOGRAPHY<br />

[Ch2] M. Cho, On the joint Weyl spectrum III, Acta Sci. Math. (Szeged) 54(1990),<br />

365–368<br />

[Ch3] M. Cho, Spectral properties of p-hyp<strong>on</strong>ormal operators, Glasgow Math. J.<br />

36(1994), 117–122<br />

[ChH]<br />

[CIO]<br />

[ChT]<br />

[ChR]<br />

M. Cho and T. Huruya, p-hyp<strong>on</strong>ormal operators (0 < p < 1 2<br />

), Commentati<strong>on</strong>es<br />

Math. 33(1993), 23–29<br />

M. Cho, M. Itoh and S. Oshiro, Weyl’s theorem holds for p-hyp<strong>on</strong>ormal<br />

operators, Glasgow Math. J. 39(1997), 217–220<br />

M. Cho and M. Takaguchi, On the joint Weyl spectrum, Sci. Rep. Hirosaki<br />

Univ. 27(1980), 47–49<br />

N.N. Chourasia and P.B. Ramanujan, Paranormal operators <strong>on</strong> Banach<br />

spaces, Bull. Austral. Math. Soc. 21(1980), 161–168<br />

[Co] L.A. Coburn, Weyl’s theorem for n<strong>on</strong>normal operators, Michigan Math. J.<br />

13(1966), 285–288<br />

[C<strong>on</strong>1] J.B. C<strong>on</strong>way, A course in functi<strong>on</strong>al analysis, Springer, New York, 1990.<br />

[C<strong>on</strong>2]<br />

[C<strong>on</strong>3]<br />

[CoM]<br />

[CoS]<br />

[Cow1]<br />

[Cow2]<br />

[Cow3]<br />

[CoL]<br />

[Cu1]<br />

J.B. C<strong>on</strong>way, The <strong>Theory</strong> of Subnormal <strong>Operator</strong>s, Math. Surveys and<br />

M<strong>on</strong>ographs, vol. 36, Amer. Math. Soc., Providence, 1991.<br />

J.B. C<strong>on</strong>way, A course in operator theory, Amer. Math. Soc., Providence,<br />

2000.<br />

J.B. C<strong>on</strong>way and B.B. Morrel, <strong>Operator</strong>s that are points of spectral c<strong>on</strong>tinuity,<br />

Integral Equati<strong>on</strong>s <strong>Operator</strong> <strong>Theory</strong> 2(1979), 174–198<br />

J.B. C<strong>on</strong>way and W. Szymanski, Linear combinati<strong>on</strong> of hyp<strong>on</strong>ormal operators,<br />

Rocky Mountain J. Math. 18(1988), 695–705.<br />

C. Cowen, More subnormal Toeplitz operators, J. Reine Angew. Math.<br />

367(1986), 215–219.<br />

C. Cowen, Hyp<strong>on</strong>ormal and subnormal Toeplitz operators Surveys of Some<br />

Recent Results in <strong>Operator</strong> <strong>Theory</strong>, I J.B. C<strong>on</strong>way and B.B Morrel Eds.,<br />

Pitman Research <str<strong>on</strong>g>Notes</str<strong>on</strong>g> in Mathematics, Vol. 171, L<strong>on</strong>gman, 1988, 155–167<br />

C. Cowen, Hyp<strong>on</strong>ormality of Toeplitz operators, Proc. Amer. Math. Soc.<br />

103(1988), 809–812<br />

C.C. Cowen and J.J. L<strong>on</strong>g, Some subnormal Toeplitz operators, J. Reine<br />

Angew. Math. 351(1984), 216–220.<br />

R.E. Curto, Fredholm and invertible n-tuples of operators. The deformati<strong>on</strong><br />

problem, Trans. Amer. Math. Soc. 266(1981), 129–159<br />

215


BIBLIOGRAPHY<br />

[Cu2]<br />

[Cu3]<br />

[CuD]<br />

[CuF1]<br />

[CuF2]<br />

[CuF3]<br />

[CHL]<br />

[CuJ]<br />

[CJL]<br />

[CJP]<br />

[CLL]<br />

[CuL1]<br />

[CuL2]<br />

[CuL3]<br />

[CuL4]<br />

R.E. Curto, Quadratically hyp<strong>on</strong>ormal weighted shifts, Integral Equati<strong>on</strong>s<br />

<strong>Operator</strong> <strong>Theory</strong>, 13(1990), 49–66.<br />

R.E. Curto, Joint hyp<strong>on</strong>ormality:A bridge between hyp<strong>on</strong>ormality and subnormality,<br />

<strong>Operator</strong> <strong>Theory</strong>: <strong>Operator</strong> Algebras and Applicati<strong>on</strong>s(Durham,<br />

NH, 1988)(W.B. Arves<strong>on</strong> and R.G. Douglas, eds.), Proc. Sympos. Pure<br />

Math., American Mathematical Society, Providence, Vol.51, part II, 1990,<br />

69–91.<br />

R.E. Curto and A.T. Dash, Browder spectral systems, Proc. Amer. Math.<br />

Soc. 103(1988), 407–412<br />

R.E. Curto and L.A. Fialkow, Recursiveness, positivity, and truncated moment<br />

problems, Houst<strong>on</strong> J. Math. 17(1991), 603–635.<br />

R.E. Curto and L.A. Fialkow, Recursively generated weighted shifts and<br />

the subnormal completi<strong>on</strong> problem, Integral Equati<strong>on</strong>s <strong>Operator</strong> <strong>Theory</strong>,<br />

17(1993), 202–246.<br />

R.E. Curto and L.A. Fialkow, Recursively generated weighted shifts and<br />

the subnormal completi<strong>on</strong> problem II, Integral Equati<strong>on</strong>s <strong>Operator</strong> <strong>Theory</strong>,<br />

18(1994), 369–426.<br />

R.E. Curto, I. S. Hwang and W. Y. <str<strong>on</strong>g>Lee</str<strong>on</strong>g>, Weak subnormality of operators,<br />

Arch. Math. (Basel), 79(2002), 360–371.<br />

R.E. Curto and I.B. Jung, Quadratically hyp<strong>on</strong>ormal weighted shifts with<br />

two equal weights,Integral Equati<strong>on</strong>s <strong>Operator</strong> <strong>Theory</strong>, 37(2000), 208–231.<br />

R.E. Curto, I.B. Jung and W.Y. <str<strong>on</strong>g>Lee</str<strong>on</strong>g>, Extensi<strong>on</strong>s and extremality of recursively<br />

generated weighted shifts, Proc. Amer. Math. Soc. 130(2002), 565–<br />

576.<br />

R.E. Curto, I.B. Jung and S.S. Park, A characterizati<strong>on</strong> of k-hyp<strong>on</strong>ormality<br />

via weak subnormality, J. Math. Anal. Appl. 279(2003), 556–568.<br />

R.E. Curto, S.H. <str<strong>on</strong>g>Lee</str<strong>on</strong>g> and W.Y. <str<strong>on</strong>g>Lee</str<strong>on</strong>g>, Subnormality and 2-hyp<strong>on</strong>ormality for<br />

Toeplitz operators, Integral Equati<strong>on</strong>s <strong>Operator</strong> <strong>Theory</strong>, 44(2002), 138–148.<br />

R.E. Curto and W.Y. <str<strong>on</strong>g>Lee</str<strong>on</strong>g>, Joint hyp<strong>on</strong>ormality of Toeplitz pairs, Memoirs<br />

Amer. Math. Soc. no. 712, Amer. Math. Soc., Providence, 2001.<br />

R.E. Curto and W.Y. <str<strong>on</strong>g>Lee</str<strong>on</strong>g>, Towards a model theory for 2–hyp<strong>on</strong>ormal operators,<br />

Integral Equati<strong>on</strong>s <strong>Operator</strong> <strong>Theory</strong>, 44(2002), 290–315.<br />

R.E. Curto and W.Y. <str<strong>on</strong>g>Lee</str<strong>on</strong>g>, Subnormality and k–hyp<strong>on</strong>ormality of Toeplitz<br />

operators: A brief survey and open questi<strong>on</strong>s, <strong>Operator</strong> theory and Banach<br />

algebras (Rabat, 1999), 73–81, Theta, Bucharest, 2003.<br />

R.E. Curto and W.Y. <str<strong>on</strong>g>Lee</str<strong>on</strong>g>, Soluti<strong>on</strong> of the quadratically hyp<strong>on</strong>ormal completi<strong>on</strong><br />

problem, Proc. Amer. Math. Soc. 131(2003), 2479–2489.<br />

216


BIBLIOGRAPHY<br />

[CuL5]<br />

[CMX]<br />

[CP1]<br />

[CP2]<br />

R.E. Curto and W.Y. <str<strong>on</strong>g>Lee</str<strong>on</strong>g>, k-hyp<strong>on</strong>ormality of finite rank perturbati<strong>on</strong>s of<br />

unilateral weighted shifts, Trans. Amer. Math. Soc. ( ), .<br />

R.E. Curto, P.S. Muhly and J. Xia, Hyp<strong>on</strong>ormal pairs of commuting operators,<br />

C<strong>on</strong>tributi<strong>on</strong>s to <strong>Operator</strong> <strong>Theory</strong> and Its Applicati<strong>on</strong>s(Mesa, AZ,<br />

1987) (I. Gohberg, J.W. Helt<strong>on</strong> and L. Rodman, eds.), <strong>Operator</strong> <strong>Theory</strong>:<br />

Advances and Applicati<strong>on</strong>s, vol. 35, Birkhäuser, Basel–Bost<strong>on</strong>, 1988, 1–22.<br />

R.E. Curto and M. Putinar, Existence of n<strong>on</strong>-subnormal polynomially hyp<strong>on</strong>ormal<br />

operators, Bull. Amer. Math. Soc. (N.S.), 25(1991), 373–378.<br />

R.E. Curto and M. Putinar, Nearly subnormal operators and moment problems,<br />

J. Funct. Anal. 115(1993), 480–497.<br />

[Da1] A.T. Dash, Joint essential spectra, Pacific J. Math. 64(1976), 119–128<br />

[Da2]<br />

[Do1]<br />

[Do2]<br />

[DPY]<br />

[DP]<br />

[Emb]<br />

[Fa1]<br />

[Fa2]<br />

[FL1]<br />

[FL2]<br />

[FM]<br />

[Fia]<br />

A.T. Dash, Joint Browder spectra and tensor products, Bull. Austral. Math.<br />

Soc. 32(1985), 119–128<br />

R.G. Douglas, Banach algebra techniques in operator theory, Springer, New<br />

York, 2003<br />

R.G. Douglas, Banach Algebra Techniques in the <strong>Theory</strong> of Toeplitz <strong>Operator</strong>s<br />

CBMS 15, Providence, Amer. Math. Soc. 1973<br />

R.G. Douglas, V.I. Paulsen, and K. Yan, <strong>Operator</strong> theory and algebraic<br />

geometry Bull. Amer. Math. Soc. (N.S.) 20(1989), 67–71.<br />

H.K. Du and J. Pan, Perturbati<strong>on</strong> of spectrums of 2 × 2 operator matrices,<br />

Proc. Amer. Math. Soc. 121 (1994), 761–776.<br />

M. Embry, Generalizati<strong>on</strong> of the Halmos-Bram criteri<strong>on</strong> for subnormality,<br />

Acta. Sci. Math. (Szeged), 35(1973), 61–64.<br />

P. Fan, Remarks <strong>on</strong> hyp<strong>on</strong>ormal trig<strong>on</strong>ometric Toeplitz operators, Rocky<br />

Mountain J. Math. 13(1983), 489–493.<br />

P. Fan, Note <strong>on</strong> subnormal weighted shifts, Proc. Amer. Math. Soc.,<br />

103(1988), 801–802.<br />

D.R. Farenick and W.Y. <str<strong>on</strong>g>Lee</str<strong>on</strong>g>, Hyp<strong>on</strong>ormality and spectra of Toeplitz operators,<br />

Trans. Amer. Math. Soc., 348(1996), 4153–4174.<br />

D.R. Farenick and W.Y. <str<strong>on</strong>g>Lee</str<strong>on</strong>g>, On hyp<strong>on</strong>ormal Toeplitz operators with polynomial<br />

and circulant-type symbols, Integral Equati<strong>on</strong>s <strong>Operator</strong> <strong>Theory</strong>,<br />

29(1997), 202–210.<br />

D.R. Farenick and R. McEachin, Toeplitz operators hyp<strong>on</strong>ormal with the<br />

unilateral shift, Integral Equati<strong>on</strong>s <strong>Operator</strong> <strong>Theory</strong> 22(1995), 273–280<br />

L.A. Fialkow, The index of an elementary operator, Indiana Univ. Math.<br />

J., 35(1986), 73–102<br />

217


BIBLIOGRAPHY<br />

[Fin] J.K. Finch, The single valued extensi<strong>on</strong> property, Pacific J. Math. 58(1975),<br />

61–69<br />

[Fo]<br />

[FF]<br />

[FIY]<br />

S.R. Foguel, Normal operators of finite multiplicity, Comm. Pure Appl.<br />

Math. 11(1958), 297–313<br />

C. Foiaş and A. Frazho, The Commutant Lifting Approach to Interpolati<strong>on</strong><br />

Problems, <strong>Operator</strong> <strong>Theory</strong>: Adv. Appl., Vol 44, Birkhäuser-Verlag, Bost<strong>on</strong>,<br />

1990.<br />

T. Furuta, M. Ito, T. Yamazaki, A subclass of paranormal operators including<br />

class of log hyp<strong>on</strong>ormal and several related classes, Sci. Math. 1(1998),<br />

389–403<br />

[Ga] J. Garnett, Bounded Analytic Functi<strong>on</strong>s, Academic Press, New York, 1981.<br />

[Ge]<br />

[GGK1]<br />

[GGK2]<br />

R. Gelca, Compact perturbati<strong>on</strong>s of Fredholm n-tuples, Proc. Amer. Math.<br />

Soc. 122(1994), 195–198<br />

I. Gohberg, S. Goldberg and M.A. Kaashoek, Classes of Linear <strong>Operator</strong>s,<br />

Vol I, OT 49, Birkhäuser, Basel, 1990<br />

I. Gohberg, S. Goldberg and M.A. Kaashoek, Classes of Linear <strong>Operator</strong>s,<br />

Vol II, OT 63, Birkhäuser, Basel, 1993<br />

[Go] S. Goldberg, Unbounded Linear <strong>Operator</strong>s, McGraw-Hill, New York, 1966<br />

[GL]<br />

[GGK]<br />

[Gu1]<br />

B. Gramsch and D. Lay, Spectral mapping theorems for essential spectra,<br />

Math. Ann. 192(1972), 17-32<br />

I. Gohberg, S. Goldberg, and M.A. Kaashoek, Classes Linear <strong>Operator</strong>s, II,<br />

Birkhäuser-Verlag, Bost<strong>on</strong>, 1993.<br />

C. Gu, A generalizati<strong>on</strong> of Cowen’s characterizati<strong>on</strong> of hyp<strong>on</strong>ormal Toeplitz<br />

operators, J. Funct. Anal., 124(1994), 135–148.<br />

[Gu2] C. Gu, N<strong>on</strong>-subnormal k-hyp<strong>on</strong>ormal Toeplitz operators, preprint, 2001.<br />

[Gus]<br />

[GuP]<br />

[Ha1]<br />

[Ha2]<br />

K. Gustafs<strong>on</strong>, Necessary and sufficient c<strong>on</strong>diti<strong>on</strong>s for Weyl’s theorem,<br />

Michigan Math. J. 19(1972), 71–81<br />

B. Gustafss<strong>on</strong> and M. Putinar, Linear analysis of quadrature domains II,<br />

Israel J. Math., 119(2000), 187-216.<br />

P.R. Halmos, Ten problems in Hilbert space, Bull. Amer. Math. Soc.,<br />

76(1970), 887–933.<br />

P.R. Halmos, Ten years in Hilbert space, Integral Equati<strong>on</strong>s <strong>Operator</strong> <strong>Theory</strong>,<br />

2(1979), 529–564.<br />

[Ha3] P.R. Halmos, A Hilbert space problem book, Springer, New York, 1982.<br />

218


BIBLIOGRAPHY<br />

[HLL]<br />

[HanL1]<br />

[HanL2]<br />

[Har1]<br />

[Har2]<br />

J.K. Han, H.Y. <str<strong>on</strong>g>Lee</str<strong>on</strong>g> and W.Y. <str<strong>on</strong>g>Lee</str<strong>on</strong>g>, Invertible completi<strong>on</strong>s of 2 × 2 upper<br />

triangular operator matrices, Proc. Amer. Math. Soc. 128(2000), 119–123<br />

Y.M. Han and W.Y. <str<strong>on</strong>g>Lee</str<strong>on</strong>g>, Weyl’s theorem for algebraically hyp<strong>on</strong>ormal operators,<br />

Proc. Amer. Math. Soc. 128(2000), 2291–2296<br />

Y.M. Han and W.Y. <str<strong>on</strong>g>Lee</str<strong>on</strong>g>, Weyl spectra and Weyl’s theorem, Studia Math.<br />

148(2001), 193–206<br />

R.E. Harte, Invertibility, singularity and Joseph L. Taylor, Proc. Roy. Irish<br />

Acad. 81(A)(1981), 71–79<br />

R.E. Harte, Fredholm, Weyl and Browder theory, Proc. Royal Irish Acad.<br />

85A (2) (1985), 151-176<br />

[Har3] R.E. Harte, Regular boundary elements, Proc. Amer. Math. Soc. 99(2)<br />

(1987) 328-330<br />

[Har4]<br />

R.E. Harte, Invertibility and Singularity for Bounded Linear <strong>Operator</strong>s,<br />

Dekker, New York, 1988.<br />

[Har5] R.E. Harte, The ghost of an index theorem, Proc. Amer. Math. Soc. 106<br />

(1989), 1031-1034<br />

[HaL1]<br />

[HaL2]<br />

R.E. Harte and W.Y. <str<strong>on</strong>g>Lee</str<strong>on</strong>g>, The punctured neighbourhood theorem for incomplete<br />

spaces, J. <strong>Operator</strong> <strong>Theory</strong> 30(1993), 217–226<br />

R.E. Harte and W.Y. <str<strong>on</strong>g>Lee</str<strong>on</strong>g>, Another note <strong>on</strong> Weyl’s theorem, Trans. Amer.<br />

Math. Soc. 349(1997), 2115–2124<br />

[HaLL] R.E. Harte, W.Y. <str<strong>on</strong>g>Lee</str<strong>on</strong>g>, and L.L. Littlejohn, On generalized Riesz points, J.<br />

<strong>Operator</strong> <strong>Theory</strong> 47(2002), 187–196<br />

[Ho]<br />

[HKL1]<br />

[HKL2]<br />

T.B. Hoover, Hyperinvariant subspaces for n-normal operators, Acta Sci.<br />

Math. (Szeged) 32(1971), 109–119<br />

I.S. Hwang, I.H. Kim and W.Y. <str<strong>on</strong>g>Lee</str<strong>on</strong>g>, Hyp<strong>on</strong>ormality of Toeplitz operators<br />

with polynomial symbols, Math. Ann. 313(1999), 247–261.<br />

I.S. Hwang, I.H. Kim and W.Y. <str<strong>on</strong>g>Lee</str<strong>on</strong>g>, Hyp<strong>on</strong>ormality of Toeplitz operators<br />

with polynomial symbols: An extremal case, Math. Nach. 231(2001), 25–38.<br />

[HwL1] I.S. Hwang and W.Y. <str<strong>on</strong>g>Lee</str<strong>on</strong>g>, The spectrum is c<strong>on</strong>tinuous <strong>on</strong> the set of p-<br />

hyp<strong>on</strong>ormal operators, Math. Z. 235(2000), 151–157<br />

[HwL2]<br />

[HwL3]<br />

I.S. Hwang and W.Y. <str<strong>on</strong>g>Lee</str<strong>on</strong>g>, The boundedness below of 2 × 2 upper triangular<br />

operator matrices, Integral Equati<strong>on</strong>s <strong>Operator</strong> <strong>Theory</strong> 39(2001), 267–276<br />

I.S. Hwang and W.Y. <str<strong>on</strong>g>Lee</str<strong>on</strong>g>, Hyp<strong>on</strong>ormality of trig<strong>on</strong>ometric Toeplitz operators,<br />

Trans. Amer. Math. Soc. 354(2002), 2461–2474.<br />

219


BIBLIOGRAPHY<br />

[Ist]<br />

[IW]<br />

[KL]<br />

[JeL]<br />

[La]<br />

[Le1]<br />

[Le2]<br />

V.I. Istratescu, On Weyl’s spectrum of an operator. I Rev. Roum. Math.<br />

Pures Appl. 17(1972), 1049–1059<br />

T. Ito and T.K. W<strong>on</strong>g, Subnormality and quasinormality of Toeplitz operators,<br />

Proc. Amer. Math. Soc. 34(1972), 157–164<br />

I.H. Kim and W.Y. <str<strong>on</strong>g>Lee</str<strong>on</strong>g>, On hyp<strong>on</strong>ormal Toeplitz operators with polynomial<br />

and symmetric-type symbols, Integral Equati<strong>on</strong>s <strong>Operator</strong> <strong>Theory</strong>,<br />

32(1998), 216–233.<br />

I.H. Je<strong>on</strong> and W.Y. <str<strong>on</strong>g>Lee</str<strong>on</strong>g>, On the Taylor-Browder spectrum, Commun. Korean<br />

Math. Soc. 11(1996), 997-1002<br />

K.B. Laursen, Essential spectra through local spectral theory, Proc. Amer.<br />

math. Soc. 125(1997), 1425–1434<br />

W.Y. <str<strong>on</strong>g>Lee</str<strong>on</strong>g>, Weyl’s theorem for operator matrices, Integral Equati<strong>on</strong>s <strong>Operator</strong><br />

<strong>Theory</strong>, 32(1998), 319–331<br />

W.Y. <str<strong>on</strong>g>Lee</str<strong>on</strong>g>, Weyl spectra of operator matrices, Proc. Amer. Math. Soc.<br />

129(1)(2001), 131–138<br />

[LL] W.Y. <str<strong>on</strong>g>Lee</str<strong>on</strong>g> and H.Y. <str<strong>on</strong>g>Lee</str<strong>on</strong>g>, On Weyl’s theorem, Math. Jap<strong>on</strong>. 39 (1994), 545-<br />

548<br />

[LeL1] S.H. <str<strong>on</strong>g>Lee</str<strong>on</strong>g> and W.Y. <str<strong>on</strong>g>Lee</str<strong>on</strong>g>, On Weyl’s theorem (II), Math. Jap<strong>on</strong>. 43(1996),<br />

549–553<br />

[LeL2]<br />

S.H. <str<strong>on</strong>g>Lee</str<strong>on</strong>g> and W.Y. <str<strong>on</strong>g>Lee</str<strong>on</strong>g>, A spectral mapping theorem for the Weyl spectrum,<br />

Glasgow Math. J. 38(1996), 61–64<br />

[LeL3] S.H. <str<strong>on</strong>g>Lee</str<strong>on</strong>g> and W.Y. <str<strong>on</strong>g>Lee</str<strong>on</strong>g>, Hyp<strong>on</strong>ormal operators with rank-two selfcommutators<br />

(preprint)<br />

[McCYa] J.E. McCarthy and L. Yang, Subnormal operators and quadrature domains,<br />

Adv. Math. 127(1997), 52–72.<br />

[McCP]<br />

[Mor]<br />

[Mur]<br />

[NaT]<br />

S. McCullough and V. Paulsen, A note <strong>on</strong> joint hyp<strong>on</strong>ormality, Proc. Amer.<br />

Math. Soc. 107(1989), 187–195.<br />

B.B. Morrel, A decompositi<strong>on</strong> for some operators, Indiana Univ. Math. J.,<br />

23(1973), 497-511.<br />

G. Murphy, C ∗ -algebras and operator theory, Academic press, New York,<br />

1990.<br />

T. Nakazi and K. Takahashi, Hyp<strong>on</strong>ormal Toeplitz operators and extremal<br />

problems of Hardy spaces, Trans. Amer. Math. Soc. 338(1993), 753–769<br />

[Ne] J.D. Newburgh, The variati<strong>on</strong> of spectra, Duke Math. J. 18(1951), 165–176<br />

[Ni] N.K. Nikolskii, Treatise <strong>on</strong> the Shift <strong>Operator</strong>s, Springer, New York, 1986<br />

220


BIBLIOGRAPHY<br />

[Ob1] K.K. Oberai, On the Weyl spectrum, Illinois J. Math. 18 (1974), 208-212<br />

[Ob2] K.K. Oberai, On the Weyl spectrum II, Illinois J. Math. 21 (1977), 84-90<br />

[Ol]<br />

[OTT]<br />

[Pe]<br />

[Pr]<br />

[Pru]<br />

R.F. Olin, Functi<strong>on</strong>al relati<strong>on</strong>ships between a subnormal operator and its<br />

minimal normal extensi<strong>on</strong>, Pacific. J. Math. 63(1976), 221-229.<br />

R.F. Olin, J.E. Thoms<strong>on</strong> and T.T. Trent, Subnormal operators with finite<br />

rank self-commutator, (preprint 1990)<br />

C. Pearcy, Some recent developments in operator theory, C.B.M.S. Regi<strong>on</strong>al<br />

C<strong>on</strong>ference Series in Mathematics, No. 36, Amer. Math. Soc., Providence,<br />

1978<br />

S. Prasanna, Weyl’s theorem and thin spectra, Proc. Indian Acad. Sci.<br />

91(1982), 59–63<br />

B. Prunaru, Invariant subspaces for polynomially hyp<strong>on</strong>ormal operators,<br />

Proc. Amer. Math. Soc. 125(1997), 1689–1691.<br />

[Pu1] M. Putinar, Linear analysis of quadrature domains, Ark. Mat., 33(1995),<br />

357-376.<br />

[Pu2]<br />

[Pu3]<br />

[Pu4]<br />

[Put1]<br />

[Put2]<br />

[RR]<br />

[Sa]<br />

[Sch]<br />

[Shi]<br />

M. Putinar, Extremal soluti<strong>on</strong>s of the two-dimensi<strong>on</strong>al L-problem of moments,<br />

J. Funct. Anal., 136(1996), 331-364.<br />

M. Putinar, Extremal soluti<strong>on</strong>s of the two-dimensi<strong>on</strong>al L-problem of moments<br />

II, J. Approximati<strong>on</strong> <strong>Theory</strong>, 92(1998), 32-58.<br />

M. Putinar, Linear analysis of quadrature domains III, J. Math. Anal.<br />

Appl., 239(1999), 101-117.<br />

C. Putnam, On the structure of semi-normal operators, Bull. Amer. Math.<br />

Soc., 69(1963), 818–819.<br />

C. Putnam, Commutati<strong>on</strong> properties of Hilbert space operators and related<br />

topics, Springer, New York, 1967.<br />

H. Radjavi and P. Rosenthal, Invariant Subspaces, Springer, New York,<br />

1973<br />

D. Saras<strong>on</strong>, Generalized interpolati<strong>on</strong> in H ∞ , Trans. Amer. Math. Soc.<br />

127(1967), 179–203.<br />

I. Schur, Über Potenzreihen die im Innern des Einheitskreises beschränkt,<br />

J. Reine Angew. Math.147(1917), 205–232.<br />

A.L. Shields, Weighted shift operators and analytic functi<strong>on</strong> theory, Mathematical<br />

Surveys 13(1974), 49–128 (C. Pearcy, ed., Topics in <strong>Operator</strong> <strong>Theory</strong>,<br />

A.M.S. Providence, 1974)<br />

221


BIBLIOGRAPHY<br />

[Smu]<br />

[Sn]<br />

J.L. Smul’jan, An operator Hellinger integral (Russian), Mat. Sb. (N.S.)<br />

91(1959), 381–430.<br />

M. Snow, A joint Browder essential spectrum, Proc. Roy. Irish Acad.<br />

75(A)(1975), 129–131<br />

[Sta1] J.G. Stampfli, Hyp<strong>on</strong>ormal operators, Pacific J. Math. 12(1962), 1453–1458<br />

[Sta2]<br />

[Sta3]<br />

[StX]<br />

[Sun]<br />

[Ta1]<br />

[Ta2]<br />

J.G. Stampfli, Hyp<strong>on</strong>ormal operators and spectral density, Trans. Amer.<br />

Math. Soc. 117 (1965), 469-476<br />

J. G. Stampfli, Which weighted shifts are subnormal, Pacific J. Math.<br />

17(1966), 367–379.<br />

S. A. Stewart and D. Xia, A class of subnormal operators with finite rank<br />

self-commutators, Integral Equati<strong>on</strong>s <strong>Operator</strong> <strong>Theory</strong>, 44(2002), 370–382.<br />

S. Sun, Bergman shift is not unitarily equivalent to a Toeplitz operator,<br />

Kexue T<strong>on</strong>gbao(English Ed.) 28(1983), 1027–1030.<br />

J.L. Taylor, A joint spectrum for several commuting operators, J. Funct.<br />

Anal. 6(1970), 172–191<br />

J.L. Taylor, The analytic functi<strong>on</strong>al calculus for several commuting operators,<br />

Acta Math. 125(1970), 1–38<br />

[Va] F. Vasilescu, On pairs of commuting operators, Studia Math. 62(1978),<br />

203–207<br />

[Va] F. Vasilescu, On pairs of commuting operators, Studia Math. 62(1978),<br />

203–207<br />

[Wer] J.Wermer, Banach algebra and analytic functi<strong>on</strong>s, Adv. Math. 1(1961), 51–<br />

102.<br />

[Wes]<br />

[We]<br />

T.T. West, The decompositi<strong>on</strong> of Riesz operators, Proc. L<strong>on</strong>d<strong>on</strong> Math. Soc.<br />

16(1966), 737–752<br />

H. Weyl, Über beschränkte quadratische Formen, deren Differenz vollsteig<br />

ist, Rend. Circ. Mat. Palermo 27(1909), 373–392<br />

[Wi1] H. Widom, Inversi<strong>on</strong> of Toeplitz matrices (II), Illinois J. Math. 4(1960),<br />

88–99<br />

[Wi2]<br />

[Xi1]<br />

[Xi2]<br />

H. Widom, On the spectrum of a Toeplitz operator, Pacific J. Math.<br />

14(1964), 365–375<br />

D. Xia, Analytic theory of subnormal operators, Integral Equati<strong>on</strong>s <strong>Operator</strong><br />

<strong>Theory</strong>, 10(1987), 880–903.<br />

D. Xia, On pure subnormal operators with finite rank self-commutators and<br />

related operator tuples, Integral Equati<strong>on</strong>s <strong>Operator</strong> <strong>Theory</strong>, 24(1996), 107–<br />

125.<br />

222


BIBLIOGRAPHY<br />

[Xi3]<br />

[Yak1]<br />

[Yak2]<br />

[Yak3]<br />

D. Xia, Hyp<strong>on</strong>ormal operators with rank <strong>on</strong>e self-commutator and quadrature<br />

domains, Integral Equati<strong>on</strong>s <strong>Operator</strong> <strong>Theory</strong>, 48(2004), 115-135.<br />

D. Yakubovich, Subnormal operators of finite type. I, Rev. Mat. Iberoamericana,<br />

14(1998), 95-115.<br />

D. Yakubovich, Subnormal operators of finite type. II, Rev. Mat. Iberoamericana,<br />

14(1998), 623-681.<br />

D. Yakubovich, A note <strong>on</strong> hyp<strong>on</strong>ormal operators associated with quadrature<br />

domains, <strong>Operator</strong> <strong>Theory</strong>: Advances and Applicati<strong>on</strong>s, 123, 513-525,<br />

Birkhaüser, Verlag-Basel, 2001.<br />

[Ya1] K.-W. Yang, Index of Fredholm operators, Proc. Amer. Math. Soc. 41<br />

(1973), 329–330.<br />

[Ya2]<br />

[Zh]<br />

K.-W. Yang, The generalized Fredholm operators, Trans. Amer. Math. Soc.<br />

216 (1976), 313–326.<br />

K. Zhu, Hyp<strong>on</strong>ormal Toeplitz operators with polynomial symbols, Integral<br />

Equati<strong>on</strong>s <strong>Operator</strong> <strong>Theory</strong>, 21(1995), 376–381<br />

223

Hooray! Your file is uploaded and ready to be published.

Saved successfully!

Ooh no, something went wrong!